You are on page 1of 193

‫بسم هللا الرمحن الرحمي‬

‫‪J[L‬‬

‫(فَأَ َّما َّالزبَدُ فَيَ ْذه َُب ُجفَ ًاء َوأَ َّما َما ي َ ْنفَ ُع النَّ َاس فَيَ ْم ُك ُث يِ‬
‫اَّلل ْ َاْل ْمث َا َل)‬ ‫ْ َاْل ْر يض كَ َذ ي َِل ي َ ْ ي‬
‫ْض ُب َّ ُ‬

‫قال رسول هللا صيل هللا عليه "اذا مات ابن أدم انقطع معهل‬
‫اال من ثالث‪ :‬صدقة جارية‪ ،‬أو عمل ينتفع به‪ ،‬أو ودل صاح‬
‫يدعو هل"‬

‫اللهم اجعل هذا العمل خالصا لوهجك وان يرباان ممن يس تغهل‬
‫ملصاح ماديه او معنويه وان يرزقنا حسن العمل‪.‬‬
‫ال نرجو منمك سوي دعوة صادقه من القلب عيس ان تكون‬
‫من نصيبنا فاحب الاعامل ايل هللا ادلعاء‪.‬‬

‫نسأل هللا التوفيق والنجاح والسداد وان يرزقمك ما تمتنون‬


‫واس تغفر هللا يل ولمك‬

‫(امللف الرئييس)‬

‫‪EMS‬‬
{1}

1( After subtotal thyroidectomy for patient. He complains restlessness the nurse is assessing his vital signs.
BP is 150/100 Heart rate is 120. Temperature 40
The nurse should expect that the cause for his condition is:
A. Hyper-metabolism due to increase in T3&T4 B. Hypo metabolism due to decrease T3&T4
C. Hypo metabolism due to increase in T3&T4 D. Hyper metabolism due to decrease in T3&T4 Answer: A
2) Calculate Total intake and output Patient in OR
1500ml Ringer,05 ml antibiotic, Nasogastric tube feeding 50ml, Blood loss 500ml,
Urine out 120
Answer: Total Intake = 1600 / Total Output = 620
3) Mother with hypothyroidism high risk for??
A. Preterm labor B. Hemorrhage C. Congenital anomalies D. Eclampsia Answer: A
4) Mother with hyperthyroidism high risk for??
A. Preterm labor B. Pre-eclampsia C. Hemorrhage D. Congenital anomalies Answer: B
5) Post right-side mastectomy where to take blood pressure?
A. Right brachial B. Left brachial. C. Right radial D. Left radial. Answer: B
6) one-week short-term goal for a total hip replacement patient
A sits up by himself three times a day B. state pain control Answer: B
7) Broken infusion machine - what to do
A. Put a sticker and report B. Put in the broken machines area Answer: A
8) ABG results - for an underweight girl 2 times?
A. one is ABG disorder - Metabolic Alkalosis. B. uncompensated metabolic alkalosis
C. Compensated metabolic acidosis. Answer: C
9) How to know it the left shoulder dislocated??
A. left arm longer than right arm B. patient cannot move the left shoulder.
C. swelling in the left shoulder Answer: A
10) A patient is admitted to the ER after sustaining abdominal injuries and a broken femur from a motor
vehicle accident. He is pale, diaphoretic, and is not talking coherently. Vital signs upon admission are
temperature 98.0 F (36.3 C), Pulse: 130 beats/minute, RR: 34 B/min, Bl. pressure 50/40mmHg.The healthcare
provider suspects which type of shock?
A. Distributive B. Neurogenic C. Cardiogenic D. Hypovolemic Answer: D
11) Which of the following the arranged assessment for postoperative patient???
A. Cardiovascular, Respiration, Surgery site, Neurological signs.
B. Respiration, Cardiovascular, Neurological signs, Surgery site.
C. Neurological signs, Cardiovascular, Respiration, Surgery site.
D. Surgery site, Respiration, Cardiovascular, Neurological signs. Answer: B
12) A woman at 24 weeks' gestational age has fever body ache and has coughing last 5 days she is sent to
hospital with admission prescriptions for H1N1influenza. which prescription has the highest priority?
A. Assign private room B. Vital signs every 4 hours
C. Obtain specimens for culture D. Ringer lactate IV Answer: A
13) What is most appropriate blood test for patient who is going to surgery?
A.WBC B. Albumin C. Prothrombin time D. Hemoglobin Answer: C

ِ‫يِاْلَ أرض‬
‫أ‬ ُ ‫َم‬
ُ ‫ك‬
‫ثِِف‬ َ ‫اس‬
‫ِِفي أ‬َ َّ ‫ع‬
‫ِالن‬ ُ ‫ف‬
َ ‫ماِيَ أن‬ َّ َ‫وأ‬
َ ِ‫ما‬ َ
{2}

14) Physician order three times the normal dose - when the nurse calls him, he insists on giving the ordered
medication - what does the nurse do?
A. Call pharmacy B. Call another doctor C. Call nurse supervisor Answer: C
15) Doctor's order excessed the recommended dose what to do?
A. Call the doctor to clarify. B. Administer the dose.
C. Hold the dose. D. Talk with supervisor. Answer: A
16) Major depression activities
A. Watch a movie with other patients. B. Plan today's activities with the patient Answer: B
17) Gum and tooth pain 一 what is the cranial nerve?
A. Third nerve B. Fifth nerve C. Seventh nerve D. Ninth nerve Answer: B
18) Patient is 6 cm dilated what stage of labor?
A. First stage B. Second stage C. Third stage D. Fourth stage Answer: A

19) The nurse was caring for patient wound staples after operation. While she was preparing the patient
sneezed and one staple fell down the floor. What should the nurse do?
A. Bring the staple from floor B. Bring it and put back on wound
C. Apply dressing to the wound✅ Answer: C
20) Recurrent urinary tract infection what advice to give?
A. Drink 2-3 liters of water B. Wipe from back to front
C. Urinate only when you have the urge. Answer: A
21) What is the complication that needs observation after the birth of the child for diabetic mother?
A. Preterm labor B. Term labor C. Macrosomia D. Hyperglycemia Answer: C
22) After cardiac catheterization, the nurse should do first ?
A. Assess blood pressure B. Assess apical pulse /distal site✅
C. Elevated the head of the bed Answer: B
23) post- operative patient to be discharged and his temperature is 37.6 at 8:00 - everything else is normal
what to do?
A. informs the physician to delay discharge.
B. gives aspirin dose earlier than scheduled.
C. temperature is normal because of the time.
D. suspected infection Answer: C
24) pregnant women Said to nurse I feel I depends on my husband and my family?
A-self concern B-ambivalence C-normal D-esteem Answer: A
25) What is the incidence rate Definition?
A. All new cases of specific disease during giving time. B. All old cases of specific disease during giving time.
C. All new and old cases of specific disease during giving time. Answer: A
26) What is the definition of prevalence rate?
A. All new cases of specific disease during giving time. B. All old cases of specific disease during giving time.
C. All new and old cases of specific disease during giving time. Answer: C
27) Woman went to gynecological clinic for routine examination then she diagnosed for positive Chlamydia.
What is this considered? A. Clinical B. Sub clinical C. Suspect Answer: B

ِ‫يِاْلَ أرض‬
‫أ‬ ُ ‫َم‬
ُ ‫ك‬
‫ثِِف‬ َ ‫اس‬
‫ِِفي أ‬َ َّ ‫ع‬
‫ِالن‬ ُ ‫ف‬
َ ‫ماِيَ أن‬ َّ َ‫وأ‬
َ ِ‫ما‬ َ
{3}

28) What is New-born examination after delivery?


A. Vital Signs. Inspection. Auscultation. Percussion. Palpation.
B. Inspection. Auscultation. Percussion. Palpation. Vital Signs
C. Palpation Inspection. Auscultation. Percussion. Vital Signs
D. Auscultation. Inspection. Palpation. Percussion. Vital Signs Answer: A
29) The doctor order 0.2 g, the available 400 mg in quantity 10 ml, how many
ml needed??
A- 2.5ml B- 5ml
C- 2ml D- 4ml Answer: B

30) A 28-year man admitted to orthopedic ward complaining of throbbing pain


in casted leg. Which of the following nursing intervention should be taken first?
A. Remove the cast B. Notify a doctor
C. Assess pedal pulse. D. Administer PRN medication Answer: C
31) Doctor writes new order restrain (PRN) for Psychiatric patient that he is
overly aggressive. Which of the following indicate that the nurse should do??
A. Call doctor while patient aggressive to do that by himself.
B. Apply the order one time only or if needed.
C. Close patient room and do not apply restrain.
D. Express that is not acceptable and complains about your supervisor. Answer: B
32) The Oral contraceptives increase risk for any cancer for long-term using??
A. Cervical cancer B. Ovarian cancer
C. Skin cancer D. Breast cancer Answer: D
33) Yemeni soldier caught in the war and he got many injuries. The patient anxious and he is speaking about
death for the nurse. What is the most appropriate action for the nurse??
A -You say no or yes. B. Why do you say you are dying?
C. Why you say you will die. D. Restating for patient wards Answer: D
34) There is someone his father is dead. Then he turned his sadness and anger by learning martial arts. What
is the defense mechanism that he used?
A. Displacement B. Sublimation
C. Projection D. Denial Answer: B
35) In teaching a female client who is HIV-positive about pregnancy, the nurse would know more teaching is
necessary when the client says:
A. The baby can get the virus from my placenta.” B. “I’m planning on starting on birth control pills.”
C. “Not everyone who has the virus gives birth to a baby who has the virus.”
D. “I’ll need to have a C-section if I become pregnant and have a baby.” Answer: D
36) The neonate is delivered by cesarean section the baby should be transported to NICU. Which type of
baby incubator should be used?
A - Closed box incubators B - Portable incubators
C - Double - walled incubators D- Servo - control incubators Answer: B
37) Therapeutic communication is used by the nurse to create a beneficial outcome in the delivery of quality
nursing care.Which of the following statement best describe the term ‘’therapeutic communication’’?
A. interactive process of socialization. B. Assessment component of nursing process
C. Communication with patient through words and language

ِ‫يِاْلَ أرض‬
‫أ‬ ُ ‫َم‬
ُ ‫ك‬
‫ثِِف‬ َ ‫اس‬
‫ِِفي أ‬َ َّ ‫ع‬
‫ِالن‬ ُ ‫ف‬
َ ‫ماِيَ أن‬ َّ َ‫وأ‬
َ ِ‫ما‬ َ
{4}

D. Process to encourage the expression of feelings and needs. Answer: D

38) Click image to enlarge A 56 - year - old patient presents with psoriasis. Secondary lesions are present on the
upper and lower extremities (see image). Which of the following best describes the appearance of psoriasis?
A. Scales B. Crusts
C. Scars D. Fissures Answer: A
"‫” نركز في كل األنواع بشكل الصور ألنه بيجيب الصورة ويسأل على االسم‬
39) While taking the history from a new patient, the
nurse densified that he had hypomanic episode which was
alternating with a mirror depressive\episode for the last
two years. what is the most likely diagnosis?
A. Bipolar I disorder B. Bipolar II disorder
C. Dysthymic disorder D. Cyclothymic disorder
Answer: D

40) Talking with the parents of a child with Down


syndrome, which of the Police would the appropriate foal
for the care of the child?
A. Encourage self-care skills in the child.
B. Teaching the child something new each day
C. Encourage more lenient behavior limits for the Cho.
D. Achieving age-appropriate social skills Answer: D
41) 28-year-old married woman is admitted in the Gynecology Ward for the observation after her
miscarriage. She has not been able to sustain any pregnancy since she got married for the past three years and
she had three miscarriages during this time.
What intervention is the most appropriate?
A. Family history B. Past medical history
C. Physical examination D. Laboratory investigations Answer: B
42) It is a managerial function that indicates leading the staff in the most effective method.
A: Planning B: Directing
C: Organizing D: Controlling Answer: B
43) Which of the following best describes the role of a case management?
A. promotes healthy lifestyle. B. provides coordination of care to ensure continuity.
C. possesses highly skilled communication methods.
D. promotes the comfort of the patient. Answer: B
44) A 30-year-old female patient went to the clinic for a consultation after her work due to Pain no both legs
during prolongs standing and sitting Doppler ultrasound was cone and confirmed the initial diagnosis of
varicose veins. The patient asked what is the possible surgical procedure in relation to her case.
A. Amputation B.Sclerotherapy C. Thermal ablation D.Ligation and stripping Answer: D
45) Leaving the tourniquet for a long-time cause?
A- Low blood flow B- Limb embolism C- Hemoconcentration Answer: C
46) Which drug would be used to treat a patient who has increased intracranial pressure (ICP) resulting from
head trauma after an accident:
a-Mannitol b-Atropine sulphate c- epinephrine hydrochloride d-Sodium bicarbonate Answer: A
47) Leaving the tourniquet for a long-time cause:

ِ‫يِاْلَ أرض‬
‫أ‬ ُ ‫َم‬
ُ ‫ك‬
‫ثِِف‬ َ ‫اس‬
‫ِِفي أ‬َ َّ ‫ع‬
‫ِالن‬ ُ ‫ف‬
َ ‫ماِيَ أن‬ َّ َ‫وأ‬
َ ِ‫ما‬ َ
{5}

A. Embolism B. Low blood flow C. Ischemia Answer: B


48) When the nurse on duty accidentally bumps the bassinet. the neonate throws out its arms. hands opened.
and begins to cry. The nurse interprets this reaction as indicative of which of the following reflexes?
A. Startle reflex B. Babinski reflex
C. Grasping reflex D. Tonic neck reflex Answer: A
49) Women come to ER with her husband and the husband demonstrate she not talk or voluntary eat since
the son died in accident, what the women need according to Maslow?
A. loving and belonging B. psychological needs
C. spiritual support D. family support Answer: A
50) Jizan province is poised to achieve malaria elimination. There is a need to change from a policy of passive
case detection to reactively and proactively detecting infectious reservoirs that require new approaches to
surveillance. For discovering health problems and diseases inside the community. What is the priority of
surveillance?
A. Social surveillance B. Financial surveillance and previous health diseases.
C. Incidence and prevalence of the health diseases. Answer: C
51) Jizan province is poised to achieve malaria elimination. There is a need to change from a policy of passive
case detection to reactively and proactively detecting infectious reservoirs that require new approaches to
surveillance. For discovering health problems and diseases inside the community. What is the priority of
surveillance?
A. Social surveillance B. Public surveillance and prevent infection.
C. Financial surveillance and previous health diseases. Answer: B
52) Jizan province is poised to achieve malaria elimination. There is a need to change from a policy of passive
case detection to reactively and proactively detecting infectious reservoirs that require new approaches to
surveillance. For discovering health problems and diseases inside the community. What is the priority of
surveillance?
A. Social surveillance B. Surveillance for previous and new health problems
C. Financial surveillance and previous health diseases. Answer: B
53) Why do we use “log rolling" Technique?
A) Facility good circulation B) Avoid movement
C) To protect the spinal cord from moving Answer: C
54) A 38-year- old patient is about to have lumber disk surgery. during preoperative care, the nurse instructs
the patient including the family. members how to do "log rolling" to change patient position. One of the
family members ask why they must do such action in turning the patient. postoperative.
Which of the following is the nurse best response?
A. Facility good circulation B. Avoid movement
C. Prevent post-operative bed sore. D. Makes changing of patient position easier. Answer: C
55)The nurse is caring for 6 patients. Providing for them all care and activities. What is the type of nurse
according to nursing care delivery system?
A⁃ primary B⁃ Case management
C⁃ Functional Answer: A
56) The patient is receiving hemodialysis. The patient is experiencing disequilibrium syndrome. What is the
cause of the syndrome?
A. Increase urea and Na B. Decrease urea and Na Answer: B

ِ‫يِاْلَ أرض‬
‫أ‬ ُ ‫َم‬
ُ ‫ك‬
‫ثِِف‬ َ ‫اس‬
‫ِِفي أ‬َ َّ ‫ع‬
‫ِالن‬ ُ ‫ف‬
َ ‫ماِيَ أن‬ َّ َ‫وأ‬
َ ِ‫ما‬ َ
{6}

57) The pregnant woman is experiencing muscle cramps in her leg. She has less calcium intake. What should
the nurse instruct her during spasm?
A. knee flexion B. knee extension Answer: B

58) Classical sign of adrenal insufficiency?


A. Hypernatremia B. Hypotension C. Hyperpigmentation Answer: C
59) position after appendectomy?
A-high fowlers B-semi fowler
C-sitting D-Dorsal recumbent Answer: B
60) How the nurse ensure that the mother is breast feeding well?
A. It is normal that suppose you know B. Referral to breast feeding educator for one day
C. Check mother while she is breast feeding and progression✅ Answer: C
61) What is type of transport that carries oxygen and carbon dioxide from and to placenta during pregnancy?
A. Simple diffusion B. Active transport C. Simple transport Answer: A
62) Patient has difficulty in walking and lethargic, there decrease in capillary refill and urine incontinence,
has risk for?
A-aspiration B-infection
C-skin breakdown D-body disturbance Answer: C
63) Child with thalassemia pale and activity intolerance what the priority?
A-RBC transfusion B-oxygen administration Answer: B
64) The nurse is giving instruction for a pregnant woman. Which of the following should be avoided during
pregnancy?
A-raw uncooked met. B-clean well fruit and vegetables
C-cooked fish and chicken Answer: A
65) child has urine incontinence two day ago with WBC in urine.?
A-psychological abuse B-urinary tract infection
C-genitalia defect Answer: B
66) pt. has difficulty taking liquid medications from a cup. How should the nurse administer the medications?
A. Request that the physician change the order to the IV route.
B. Administer the medication by the IM route.
C. Use a needleless syringe to place the medication in the side of the mouth.
D. Add the dose to a small amount of food or beverage to facilitate swallowing. Answer: C
67) Patient diagnosed with appendicitis which statement by patient indicates absolute ruptured of
appendicitis?
A -I feel like I want to vomit. B- Pain in the epigastric
C - Painless in right quadrant / No hurts suddenly ✔️ Answer: C
68) Nursing supervisor newly elected Wants to use" disciplinary evaluation to nurses’behavior "
What type of power she is using?
A. Reward B. Coercive C. legitimate D. Formal Answer: Bbb
69) The patient weight 67 kg. He is receiving drug 30mg /kg/24hr. How many grams should the nurse give for
24 hr.?
A.5 g B. 1.5g
C. 1 g D. 2g Answer: D

ِ‫يِاْلَ أرض‬
‫أ‬ ُ ‫َم‬
ُ ‫ك‬
‫ثِِف‬ َ ‫اس‬
‫ِِفي أ‬َ َّ ‫ع‬
‫ِالن‬ ُ ‫ف‬
َ ‫ماِيَ أن‬ َّ َ‫وأ‬
َ ِ‫ما‬ َ
{7}

70) Based upon the legal guidelines for documentation, which of following corrective action is correct?
A. Never erase entries do line under ward and write correct
B. erases the incorrect notes then writes the correct Answer: A
71) The nurse is caring for child regarding his activity. Which of the following is considering fine? Motor
activity? A. Ride bicycle B. Walking
C. Holding cup Answer: C

72) When the Zygote enter to the uterus after fertilization?


A. 2 day B. 4 day
C. 6 day D. 8 day Answer: B
*Take care:
*Zygotes enter to the uterus 3 _ 5 day
* Implant 7 _ 10 day after fertilization

73) A nurse is assisting during a normal vaginal delivery on


a 22-year diabetic patient. The head was delivered without any
Complication head suddenly retracts against the perineum
Prompting the physician to immediately ask for the nurse assistance with this dystocia. Which of the following
will be the nurse appropriate action to impacted shoulders of the infant?
A. Fracture the infant’s clavicle.
B. Prepare patient for immediate caesarean section.
C. Apply fundal pressure to displace anterior shoulder.
D. Perform supra public pressure to release anterior shoulder. Answer: D
74) The community nurse was providing home visit for a family to give health instruction. The nurse observed
while she entered the garage, she was finding the car's oil had expired. What would you do?
A. get rid of oil and throw it away junk.
B. Call to the company and change it.
C. Use the oil
D. Give it to the neighbors. Answer: B
75) What are the symptoms of the BCG vaccination side effect?
A. Diarrhea B. Skin ulcer or scar C. no symptoms D. Seizure Answer: B

76) How many times shower the baby?


A- twice a week with warm water B- Every day with warm water
C- Every day with warm water and acidic soap D- Every day with warm water and alkaline soap Answer: A

77) A nurse is caring for a 72-year-old man patient, is unsteady. The patient requests the nurse to help nurse
asked the patient to wait few minutes' device to transfer him. Which of the following transfer devices is the
nurse to use? A. Belt B. Board C. Handle D. Mechanical lift Answer: A
78) Neonate is near to cold window what is the type of heat loss?
A. Radiation B. Convection C. Conduction D. Evaporation Answer: A

ِ‫يِاْلَ أرض‬
‫أ‬ ُ ‫َم‬
ُ ‫ك‬
‫ثِِف‬ َ ‫اس‬
‫ِِفي أ‬َ َّ ‫ع‬
‫ِالن‬ ُ ‫ف‬
َ ‫ماِيَ أن‬ َّ َ‫وأ‬
َ ِ‫ما‬ َ
{8}

79) The nurse has been teaching a new mother how to feed was born with a cleft lip and palate before surgical
repair of Which of the following action from the mother indicate teaching has been successful?
A. burping the baby frequently ✅✅ B. Prevent the infant from crying
C. Placing the baby flat during feeding D. Keep the infant prone following feedings Answer: A
80) The age of Cleft lip surgery is 2 – 6 months.
81) Definition of Word Salad: Words or phrases are connected meaninglessly being forced.
82) A 49-year-old patient was admitted in the Medical Ward to rule out his pale appearance, loss of appetite,
abdominal pain in the right upper quadrant, dark urine and grey colored stools for the last few days. He was
diagnosed with Hepatitis C and was discharge from the hospital when his condition gets stable. He was given
the advice to take his medication regularly. What should be the nurse’s prime focus in the discharge teachings
for the patient?
A. Family support
B. Dietary counselling
C. Activity and exercise
D. Understanding of the disease Answer: D
83) The nurse is assessing a 2 -years-old child with Wilms surgery Which of the following should the nurse
avoid?
A. Putting the child in lateral position. B. Palpating the child's abdomen.
C. Putting the child in a private room. D. Provide mouth hygiene 30 minutes after meal. Answer: B
84) Mother came to the Outpatient Department with an infant having cleft and palate. The infant was
underweight, so the nurse has to consider Teaching the proper way of feeding the child in the treatment plan.
Which of the following is the proper way of feeding
A. Use a non-squeezable bottle during feeding
B. Feed infant in an upright, sitting position ✅✅
C. Enlarge nipple holes of bottle to allow more milk to pass through
D. Feed infant longer than 45 minutes to allow more food to be small Answer: B
85) What is the type of umbilical cord care for neonate?
A. Curing and treatment B. Infection prevention Answer: B
86) After lumbar puncture position?
A. Supine B. Prone Answer: A
87) Elderly Immunization - - - - - - Pneumonia, Influenza
88) What Is The Action To Prevent Cleft Lip & Palate?
A. Give immunizations B. Advice pregnant folic acid Answer: B
89) Which of the following vitamin supplements can decrease the incidence of Neural tube defects such as
anencephaly and spina bifida new-born?
A. Vitamin A B. Riboflavin C. Folic Acid D. Vitamin K Answer: C
90) As identified by DR Elizabeth Kubler which stage of dying is characterized by the transition from ‘NO’
not me to “yes, me but....”
A. Anger B. Depression
C. Acceptance D. Bargaining Answer: D

ِ‫يِاْلَ أرض‬
‫أ‬ ُ ‫َم‬
ُ ‫ك‬
‫ثِِف‬ َ ‫اس‬
‫ِِفي أ‬َ َّ ‫ع‬
‫ِالن‬ ُ ‫ف‬
َ ‫ماِيَ أن‬ َّ َ‫وأ‬
َ ِ‫ما‬ َ
{9}

91) The head nurse gives low evaluation to the nurse because of her late. What is the type of her evaluation
effect?
A. Halo effect B. Horn effect C. Central tendency Answer: B
92) The head nurse is too busy to give evaluation to her staff. She gave to everyone average evaluation. What
is the evaluation type that the head nurse used?
A. Halo effect B. Horn effect C. Central tendency Answer: C
93) nurse educator said to a new nurse" our hospital provides the highest level of specialized healthcare in an
integrated educational and research setting" which of the following aspects of the hospital's strategic plan is
indicated by this statement?
A. Vision B. Policy C. Values D. Mission Answer: D
94) New-born has small, whitish, pinpoint spots over the nose, which the nurse knows are caused by retained
sebaceous secretions. When charting this observation, the nurse identifies it as:
A. Milia B. Lanugo
C. Whiteheads D. Mongolian spots Answer: A
95) The community health care Facility observed low response for immunizations activities. Then it decided to
teach people about immunizations and the importance for applying it. What would improve the effect of
response?
A. Modify immunizations program schedule.
B. Provide more health teaching programs. Answer: B
96) Normal Fetal Hr 120-160
97) Which patient to see first:
A. Patient with Potassium level 7.5 B. Anemia client with pain level 6/10 Answer: A
98) The speed of transmission of the disease from person to person?
A. Infective✅ B. Infection C. Infectivity D. Precaution Answer: A
99) Why should the community nurse know about the disease in other countries?
A-To give patient needs accordingly ✔️✔️.
B-To know the perveance of the disease in certain country.
C- To help countries for solutions D- To make research and studies Answer: A
100) HIV patient will be risk for which life threading disease?
A. Pneumonia B. Tuberculosis Answer: B
101) couple has a son and a daughter, and both moved out and got family on their own. Which type of family
structure? A. Extended B. Blended Answer: A
102) Patient with cardiac tamponed muffled heart sound which of the following the nurse monitor?
A. Pulse and respiratory rate. B. Paradoxical pulse 40 in inspiration. Answer: B
103) Treatment of pyloric stenosis? * Rehydration and pyloromyotomy
104) Which medicine prescription the nurse should question?
A. Lasix 40mg oral BID B. Codeine oral PRN Answer: B
105) Float nurse from medicine floor. Came to telemetry unite which patient she receives?
A. MI patient in heparin infusion B. Hypertensive patient on Lasix.
C. Atrial Fibrational with anticoagulant drug. Answer: C
106) Cause of death in Saudi Arabia? *ischemic heart disease and road accident.

ِ‫يِاْلَ أرض‬
‫أ‬ ُ ‫َم‬
ُ ‫ك‬
‫ثِِف‬ َ ‫اس‬
‫ِِفي أ‬َ َّ ‫ع‬
‫ِالن‬ ُ ‫ف‬
َ ‫ماِيَ أن‬ َّ َ‫وأ‬
َ ِ‫ما‬ َ
{ 10 }

107) Diphtheria which type of isolation? A. Droplet B. Airborne Answer: A


108) Patient with schizophrenia seen with which of the following signs?
A -Word salad B -Associates lessness Answer: B
109) Why hemorrhage occurs postpartum?
A-prolonged labor last 12 hours B-excusive analgesics given during labor. Answer: B
110) How measure Orthostatic hypotension? * The Systolic is <20, and the Diastolic is <10.
111) A male client has received a prescription for orlistat for weight and nutrition management. In addition to
the medication, client states plan to take a multivitamin. what teaching should a nurse provide?
A. Following a well-balanced diet is a much healthier approach to good nutrition than depending on a
multivitamin. Be sure to take the multivitamin and the medication at least two hours apart.
B. As a nutritional supplement, orlistat contains all the recommended daily vitamins and minerals.
C. Multivitamins are contraindicated during treatment with weight control medications such.
D. as orlistat Answer: A

*Take care: Disfral or Orlistat is medication of (Beta-thalassemia) - - - - - take multiple vitamins and
this medicine 2 hour apart.
112) How to prevent cleft lip and palate from occurring?
A-eat green vegetables and citrus fruit B- Drink too much juice Answer: A
113) IQ 45 what level of mental retardation?
A-moderate mental retardation B-severe mental retardation.
C-mild mental retardation. Answer: A
114) IQ is mild retardation characteristic?
A-needs assistance all time.
B-can learn some of daily life activities but still needs helps. Answer: B
115) Nurse plan community program to those who experienced a disease And assess the recovery or
rehabilitation or dis ability Which level of prevention?
A-primary B-secondary C-tertiary Answer: C
116) What is an example of primary level of prevention?
A-how to be smoking cessation. B-Given education on HIV to high school students. Answer: B
117) During haji nurse is caring for non-Arabic speaker What is most appropriate action?
A-Use google translation. B-ask family members to interpret.
C-call hospital Interpreter services Answer: C

118) A5 - year - old child was brought to the Emergency Room with a fractured of forearm. He had several
bruises on his body but showed no signs of pain while palpating them. He seemed scared and did not answer
any questions asked. How should the nurse initiate therapeutic communication with the child?
A. Start interviewing B. Encourage him to speak. C. Explain about the fracture
D. Greet and show gentleness - nurse gives health. Answer: D

119) The nurse administered a prescribed intramuscular medication to a patient during a home health visit.
How should the nurse dispose of the used needle and syringe?
A) Recap the needle, then place the needle and syringe into a waterproof. container until safe disposal can be made.
B) Bend the needle back towards the barrel of the syringe before putting. the needle and syringe in a metal trash
container.

ِ‫يِاْلَ أرض‬
‫أ‬ ُ ‫َم‬
ُ ‫ك‬
‫ثِِف‬ َ ‫اس‬
‫ِِفي أ‬َ َّ ‫ع‬
‫ِالن‬ ُ ‫ف‬
َ ‫ماِيَ أن‬ َّ َ‫وأ‬
َ ِ‫ما‬ َ
{ 11 }

C) Wrap the needle and syringe in disposable paper before putting the needle and syringe into the dirty section of
the nurse’s equipment bag.
D) Put the needle and syringe directly into a puncture-resistant plastic container that has a lid. Answer: D

120) Patients eat contaminated food and the diagnosed with hepatitis A as result of that food which of the
following s related to hepatitis A disease: -
A-The disease is transmitted through the blood. B- The disease is transmitted through sexual contact.
C- Incubation period 3-5weeks. D- Incubation period 4-5 month Answer: C
121) A 16-year-old girl developed an infection over the surface of the heart after having her nose pierced to
place jewelry on admission, she had developed pyrexia of 38C, a heart murmur and petechiae over the whole
body. She was admitted to the hospital and treated with intravenous antibiotics. The nurse explains to the
patient that she is at high-risk for re-infection and provides discharge teaching on preventive measures.
Which of the following would most likely require prophylactic treatment?
A. pelvic examination B. dental Care C. bronchoscopy D. urinary catheterization Answer: B
122) 5-year-old child was admitted with Nephrotic Syndrome. A nurse noticed that the child has slight facial
puffiness with mild pitting edema on his hands and feet. there was no distended abdomen diet the nurse
should order for the child.
A. High protein, high salt diet
B. Low protein, low fiber diet
C. Low protein, normal salt diet
D. Normal protein, low salt diet Answer: D
123) Managers create a new position patient educator work with head nurse, what organizational
relationship?
A. Square B. Solid line C. Matrix D. Dotted line Answer: D
124) There is a new community nurse assigned to work in a Makkah, one of the most multicultural cities in
Saudi Arab has to interact with multicultural population every day. What is the most important step the
nurse should do before with the clients?
A. Asking the client about his/her cultural background
B. Conducting an appropriate culturological assessment.
C. Ensuring that the client has his/her a privacy.
D. Looking at the client's file to take history. Answer: B
125) Doctor writes DNR on a patient with end stage cancer, the patient tells the nurse and express his
feelings he wants to be resuscitation, what is your nursing action?
A. explain to the patient to except DNR.
B. ignore the patient feeling.
C. Follow the hospital policy.
D. Report the doctor to Saudi Commission for Health Specialties Answer: C
126) A nurse is taking care of a woman who is in active labor. She is multipara at 38 weeks gestation. Her
cervix is 5 cm dilated at -2 station with ruptured membranes. On the fetal heart rate tracing, the nurse
observes a fetal heart rate baseline of 150 beats per minute with Deceleration in fetal heart rate to as low as
100 beats per minute. Deceleration in heart rate occurs both during and between contractions. With each
deceleration heart rate quickly returns to baseline after a few seconds. The nurse has anticipated which of the
following.?

ِ‫يِاْلَ أرض‬
‫أ‬ ُ ‫َم‬
ُ ‫ك‬
‫ثِِف‬ َ ‫اس‬
‫ِِفي أ‬َ َّ ‫ع‬
‫ِالن‬ ُ ‫ف‬
َ ‫ماِيَ أن‬ َّ َ‫وأ‬
َ ِ‫ما‬ َ
{ 12 }

A. Client will probably deliver quickly.


B. Client may need rapid intervention for a prolapsed cord.
C. Client has a normal fetal heart rate tracing for a multipara in active labor.
D. Client has a fetal heart rate tracing consistent with poor uteroplacental blood flow. Answer: B

127) Mother of nine children, three of them with congenital anomalies and one down syndrome: she is a
primary school graduate, with low. financial status. She is not using any method of family planning. So, the
primary health care nurse has referred her for counseling. Which of the following indicates that the
summarizes the important points during counseling?
A. Provision broad ideas to the client
B. Restatement for better understanding
C. Keeping silent while the client asking questions.
D. Interpreting feelings and resistance of the client Answer: B
128) What percentage of patients with ectopic pregnancy will have normally rising HCG levels?
A. 10%
B. 25%
C. 50%
D. 95% Answer: B
129) An old patient admitted to ICU with signs and symptoms of Dyspnoea, cough, expectoration, weakness,
and edema. Which of the following conditions are correct related these symptoms?
A. Pericarditis
B. Hypertension
C. Obliterative
D. Restrictive Answer: A
130) The nurse prepares cardiac patient for the insertion of a pulmonary artery catheter (Swan-Ganz
catheter). The nurse teaches the patient that the catheter will be inserted to provide information about.
A. Stroke volume
B. Venous pressure
C. Cardiac output
D. left ventricular functioning. Answer: Ddd
131) Which of the following conditions would a nurse not administer erythromycin?
A. Campylobacteriosis infection
B. Multiple Sclerosis
C. Legionnaires disease
D. Pneumonia Answer: B
132) A nurse is assessing a patient who just arrived in the emergency department after a motor vehicle
collision the patient has a strong smell of alcohol on breath is restless and has a bluish discoloration on the
abdomen by the umbilical. The patient vital signs or temperature of 37.2 heart rate 120 beats per minute and
blood pressure of 100/ 62 mmhg while other members of the team are evaluating the patient the nurse should
obtain.
A. A pair of elastic support stockings
B. a chest tube insertion tray
C. Supplies for peritoneal lavage
D. a Vial of Hydralazine Answer: C

ِ‫يِاْلَ أرض‬
‫أ‬ ُ ‫َم‬
ُ ‫ك‬
‫ثِِف‬ َ ‫اس‬
‫ِِفي أ‬َ َّ ‫ع‬
‫ِالن‬ ُ ‫ف‬
َ ‫ماِيَ أن‬ َّ َ‫وأ‬
َ ِ‫ما‬ َ
{ 13 }

133) A patient is attending a primary health clinic for regular checkup. He complains of constipation. After
assessment, the nurse instructed him to consume bulk-forming foods. Which of the following is the best bulk-
forming foods?
A. Fruit juice
B. Raw meat
C. Whole grains
D. Milk products Answer: C
134) A nurse in the postnatal ward is assigned for a multiparous patient has just delivered a healthy new-
born. When should the nurse plan to take the patient vital signs?
A. Every hour for the first 2 hours
B. Every 30 minutes during the first hour
C. Every 15 minutes during the first hour
D. Every 5 minutes for the first 30 minutes Answer: C

135) post-operative patient who underwent an abdominal procedure request pain medication from the nurse
and rates the pain at a level nine. There is a standing order for narcotic administration. When the nurse opera
the narcotic box and performs a count, the number of pills remaining in the box is different than the number
of pills. recorded on the sheet. What is the most appropriate initial nursing action?
A. Notify the nursing supervisor.
B. Write the finding on the narcotic sheet.
C. Administer the patient's requested medication.
D. Identify the last nurse who used the narcotic. Answer: A
136) Women should get their pap smear test each.
A-6-month B-1year C-3years D-5 years Answer: C
137) A woman was diagnosed gestational trophoblastic disease what is the lab investigation was done to
diagnose condition?
A. Cervical pap smear
B. Serum HCG levels
C. Serum estrogen level
D. Plasma thyroxin level Answer: B
138) The community midwife has just finished an educational session with a group of women regarding
maternal health and annual gynecological examination along with mammogram and Pap smear testing. What
outcome is the most desired?
A. Disease prevention.
B. Increased healthy living.
C. Early detection of related issues.
D. Strengthened marital relationship. Answer: C
139) Female patient has been advised that laboratory tests confirm herpes simplex virus (HSV), type 2. The
nurse should teach the patient that a Papanicolaou test (Pap smear) is recommended:
A. Every 6 months if symptoms persist despite treatment.
B. Every year even if asymptomatic whenever symptoms recur.
C. Every 3 years if other Pap smears have been negative. Answer: A
140) A female patient complains of abdominal discomfort. Watery stool has been leaking from her rectum.
This could be a sign of which of the following?
A. fecal impaction ✅ B. constipation C. bowel incontinence D. diarrhea Answer: A

ِ‫يِاْلَ أرض‬
‫أ‬ ُ ‫َم‬
ُ ‫ك‬
‫ثِِف‬ َ ‫اس‬
‫ِِفي أ‬َ َّ ‫ع‬
‫ِالن‬ ُ ‫ف‬
َ ‫ماِيَ أن‬ َّ َ‫وأ‬
َ ِ‫ما‬ َ
{ 14 }

141) Which of the following religion is prohibited the blood transfusion?


A. Christian B. Muslims C. Jewish D. Jehovah Answer: D
142) A patient, who had abdominal surgery six days ago, has been ambulating in the halls without much
difficulty. however, on seventh postoperative day, the patient complains of increased pain at incisional site and
is walking bent over. What is the most likely cause?
A. Intestinal inflammation B. Pulmonary edema
C. Wound infection D. Deep vein thrombosis Answer: C
143) A 29-year-old woman had been diagnosed with a 3 cm ovarian cyst. Which of the following is the
appropriate step in management?
A. Cyst aspiration
B. Hormonal therapy
C. Cyst removal by laparoscopy
D. Examination after next menstruation Answer: D
144) 25-year-old woman has a family history of breast cancer. The nurse reviews the procedure for breast
self- examination (BSE) and tells her that the best time for a woman to perform a breast self-examination is:
A. a few days before her period.
B. during her menstrual period.
C. on the last day of menstrual flow.
D. 3- 7days after the beginning of her period Answer: D
145) The nurse is instructing a female client how to do breast self-exam. Which of the following is the best
time to perform this exam?
A. After ovulation B. After period
C. Two weeks after period D. Three days before period Answer: B
146) At labor room a nurse assessed the condition of the patient and gathered the following data. Cervical
dilatation 2-5minutes lasting 40-60seconds increasing bloody show leg discomfort with heaviness what is the
significance of the data?
A. Patient on the first stage of labor
B. Patient on the third stage of labor
C. Patient on the second stage of labor
D. Patient is experiencing a prolonged labor. Answer: A
147) Establishes baby's hydration A 11-month-old boy with cleft lip & palate was discharged from the
hospital after surgery. The nurse provided the parents with teaching. Which of the following statement by the
parents indicate appropriate understanding of the teaching instruction?
A. I should avoid that my infant's cry as much as possible.
B. I will use a spoon to feed my child after the surgery.
C. I will put my child in his abdomen after feeding.
D. I will use lotion to ease my child's lip irritation. Answer: A
148) Epidural anesthesia site?
A. L3 - L4
B. L1 - L2
C. T3 - T4
D. C1 - C2 Answer: A
149) The scale used by a nurse to assess the gestational age of a
newborn is:
A. Bishop score B. Ballard score ✅
C. Bel ward score D. Apgar score Answer: B

ِ‫يِاْلَ أرض‬
‫أ‬ ُ ‫َم‬
ُ ‫ك‬
‫ثِِف‬ َ ‫اس‬
‫ِِفي أ‬َ َّ ‫ع‬
‫ِالن‬ ُ ‫ف‬
َ ‫ماِيَ أن‬ َّ َ‫وأ‬
َ ِ‫ما‬ َ
{ 15 }

150) On assessing the client pressure ulcer (see photo), the nurse would document this as which stage?
A Stage I B. Stage II
C. Stage III D. Stage IV Answer: A

151) Adult -15- years old hospitalized to relieve anxiety. What should the nurse do?
A. Increase numbers of staff during nursing care
B. Limited numbers of staff during nursing care✅ C. Allow parents visit him along daytime Answer: B
152) Nurse is caring for a client with a nasogastric tube that is attached to low suction.The nurse monitors the
client, knowing that the client is at risk for which acid-base disorder?
A. Metabolic acidosis B. Metabolic alkalosis
C. Respiratory acidosis D. Respiratory alkalosis Answer: B
153) Nurse is preparing to insert a nasogastric tube into a client. The nurse places the client in which position
for insertion:
A. Right side
B. Low Fowler's
C. High Fowler's
D. Supine with the head flat Answer: C
154) Mother of nine children, three of them with congenital anomalies Down syndrome. she is a primary
school graduate, with low status. She is not using any method of family planning. So, the health care nurse
has referred her. for counseling. Which of the following application the counsellor can help? Mrs. regarding
family planning
A. Prevention level B. Gather model.
C. Group teaching D. Rejection of Mrs.M expression Answer: B
155) Mother of nine children, three of them with congenital anomalies and down syndrome she is a primary
school graduate, with low financial status. She is not using any. method of family planning Accordingly, the
primary health care nurse has referred her for counselling the following. terms describe any restriction or
lack of ability to perform an activity?
A. Impairment
B. Abnormality
C. Handicapped
D. Disability Answer: D
156) Mother of nine children, three of them with congenital anomalies and one down syndrome; she is a
primary school graduate, with low financial status. She is not using any method of family planning.
Accordingly, the primary health care nurse has referred her for counseling. Which of the following is a
barrier facing the team responsible for providing health services to handicapped individuals?
A. Sensory limitations B. Rigid rules
C. Developmental disabilities D. Deafness & hearing limitations Answer: C
157) Mother of nine children, three of them with congenital anomalies Down syndrome; she is a primary
school graduate, with low status. She is not using any method of family planning. According to primary health
care nurse referred her for counseling Which of the following types of home visits that the community her
nurse should conduct for this client?
A. Systematic routine B. Selective
C. Follow up D. Field trip Answer: C

ِ‫يِاْلَ أرض‬
‫أ‬ ُ ‫َم‬
ُ ‫ك‬
‫ثِِف‬ َ ‫اس‬
‫ِِفي أ‬َ َّ ‫ع‬
‫ِالن‬ ُ ‫ف‬
َ ‫ماِيَ أن‬ َّ َ‫وأ‬
َ ِ‫ما‬ َ
{ 16 }

158) Mother of nine children, three of them with congenital anomalies and one down syndrome; she is
primary school graduate, with low financial status. She is not using any method of family planning. So, the
primary health care nurse has referred her for counseling. Which of the following is the best health education
method that can be used?
A. community organization
B. individual counselling
C. group discussion
D. health class Answer: B
159) Mother of nine children, three of them with congenital anomalies down syndrome; she is a primary
school graduate, with low first status. She is not using any method of family planning. According to primary
health care nurse has referred her counselling. Which of the following phases of home visit accomplishes
intervention?
A. Initial
B. Closing
C. Action
D. Terminal Answer: A
160) Mother of nine children, three of them with congenital an down syndrome; she is a primary school
graduate, with status. She is not using any method of family planning. So Health care nurse has referred her
for counselling which of the following must be focused on by the community nurse to provide an effective
health education?
A. Educate regardless realistic objectives.
B. Use clear and concise language.
C. Use scientific terms during explanation.
D. Explain the negative consequences in the family. Answer: B
161. A pregnant woman is 36 weeks pregnant was admitted to antenatal ward for observation after being
involved in a car accident the refuses when the nurse tried to touch her abdomen to perform examination
saying it is painful. She also present mild vaginal bleeding . Which of the following is the most appropriate
diagnosis
A. Placenta previa B. Tubal pregnancy
C. Abruptio placenta ✅ D. Inevitable abortion Answer: C

162) The nurse is assessing patient after craniotomy. The patient's blood pressure is 180/65 ICP is 25. What is
the patient's cerebral perfusion pressure?
A. 72mmHg
B. 81mmHg
C. 78mmHg
D. 83mmHg
Answer: C
*Explanation Question 162:
CPP = MAP _ ICP
A
n
(Cerebral perfusion Pressure) = (Mean Arterial Blood Pressure) _ (Intracranial Pressure)
s
w 103 _ 25 So CPP = 78 mmHg
e 2(Diastolic BP)+ Systolic BP ) (2×65)+180 310
MAP=
r 3
MAP = 3
= 3 = 103
:

C
ِ‫يِاْلَ أرض‬
‫أ‬ ُ ‫َم‬
ُ ‫ك‬
‫ثِِف‬ َ ‫اس‬
‫ِِفي أ‬َ َّ ‫ع‬
‫ِالن‬ ُ ‫ف‬
َ ‫ماِيَ أن‬ َّ َ‫وأ‬
َ ِ‫ما‬ َ
{ 17 }

163) year-old woman patient in the Medical Ward is in a semiconscious. Her pancreatic cancer is metastasized
to her liver and lungs and she admitted for supportive treatment. Her physicians discussed with the family
that she will not be given the Cardiopulmonary Resuscitation to save life if she goes into the cardiac arrest.
Her two sons agreed but the daughter is indecisive. Which of the following is the critical thinking behind not
providing discounted pulmonary resuscitation?
A. Unilateral judgment of health professionals
B. Refusal of patient’s right to treatment
C. Ethical dilemma and indecisiveness
D. Mercy killing to ease suffering. Answer: D
164) Which of the following medication should the nurse clarify if given to post-delivery client who had
preeclampsia?
A-magnesium sulphate B-methergine C-Panadol Answer: B
165) Which of the following is necessities for caesarean section?
A-preterm labor B-severe preeclampsia
C-total placenta previa D-partial placenta previa Answer: C
166) Female patient will do the breast implant surgery and she tell the nurse not to tell her family about the
surgery. What is the nature of this action?
A-confidentiality B-ethical dilemma
C-illegal nursing practice D-medico-legal practice Answer: A
167) Which of the following statement indicate nursing action during the first hour after delivery of the
placenta?
A. Monitor of mothers hemoglobin
B. Assess maternal vital signs every 15 minutes ✅
C. Ensure that the mother mobilize and empty her bladder
D. Administer 10 units of oxytocin via IV line to ensure uterus is well contracted. Answer: B
168) A nurse is giving health education for a mother who has mastitis. Which of the following if stated by the
mother about what she needs to do, indicate the additional education is needed?
A. Take antibiotics
B. Use analgesics
C. Wear a supportive bra.
D. Stop breast-feeding Answer: D
169) A mother is in the midwife's clinic with the complaint of, redness and swelling in her right breast. She is
breast feeding her nine months-old baby. What should be immediate intervention?
A. Start on antibiotics
B. Detailed investigations
C. Stop the breast feeding.
D. Wound cleaning and dressing Answer: A
170) The nurse was planning care for a 25-year-old primigravida post-partum mother who had engorgement
due to poor feeding technique. the left breast appeared red and swollen and was diagnosed as. Which of the
following is the best education for the mother?
A. Avoid wearing brassiere.
B. Begin suckling on the right breast.
C. Stop pumping milk from the left breast.
D. Take antibiotics till the soreness subsides. Answer: D

ِ‫يِاْلَ أرض‬
‫أ‬ ُ ‫َم‬
ُ ‫ك‬
‫ثِِف‬ َ ‫اس‬
‫ِِفي أ‬َ َّ ‫ع‬
‫ِالن‬ ُ ‫ف‬
َ ‫ماِيَ أن‬ َّ َ‫وأ‬
َ ِ‫ما‬ َ
{ 18 }

171) A patient in surgical was transferred to isolation room after the wound swab confirmed to have
methicillin Resistant staphylococcus Atreus MRSA. Which of the following measures should the nurse take to
prevent infection in the ward?
A. Clean the Room three times a day.
B. Discard all soiled dressing into waste bag.
C. Instruct the patient to wash hands regularly.
D. Wear gloves and gown on every entry into the room Answer: D

172) A nurse is providing instructions to a mother who has been diagnosed with mastitis. Which of the
following statements if made by the mother indicates a need for further teaching?
A. I need to take antibiotics. And I should begin to feel better in 24-48 hours.
B. I can use analgesics to assist in alleviating some of the discomfort.
C. I need to wear a supportive bra to relieve the discomfort.
D. “I need to stop breastfeeding until this condition resolves.” Answer: D
173) 33 old women presented to the ER with general weakness. The laboratory investigation indicated VIT D
deficiency. Which of the following nutrient should be recommended as a good source of vitamin?
A. Rice
B. Green tea
C. Orange juice
D. Fish liver oils Answer: D
174) Radon can cause any cancer.
A. skin
B. cervical
C. colon
D. prostatic Answer: A
* Cause lung or pancreas cancer also.
175) What is the action of combined oral contraceptives?
A. induce thinning of endometrium and prevent ovulation.
B. Decrease the production of luteinizing hormone and follicle and prevent ovulation. Answer: A
176) A patient who underwent a left foot amputation post-operative Care Unit and the following assess (see
lab results). Blood pressure 110/80 mmHg, Heart rate 65 /min, Respiratory rate 13 /min Temperature 37.2C,
Oxygen Saturation 98 % on room air
Test Result Normal Value
Hb 120 120-158 g/L
WBC 10.2 4-10.5 ×10
RBC 3.8 3.8-5.1 ×10
Fasting blood sugar 9.3 3.5-6.5 mm
LDL 6.5 <4.0 mm
Triglycerides 3.8 <2.16 mm
Which long-term complication is most likely?
A. Pain
B. Infection
C. Immobility
D. Bleeding Answer: B
177) What is the management/treatment for woman who has gestational diabetes?

ِ‫يِاْلَ أرض‬
‫أ‬ ُ ‫َم‬
ُ ‫ك‬
‫ثِِف‬ َ ‫اس‬
‫ِِفي أ‬َ َّ ‫ع‬
‫ِالن‬ ُ ‫ف‬
َ ‫ماِيَ أن‬ َّ َ‫وأ‬
َ ِ‫ما‬ َ
{ 19 }

A. Diet management
B. Insulin
C. Oral diabetic drugs Answer: A
178) A man is to be discharged from the General appendectomy. The precautionary measures, plans are
discussed with him. What is the most important desired outcome after discharge?
A. Remain free of post-surgical complications.
B. Report fever, redness or drainage from the wound site
C. Use pain management techniques apropos.
D. Resume gradual activities and avoid weight. Answer: A
179) A postpartum woman who was admitted for 24 hours, community nurse visited her after 2 days of
discharge. Which of these are abnormal findings?
A. Frequent urination
B. Lochia serosa
C. Uterus below umbilical level
D. Both breast full of milk Answer: D
180) A postpartum woman who was admitted for 24 hours, community nurse visited her after 2 days of
discharge. Which of these are abnormal findings?
A. frequent urination
B. lochia serosa
C. uterus below symphysis pupils
D. breast full of milk Answer: C
181) 6month-old boy with hydrocephalus is admitted to the pediatric surgical Ward for ventriculoperitoneal
Shunt (VPS) insertion. Which of the following findings should be of the most concern when assessing the child
postoperative?
A. Sunken fontanelle and irritability
B. decreased head circumference
C. poor feeding and pupillary change
D. headache and excessive sleepiness Answer: C
182) 6mont h-old boy with hydrocephalus is admitted to the pediatric surgical Ward for ventriculoperitoneal
Shunt (VPS) insertion. Which of the following findings should be of the most concern when assessing the child
postoperative?
A. Plugging fontanelle B. decreased head circumference
C. poor feeding and pupillary change. D. headache and excessive sleepiness Answer: A
183) 6 month-old boys with hydrocephalus is admitted to surgical Ward post ventriculoperitoneal Shunt
(VPS) What is the priority postoperative assessment???
A-Neurological Assessment B. decreased head circumference
C. poor feeding and pupillary change. D. headache and excessive sleepiness Answer: A
184) A 65-year-old women visited the gynecological outpatient history reveals that she had 3 pregnancies, one
abortion gestational age, had 2 normal deliveries. She smokes 20 Her complaint is that she wets herself when
she coughs embarrassing for her? Which of the following can be considered as risk factors pelvic floor
muscles?
A. Chronic coughing B. Diabetes mellitus
C. Excessive spot D. Sedentary lifestyle Answer: D
185) After accessing patients' medical records, which behavior nurse shows that patient’s confidentiality has
been breached?

ِ‫يِاْلَ أرض‬
‫أ‬ ُ ‫َم‬
ُ ‫ك‬
‫ثِِف‬ َ ‫اس‬
‫ِِفي أ‬َ َّ ‫ع‬
‫ِالن‬ ُ ‫ف‬
َ ‫ماِيَ أن‬ َّ َ‫وأ‬
َ ِ‫ما‬ َ
{ 20 }

A. Reviews patients medical record B. Read patients care plan.


C. Disclosing patient’s information. D. Documents medication administered. Answer: C
186) A 5-month-old boy has been vomiting green coloured vomit He has intermittent abdominal pain during
which he draws his chest, turns pale and cries forcefully. On observation, the in the stool which has a jelly-like
consistency. Abdominal pal along. tube-like mass. There is no fever, rash nor diarrhoea are hyperactive in all
quadrants. Which is the most likely form of initial treatment?
A. Manual manipulation B. Surgical resection
C. Normal saline enema D. Laparoscopy Answer: C

187) A 22-year-old gravida 2 para 1 with gestational age 38 week admitted to the hospital. The chief complaint
is decreased the fetal non-stress test revealed decreased variability and fetal movement. The next morning as
part of the antenatal the nurse checks the fetal heart rate by Doppler Sonic aid decreased the fetal heart rate
to less than 100 /min. which of the following action the nurse should do first?
A. Reassure the mother that the FHR is Ok.
B. immediately notifies the physician or midwife.
C. Reposition the patient to left lateral position.
D. Ask the mother about the pattern of fetal movement. Answer: B
188) A child was admitted to the hospital three hours ago with an injury. The child responds appropriately,
but sluggishly to drifts in and out of sleep Which of the following best describes this patient's level of?
A. Lethargic B. Obtruded C. Comatose D. Semi-comatose Answer: A
189) A patient was on a course of lithium carbonate drug. During the nurse found that he complained from
nystagmus visual hallucination, and oliguria Which of the following drug related complications best
symptoms?
A. Overdose B. Mild toxicity C. Severe toxicity D. Moderate toxic Answer: C
190) A nurse is planning to discharge a known HIV, the Isolation Unit after the recovery from upper, which
of the following nursing problem requires?
A. Risk of infection due to altered immune.
B. Fluid volume deficit due to frequent diarrhea
C. Anxiety due to disease, fear and social
D. Weight loss due to higher metabolism rate Answer: A
191)An elderly patient who has an aortic aneurysm Intensive Care Unit to a Medical Surgical Unit on day.
While assessing the client, a nurse notes extremity and is unable to palpate the pedal pulse. Which
intervention should the nurse implement?
A. Wrap the lower extremities with warm.
B. Use a Doppler ultrasound to reassess the
C. Elevate the extremities above heart level.
D. Place a bed cradle over the bed to levitate. Answer: B
192) The head nurse of a Coronary Care Unit delegated the staff a senior nurse in that unit What initial step
must the head nurse implement before?
A. Check the hospital policies for delegating tasks.
B. Explain the task to the senior nurse.
C. Negotiate with the senior nurse.
D. Take the signature of the senior nurse. Answer: A

ِ‫يِاْلَ أرض‬
‫أ‬ ُ ‫َم‬
ُ ‫ك‬
‫ثِِف‬ َ ‫اس‬
‫ِِفي أ‬َ َّ ‫ع‬
‫ِالن‬ ُ ‫ف‬
َ ‫ماِيَ أن‬ َّ َ‫وأ‬
َ ِ‫ما‬ َ
{ 21 }

193) 34-weeks-pregnant mother experiences a sudden gush o from her vagina and mild uterine contractions.
She informs about her condition and requests if she could wait until the delivery. Which of the following is the
best desired response for report to the hospital?
A. Intravenous fluids and medicines need to be administered.
B. Observation is necessary to identify premature labour.
C. Pain and fluid flow both need to be controlled.
D. Fetal heart sound monitoring is necessary. Answer: B
194) The nurse is assisting a patient to ambulate in hall. The patient has a history of coronary artery disease
(CAD) and had coronary artery bypass graft surgery (CABG) 3 days ago. The patient reports chest pain rated
3 on a scale of 0 (no pain) to 10 (severe pain). The nurse should first:
A. Determine how long it has been since the patient's last dose of aspirin
B. Obtain a chair for the patient to sit down
C. Assess the patient's radial pulse
D. Ask the patient to take several slow, deep breaths Answer: B
195) What is the benefit of Kegel exercise?
A- Increase strength of pelvic muscle B- Induce cervix contraction. Answer: A
196) Bioeffects report claims that obstetrical scanning may be harmful to a particular group of patients. What
should be the response of the medical community?
A. Perform the exams on all patients when the risks outweigh the benefits.
B. Stop all diagnostic exams. C. Ignore the report
D. Perform exams on all patients when the benefits outweigh the risks. Answer: D
*Beneficence: benefits out weight risks.
197) 45 years old Woman with uterine fibroid. The doctor prescribed for her Gonadotropin-
releasing hormone (GnRH) agonists. What is the side effect for hcg hormone?
A. Depression ✅ B. Anorexia
C. Osteoarthritis D. Menopause Answer: A
198) The first step in the qualitative research process?
A. Data analysis B. Review of literature
C. Sample D. Study design Answer: B
199) In an article’s abstract, which research process steps may be noted?
A-Literature review
B-Data-collection procedure
C-Research question & study purpose
D-Legal-ethical issues Answer: C
200) During the research process, when should a hypothesis be developed by the researcher?
A-Before any statistical analysis
B-After a research design is determined.
C-Before development of the research question
D-After development of the research question Answer: D
201) All the following are considered steps in the qualitative research process, except?
A-Literature review B-Data collection
C-Sample D-Hypothesis Answer: D
202) The doctor decided DNR for patient. The patient refused that. What should the nurse do?
A-discuss patient about DNR

ِ‫يِاْلَ أرض‬
‫أ‬ ُ ‫َم‬
ُ ‫ك‬
‫ثِِف‬ َ ‫اس‬
‫ِِفي أ‬َ َّ ‫ع‬
‫ِالن‬ ُ ‫ف‬
َ ‫ماِيَ أن‬ َّ َ‫وأ‬
َ ِ‫ما‬ َ
{ 22 }

B-comfort and palliative care ✅


C_ prepare spiritual equipment
D- stop all medication Answer: Bbb

203) For old man patient with poor prognoses the health care provider decides to put him DNR, Patient have
3 children 2 of them agree with DNR the 3rd one refused and ask to continue treatment.
What the nurse should do:
A. provide care and comfort B. provide palliative care
C. Discuss with family regarding DNR and patient needs✅ Answer: C
204) The discussion section addressing the…
A-Problem B-Question C-Objectives D-All the above Answer: D
205) The nurse is caring woman that has cancer, and she is under Chemotherapy. She is complaining anorexia
and the patient has low weight. What should the nurse instruct her???
A. Eat small meals every day.
B. Eat large meals every day.
C. Eat if you are hungry.
D. Eat your favourite food. Answer: A
206) 56-year-old present to the emergency department experiencing left sides eye discomfort for the past 3
hours, left eye was blurred while vision in the right eye remained examination showed increased intra-ocular
pressure in the left eye pupil of the left also reacted slowly to light. Which is the most likely health problem?
A. Detached retina B. Macular hole C. Glaucoma Answer: C
207)10 years' boy with polyuria and dysuria after assessment diagnosed with urinary tract infections what
should do to take urine sample?
A. Increase fluid intake
B. Decrease urine intake
C. Regular intake of fluid
D. Zero intake of fluid Answer: A
208) After assessment of new-born nurse instruct parent to use kangaroo roll which of the following should
be useful
A. Heart rate 150
B. Respiration 55
C. Temperature 34.5 Answer: C
209) Nurse notes some students with anorexia nervosa what does she do??
A. Push them to eat.
B. One to one superior during eating
C. Let students eat with them the same meals. Answer: C
210) A nurse is teaching a group of women about the side effects of different types of contraceptives. What
common side effect associated with the use of an intrauterine device (IUD) should the nurse discuss during the
teaching session?
A: Tubal pregnancy
B: Rupture of the uterus
C: Expulsion of the device
D: Excessive menstrual flow Answer: D
211) What does the mechanism of action result in for Inhaled corticosteroids for asthmatic patient??

ِ‫يِاْلَ أرض‬
‫أ‬ ُ ‫َم‬
ُ ‫ك‬
‫ثِِف‬ َ ‫اس‬
‫ِِفي أ‬َ َّ ‫ع‬
‫ِالن‬ ُ ‫ف‬
َ ‫ماِيَ أن‬ َّ َ‫وأ‬
َ ِ‫ما‬ َ
{ 23 }

A. Reduced inflammatory response and fewer exacerbations, reduced oedema and mucus. production, Increase
responsiveness to β-agonist.
B. Increase broncho dilatation inside lung. Answer: A

212) The nurse is caring for patient with asthma. What is the appropriate treatment for asthma?
A. Alpha antagonist, Corticosteroid B. Beta antagonist, Corticosteroid
C. Alpha agonist, Corticosteroid D. Beta agonist, Corticosteroid Answer: D
213)The patient with sign of severe asthma. What is the treatment you will administer first?
A. Corticosteroids. B. Bronchodilator. C. Beta agonist. Answer: A

214) The nurse is caring for patient with deep vein thrombosis (DVT). The patient’s heparin sodium infusion
has been discontinued and the patient is receiving prescribed warfarin sodium (Coumadin). The nurse should
advise the patient that which of the following needs to be continued?
A. Daily complete blood count (CBC) B. Laboratory tests for partial thromboplastin time (PTT)
C. Strict bed rest D. Wearing elasticized support stockings Answer: C
215) Amenorrhea for 5 weeks and breast tenderness?
A-Positive sign B-presumptive sign Answer: B

216) The nurse is reviewing the health care provider’s prescriptions for a child with a streptococcal infection
and notes that an antistreptolysin O titer is prescribed. Based on this prescription, which disorder would the
nurse suspect in the child?
A- Rheumatic Fever RF
B-Aortic valve diseases
C-pulmonic valve diseases
D-congestive heart failure
Answer: A

ِ‫يِاْلَ أرض‬
‫أ‬ ُ ‫َم‬
ُ ‫ك‬
‫ثِِف‬ َ ‫اس‬
‫ِِفي أ‬َ َّ ‫ع‬
‫ِالن‬ ُ ‫ف‬
َ ‫ماِيَ أن‬ َّ َ‫وأ‬
َ ِ‫ما‬ َ
{ 24 }

217) Patient with Rheumatic fever. what the accurate test for Rheumatic Fever?
A. Ant streptolysin Test
B. Blood cultures C. Urine culture Answer: A
218) How to prevent Rheumatic fever?
A. Give vaccines the pregnant women during pregnancy.
B. Isolation the children with tonsillitis
C. Give 9 months vaccination.
D. Treat the children with rheumatic fever antibiotics full course. Answer: D
219) In determining the one-minute APGAR score of a male infant the nurse assesses a heart rate of 120 beats
per minute and respiratory rate of 44 per minute. He has flaccid muscle tone with slight flexion and resistance
to straightening. He has a loud cry with colour is acrocyanotic What is the APGAR score for the infant?
A. 7 B. 8 C.9 D.10 Answer: B
220) year-old child was admitted with suspected appendicitis. A nurse was evaluating the child’s condition
and the mother stated that the child did have his bowel movement for the past two days and requested for
natives. What is the risk of giving laxative to patient with appendicitis?
A. Pain
B. Fever
C. Rupture
D. Diarrhoea Answer: C
221) What is The term that use to describe people communicate with same culture, goals, values and ethical?
A. Intrapersonal communication
B. Interpersonal communication✅
C. Intergroup communication
D. Interorganization communication Answer: B
222) A patient took his NPH insulin at 6 AM, when will he be hypoglycemia?
A-7am B-8am C-9am D-10am Answer: D
223) A 56-year-old man was admitted with complaint working for three days. The nurse is preparing to
administer infusion of saline 1000 ml over six hours. What is the hourly infusion rate that the nurse needs to
infusion therapy in mL/hour? A.155 B.167 C.190 D.217 Answer: B
224) Suspect patient with TB what is the highest priority nursing action?
A. Isolate the patient in private negative pressure room. B. Take nasal swab Answer: A
225) At 6:00 PM while admitting a woman for a scheduled repeat C section1, a patient tells a nurse that she
drank a cup of coffee at 4:00OM because she wanted to avoid getting a headache. Which of the following
actions should the take first?
A. Inform anesthesia care provider.
B. Ensure preoperative lab results are available.
C. Start prescribed IV with lactated ringers.
D. Contact patient's obstetrician Answer: A
226) A patient is transferred to the Intensive Care following a craniotomy. The Patient is difficult to arouse,
and the pupils are pinpoint and non-reactive. Blood pressure 118/70 mmHg, Heart rate 58/min, Respiratory
rate 11/min, Temperature 37.2°C. Which medication should the nurse prepare administer?
A- Adrenaline B- Thiamine C- Naloxone D- Dextrose 50% Answer: C

ِ‫يِاْلَ أرض‬
‫أ‬ ُ ‫َم‬
ُ ‫ك‬
‫ثِِف‬ َ ‫اس‬
‫ِِفي أ‬َ َّ ‫ع‬
‫ِالن‬ ُ ‫ف‬
َ ‫ماِيَ أن‬ َّ َ‫وأ‬
َ ِ‫ما‬ َ
{ 25 }

227) Patient with schizophrenia. He complains with anxiety episodes which needs is highest priority in
Maslow hierarchy?
A. Physiological needs B. Self esteem C. Safety D. actualization Answer: C
228) How can we prevent infection for preterm neonate?
A. Schedule for immunizations
B. Give influenza and pneumonia vaccine.
C. Wear gloves and gown during the care of neonate Answer: C
229) A nurse who works in the surgical unit at one of the hospitals was asked by the home health care nurse to
make a home visit to a patient with colostomy, who had been discharged the previous day in order to give him
a follow-up care and education which of the following nurses should do the assigned task?
A. Critical care nurse B. Psychiatric nurse
C. Surgical nurse D. Community nurse Answer: D
230) A nurse is caring for child who is pot tonsillectomy and adenoidectomy the nurse should plan to assess
which of the following complication?
A. Pulmonary hypertension B. Hemorrhage
C. Hearing loss D. Orthopnea Answer: B
231) The infant after circumcision. What are the non-pharmacology techniques to relieve pain??
A. Breastfeeding decrease pain. Answer: A
232) A conscious victim of a motor vehicle accident arrives at the emergency department. The patient is
Gasping for air, is extremely anxious, and has a deviated trachea. What diagnosis should the nurse
Anticipate?
A. pleural effusion.
B. tension pneumothorax✅
C. pneumothorax
D. cardiac tamponed Answer: B
233) What is the position for adenoidectomy??
A. Semi Fowler
B. Sitting
C. prone Answer: A
234) A 20 weeks pregnant, primary gravid woman visits the antenatal has sickle cell anemia trait and worried
this disease transmitted to her baby which of the following should be initial intervention?
A. Plan for the fetal genetic screening
B. Educate mother that her disease is inactive.
C. Discuss the chances of genetic disease in the fetus.
D. Gather data about the other family members having the disease. Answer: D
235) A 62-year-old women admitted to the emergency department for the fourth time this year, each time the
patient comes with severe injuries and bruises in the body. What is your responsibility as a nurse to prevent
such incident to happen again?
A. Reports the assault to the local police and write a report.
B. Provides information about safe shelter and support.
C. Instructs the women to move away from her home.
D. Discharge the patient to a safe shelter Answer: A
236) An 82-year-old woman with Alzheimer’s disease had moved into a long-term care facility two weeks
previously. Since then, the staff has found her wondering in the hallways in middle of the night. When

ِ‫يِاْلَ أرض‬
‫أ‬ ُ ‫َم‬
ُ ‫ك‬
‫ثِِف‬ َ ‫اس‬
‫ِِفي أ‬َ َّ ‫ع‬
‫ِالن‬ ُ ‫ف‬
َ ‫ماِيَ أن‬ َّ َ‫وأ‬
َ ِ‫ما‬ َ
{ 26 }

approached, she is confused and frustrated, often forgetting where she is. Which intervention would most
likely decrease the patient's confusion?
A. Administer a sleeping sedative.
B. Provide full-time nursing care.
C. Place a nightlight in the room
D. Provide a large meal before bed Answer: C

237) Nurse is completing the preoperative checklist for one of the patients who ring is wearing a ring. What is
the most appropriate action?
A. Give ring to security office.
B. Lock ring with patient's valuables
C. Call patient's family to give them the ring.
D. Respect patient's choice and leave ring on patient's finger. Answer: B

238) An Indian patient, who is vegetarian, is being discharged from after an elective surgical procedure. The
unit nurse is teaching using visual aids and pictures about food combinations complete protein. Which of the
following food items should the nurse recommended dietary list of the patient?
A. Lentils B. Potatoes
C. Macaroni D. Green salad Answer: A
239) which of the following indication for patient with MI:
A. ST depression
B. ST elevation * ST elevation = MI
C. short ST *ST depression = ischemia
D. none of the above Answer: B
240) The nurse is preparing to transfer a 55-year-old patient on the CT-scan unit. The patient is too heavy for
the nurse to transport, nurse went to get an assistive device to transfer the patient. Which of the following
transfer device is the most appropriate for the nurse to use?
A. Board B. Handle
C. Trapeze D. Mechanical lift Answer: D
241) During meningitis outbreak in one of the hospitals, the nun-immune staff were given an immunoglobulin
in order to prevent them from the infection. What is the type of immunity that will be developed by these
staff?
A. active B. long-term
C. natural D. passive Answer: D
242) patient is being weaned off from the mechanical ventilator is about to hook the endotracheal tube to
oxygen at FiO2 of 40 Which of the following oxygen administration device is the best the nurse in this
situation?
A. Ambo bag
B. Ventura mask
C. Tracheostomy collar
D. T-piece/ Briggs Answer: D
243) Basal skull fracture signs: Racoon eyes
244)For physical assessment steps for the foot I Start by:
A. Inspection, palpitation
B. Palpation, auscultation

ِ‫يِاْلَ أرض‬
‫أ‬ ُ ‫َم‬
ُ ‫ك‬
‫ثِِف‬ َ ‫اس‬
‫ِِفي أ‬َ َّ ‫ع‬
‫ِالن‬ ُ ‫ف‬
َ ‫ماِيَ أن‬ َّ َ‫وأ‬
َ ِ‫ما‬ َ
{ 27 }

C. Percussion, palpation Answer: A


245) A 43-year-old man in the post-surgical area complains of abdominal pain radiating to the naval which is
increasing with examination his abdomen is guarded with marked tender lower quadrant. What is the
immediate goal of care to do?
A. Teach abdominal splinting during coughing.
B. Administer pain medication as ordered.
C. Assess pain and report immediately.
D. Position on the left lateral side Answer: C
246) A 21-year-old in oversized clothing presents to the hospital with of felling dizzy and faint. The hair and
nail appear thin and dry. The skin appears pale, and she has sunken eye sockets and tenting skin. Her body
mass index is 16. She often induces vomiting after eating blood is collected for analysis (see lab results).
*Blood pressure 90/52 mmHg, Heart rate 118 /min, Respiratory rate 26/min, * Temperature 37.2 C ͦ , Oxygen
saturation 97%. : ABG Test Result Normal value
HCO₃ 31 22 – 88mmol/L
PCO₂ 10.3 4.7-6.0 kPa
PH 7.50 7.36-7.45
Which nursing problems stem is the most appropriate?
A. Impaired nutrition
B. Decreased cardiac output.
C. Infective airway clearance
D. Ineffective breathing pattern Answer: B
247) Patient has heart failure and suddenly has edema. What is the lung sound will be hearing?
A. Rales B. Rhonchi C. Wheezing Answer: A
248) Patient is receiving oxygen therapy by face mask. What is the oxygen saturation should be achieved?
A. 85% and above B. 95% and above Answer: B
249) What is the normal range for oxygen saturation in patient who is receiving oxygen therapy?
A- 60-69% B- 70-79% C- 80-89% D- 94- 98% Answer: D
250) Soldier was brought to triage area after being exposed to chemical weapons. Signs and symptoms of
nerve gas exposure were noticed. A nurse prepares for medical management. Which medication should the
nurse prepare for the patient?
A. Atropine B. Adrenaline C. Sodium nitrate D. Sodium thiosulphate Answer: A
251) A patient comes to the emergency unit with cough and severe dyspnea. The patient’s medical history
revealed a diagnosis of chronic heart failure and chronic obstructive pulmonary disease. Blood pressure
110/70 mmhg Heart rate 87/min Respiratory rate 23/min Temperature 37.3 C
Which of the following diagnostic tests will be most beneficial to a nurse to figure out if there is an
exacerbation of heart failure?
A. B-type natriuretic peptide (BNP) B. arterial blood gas (ABG)
C. cardiac enzymes (CK-MB) D. chest x-ray Answer: A
252) 20 years old came to ER department in motor accident with one of his relative, all medical team attempts
failed and the PT died, what is the best action to his relative ?
A-Ask the relative sitting down B-Assess the relative vital signs
C-Let the relative see the PT✅ D-Ask the relative to be patient Answer: C
253) A patient returned to the Surgical Unit from the thyroidectomy. The nurse observed that the arousable.
Blood pressure 90/60 mmHg Heart rate 108 /min What immediate action should the nurse take?

ِ‫يِاْلَ أرض‬
‫أ‬ ُ ‫َم‬
ُ ‫ك‬
‫ثِِف‬ َ ‫اس‬
‫ِِفي أ‬َ َّ ‫ع‬
‫ِالن‬ ُ ‫ف‬
َ ‫ماِيَ أن‬ َّ َ‫وأ‬
َ ِ‫ما‬ َ
{ 28 }

A. Recheck pulse and blood pressure. B. Administer intravenous fluids as ordered.


C. Place client in modified Trendelenburg's
D. Assess the back of neck surgical dressing for bleeding. Answer: D
254) new-born has small, whitish, pinpoint spots over the nose, which the nurse knows are caused by retained
sebaceous secretions. When charting this observation, the nurse identifies it as:
A. Milia B. Lanugo C. Whiteheads D. Mongolian spots Answer: A

255) A nurse is caring for a client with syndrome of inappropriate antidiuretic hormone (SIDH). Which of the
following should be a priority intervention for the client?
A. Monitoring hourly intake and output
B. Pressure ulcer prevention strategies
C. Encourage client to eat foods rich in potassium.
D. Restricting fluid intake of the client to less than 1000ml per day Answer: D
256) A postdate pregnant woman is admitted for the induction of labour. Her fetal heart rate and vital signs
are within normal range her intravenous line is maintained and she is to be started on low doses of labour
inducing medication. Which of the following medication the mother is likely to receive intravenously?
A. Oxytocin B. Cervidil C. Cytotec D. Cytoxan Answer: A
257) A 40-year-old man was admitted to a male medical department with pneumonia. A nurse in the
department explained and obtained informed consent for a chest CT scan procedure with contrast. with of the
following ethical principles underpinning informed consent.?
A. Autonomy B. Non maleficence C. Beneficence D. Respect Answer: A
258) While a nurse is assessing vital signs of new-born infant first hour of delivery. HR 170 RR 70 TEM 36 the
nurse would interpret these finding as in the discharge instruction.
A. Anaemia B. Cold distress
C. Heart defects D. Hyperglycemia Answer: B
259) Which of the following statements describe the function of Immunoglobulin (IgG)?
A. It does not cross the placenta and is present in colostrums and breast milk.
B. It protects against gastrointestinal and respiratory system infections.
C. It provides temporary immunity to bacteria and toxins to which the mother has developed immunity
D. It protects against gram negative bacteria and can be found cord blood. Answer: C
260) Fertilization occurs in the fallopian tube, egg and sperm united to form which moves towards the uterus
in a long journey to implant in uterus, how long the zygote takes to implant in the uterus?
A )3-8 day B) 3-7 C) 7-10 D) 10-14 Answer: C
261) A lot of theories took about early skin contact in 3rd stage of labour what is the purpose from early skin
to skin contact between mother and child?
A-at warm baby B- early breast feeding
C- improve uterine contraction. D-decrease maternal pulse rate Answer: B
262) 22 years old girl went her that voice is bad she in music institute which defends mechanism she used?
A) Denial B) displacement C) projection D) repression Answer: C
263) Patient immobile 3days science total hip replacement. She will ambulate for the first time after surgery
the nurse told her to descent her leg and sit at the side of the bed before bearing her weight in foot. The nurse
instructs her to do this to avoid which of the following?
A- hypotension B- dislocation C-hypertension D-headache Answer: A

ِ‫يِاْلَ أرض‬
‫أ‬ ُ ‫َم‬
ُ ‫ك‬
‫ثِِف‬ َ ‫اس‬
‫ِِفي أ‬َ َّ ‫ع‬
‫ِالن‬ ُ ‫ف‬
َ ‫ماِيَ أن‬ َّ َ‫وأ‬
َ ِ‫ما‬ َ
{ 29 }

264) Pt when entered ER he said, my heart will get out of my chest I fell that I will die he diagnosed with panic
attack what is the medical problem that will be developed if panic not controlled?
A) Respiratory acidosis B) Respiratory alkalosis
C) Metabolic acidosis D) Metabolic alkalosis Answer: B
265) If You lose an ampule of morphine what you will do?
A) write an incident report B) call the nurse manager and told her to deal with this situation
C) call the primary nurse who was in the previous shift
D) counselling the nurse who count and sign Answer: B
266) Nurses meet another nurse college after failed in resuscitation of child, she was crying and depressed.
What is the suitable response in this situation?
A) you need to take vacation some days until you feel better
B) just return to your home and hugging your children
C)there was nothing in your hand to do.
D) let her crying and talk about the matter Answer: D
267) A gravida 3 para 2 presents to the Maternity Triage Unit after the amniotic membranes ruptured at
home. The fluid is noted to be clear. The neonates head is engaged into the pelvis and the patient is having
contractions every 5 to 7 minutes. Each contraction lasts for 60-90 seconds. An examination of the cervix
finds 4 centimeters dilatation and 90% effacement. She is. uncomfortable during contractions and rates the
pain at a level 7, on pain scale of 1-10.
Which of the following is most indicative that she is in true labour?
A. Level of pain B. Cervical dilatation and effacement
C. Engagement of presenting part D. Frequency and length of contractions Answer: B
268) The nurse is teaching 32-week pregnant women how to distinguish between pre labor (false) contraction
and true labor contractions. Which statement about pre labor contractions accurate?
A. they are regular and increase gradually. B. they are felt in the abdomen.
C. they start at the back and radiate to the abdomen.
D. they become more intense during walking. Answer: B
269) A patient was complaining of depression, muscle weakness and continuous fatigue. which of the
following deficiencies is suspected?
A. Deficiency of vitamin B. B. Deficiency of vitamin D.
C. Deficiency of vitamin E D. Deficiency of vitamin K Answer: B
270) The nurse is assessing a child who has Tetralogy of Fallot observed that the child is having clubbing in
his fingernails Which of the flowing might be the reason for this clubbing?
A. Prolonged tissue hypoxia B. Delayed physical growth.
C. Inactive bone marrow D. Pulmonary fibrosis Answer: A’
271) Which of the following related to Low pitched breath sound??
A. Crackles B. Rhonchi C. Wheezing D. Vascular Answer: B
272) Which of the following related to High pitched breath sound??
A. Crackle B. Rhonchi C. Wheezing D. Bronchial Answer: C
273) Which of the following related to harsh breath sound??
A. Crackles B. Bronchial C. Wheezing D. Stridor Answer: D
274) Which of the following related to harsh breath sound??
A. Crackles B. Tracheal C. Wheezing D. Rhonchi Answer: B

ِ‫يِاْلَ أرض‬
‫أ‬ ُ ‫َم‬
ُ ‫ك‬
‫ثِِف‬ َ ‫اس‬
‫ِِفي أ‬َ َّ ‫ع‬
‫ِالن‬ ُ ‫ف‬
َ ‫ماِيَ أن‬ َّ َ‫وأ‬
َ ِ‫ما‬ َ
{ 30 }

275) A newly graduated nurse is inserting an intravenous cannula into the mid-cephalic vein of a patient who
is being admitted to the unit. As she withdraws the needle, a nurse calls out for help from another patient’s
room. The new nurse rushes to help. She secured the intravenous catheter and threw the needle into the waste
basket instead of the sharp’s container. When asked who had thrown the needle into the bin, the new nurse
admits that she had made that mistake. Which professional act best describes the newly graduated nurse’s
response?
A. Responsibility B. Accountability C. Assertiveness D. Leadership Answer: B

276) He wants to influence the customary way of thinking and behaving that is shared by the members of the
department. Which of the following terms refer to this?
A. Organizational chart B. Cultural network
C. Organizational structure D. Organizational culture Answer: D
277) After a code blue, a doctor announced the death of a patient. A signature other begins to show signs of
decreased level of consciousness. What is the priority intervention of the nurse at this time?
A. Offer a meal B. Ensure safety ✅
C. Set up an IV line D. Offer a glass of water Answer: B
278) Stephanie is a new Staff Educator of a private tertiary hospital. She conducts orientation among new
staff nurses in her department. Joseph, one of the new staff nurses, wants to understand the channel of
communication, span of control and lines of communication. Which of the following will provide this
information?
A. Organizational structure B. Policy
C. Job description D. Manual of procedures Answer: A
279) Stephanie is often seen interacting with the medical intern during coffee breaks and after duty hours.
What type of organizational structure is this?
A. Formal B. Informal
C. Staff D. Line Answer: B
280) She takes pride in saying that the hospital has a decentralized structure. Which of the following is NOT
compatible with this type of model?
A. Flat organization B. Participatory approach
C. Shared governance D. Tall organization Answer: D
281) you find a victim arrested in the street according to car accident you found a clear fluid dropping from
his ear, and you must give him rescues breathing, what you will do?
A. chest thrust B. jaw thrust ✅ C- Head tilt chin lift Answer: B
282) Patient has Colostomy we transfer the Pt:
A-Belt abdomen B- Belt chest ✅ C- Restrain Answer: B
283) A 4-month-old infant returned immediately from OR room post cleft lip repair which of the following
nursing intervention should be considered?
A. Apply elbow restrain ✅ B. Apply suction when needed
C. Measure temperature D. Put infant in prone position Answer: A
284) Which of the following guidelines should be least considered in formulating objectives for nursing care?
A. Written nursing care plan B. Holistic approach
C. Prescribed standards D. Staff preferences Answer: D

ِ‫يِاْلَ أرض‬
‫أ‬ ُ ‫َم‬
ُ ‫ك‬
‫ثِِف‬ َ ‫اس‬
‫ِِفي أ‬َ َّ ‫ع‬
‫ِالن‬ ُ ‫ف‬
َ ‫ماِيَ أن‬ َّ َ‫وأ‬
َ ِ‫ما‬ َ
{ 31 }

285) Stephanie considers shifting to transformational leadership. Which of the following statements best
describes this type of leadership?
A. Uses visioning as the essence of leadership.
B. Serves the followers rather than being served.
C. Maintains full trust and confidence in the subordinates.
D. Possesses innate charisma that makes others feel good in his presence. Answer: A

286) As a manager, she focuses her energy on both the quality of services rendered to the patients as well as
the welfare of the staff of her unit. Which of the following management styles does she adopt?
A. Country club management
B. Organization man management
C. Team management
D. Authority-obedience management Answer: C
287) The statement, “The Holy Spirit Medical Centre aims to provide patient-centered care in a total healing
environment” refers to which of the following?
A. Vision B. Goal C. Philosophy D. Mission Answer: B
288) Joey plans to revisit the organizational chart of the department. He plans to create a new position of a
Patient Educator who has a coordinating relationship with the head nurse in the unit. Which of the following
will likely depict this organizational relationship?
A. Box B. Solid line
C. Broken line (Dotted) D. Matrix Answer: C
289) He likewise stresses the need for all the employees to follow orders and instructions from him and not
from anyone else. Which of the following principles does he refer to?
A. Scalar chain B. Discipline
C. Unity of command D. Order Answer: C
290) Joey orients his staff on the patterns of reporting relationship throughout the organization. Which of the
following principles refer to this?
A. Span of control B. Hierarchy
C. Esprit d’ corps D. Unity of direction Answer: B
291) He emphasizes to the team that they need to put their efforts together towards the attainment of the
goals of the program. Which of the following principles refers to this?
A. Span of control
B. Unity of direction
C. Unity of command
D. Command responsibility Answer: B
292)What is drug that prevent recurrence of rheumatic fever?
A-Penicillin B- Corticosteroids C. Salicylates Answer: A

293) He discusses the goal of the department. Which of the following statements is a goal?
A. Increase the patient satisfaction rate.
B. Eliminate the incidence of delayed administration of medications.
C. Establish rapport with patients.
D. Reduce response time to two minutes. Answer: A

ِ‫يِاْلَ أرض‬
‫أ‬ ُ ‫َم‬
ُ ‫ك‬
‫ثِِف‬ َ ‫اس‬
‫ِِفي أ‬َ َّ ‫ع‬
‫ِالن‬ ُ ‫ف‬
َ ‫ماِيَ أن‬ َّ َ‫وأ‬
َ ِ‫ما‬ َ
{ 32 }

294) Ms. Caputo is newly promoted to a patient care manager position. She updates her knowledge on the
theories in management and leadership in order to become effective in her new role. She learns that some
managers have low concern for services and high concern for staff. Which style of management refers to this?
A. Organization Man B. Impoverished Management
C. Country Club Management D. Team Management Answer: C
295) Her former manager demonstrated passion for serving her staff rather than being served. She takes time
to listen, prefers to be a teacher first before being a leader, which is characteristic of
A. Transformational leade B. Transactional leader
C. Servant leader D. Charismatic leader Answer: C

296) She reads about Path Goal theory. Which of the following behaviours is manifested by the leader who
uses this theory?
A. Recognizes staff for going beyond expectations by giving them citations.
B. Challenges the staff to take individual accountability for their own practice.
C. Admonishes staff for being laggards.
D. Reminds staff about the sanctions for non-performance. Answer: A
297) One leadership theory state that “leaders are born and not made,” which refers to which of the following
theories? A. Trait B. Charismatic C. Great Man D. Situational Answer: C
298) She came across a theory which states that the leadership style is effective dependent on the situation.
Which of the following styles best fits a situation when the followers are self-directed, experts and are matured
individuals?
A. Democratic B. Authoritarian
C. Laissez faire D. Bureaucratic Answer: C
299) The nurse was giving sufficient information for parent regarding health for their baby. What is this
considering??
A. Positive denial B. Detachment C. Empowerment✅ Answer: C
300) A nurse manager in medical unit is not satisfied with the way things in her unit. Patient satisfaction rate
seductive months and staff morale is at its lowest. He decides to plan and initiate changes that will push for a
turnaround in the condition of the unit. Which of the following actions is a priority for Henry?
A. Call for a staff meeting and take this up in the agenda.
B. Seek help from her manager.
C. Develop a strategic action on how to deal with these concerns.
D. Ignore the issues since these will be resolved naturally. Answer: A
301) He knows that there are external forces that influence changes in his unit. Which of the following is NOT
an external force?
A. Memo from the CEO to cut down on electrical consumption.
B. Demands of the labour sector to increase wages.
C. Low morale of staff in his unit
D. Exacting regulatory and accreditation standards Answer: C
302) After discussing the possible effects of the low patient satisfaction rate, the staff started to list down
possible strategies to solve the problems head-on. Should they decide to vote on the best change strategy,
which of the following strategies is referred to this?
A. Collaboration B. Majority rule
C. Dominance D. Compromise Answer: B

ِ‫يِاْلَ أرض‬
‫أ‬ ُ ‫َم‬
ُ ‫ك‬
‫ثِِف‬ َ ‫اس‬
‫ِِفي أ‬َ َّ ‫ع‬
‫ِالن‬ ُ ‫ف‬
َ ‫ماِيَ أن‬ َّ َ‫وأ‬
َ ِ‫ما‬ َ
{ 33 }

303) One staff suggests that they review the pattern of nursing care that they are using, which is described as
a:
A. job description B. system used to deliver care.
C. manual of procedure D. rules to be followed. Answer: B

304) Which of the following is TRUE about functional nursing?


A. Concentrates on tasks and activities
B. Emphasizes use of group collaboration.
C. One-to-one nurse-patient ratio
D. Provides continuous, coordinated, and comprehensive nursing services. Answer: A

305) Elderly non-Saudi patient has COPD Stay in KSA. He admitted hospital for caring and treatment, but
he needed to stay long time to receive health services. The doctor informed his family about that. They
decided to discharge the patient because they cannot pay hospital cost period. It was awfully expensive. What
is the most common cause that force the family to take this decision specially with elderly patient?
A. Because the patient is elderly he will die and no need to pay too much money
B. There is no private health care facility for geriatric patient with less expensive.
C. There are too many geriatric patients over 80 years old in hospitals
D. There is no hospital focus more on long term care for Non Saudi patients Answer: B
306) He raised the issue on giving priority to patient needs. Which of the following offers the best way for
setting priority?
A. Assessing nursing needs and problems.
B. Giving instructions on how nursing care needs are to be met.
C. Controlling and evaluating the delivery of nursing care.
D. Assigning safe nurse: patient ratio Answer: A
307) Which of the following is the best guarantee that the patient’s priority needs are met?
A. Checking with the relative of the patient
B. Preparing a nursing care plan in collaboration with the patient.
C. Consulting with the physician
D. Coordinating with other members of the team Answer: B
308) When Henry uses team nursing as a care delivery system, he and his team need to assess the priority of
care for a group of patients, which of the following should be a priority?
A. Each patient as listed on the worksheet.
B. Patients who need least care
C. Medications and treatments required for all patients.
D. Patients who need the most care. Answer: D
309) He is hopeful that his unit will make a big turnaround in the succeeding months. Which of the following
actions of Henry demonstrates that he has reached the third stage of change?
A. Wonders why things are not what it used to be.
B. Finds solutions to the problems
C. Integrate the solutions to his day-to-day activities.
D. Selects the best change strategy. Answer: C
310) Saudi Arabia has noticed the spread of a disease osteoporosis among people, low attitude towards the
problem and lack of commitment of people, what do we do ?
A. Apply strict decisions
B. Provide educational program for population
C. Cure all affected Answer: B

ِ‫يِاْلَ أرض‬
‫أ‬ ُ ‫َم‬
ُ ‫ك‬
‫ثِِف‬ َ ‫اس‬
‫ِِفي أ‬َ َّ ‫ع‬
‫ِالن‬ ُ ‫ف‬
َ ‫ماِيَ أن‬ َّ َ‫وأ‬
َ ِ‫ما‬ َ
{ 34 }

311) The nurse is performing an admission assessment for a client who has schizophrenia. The nurse notices
that the client's appearance is unkempt, and he appears to be actively hallucinating. Which of the following
should be the nurse's priority assessment?
A. Perception of reality B. Ability to follow directions.
C. Physical needs D. Mental status Answer: C
312) A child has an abdominal pain and Hepatomegaly, what should the nurse expect?
A- Hepatitis B- Cancer C- Liver damage D- Low level of hemoglobin Answer: A

313) Nurse A want to take a break and delegate her task to Nurse B, during that time the patient falling. As a
nurse, who is responsible and be accountable for fall.
A) Head nurse B) Nurse A
C) Nurse B D) Nurse A and B Answer: D
314) which of the following the role of informatic nurse?
A) software B) design system Answer: B
315) Community Nursing nurse is planning a follow-up visit to a family after their firstborn 18) child died
from Sudden Infant Death-Syndrome (SIDS). Which action is the most important for the nurse to include in
the initial visit??
A. Help the family in planning for future children.
B. Make a referral for genetic counselling and education.
C. Allow time for the parents to express their anger an
D. Educate the family on the causes of SIDS Answer: C
316) After the pericardiocentesis, the doctor inserted a polyethylene catheter left it in the pericardial sac. The
patient asked the nurse about the response of the catheter. What should be the nurse's response?
A. Monitor consistency of drainage
B. Prevent movement of pericardial sac.
C. Prevent recurrence of cardiac tamponed.
D. Prevent increase in venous and blood pressure. Answer: A
317) After tonsillectomy, a child begins to vomit bright red blood. the initial nursing action is to?
A. Notify the physician
B. Turn the child to the side.
C. Maintain an NPO status
D. Administer the prescribed antiemetic. Answer: B
318) A 17-year-old arrived at the emergency room complaining abdominal pain on right lower quadrant. Pain
was rated as 9 numeric scale with positive rebound tenderness over the pain.Blood pressure 120/70 *Heart
rate 100* Respiratory rate 22 Which of the following interventions has the highest priority?
A. keep NPO.
B. secure an IV access.
C. prepares for ultrasound.
D. prepares for abdominal surgery. Answer: A

319) A 32-year-old prim pare attended the postnatal clinic 4days post – she says she is keen to breastfeed but
the baby to the painful. The nurse examined the breasts and found that the red and cracked. Which will be
nurse advice to her to help the women situation?
A. apply antibiotic noodle cream to prevent infection.
B. use correct positioning of the infant to latch on nipple.

ِ‫يِاْلَ أرض‬
‫أ‬ ُ ‫َم‬
ُ ‫ك‬
‫ثِِف‬ َ ‫اس‬
‫ِِفي أ‬َ َّ ‫ع‬
‫ِالن‬ ُ ‫ف‬
َ ‫ماِيَ أن‬ َّ َ‫وأ‬
َ ِ‫ما‬ َ
{ 35 }

C. use the same position when feeding not to confuse


D. use breast pads with plastic lining to prevent leaking Answer: B

320) Patient diagnosed with nephrotic syndrome his lab results showed that he had protein in urinalysis
Which of the following medication should give.
A. Immune spirits B. Cortisone C. Diuretic Answer: B

321) How do I collect sputum? Collect your sputum in the early morning
322) Bacteria: The bacteria Streptococcus pneumoniae and Neisseria meningitides are responsible for 80% of
cases of meningitis in adults
323) Vitamin k: Intramuscular injection 0.5-1mg IM at birth (preferred route)
324. A nurse prepares to administer a vitamin K injection to a full term the mother wants to know the importance of
the injection Which of the following is the best nurse response to the mother
A. needed for blood clotting to prevent hemorrhage ✅
B. accelerate the growth and development of infants
C. help in maintain healthy gut and passage of meconium
D. protect the infant from developing sever respiratory distress Answer: A
325) SIDH\ Nursing management is: Fluid restriction
326) A 24-year-old woman diagnosed as having hemolytic anent physician ordered to transfuse one unit of
packet red blood transfusion, the patient start flushing and complained of dyspnea, generalized body itching
(see lab results)
TEST Result Normal values
Hb 130 142-176 g/L
WBC 4.5 4-10.5 x10^2
RBC 3.0 3.8-5.1 x 10^2 /L
Which of the following nursing diagnosis is related to blood an occludes the patient’s problem?
A. Bacteremia
B. Fluid overload
C. Hypovolemic shock
D. Transfusion reaction Answer: D
327) A post-operative patient who underwent an abdominal procedure requests medication from the nurse
and rates the pain at a level nine. They are standing order for narcotic administration. When the nurse opera
the narcotic box and preforms a count, the number of pills recorded on the sheet. What is the most
appropriate initial nursing action?
A. Notify the nursing supervisor.
B. write the finding on the narcotic sheet.
C. Administer the patient requested medication.
D. Identify the last nurse who used the narcotic box. Answer: A

328) A mother of 8 months' child wants to give him an egg to eat what is the kind of egg she should give?
A. Egg yolk B. Egg whites (> 1yrs) Answer: A
329) What are the 3 stages of Separation Anxiety in order?
A. Protest, despair, detachment.
B. Despair, protest detachment Answer: A
330) Patient will start TPN and should started gradually to avoid which:
A-hyperinsulinemia B-hyperinsulinemia
C-hypoglycemia D-hyperglycemia Answer: D

ِ‫يِاْلَ أرض‬
‫أ‬ ُ ‫َم‬
ُ ‫ك‬
‫ثِِف‬ َ ‫اس‬
‫ِِفي أ‬َ َّ ‫ع‬
‫ِالن‬ ُ ‫ف‬
َ ‫ماِيَ أن‬ َّ َ‫وأ‬
َ ِ‫ما‬ َ
{ 36 }

331) A nurse has an order to transfuse a unit of packed red blood cells to a client who does not currently have
an intravenous (IV) line inserted. When obtaining supplies to start the IV infusion, the nurse selects an Angio
catheter with a size of:
A. 18-gauge B. 21 gauge C. 22-gauge D. 24 gauge Answer: A

332) A nurse develops a plan of care for a child at risk for tonic-colonic seizures. In the plan of care, the nurse
identifies seizure precautions and documents that which item(s) need to be placed at the child's bedside?
A. Emergency cart
B. Tracheotomy set
C. Padded tongue blade
D. Suctioning equipment and oxygen Answer: D.
333) A client with myocardial infarction suddenly becomes tachycardic, shows signs of air hunger, and begins
coughing frothy, pink-tinged sputum. Which of the following would the nurse anticipate when auscultating
the client's breath sounds?
A. Stridor B. Crackles
C. Scattered rhonchi D. Diminished breath sounds Answer: B
334) The physician prescribes captopril (Capo ten) 25mg po tid for hypertension. Which of the following
adverse reactions can o Capo ten?
A. Tinnitus B. Persistent cough
C. Muscle weakness D. Diarrhea Answer: B
335) The nurse wants to assess growth of baby 9 month what is the most appropriate measure?
A. Weight B. Height C. Development Answer: A
336) The nurse wants to assess nutritional status of baby 9 month what is the most appropriate measure?
A. Head circumference
B. Arm circumference
C. Chest circumference Answer: B
337) The supervisor observes a new graduate nurse suctioning a client. Which of the following techniques
requires an intervention?
A. Suction is applied when the catheter is withdrawn.
B. Suction is applied when the catheter is inserted.
C. Suction is applied for 10 seconds.
D. Suction is applied while rotating the catheter 360 degrees. Answer: A
338) Which of these following indicates signs of severe COPD?
A. High p02 and high pC02
B. Low p02 and low pC02
C. Low p02 and high pC02
D. High p02 and low pC02 Answer: C
339) Multiple myeloma is a neoplastic proliferation of:
A- lymphocytes B- Granulocytes C- Plasma cells D- Monocytes Answer: C
340) How much of blood volume for patient has a 10% blood loss and his weight is 50kg
A- 300ml
B- 400ml Formula of Blood Transfusion = 10 ml/ kg

ِ‫يِاْلَ أرض‬
‫أ‬ ُ ‫َم‬
ُ ‫ك‬
‫ثِِف‬ َ ‫اس‬
‫ِِفي أ‬َ َّ ‫ع‬
‫ِالن‬ ُ ‫ف‬
َ ‫ماِيَ أن‬ َّ َ‫وأ‬
َ ِ‫ما‬ َ
{ 37 }

C- 3000ml
D- 4000ml Answer: B
341) Community health nurse in any country should know the statistical morbidity and mortality rates for the
most common disease for 4th data about national and international levels. What is the main reason for this
need?
A. Identify patient’s needs accordingly. B. Offer a few accessible services to the people.
C. Compare the prevalence of disease in the communities.
D. Publicize previous health problems and suggest appropriate action. Answer: D
342) Nursing staff If they take Saudi commission for health specialties. Which is the Ministry is responsible
for that ?
A. Ministry of Health✅ B. Saudi nursing Authority Answer: A
343) Head nurse of an Intensive Care Unit oriented a new staff nurse about the hospital's flat organizational
structure. Which of the following expectations will the nurse have about this type of organization?
A- Hierarchy of authority B- Limit authority for workers
C- Low level of formalization D- Centralized decision making Answer: C
344) When developing a plan care for a hospitalized child, nurse Mica knows that children in which age group
are most likely to view illness as a punishment for misdeeds?
A. Infancy B. Preschool age
C. School age D. Adolescence Answer: B
345) Which hormone that detect the gender of the fetus??
A. Progesterone B. testosterone
C. Genetics D. Estrogenic Answer: B
346) A nurse is starting the morning shift and has received the patient’s handover from night shift nurses and
completed the morning rounds. The nurse is preparing to perform physical assessment for the patients Which
of the following patients should be assessed first?
A. A postoperative patient with total hip replacement
B. A patient with a stroke who is on nasogastric tube.
C. A patient with congestive heart failure
D. A patient scheduled for appendectomy who was admitted. Answer: A
347) Rayed is provided new wheelchair program needs. Which of the following Priority should include in the
program??
A. Elderly
B. Psychiatric
C. Pre-eclampsia
D. MeaslesAnswer: A
348) Rayed is provided framework wheelchair program needs. Which of the following Priority should include
in the program??
A. School age
B. Psychiatric
C. Pre-eclampsia
D. MeaslesAnswer: C
349) Which of the following is consider the type of leadership in X theory?
A. Authoritaria B. Autocratic C. Democratic Answer: B
350) Which of the following is consider the type of leadership in Y theory?
A. Authoritarian B. Autocratic C. Democratic Answer: C

ِ‫يِاْلَ أرض‬
‫أ‬ ُ ‫َم‬
ُ ‫ك‬
‫ثِِف‬ َ ‫اس‬
‫ِِفي أ‬َ َّ ‫ع‬
‫ِالن‬ ُ ‫ف‬
َ ‫ماِيَ أن‬ َّ َ‫وأ‬
َ ِ‫ما‬ َ
{ 38 }

351) Which of the following is consider the type of leadership of HR (Human Relationship)?
A. Authoritarian B. Autocratic C. Democratic Answer: A
352) The emotions that is happened during pregnancy and allow pregnant to cry without any cause it is called
which of the following??
A. Sadness B. Mood swing C. Ambivalence Answer: B

353) When the employees assess the performance of their managers. They give positive feedback for them.
What is the type of the evaluation?
A. Supervisor evaluation B. Subordinate evaluation
C. Peer evaluation D. Team evaluation Answer: B
354) What is the most common Complication of NGT??
A. Hemorrhage B. Aspiration
C. Infection Answer: B
355) What is rational of insertion therapeutic NGT of pt. in the hospital?
A. lavage B. decompression
C. juice suction Answer: A
356)What is the goal for NGT insertion after cholecystectomy?
A. Prevent aspiration B. Compress stomach
C. Suction gastric content Answer: B
357) PKU screen test for what?
A-blood disorders B-glucose C-metabolism disorders Answer: C
358) What is the authority that issues licenses for workers in the health sector in Saudi Arabia?
A. Saudi commission for health specialties (SCFHS) B. Saudi hospitals Answer: A
359) Saudi Arabia neonatal screening for?
A. Hypothyroidism diseases✅ B. Congenital diseases
C. Immunodeficiency diseases Answer: A
360) How take Urine sample from patient with catheter?
A-with syringe B-clamp for 30 minutes then take it
C-Early morning Answer: B
361) Subinvolution medication?
A-methergine B-oxytocin C-tramadol iv Answer: A
362) The pregnant 24 week came to antenatal clinic with spot of blood from vagina for 3 days. She was
worried that Maybe she need curettage because she had 3 previous abortions before. What should the
nurse do?
A. Call the doctor B. Observation and make more investigation
C. Prepare for immediate curettage and dilatation Answer: B
363) Meningocele in the ICU what we should Check?
A-infection B-dehydration Answer: A
364) What is the type of isolation to Pt with Meningitis?
A-Negative pressure B-Positive pressure ✅✅
C-Reserve Answer: B
365) Pregnant woman in Supine position what is the effect?

ِ‫يِاْلَ أرض‬
‫أ‬ ُ ‫َم‬
ُ ‫ك‬
‫ثِِف‬ َ ‫اس‬
‫ِِفي أ‬َ َّ ‫ع‬
‫ِالن‬ ُ ‫ف‬
َ ‫ماِيَ أن‬ َّ َ‫وأ‬
َ ِ‫ما‬ َ
{ 39 }

A. Decrease oxygen to placenta Answer: A


366) Position post vulva procedure (vulvectomy)??
A. Recumbent B. Semi Recumbent C. Supine Answer: A
367) Position of flatulent??
A. Flat B. Side lying C. Knee chest Answer: C
368) Dexamethasone function? - promote lung Maturation
369) The nurse was giving health education for elderly patient. The patient asked her to repeat again many
times. What is the cause?
A. Patient does not care.
B. Patient confused
C. Sensory defect
D. Mental problem Answer: C
370) Neonate in well nursery. They suspect that he has diaphragmatic hernia. What does the Diagnosis
confirm diaphragmatic hernia?
A. Ultrasound *Prenatal - - - - - ultrasound
B. MRI
C. Radiography Answer: C *Postnatal - - - - - Radiography
171 ) Pregnant complain vaginal blood spots happened 3 days back and repeat again today. What should
the nurse instruct her?
A. Rest on her back and elevate the leg
B. Prepare her for vaginal examination Answer: A
372) A head nurse of an intensive care unit wrote a report about the needs of her department and gave it to
the nursing supervisor. Which of the following types of communication is this?
A. Upward B. Horizontal
C. Diagonal D. Downward Answer: A
373) The nurse is assigned to care for an 8-year-old child with a diagnosis of a basilar skull fracture. The
nurse reviews the health care provider's (HCP's) prescriptions and should contact the HCP to question which
prescription?
A. Obtain daily weight. B. Provide clear liquid intake.
C. Nasotracheal suction as needed. D. Maintain a patent intravenous line. Answer: C
374) What precautions are necessary when caring for a patient with Hepatitis A to protect family from
infected?
A. wash dish and do not share.
B. not sharing the toilet.
D. Placing the patient in a private room. Answer: A
375) child with rheumatic fever how can prevent for recurrent?
A- Isolate the infected child.
B- Give vaccine for mother.
C- Let them complete the antibiotic course.
D- Give vaccine in 9 months. Answer: C
376) What is Thyroxine side effect?
A. Depression B. Fever C. Weight gain D. Increase appetite ✅ Answer: D
377) A neonatal is admitted to the NICU with a meningomyelocele. HR130 .. RR 28….. TEM 36.7 which of the
following action the nurse should perform to prevent infection of the meningomyelocele sac ?
A. Wash the sac with betadine every shift

ِ‫يِاْلَ أرض‬
‫أ‬ ُ ‫َم‬
ُ ‫ك‬
‫ثِِف‬ َ ‫اس‬
‫ِِفي أ‬َ َّ ‫ع‬
‫ِالن‬ ُ ‫ف‬
َ ‫ماِيَ أن‬ َّ َ‫وأ‬
َ ِ‫ما‬ َ
{ 40 }

B. Expose the defect to the room air


C. Apply antibiotic cream every 24 hours
D. Cover he sac with moist sterile saline dressing ✅✅ Answer: D
378) The nurse has problem in her schedule then she contacts with manager she follows chain of command.
She has meeting with her director. Which the appropriate approach of manager to deal with this situation??
A. Assertive B. Calming C. Reflective ✅✅ Answer: C
379) Child with Attention deficit hyperactivity disorder (ADHD). The parent complains that child hyperactive
and too much moving. What is your action?
A. Instruct parent to provide private school for child.
B. Instruct parent to provide normal school for child.
C. Instruct parent to allow child to share in activities that need more movement. ✅✅ Answer: C
380) What is the fetus’s name from 3 weeks to 8 weeks? Embryonic

381) The patient diagnosed with syphilis and He complains a


sore (chancre) that is usually painless. Which phase of syphilis
for patient?
A. Primary phase
B. Secondary phase
C. Tertiary phase Answer: A
382) MR Ahmed admitted to ICU with congestive heart
disease his vital signs BP 110/60 HR 120. Stroke volume 80 the
nurse expected cardiac output to be.
A. 180 B. 1700
C. 2400 D. 9600 Answer: D
COP (cardiac output) = HR (Heart Rate) × STROKE VOLUME
=120 × 80 = 9600
383) Nurse was providing research study for 200 people, the research is answered by 120 people only,
calculate how many people that shared in study in percentage?
A-20 % B-40 % C-80 % D-60 % ✅✅ Answer: D
384) The director is planning to develop hospital system. They will improve weakness point in the system.
What is the type of system?
A. Swot analysis system
B. Delphi analysis system Answer: A
385)infant who weighs 9 kg (19.8 lbs.) requires 900ml of fluids per day for maintenance fluids. The infant

ِ‫يِاْلَ أرض‬
‫أ‬ ُ ‫َم‬
ُ ‫ك‬
‫ثِِف‬ َ ‫اس‬
‫ِِفي أ‬َ َّ ‫ع‬
‫ِالن‬ ُ ‫ف‬
َ ‫ماِيَ أن‬ َّ َ‫وأ‬
َ ِ‫ما‬ َ
{ 41 }

typically consumes 120ml during each feeding. The infant must have how many feedings per day to meet the
fluid maintenance needs?
A. 4 B.8 C.10 D.12 Answer: B
386) After tonsillectomy / thyroidectomy which of the following should be provided for the patient??
A. Orange juice B. Apple juice C. Tomato juice D. Strawberry juice Answer: B
387) Most common cause for acute renal failure?
A. pyelonephritis B. Tubular destruction C. Urinary tract obstruction
D. Dehydration Answer: D
388) Qualitative research with evidence
A. Survey with ranking B. Interview one to one C. Collects data from system. Answer: B
389) In the middle of bathing a client, the unit secretary notifies the nurse that there is an emergency
telephone call. Which action should the nurse implement to best assure client safety?
A. Quickly finish the bath before answering the call.
B. Immediately leave the client’s room and answer the call.
C. Cover the client, place the call light within reach, and then leave to answer the call.
D. Leave the door open and ask staff to monitor the client, and then leave to answer the call. Answer: C
390) A nurse is caring for 14 - month - old immediately after a surgical repair of cleft palate. In which of the
following positions should the nurse put the child?
A. Prone ✅✅ B. lateral C. Supine D. Lithotomy Answer: A

391) What is the function of breathing exercise before anesthesia and preoperative?
A. To prevent respiratory complication after operation. B. To prevent Aspiration.
C. To prevent atelectasis. Answer: A
392) When put baby with high temp. In cold water to lower his temp.
A) convention B) evaporation C) conduction Answer: B
393) What is COPD complication?
A. Cardiac tamponade B. Pneumothora C. Cor pulmonale✅ Answer: C
394) The nurse forgot to raise side rails for patient.?
A. Breach in duty B. Nature in duty C. Assault Answer: A
395) patient is to receive a prescribed blood transfusion using a subclavian catheter. Catheter was inserted
about 30 minutes ago. Which of the following is the most appropriate step before starting blood transfusion?
A. Check patency of catheter by flushing with normal saline solution.
B. Position patient with the head of the bed elevated 30 degrees.
C. Review result of chest radiograph completed 15 minutes ago.
D. Obtain patient's most recent complete blood count result. Answer: C
396) What is the best measure to ensure privacy and protect patient information during uploading
password??
A. Write the password in a paper and keep it.
B. Use alphabetic and numbers for password✅
C. Expose patient information record and don't share password
D. Refrain use of the name and Date of birth for the patient Answer: B
397) Patient habitually worries about any symptoms, which is defense mechanism?
A. Conversion defense mechanism. Answer: A

ِ‫يِاْلَ أرض‬
‫أ‬ ُ ‫َم‬
ُ ‫ك‬
‫ثِِف‬ َ ‫اس‬
‫ِِفي أ‬َ َّ ‫ع‬
‫ِالن‬ ُ ‫ف‬
َ ‫ماِيَ أن‬ َّ َ‫وأ‬
َ ِ‫ما‬ َ
{ 42 }

398) Which of the following can lead to infertility in adult males?


A. Orchitis B. German measles
C. Rubella D. Chicken pox Answer: A
399) What is the most common cause of fetal hypoxia??
A. fetal descent B. Full Dilation of cervical
C. Contraction of uterine D. cervical effacement Answer: C
400) Which of the following the most common cause of dissociative disorder?
A. Family history B. Drug abuse
C. Traumatic Event D. Behavioral changes Answer: C
401) Cause of hemodilution in pregnancy?
A. Anemia B. Decrease RBC
C. Increase WBC D. Increase plasma Answer: D

402) During suction of child, ensure that each pass of the suction catheter take no more than?
A-5 Sec✅ B-10 Sec C-15 Sec Answer: A
403) What is the food should be limited during pregnancy??
A. Pasteurized Milk B. Processed Cheese
C. Soft cheese D. Yogurt Answer: C
404) Patient after cardiac surgery Recovery return back to hospital with complication. He stated After I have
discharged from hospital to my house, I had thrown all medication because I felt that I improved no need for
drug. Which of the following action should the nurse take?
A. Assess patient reasons for his action B. Add new nursing diagnosis of None compliance✅
C. Give Instruction to the patient Answer: B\

405) who is pt. high risk for electrolytes imbalance?


A. 2 years pt. with gastrostomy feeding.
B. Pt with ileostomy
C. Pt with colostomy Answer: B
406) when caring for a patient with an ostomy the nurse knows that extra skin protection for the personal skin
is MOST important for those patients with a (an):
a- Ileostomy
b- Ascending colostomy
c- Transverse colostomy
d- Sigmoid colostomy Answer: A
407) a patient with long-standing diabetes mellitus (type I) is scheduled for surgical amputation of 4
gangrenous toes on the right foot. Which surgical intervention would this be classified as?
A. Palliative
B. Curative
C. Reconstructive
D. Diagnostic Answer: A
408) the nurse is teaching a group of nursing students how provide care to clients with anger management and
aggression problem. Which statement made by the nurse is most indicative of effective teaching?
A. Avoid taking the client's anger personally.
B. Defend yourself from the client's anger.
C. Take responsibility for the client's anger.
D. Respond firmly to the client to relieve your own stress. Answer: A

ِ‫يِاْلَ أرض‬
‫أ‬ ُ ‫َم‬
ُ ‫ك‬
‫ثِِف‬ َ ‫اس‬
‫ِِفي أ‬َ َّ ‫ع‬
‫ِالن‬ ُ ‫ف‬
َ ‫ماِيَ أن‬ َّ َ‫وأ‬
َ ِ‫ما‬ َ
{ 43 }

409) mother asked the nurse that while she was changing the diaper for her female newborn, she noticed a
brick red stain on it. What is the best response by the nurse?
A. It is a sign of low iron excretion. B. It is expected in female newborn.
C. It is due to medication given to the mother.
D. it due to medication given to the newborn. Answer: B

410) A 18-year-old girl was under weight and malnourished her height was 163 cm, her body weight was 45 kg
and her BMI was 16.9. She was admitted in the Female Medical Word for severe epigastric pain. The nurse
observed that on the second day she induced vomiting after having her dinner. On asking, she replied that
inducing vomiting from time to time helps clear the stomach bacteria.
Which aspect of the nursing care area is altered?
o A. Self-concept
o B. Health perception
o C. Value-belief system
o D. Nutrition management Answer: B
411) Which of the following action is correct when college a urine specimen from a client’s indwelling urinary
catheter?
A. Collect urine from the drainage collecting bag using sterile. technique.
B. Disconnect the catheter from the drainage tubing to collect urine. using clean technique.
C. Remove the indwelling catheter and insert a sterile straight catheter. to collect urine.
D. Aspirate specimen from the tubing draining port using needle and syringe with sterile technique Answer: D\

412) A patient with chronic obstructive pulmonary disease (COPD) Experiencing frequent dyspnea which of
the following exercise would teach the patient how to BETTER control breathing?
a. Lower side rib
b. Segmental
c. Pursed lip
d. Diaphragmatic Answer: C

413) Pregnant mothers in 9 weeks she came for ER.with vaginal bleeding and lower abdominal cramp ...on
U/S ...the concept products in lower uterus
A. Missed abortion
B. Threaten abortion
C. Inevitable abortion
D. Complete abortion Answer: C

414) How can the nurse detect the poor oxygenation for fetus??
A. Absent Deceleration
B. Absent acceleration
C. Non-reactive fetal heart ctg tracing
D. Breathing movement Answer: B

415) A 29-year-old woman patient was brought to the Outpatient for the removal of stitches on her left cheek
which was treated nine days back after being involved in road traffic accident. She covers her face completely
and requests to be seen by a female doctor. The site of the wound was red, swollen and some pussy points were
visible. She states that she did not wash her face since her accident and kept her face covered all the time as
she did not want anyone to see it. Which of the following should the nurse do to elevate the patient's self-
esteem?
A. Involve her husband in her care.

ِ‫يِاْلَ أرض‬
‫أ‬ ُ ‫َم‬
ُ ‫ك‬
‫ثِِف‬ َ ‫اس‬
‫ِِفي أ‬َ َّ ‫ع‬
‫ِالن‬ ُ ‫ف‬
َ ‫ماِيَ أن‬ َّ َ‫وأ‬
َ ِ‫ما‬ َ
{ 44 }

B. Explain that stitches marks will heal soon.


C. Ask her to wash face and wear light make up.
D. Discuss that it is normal to feel low temporarily. Answer: D

416)-year-old woman patient was brought to the Outpatient for the removal of stitches on her left cheek which
was treated nine days back after being involved in road traffic accident. She covers her face completely and
requests to be seen by a female doctor. The site of the wound was red, swollen and some pussy points were
visible. She states that she did not wash her face since her accident and kept her face covered all the time as
she did not want anyone to see it. What is the most appropriate nursing diagnosis?
A. Hopelessness
B. Social isolation
C. Anxiety
D. Powerlessness Answer: B
417) Pregnant complain from heart burn. What should the nurse advise her?
A. Drink water after ever meal
B. Elevate head of the bed Answer: B
418) The head nurse notice conflict between the stuff nurses and try to solve this conflict. She told them that
one side give in the pt. satisfaction what type of this solution??
a- avoidance
B- accommodation
C. Compromise
D. Collaboration Answer: B

419) Which model is most useful in examining the cause of disease in an individual, based upon external
factors?
A. The preventive health problem model
B. The Agent-Host-Environment Model Answer: B

420) Recurrent Urinary Tract infection by Escherichia coli bacteria. Which of the following external factor is
the cause?
A. Host
B. Agent
C. Environment Answer: B

421) A patient is admitted to the hospital for a right total knee replacement. during the preoperative
assessment the patient reports no known during allergies, but the patient does. report a severe allergic
reaction to shellfish. Why is this allergic reaction to shellfish of special? importance?
a. Shellfish allergies could indicate a negative reaction to anesthesia.
b.This patient will not be able to receive donor packed red blood cells if needed.
c.A seafood allergy can be an indication of allergy to provide one-iodine.
d.A seafood allergy can be precursor to malignant hyperthermia. Answer: C

422) An 84-year-old bedridden patient with Alzheimer's disease lives in a long-term care facility. A nurse
enters the patient's room at 7:30 and finds the patient lying towards the foot of the bed on wet she draw sheet
beneath t. Another nurse is called to the room to assist. They place a patient and prepare to first move him
upwards in the bed which of the following is the best explanation for using the sheet?
A- provide material for grasping.
B- Keep the patient dry.
C- prevent skin rubbing.

ِ‫يِاْلَ أرض‬
‫أ‬ ُ ‫َم‬
ُ ‫ك‬
‫ثِِف‬ َ ‫اس‬
‫ِِفي أ‬َ َّ ‫ع‬
‫ِالن‬ ُ ‫ف‬
َ ‫ماِيَ أن‬ َّ َ‫وأ‬
َ ِ‫ما‬ َ
{ 45 }

D- Minimize patient confuse. Answer: C

423) Patient came to a scheduled appointment for follow up when the nurse checks the vital signs BP: 130/92
RR: 18 HR: 86T: 37Which of the vital signs is abnormal?
A- Temperature
B- Heart rate
C- Respiratory rate
D- Blood pressure Answer: D Pre hypertensive
424(Which hormones is responsible for uterine wall change and breast enlargement during pregnancy,
leucorrhea and hyperemesis. Which of the following placental hormone is responsible??
A. Progesterone
B. Estrogen ✅✅
C. HCG Answer: B
425) Which of the following hormones are responsible for implantation of ovum inside Uterine wall??
A. Progesterone and estrogen ✅✅
B. LH and. FSH Hormones
C. Oxytocin Answer: A
426) There is crisis in hospital. Which of the following category of patients should be evacuated first during
fire???
A. Infant
B. Ambulatory✅
C. elderly
D. Critically ill Answer: B
427) – Patient came to ER complain from blood in stool. What is the most common diagnosis??
A. GERD
B. Gastroenteritis
C. BID bowel inflammatory disease ✅✅ Answer: C
428) Pregnant complain nausea too much early morning. The nurse should advise her?
A. Instruct her to increase activity
B. Advise her to get biscuits and cracker before getting out bed✅ Answer: B

429) Patient has cystic fibrosis and the nurse wants to perform postural drainage when is the best time for
posture drainage?
A-Before meals
B-After meals after half an hour of meals✅
C-Before sleeping
D. Afternoon Answer: B
430) nurses are caring for a client who has just returned from having a right-sided renal biopsy. The nurse
should intervene if a nursing assistant is observed doing which of the following?
A. Obtaining the client's vital signs
B. Positioning the client on the left side
C. Positioning the client on the right side
D. Providing the client with reading materials Answer: B

ِ‫يِاْلَ أرض‬
‫أ‬ ُ ‫َم‬
ُ ‫ك‬
‫ثِِف‬ َ ‫اس‬
‫ِِفي أ‬َ َّ ‫ع‬
‫ِالن‬ ُ ‫ف‬
َ ‫ماِيَ أن‬ َّ َ‫وأ‬
َ ِ‫ما‬ َ
{ 46 }

431) A nurse is a member of the Nursing Research Council of the hospital. His first assignment is to determine
the level of patient satisfaction on the care they received from the hospital. He plans to include all adult
patients admitted from April to May, with average length of stay of 3-4 days, first admission, and with no
complications. Which of the following is an extraneous variable of the study?
A. Date of admission
B. Length of stay
C. Age of patients ✅
D. Absence of complications Answer: C

432) A nurse is caring for a child who sustained a head injury after falling from a tree. On assessment of the
child, the nurse notes the presence of a watery discharge from the child's nose. The nurse will immediately test
the discharge for the presence of which of the following substance?
A. Glucose✅
B. b. Protein
C. c. White blood cells
D. d. Neutrophils Answer: A

433) Pregnant 36 weeks gestational with irregular painless contraction this signs called ?
A-Hegar's sign
B- Chadwick sign
C-Braxton Hicks (false labor) ✅
D-True labor Answer: C

434) The nurse is preparing to insert peripheral Cannula for patient. Which of the following patients avoid
insert cannula in lower extremity for them it is risky?
A. DM✅ B. Renal failure
C. COPD D. Heart failure Answer: A
435) To make cornea reflux you should use?
A. Straily gloves B. Swap cotton✅
C. Cotton tap D. Sterile tongue depressor Answer: B
436) Alkhurma hemorrhagic fever?? Flaviviridae virus ✅
437) Patient with TB going on Isoniazid medication. To prevent peripheral Neuropathy What should be
included in care plan advice?
A. Instruct low protein diet
B. Avoid sun exposure too much
C. Provide vitamin B6 intake✅
D. Increase fluid intake Answer: C
438) Which of the following circumstances is most likely to cause uterine atony and lead to postpartum
hemorrhage?
A- endometriosis
B- urine retention
C- cervical and vaginal tears
D- hypertension Answer: B
439) COMPLICATION OF Aortic Aneurysm - - - - - - Bleeding and hypotension
440) Aortic aneurysm - - - - - - - - - Emergency Care

ِ‫يِاْلَ أرض‬
‫أ‬ ُ ‫َم‬
ُ ‫ك‬
‫ثِِف‬ َ ‫اس‬
‫ِِفي أ‬َ َّ ‫ع‬
‫ِالن‬ ُ ‫ف‬
َ ‫ماِيَ أن‬ َّ َ‫وأ‬
َ ِ‫ما‬ َ
{ 47 }

441) Patient diagnosed MI then suffered from cardiogenic shock. What should the nurse observe in
symptoms?
A. Hypertension and hyperventilation
B. Hypotension and shortness of breath Answer: B
442) Risks of MI - - - - - - hyperlipidemia
443) The nurse receives ECG for patient via Internet. What is consider?
A. Telehealth B. Telecommunication
C. Telemedicine Answer: B
444) Group of health care givers provide care for group of patients have same diagnosis at same setting. What
is this considering?
A. Team B. Case management
C. Primary D. Functional Answer: B
445) The nurse received patient from Recovery room PACU. What is the first assessment should the nurse
observe?
A. Oxygen B. Level of consciousness Answer: A
446) The disease divided According to three factors host ،Environment ،Agent. The patient complete
antibiotics course. Then the disease returns again. What is the causative factor ??
A. Agent B. Environment
C. Host D. Reaction between host and environment Answer: D
447) The patient with central venous line. CVP measure 1 mmhg. The patient hemoglobin 9.8 g/dl. What
should give first ?
A. Give blood transfusion B. Give 2 L N/S Answer: B
448) Why should the nurse not keep drainage bag at the body level of the patient during transporting him?
A. Prevent entry of air B. Prevent entry of fluid to patient
C. Prevent tension pneumothorax Answer: B
449) The patient admitted hospital for surgical procedure. The preoperative preparing phase started. What is
the first action in this phase?
A. Obtain informed consent B. Surgical procedure explanation ✅ Answer: B
450)Patient with COPD uses pursed lip. What is the function for using this maneuver?
A. Relax respiratory muscle ✅✅ B. Facilitate inhalation to medication. Answer: A
451) Nursing diagnosis done and the nurse set priorities for all NANDA. Which step in nursing process the
nurse doing??
A. Planning ✅✅ B. Diagnosis C. Evaluation D. Implementation Answer: A

452) The nurse is teaching a patient about spironolactone (Aldactone).Which of the following instructions
should review with the patient?
A. Increasing the intake of foods that are high in potassium.
B. Taking the medication right before going to sleep.
C. Avoid seasonings that are labeled as salt substitutes.
D. Scheduling the medication so that a multivitamin is taken an hour later. Answer: C
placenta ‫👇👇 سؤال االمتحان بييجي بالشكلين دول ده سؤال وظيفة ال‬
453) A nurse explains some of the purposes of the placenta to a client during a prenatal visit. The nurse
determines that the client understands some of these purposes when the client states that the placenta:

ِ‫يِاْلَ أرض‬
‫أ‬ ُ ‫َم‬
ُ ‫ك‬
‫ثِِف‬ َ ‫اس‬
‫ِِفي أ‬َ َّ ‫ع‬
‫ِالن‬ ُ ‫ف‬
َ ‫ماِيَ أن‬ َّ َ‫وأ‬
َ ِ‫ما‬ َ
{ 48 }

A. Cushions and protects the baby.


B. Maintains the temp of the baby.
C. Is the way the baby gets food and oxygen✅✅?
D. Prevents all antibodies and viruses from passing to the baby. Answer: C

454 (Which purposes of placental functioning should the nurse include in a prenatal class? Select all that
apply.
A.it cushions and protects the baby.
B.it maintains the temperature of the baby.
C.it is the way the baby gets food and oxygen✅✅
D.it prevents all antibodies and viruses from passing to the baby.
E.it provides an exchange of nutrients and waste products between the mother and developing fetus✅✅ Answer:
C+E
Amniotic fluid ‫هي مش بتحمي البيبي ✋ اللي بيحمي البيبي‬

455)

Answer: premature ventricular complex


456) A 67-year-old man is admitted to the Post-Anesthesia Recovery unit following chest surgery. The patient
has a right chest tube that is attached to low suction. Three hours after admission to the unit, the nurse
observes the drainage output. from the chest tube is 300 milliliters. What is the most appropriate initial
intervention?
A. Notify the doctor. B. Reduce IV infusion rate.
C. Strip tube with roller device D. Re-position in left lateral decubitus Answer: A
457) 21-You are assisting a MD with the removal of a chest tube. What activity may the MD have the patient
perform while the chest tube is being remove??
A. Valsalva maneuver.
B. Leopold Maneuver
C. Chest Physiotherapy
D. Huff Cough Technique Answer: A
458) The nurse wittiness for another nurse while entering to the isolated pt room with airborne precautions
and she wear only gown and gloves, what is the appropriate action from the wittiness nurse to do?
1. Talk with the nurse about wearing mask
2. Talk with the education nurse to introduce lecture for staff about PPE
3. Inform the head department about the nurse
4. Ignored the nurse action Answer: A
459) Which of the following should be included in the nursing care to a woman during 2nd stage of labor?
A. Shave the perineum B. Administer enema to the woman.
C. Careful evaluation of prenatal history

ِ‫يِاْلَ أرض‬
‫أ‬ ُ ‫َم‬
ُ ‫ك‬
‫ثِِف‬ َ ‫اس‬
‫ِِفي أ‬َ َّ ‫ع‬
‫ِالن‬ ُ ‫ف‬
َ ‫ماِيَ أن‬ َّ َ‫وأ‬
َ ِ‫ما‬ َ
{ 49 }

D. Watch breathing, bear down with each contortion. Answer: D


460) MD writes an order for Ibuprofen 3 mg/kg by mouth every 6 hours for pain for a child. The child weighs
73 lb. Pharmacy dispenses you with 50 mg/2 ml. How many ml will you administer per dose? *
A. 1 ml/dose B. 3.9 ml/dose
C. 2.9 ml/dose D. 6 ml/dose Answer: B
461) A patient’s wound is draining thick yellow material. The nurse correctly describes the drainage as which
of the following?

A. Serous
B. Purulent
C. Sanguineous
D. Serous-sanguineous Answer: B
462) patient came to ER with complain abdominal patient history appendicitis patient now with change pass
stool distention abdominal and diharria which most diagnosis?
A. colitis
B. peritonitis
C. gastritis
D. none Answer: A
463) Schizophrenic patient with auditory hallucination. What should the nurse do?
A. Accept the patient B. Put patient in isolated room
C. Let him share in activity D. Say Don't hallucinate Answer: C
464) Patient has cast on his arm. The patient complaining sever pain. The nurse not palpating the pulse. What
is the first action?
A. It is normal B. Continue observation
C. Report the physician D. It is not normal Answer: C
465) The doctor decided DNR for patient. The patient refused that. What should the nurse do?
A. Follow hospital policy and respect his desire B. Give the care, continue treatment
C. Ignore the patient need D. Stop all medication Answer: A

466) An older-adult patient has developed acute confusion. The patient has been on tranquilizers for the past
week. The patient’s vital signs are normal. What should the nurse do?
A. Take into account age-related changes in body systems that affect pharmacokinetic activity.
B. Increase the dose of tranquilizer if the cause of the confusion is an infection.
C. Note when the confusion occurs and medicate before that time.
D. Restrict phone calls to prevent further confusion. Answer: A

467) The doctor decided DNR for patient. The patient refused that. He insisted to do CPR. What should the
nurse do?
A. Continue to produce care and reduce pain B. Discontinue the treatment Answer: A
468) what complications after cleft palate repair?
A) Teeth pain B) Deviated septum.
C) speech difficulty D)recurrent tonsillitis Answer: C]
469) what predisposing factor of child with autism
A) family history B) child abuse C) history of AHDH and mental illness Answer: A
470) The nurse is caring for Postpartum mother has DVT. What is the most appropriate action for her??

ِ‫يِاْلَ أرض‬
‫أ‬ ُ ‫َم‬
ُ ‫ك‬
‫ثِِف‬ َ ‫اس‬
‫ِِفي أ‬َ َّ ‫ع‬
‫ِالن‬ ُ ‫ف‬
َ ‫ماِيَ أن‬ َّ َ‫وأ‬
َ ِ‫ما‬ َ
{ 50 }

A. Ambulation
B. Elevate left leg.
C. Apply ice back.
D. Give prescribed anticoagulant Answer: B

471) The nurse should avoid the use of the dorsolateral site for an intramuscular injection in children because
of the risk of injury to which of the following nerves?
A. Vigus
B. Sciatic
C. Ilioinguinal
D. Lumbar plexus
Answer: B

472) Persons try to stop smoking after that smoking cessation done successfully and continue in cessation
which stage of change the patient achieved?
A. He stop applying change process in action stage.
B. He stop applying change process in preparation stage.
C. He stop applying change process before maintenance stage complete.
D. He stop applying change process after Maintenance complete Answer: D

473) Nurse manger float RN nurse from surgical ward to CCU which patient should be assigned to this nurse
A. Patient just transferred after coronary angiography.
B. Patient postoperative after open heart surgery
C. Patient on I. V Lasix have congestive heart failure✅✅
D. Patient had ventricular tachycardia and connected to cardiac monitor Answer: C

474) Nurse mangers pull out (float) nurse from medical ward to CCU which of the following Patients should
be assigned to the nurse:
A. Patient just transferred from cardiac catheterization✅✅
B. Patient just received with unstable angina on heparin infusion.
C. Patient need discharge education about coronary stenting.
D. Patient on Lasix I. V have acute left ventricular failure. Answer: A

ِ‫يِاْلَ أرض‬
‫أ‬ ُ ‫َم‬
ُ ‫ك‬
‫ثِِف‬ َ ‫اس‬
‫ِِفي أ‬َ َّ ‫ع‬
‫ِالن‬ ُ ‫ف‬
َ ‫ماِيَ أن‬ َّ َ‫وأ‬
َ ِ‫ما‬ َ
{ 51 }

RN ‫لو ساب السؤال ومحددليش‬


LPN‫يبقى هختار على اساس‬
‫ هيحددلي في السؤال ويكتبه‬RN ‫لكن لو عايز‬
‫ دول الناس اللي بيتفوضلهم بس‬As a general
‫✋✋نفهم ونختار عموما الن مش دول السؤالين اللي هيجوا بس هيجيبو أسئلة تانيه مش عايزين نستقبل اإلجابة الصح وخالص‬
475) A newly nursing director assigned to a hospital. That is the first should he do?
A. Evaluates the staff
B. Change the head nurses.
C. Change the roles
D. Nothing to do Answer: A
476) A patient is admitted to the hospital with klebsiella pneumoniae. During the initial intravenous dose of
Amikacin (amikacin sulfate), the patient develops severe respiratory distress. This is most likely:
A) A side effect
B) An indication of drug tolerance
C) A drug allergy
D) A toxic effect Answer: D
477)A 47-year patient received spinal ane sthesia five hours ago in the operation theatre and was transferred
to the surgical unit. Few hours later the patient complains of incisional pain. The surgical site is dry and intact
but the patient looks pale irritated. BP 175/90 HR 112 RR 27 TEM 37.6 What is the most appropriate nursing
intervention?
A. Take a 12 lead ECG
B. Administer pain medication
C. Notify treating physician ✅
D. Strat IV infusion and place patient in supine Answer: C
479) A patient with impaired hearing, He is wearing a hearing aid. The nurse is preparing him to operation,
He asked the nurse to keep his hearing aid while he is going to operation.
A. Leave patient wear his hearing aid.
B. Ask Operation Nursing staff
C. Say it is forbidden. You have to remove it.
D. He does not need it because he will be given general anesthesia. Answer: A
480) Patient is taking streptokinase. Suddenly he is complaining fever, shortness of breathing. What should
the nurse expect that effect?
A. Side effect B. Allergic reaction C. Normal response Answer: B
481) Cause of decrease the natural immunity of reproductive system after menopause??
A. Decrease cervical and vaginal mucus B. Decrease genital tissue and subcutaneous fats
C. Thinning of pubic hair Answer: B
482) According to Erickson, the psychosocial task of adolescence is the development of a sense of identity. The
nurse can best promote the development of a hospitalized adolescent by:
A. Emphasizing to follow the facility regime.

ِ‫يِاْلَ أرض‬
‫أ‬ ُ ‫َم‬
ُ ‫ك‬
‫ثِِف‬ َ ‫اس‬
‫ِِفي أ‬َ َّ ‫ع‬
‫ِالن‬ ُ ‫ف‬
َ ‫ماِيَ أن‬ َّ َ‫وأ‬
َ ِ‫ما‬ َ
{ 52 }

B. Allowing the parents and siblings to visit frequently.


C. Arranging for tutoring in schoolwork.
D. Encouraging peer visitation Answer: C

483) One day a male client with the diagnosis of borderline personality disorder describes a situation that
happened at work when his immediate supervisor reprimanded him for not completing an assignment. He
explains that it was not his fault and states, "people get angry and take it out on me." Which defense
mechanism identified by the nurse was the client using in this situation?
1. Denial 2. Projection
3. Displacement 4. Intellectualization Answer: B
484) Some of Narcotic group arranged to C1-C5. That is According to:
A. Dose B. Abuse ✅ C. Severity Answer: B
485) Pregnant complain leg cramp. What should the nurse advise her?
A. Elevate the leg above hip level B. Change her position continuously✅
C. Apply foot support during setting D. Wearing slipper Answer: B
486) Postpartum mother was complaining of mastitis on the Right breast, and she told the nurse that it's safe
feeding by my left side. What should the nurse response?
A. Breastfeed from both sides it is safe B. The left side is only safe
C. the Right side is the Safe Answer: A
487) Extrapyramidal adverse effects and symptoms are most often associated with which of the following drug
classes?
A. Antidepressants B. Antipsychotics
C. Antihypertensives D. Antidysrhythmic Answer: B
488) Sudden infant death syndrome (SIDS) is one of the most common causes of death in infants. To
prevent this syndrome, the nurse should keep infant inside incubators in which position??
A. Left side B. Back✅✅ C. Prone D. Right side Answer: B
489) Sudden infant death syndrome (SIDS) is one of the most common causes of death in infants. Which of the
following position should the nurse avoid for infant that cause SIDS??
A. Left side B. Back C. Prone✅✅ D. Right side Answer: C
490) After 3 days of breast feeding a post partial patient reports nipple soreness. To relieve her discomfort the
nurse should suggest that she:
A. lubricate her nipples with expressed milk before feeding.
B. dry her nipples with soft towel after feeding.
C. Apply warm compresses to her nipples just before feeding.
D. Apply soap directly to her nipples, and then rinse. Answer: A

ِ‫يِاْلَ أرض‬
‫أ‬ ُ ‫َم‬
ُ ‫ك‬
‫ثِِف‬ َ ‫اس‬
‫ِِفي أ‬َ َّ ‫ع‬
‫ِالن‬ ُ ‫ف‬
َ ‫ماِيَ أن‬ َّ َ‫وأ‬
َ ِ‫ما‬ َ
{ 53 }

491) What is the primary sign for ARF acute renal failure?
A. Hypertension B. Oliguria C. Face edema Answer: B
492) Infant 11 month On ECG heart rate 240 b/m. Embedded in the QRS complexes and absent p wave.
What is the expected diagnosis for the infant???
A. Ventricular tachycardia (VT) B. Bradycardia
C. Supraventricular tachycardia (SVT) D. Atrial fibrillation Answer: C
493) The nurse is caring for a client who has had a right modified radical mastectomy this morning. Which
exercise should the nurse encourage the? client to perform this evening.
a- Hair combing exercises with the right arm. b- Wall climbing exercises with the right arm.
c- Movement of the fingers and wrists of the right arm d- Exercises of the left arm only Answer: C

494) There is lecture about health model and illness model, and in the lecture included the agent / host /
environment model.What is considered that model to be classified as:
A. Stage of illness B. Risk factor✅ Answer: B
495) Patient is complaining from hypostatic hypotension Before that, his blood pressure was 110/70, the pulse
was 76 b/m. What is the expected vital signs?
A. BP 90/60 HR 69 B. BP 88/60 HR 100✅ Answer: B
496) A 9 -year-old child is admitted to the Emergency Department injury. The child is oriented to the place,
person and time, spontaneously, obeys commands. The nurse is doing a ped Coma Scale (PGCS). Which of the
following score the nurse should record?
A. 3 B. 8 C. 12 D. 15 Answer: D
497) Pregnant women in16 week sever hyperemesis garvidurm what is the complication for pregnant?
A. Preeclampsia B. Electrolyte imbalance C. Miscarriage Answer: B
498) The nurse is caring for a client who has had a chest tube inserted and connected to water seal drainage.
The nurse determines the drainage system is functioning correctly when which of the following is observed:
A. Continuous bubbling in the water seal chamber B. Fluctuation in the water seal chamber✅
C. Suction tubing attached to a wall unit. D. Vesicular breath sounds throughout the lung fields.
Answer: B
499) The nurse is caring for a client who has just had a chest tube attached to a water seal drainage system.
To ensure that the system is functioning effectively the nurse should:
A. Observe for intermittent bubbling in the water seal chamber✅
B. Flush the chest tubes with 30-60 ml of NSS every 4-6 hours
C. Maintain the client in an extreme lateral position.
D. Strip the chest tubes in the direction of the client Answer: A
500) client chest tube is connected to a chest tube drainage system with a water seal. The nurse noted that the
water seal c is fluctuating with each breath that client takes. The fluctuation means that.
A. There is an obstruction in the chest tube. B. The client is developing emphysema.
C. The chest tube system is functioning properly D. There is leak in the chest tube system. Answer: C
501) What the purpose of a late vaginal_rectal culture during ante natal screening?
A. To screen for group B streptococci B. To detect a Chlamydia infection.
C. To screen for gonorrhea D. To screen for syphilis Answer: A

ِ‫يِاْلَ أرض‬
‫أ‬ ُ ‫َم‬
ُ ‫ك‬
‫ثِِف‬ َ ‫اس‬
‫ِِفي أ‬َ َّ ‫ع‬
‫ِالن‬ ُ ‫ف‬
َ ‫ماِيَ أن‬ َّ َ‫وأ‬
َ ِ‫ما‬ َ
{ 54 }

502) What is the Noninvasive test for Hirschsprung disease?


A. Ultrasound B. Anorectal manometry C. X- ray Answer: B

503) Nurse is watching the cardiac monitor, and a client’s rhythm suddenly changes. There are no P waves;
instead, there are wavy lines. The QRS complexes measure 0.08 second, but they are irregular, with a rate of
120 beats a minute. The nurse interprets this rhythm as:
A-Sinus tachycardia B-Atrial fibrillation
C-Ventricular tachycardia D-Ventricular fibrillation Answer: B
504) The patient is postoperative cardiac surgery. The pulse rate is changed to bradycardia. What is the most
common cause of decrease Heart rate after operation?
A. Pain B. Stimulate Vegas nerve C. Anxiety Answer: B
505) child with iron deficiency complains of feeling tired all the Times. The nursing diagnosis of fatigue is
related to:
A- Decreased ability of the blood to transparent oxygen to the Tissues
B-An increased paroxysmal abdominal pain and distension to the stomach
C-A decreased anxiety level during hospitalization.
D-A decreased nutritional intake with mal absorption of Answer: A
506) 35 years old female patient after taking breast biopsy she diagnosed with 3rd stage breast cancer
She asked what is the purpose of chemotherapy?
A-it make no need for surgery.
B- destroy cancer cells and prevent it from spreading.
C- prevent recurrence of cancer. Answer: B
507) An old Man 72 years old with insomnia which most likely If patient said will need further instructions?
A-Avoid nap B-Read book before sleep
C- Put the temperature room cold. D-avoid drink caffeine after the dinner. Answer: c
508) When should stop performing sponges bathing for febrile infant?
A. Shivering B. Tachycardia
C. Mettled skin D. Increase respiratory rate Answer: A
509) The doctor is allow woman to go in with her husband during CPR ressustation. What should the
nurse tell her?
A. Try to stand away in seperate place to prevent stoping work because the work at this time very fast✅
B. Don't cry or move during CPR and don't try to share Answer: A

510) A nurse is reviewing blood chemistry result of a client magnesium 1.7 0.7-1.2 With of the following
symptoms the nurse be monitored.
A. Trousseau sign B. Depressed respirations

ِ‫يِاْلَ أرض‬
‫أ‬ ُ ‫َم‬
ُ ‫ك‬
‫ثِِف‬ َ ‫اس‬
‫ِِفي أ‬َ َّ ‫ع‬
‫ِالن‬ ُ ‫ف‬
َ ‫ماِيَ أن‬ َّ َ‫وأ‬
َ ِ‫ما‬ َ
{ 55 }

C. Elevated blood pressure D. Increased tendon reflex Answer: B


511) What is the priority nursing assessment before administering methergine for management of postpartum
hemorrhage?
A. Blood pressure B. Uterine atony C. Amount of lochia D. Deep tendon reflex Answer: A
512) A pregnant woman informed a nurse that she was never vaccinated against rubella. Which of the
following is the best? nursing advice?
A. No need for her to be distress, rubella is not harmful to the fetus.
B. The vaccine can be administered any time during her pregnancy.
C. She can get pregnancy any time after receiving the vaccine.
D. She should be vaccinated after delivery of the baby she get discharge. Answer: D
513) The antenatal clinic nurse was assessing a 32yaers old gravida 2, para1 pregnant mother on the fundal
height at 36 weeks. What is the expected fundus position?
A. Umbilicus B. Xiphoid process C. Symphysis pubis Answer: B
514) A mother is asking the nurse how sickle cell is could be deducted. Which of the following is at test
detected?
sickle cell disease of traits?
A. Hemoglobin electrophoresis B. Bone marrow aspiration
C. Complete blood count D. Free erythrocyte protoporphyrin Answer: A
515) A child with thalassemia was given deferoxamine (Deferral); which of the following should alert the
nurse to notify the physician?
A. Decreased hearing B. Hypertension
C. Red urine D. Vomiting Answer: A
516) The nursing assistant with 20 years of experience approaches a recently graduated nurse who recently
passed the licensing examination. The nursing assistant states “the only difference between you and me is the
size of our pay. checks”. Which of the following is the most appropriate? response for the newly graduated
nurse?
A. assert a hierarchical position.
B. emphasize the additional education received.
C. explain the legal difference in the scope of practice.
D. focus on the need to work together for quality client care. Answer: D

517) During clinical assessment, the nurse suspected that a child has been psychological abuse. Which
statement by the child is most likely support the suspicion.
A. My parents tell me that I am stupid.
B. My parents hurt me to get attention from the doctors.
C. My mother forces do not give meals on times.
D. My uncle shows me picture of nude people. Answer: A
518) A nurse accidentally dropped a medication ampule, informed the charge nurse and complicated an
incident. report form. Which of the following best describes the nurse role?

ِ‫يِاْلَ أرض‬
‫أ‬ ُ ‫َم‬
ُ ‫ك‬
‫ثِِف‬ َ ‫اس‬
‫ِِفي أ‬َ َّ ‫ع‬
‫ِالن‬ ُ ‫ف‬
َ ‫ماِيَ أن‬ َّ َ‫وأ‬
َ ِ‫ما‬ َ
{ 56 }

A. Responsiveness B. Timely decision making


C. Profession accountability D. Abiding by moral obligation Answer: C

519) What is the Priority of patient with schizophrenia?


A. Self esteem B. Medication compliance
C. Family support D. Impaired thought process Answer: B

520) Which of the following is the leading cause of injury for primary school?
A. accidental suffocation B. motor vehicle✅ C. riding bicycle D. drowning Answer: B
‫👇👇 صيغة السؤال بطريقة تانيه‬
521) which of the following is the leading cause of injury for children who are more than five years old?
A. accidental suffocation B. motor vehicle✅ C. congenital anomalies D. drowning Answer: B
522) What is the importance of vitamin K to stop bleeding. How it act ??
A. Converting prothrombin to thrombin✅ Answer: A
___Action of vitamin K increases the release of clotting factor by forming prothrombin to secrete
factors D VII, IX, X, seventh, ninth and tenth that prevent bleeding.
523) Postpartum mother used the IUD as a contraceptive method. The nurse should instruct her ?
A. Check for strips every month follow up✅ B. Avoid intercourse for 1 week Answer: A
524) Child has urine incontinence two day ago with increase WBC in urine. What is the cause??
A. Psychological abuse B. Urinary tract infection C. Genitalia defect Answer: B
525) Which of the following methods need to use calendar in Natural method of family planning??
A. 14 day from beginning of the period. B. Body TEMP measure daily at morning Answer: A
526) What is the benefit of skin to skin after delivery?
A- Control mother and baby temperature
B- Initiate and facilitate early breast-feeding during 3rd stage of labor
C- Improve circulation of baby. D. Improve Uterine Contraction Answer: B
Control baby temperature ONLY NOT mother temperature
527) A 2-year-old child is admitted to pediatric ward. The mother cannot stay the child and she will visit him
in the
weakened only. Which of the following nursing action indicates an understanding of the emotional needs this?
child?
A. Give her a warm bath to calm down. B. Allow the child to suck on a pacifier.

ِ‫يِاْلَ أرض‬
‫أ‬ ُ ‫َم‬
ُ ‫ك‬
‫ثِِف‬ َ ‫اس‬
‫ِِفي أ‬َ َّ ‫ع‬
‫ِالن‬ ُ ‫ف‬
َ ‫ماِيَ أن‬ َّ َ‫وأ‬
َ ِ‫ما‬ َ
{ 57 }

C. Ask the parents to bring the child favorite toy.


D. Tell he parents that frequently visiting is unnecessary. Answer: C

528) Full term Newborn was delivered 2 hour ago. The nurse checks him observed. mild peripheral cyanosis.
What is the first action?
A. Call for help B. Notify the doctor C. Check temperature Answer: C
529) A community Nursing nurse is planning to conduct prenatal teaching and community assembly for
pregnant adolescents. Which teaching strategy would be most effective?
A. Offering open sessions for pregnant adolescents and anyone else who wants to attend.
B. Designing posters that girls can view individually in community Nursing centre.
C. Preparing group class sessions for teaching pregnant adolescents together.
D. Conducting one to one teaching sessions for both mothers and daughters. Answer: C
530) Child 2 years old has acute otitis media. What is the sign while the nurse observing child, confirm the
diagnosis?? A. Otorrhea B. Roll head side to side Answer: B

531) Pregnant women are very tired. She is feeling anxious and anxiety for her baby. What is describe woman
situation according to Maslow hierarchy??
A. Self esteem B. Safety
C. Physiological needs D. Love and belonging Answer: B
532) An 11-year-old child has been diagnosed with type 1 diabetes mellitus. Which of the following education?
should the nurse explain about the exercise?
A. Extra insulin is requiring during exercise.
B. Extra snacks are needed before exercise.
C. Exercise will increase blood glucose.
D. Exercise should be restricted. Answer: B

533) A nurse is preparing a teaching session for elementary school students about the safety measures of
hazards. Which of the following is the most appropriate topics for safety measure for children at this age?
A. Sport activities B. Riding bicycle
C. Swimming D. Driving Answer: B

ِ‫يِاْلَ أرض‬
‫أ‬ ُ ‫َم‬
ُ ‫ك‬
‫ثِِف‬ َ ‫اس‬
‫ِِفي أ‬َ َّ ‫ع‬
‫ِالن‬ ُ ‫ف‬
َ ‫ماِيَ أن‬ َّ َ‫وأ‬
َ ِ‫ما‬ َ
{ 58 }

534) A 28-week pregnant mother is in the antenatal clinic with the complaint of vaginal irritation itching and
thick white secretion. she is diagnosed as having vaginal candidiasis she had been having the same infection
before the pregnancy for which she was prescribed the vaginal cream she is using the same cream for the past
few days. but her symptoms are not relieved what need to be emphasized to mother.
A. change clothes daily use sanitary napkins
B. Avoid self-treatment and seek doctor advise.
C. Use medicated bubble baths and reduce activates.
D. Was with warm water every two hours. Answer: B
535) A prim gravida woman who is pregnant at 30 weeks' gestation told the nurse that she is worried that
anything happens to her baby. Which of the following should be the proper nurse's response?
A. Ask the woman not to worry.
B. Ask the woman to express her concerns.
C. Attract the woman's attention to other issue.
D. Reassure the woman about the baby condition. Answer: B
536) The nurse is caring for an adult patient who is admitted to the hospital for fever and chills. The nurse
repeatedly finds a temperature of 40° C (104° F) in the morning and 38.9° C (102°F) at night. What does the
nurse infer about the fever pattern?
A. Intermittent Fever B. Remittent fever✅✅
C. Relapsing fever D. Continuous fever Answer: B
537) The nurse notes that the patient has been experiencing febrile episodes lasting more than 24 hours
interrupted by periods of normal body temperature that also last than 24 hours. What does the nurse infer
about the patient's fever pattern?
A. Intermittent Fever B. Remittent fever
C. Relapsing fever✅✅ D. Continuous fever Answer: C

538)A 10-year-old child is brought to the hospital with high fever and chills. The nurse records the vital signs
and finds that her temperature is 104° F (40° C), blood pressure is 130/85 mm Hg, and pulse rate is 120/min.
The fever remains mostly high but is interspersed with periods of normal body temperature. What pattern of
fever does the child have?
A. Sustained B. Intermittent✅✅
C. Remittent D. Relapsing Answer: B
539) Medical definition of Otic route for drug?
A. In the nose B. In the ear✅ C. In the eye Answer: B
540) The patient with CRF should perform peritoneal dialysis at home. The nurse is learning patient and
instructing the client to warm the dialyzing solution to 37 degrees Celsius. Why should the patient warm the
solution before using??
A. Dilate the peritoneal blood vessels✅✅ B. Maintain a constant body temperature.

ِ‫يِاْلَ أرض‬
‫أ‬ ُ ‫َم‬
ُ ‫ك‬
‫ثِِف‬ َ ‫اس‬
‫ِِفي أ‬َ َّ ‫ع‬
‫ِالن‬ ُ ‫ف‬
َ ‫ماِيَ أن‬ َّ َ‫وأ‬
َ ِ‫ما‬ َ
{ 59 }

C. Remove toxins from the body's cells.


D. Relax the abdominal muscles. Answer: A

541) Patient with chronic renal failure before performing peritoneal dialysis the nurse should warm the
solution before administering to:
A. Promote abdominal muscle relaxation. B. Maintain extra body warmth temperature.
C. Encourage the removal of serum urea ✅✅
D. Stimulate potassium back into body cells. Answer: C
542)The nurse is preparing a preoperative client for transfer to the operating room. The nurse should take
which of the following actions in the care of this client at this time?
a) Ensure that the client has voided b) Administer all the daily medications
c)Have the client practice postoperative breathing exercises.
d)Verify that the client has not eaten for the last 24 hours. Answer: A
543) A 6-month-old infant receives a diphtheria, tetanus, and acellular pertussis (DTAP) immunization at a
well-baby clinic. The mother returns home and calls the clinic to report that the infant has developed swelling
and redness at the site of injection. A nurse tells the mother.
a) Monitor the infant for a fever b) Bring the infant back to the clinic
c) Apply a hot pack to the injection site d) Apply an ice pack to the injection site Answer: D
544) A nurse hears a client calling out for help, hurries down the hallway to the client's room, and finds the
client lying on the floor. The nurse performs a thorough assessment, assists the client back to bed, notifies the
physician of the incident, and completes an incident report. Which of the following should the nurse document
on the incident report?
A) The client fell out of bed B) The client climbed over the side rails
C) The client was found lying on the floor
D) The client became restless and tried to get out of bed Answer: C
545) A nurse is monitoring a 3-year-old child for signs and symptoms of increased intracranial pressure (ICP)
after a craniotomy. The nurse plans to monitor for which early sign or symptom of increased ICP?
a) Excessive vomiting b) Bulging anterior fontanel
c) Increasing head circumference d) Complaints of a frontal headache Answer: A
546) The nurse is monitoring a client in the Medical- Surgical Unit who is recovering overnight following a
femoral angioplasty and observes that his leg has become pale and cool to the touch. After paging the health
care provider for an emergency evaluation, the nurse receives an angry telephone call asking why this
department always calls at 2 AM with ridiculous questions. Which of the following is the most appropriate
response?
A. "how would you feel if it was your father not getting the medical care, he needed."
B. "I am worried that this man may lose his leg unless something is done immediately."
C. "your behavior at this time is extremely unprofessional, and I will be making a formal report to my supervisor."
D. I am sorry, but it is hospital policy to report any post-operative complications to the health care provider on call.
May I ask what's has been bothering you about this department's pages”? Answer: B
547) A newborn admitted to the NICU with tracheoesophageal fistula (TOF). Which of the following nursing
intervention should be included?
A. Elevate the head for feeding. B. Elevating the head but keep the child NPO.
C. Insert a nasogastric tube for feeding. D. Encourage the mother to breast feed. Answer: B
548) A pregnant mother has been in labor for three hours with her membranes spontaneously ruptured. The
nurse observed that the liquor amnl is meconium stained. Which of the following is the nurse's best
interpretation?

ِ‫يِاْلَ أرض‬
‫أ‬ ُ ‫َم‬
ُ ‫ك‬
‫ثِِف‬ َ ‫اس‬
‫ِِفي أ‬َ َّ ‫ع‬
‫ِالن‬ ُ ‫ف‬
َ ‫ماِيَ أن‬ َّ َ‫وأ‬
َ ِ‫ما‬ َ
{ 60 }

A. Fetal hypoxia B. Low-lying placenta C. Intrauterine infection


D. It is mixed with maternal urine. Answer: A
549) Which of the following psychiatric term best describes this symptom "While talking, the psychiatric
patient repeated the same words stated by the nurse"?
A. Verbigeration. B. Echolalia. ✅✅
C. Echopraxia. D. Preservation. Answer: B
550) When the psychiatric patient repeats his words boat, boat, boat. What is the best term describing the
patient action?
A. Echolalia B. Palilalia ✅
C. Neologism D. Ward Salad Answer: B
551) A nurse is interviewing a client with schizophrenia when the client begins to say, "Kite, night, right,
height, fright." The nurse documents this as:
A. Verbigeration. B. Stilted language.
C. Clang association D. Neologisms. Answer: C
552) Which of the following is the best contraceptive for mother who is breastfeeding her baby?
A. Combined oral contraceptives. B. Contraceptive Patches
C. Estrogen only pills contraceptive D. Progesterone only "mini" pills Answer: D
553) A 2-year-old child is admitted to the pediatric unit with a diagnosed pneumonia. Which of the following
intervention would be a nursing priority?
A. Encourage coughing B. Encourage exercise
C. Perform postural drainage✅✅ D. Avoid food high in carbohydrates. Answer: C
554) A Client who underwent surgical procedure the preceding day normal assessment with an oral body
temperature of 37.5 0800 hours. If the condition remains stable the client is to be in the afternoon. which of
the following is the highest priority intervention?
A. Observation B. Inform the surgeon C. Administer a dose of aspirin.
D. Ensure proper use of the incentive spirometer. Answer: A
555) For a patient with a colostomy, which of the following-intervention is appropriate for preventing the risk
of the impaired skin related to exposure excretions?
A. Empty pouch when it is completely full B. Remove the skin barrier inspect the skin monthly
C. Recaps Skin barrier opening to size of stoma with each change
D. Cut an opening in the skin barrier then the circumference of the stoma ✅ Answer: D

556) Mastitis is an infection of the breast that occurs most often 2-4 after childbirth. Which of the following is
considered first line treatment of mastitis?
A. Drainage of breast abscess
B. Antibiotic therapy and cessation of breast feeding
C. Antibiotic therapy and continuation of breastfeeding
D. Advise mother to stop breastfeeding until infection is clean. Answer: C

557) A new community nurse was caring for a long - term disabled patient. The patient said, " I prefer to have
my care given this way “. the nurse explained more alternative approaches were clinically more effective.
However, the patient was Insisting on his opinion and said, " I am more comfortable if my care is given this
way " What should the nurse do in this case to manage the situation?
A. accept that the patient knows best what works for him.
B. review the recent research with the patient to convince him.

ِ‫يِاْلَ أرض‬
‫أ‬ ُ ‫َم‬
ُ ‫ك‬
‫ثِِف‬ َ ‫اس‬
‫ِِفي أ‬َ َّ ‫ع‬
‫ِالن‬ ُ ‫ف‬
َ ‫ماِيَ أن‬ َّ َ‫وأ‬
َ ِ‫ما‬ َ
{ 61 }

C. give evidence-based care according to current findings.


D. Compromise between patients preference and research approach of care Answer: D

558) There is psychiatric patient, every time says to a nurse, “You are my sister, always calling her my sister,”
and the nurse replied, “Your sister is not here". What should the nurse use from therapeutic communication?
A. Placing event in time or sequence B. Presenting reality
C. Refer to reality or Realization✅ Answer: C
559) After repairing for child with cleft lip left side. Which of the following position should the nurse put the
baby to prevent Aspiration?
1- Prone 2- On stomach 3- Right lateral 4- Left lateral Answer: C
560) When assessing neonate after delivery, the nurse notes the infant have caput succedaneum. What is the
nurse should do?
A. Code blue B. Order to physician C. Observation only nothing to do. Answer: C
561) What is the function of serotonin neurotransmitter receptors?
A. Motor B. Memory C. Inhibit pain Answer: B
Serotonin receptor ---- Mood + memory
Dopamine - - - - - - Motor
562) The child diagnosed with anemia and what should the nurse instruct about the food should be Limited
and avoided for child???
A. Apple juice B. Orange juice
C. Lemon juice D. Chocolate✅ Answer: D
563) Pregnant mother check the baby’s movements every day for an hour, she always feels among this hour
baby's movement 10 times or more. Once time while she was checking movement, she wasn't feel any
movement of the baby for an hour, what should the nurse do?
A. Instruct her to measure again the kick movement in another hour✅
B. Measure again movement on the next day
C. Ask her to come in clinic quickly to do a non-stress test Answer: A
564)A patient with pre-eclampsia is admitted to the unit with an order for magnesium sulfate. The nurse will
understand that the therapy is effective if?
A. Scotomas are present B. Ankle clonus is increased
C. No seizures occur D. Blood pressure drops Answer: C
565) The nurse was endorsing patient on TPN. The doctor prescribed to a certain rate, after 2 hours from
endorsement the nurse checked the TPN rate it was in low rate than the prescribed the solution was slowly.
What should the nurse do?
A. Ask doctor to write again new rate B. Return the rate to the prescribed rate
C. Keep the rate then adjust the rate to prescribed from the next bottle✅
D. Increase the rate immediately Answer: C
566)What is the food that a pregnant woman should not eat because it can lead to infection for her??
A. Pasteurized milk B. Unprocessed cheese✅
C. Processed cheese D. Yogurt Answer: B
567) The patient with a kidney transplantation, then he came after few days from discharge complaining from
fever, tenderness and edema. What should the nurse expect??
A. UTI B. Organ rejection✅ Answer: B
568) What position should the nurse place the head of the bed for heart failure patient with jugular vein
distention?

ِ‫يِاْلَ أرض‬
‫أ‬ ُ ‫َم‬
ُ ‫ك‬
‫ثِِف‬ َ ‫اس‬
‫ِِفي أ‬َ َّ ‫ع‬
‫ِالن‬ ُ ‫ف‬
َ ‫ماِيَ أن‬ َّ َ‫وأ‬
َ ِ‫ما‬ َ
{ 62 }

A. High fowler's B. Raised 10 degrees


C. Raised 30 degrees Semi fowler ✅ D. Supine position Answer: C
569) What is the priority patient for triage nurse to assess?
A. Patient with Sever chest pain B. Patient with Sever bleeding✅ Answer: B

570) Pregnant woman came to ER with rupture of membrane. Why should the nurse Limit vaginal
examination for her?
A. Prevent risk of infection✅ B. Avoid bleeding
C. Prevent further loss of membrane fluid D. Prevent fetal hypoxia Answer: A
571) The patient has joint arthroplasty surgery. The patient with urinary catheter and IV line is attached. He
was on oxygen, what needs immediate intervention?
A. Pain, redness, and swelling B. Shortness of breath and coughing✅ Answer: B
572) The nurse was doing research with two groups of smokers, the first group took Nicotine Patch and the
second group not acheived the target goal, what is the research type?
A. Case Study B. Cohort study✅ Answer: B
573) The psychiatric patient, She was speaking and laughing in a loud voice, She was wearing a brightly-
colored dress and an inconsistent make up, and She had a history of depression before, what is her diagnosis?
A. Major depression B. Bipolar disorder I✅
C. Bipolar disorder ll D. Cyclothymic disorder Answer: B
574) Woman came to ER with buffy eye (swelling)her eye disappeared she was pale and yellow, hair loss. She
was very exhausted. What should the nurse ask her?
A. Family history for same symptoms
B. Check nails and ask if she expose to any chemical substance✅ Answer: B

575) The pregnant woman is asking the nurse about the function of placenta. The nurse explains to her that
placenta provide your baby with food and Oxygen. Which another additional function for placenta??
A. Cushion and protect baby B. Produce HCG C. Produce contraction Answer: B
576) The size of uterus at the end of pregnancy
A. 900 g B. 1000 g C. 1500 g Answer: A
577) The uterus size after delivery:
A. 900 g B. 1000 g C. 1500 g Answer: B
578) What is the side effect of baclofen if given for diabetic patient?
A. Blurred vision. B. Increase of insulin demand C. Fever Answer: B
579) The patient with SLE (systemic lupus erythematous). The patient is complaining skin rash. What are the
factors affecting and cause the rash??
A. Skin rash after exposure of sun B. Skin rash increase in winter season Answer: A
580) A 45-year-old woman presents with a generalized rash that is not itchy. She reports that she has had the
problem for the past 15 years. Examination reveals a well-outlined, reddish plaque over the right gluteal fold.
The plaque has scales over it and is cracked in some areas Which of the following interventions is the most
appropriate?
A. Apply topical cream to the affected area B. Expose area to sunlight for twenty minutes' daily
C. Maintain immunosuppressant therapy regimen D. Increase dietary intake of vitamin A Answer: C
581) A patient with SLE (systemic lupus erythematous) report decreased urinary output during the past 2-4
days and chest pain.that is aggravated by breathing and coughing. The patient vital signs remain within the

ِ‫يِاْلَ أرض‬
‫أ‬ ُ ‫َم‬
ُ ‫ك‬
‫ثِِف‬ َ ‫اس‬
‫ِِفي أ‬َ َّ ‫ع‬
‫ِالن‬ ُ ‫ف‬
َ ‫ماِيَ أن‬ َّ َ‫وأ‬
َ ِ‫ما‬ َ
{ 63 }

baseline normal range s1 and s2 are present with audible friction rub. Which of the following statement would
be? appropriate for the nurse to make?
a. It sounds like SLE is being well controlled.
b. I need to get some nitroglycerine for your chest pain.
c. There may be some inflammation surrounding your heart.
d. Your symptoms may be due to a urinary tract infection. Answer: C
582) Which hormone is release from placenta?
A. HCG✅ B. Progestron C. Estrogen Answer: A
583) A patient with bowlegs due to abnormal bone formations and deformities has calcium level of 7.5
mg/100ml. Which of the following foods would the nurse most likely instruct the patient to add to a diet?
a- Organ meats b- Whole grains
c- Egg yolks d - Chicken meat Answer: C
584) According to Maslow hierarchy patient with schizophrenia has lack of motivation to see herself. What
are the needs for patient that lost??
A. Physiological need B. Self esteem C. Love and belonging D. Safety Answer: B
585) Patient with leukemia which blood component most affected?
A. RBCs B. WBC✅ C. Plasma D. Platelet Answer: B
586) Which of the following term the nurse used for explaining the normal respiratory rhythm and depth in a
patient?? A- Apnea B- Eupnea ✅ C- Brydpnea D-Tachypnea Answer: B
587) 22 years old girl people advise her that she has voice is very bad. she joined in Academic music
institution. which defense mechanism she used?
A) Denial B) displacement C) projection D) Repression Answer: C
588) Patient with chest tube drainage. What is the most appropriate nursing diagnosis?
A. High risk for infection✅ B. Disturbed body image Answer: A
589) When messure Blood pressure for adult patient with using child cuff. What is the expected result?
A. False high reading✅ B. False low reading C. Normal reading Answer: A

590) When messure Blood pressure for child patient with using adult cuff. What is the expected result?
A. False high reading B. False low reading✅ C. Normal reading Answer: B

591) The nurse was providing Education about oral Contraceptives pill for mothers What should the nurse
tell ?
A. If you take antibiotic drug change oral contraceptives ✅
B. if you forget one or more pills, take for a week two pills daily Answer: A

592) The nurse was thinking that doctor forgot to write one of drug prescriptions. Then later she discovered
that doctor wrote it and she didn't give medication to the patient. To reduce malpractice What should the
nurse do??
A. Ask the doctor write again the medication B. Write Incident report ✅ Answer: B

593)Bedridden PT nurse is doing pathing for him and she used lotion after bathing to prevent pressure ulcer
for patient. What is appropriate action??
A. Dry patient after bathing B. Use Air mattress ✅
C. Keep the sheet loosely. Answer: B

ِ‫يِاْلَ أرض‬
‫أ‬ ُ ‫َم‬
ُ ‫ك‬
‫ثِِف‬ َ ‫اس‬
‫ِِفي أ‬َ َّ ‫ع‬
‫ِالن‬ ُ ‫ف‬
َ ‫ماِيَ أن‬ َّ َ‫وأ‬
َ ِ‫ما‬ َ
{ 64 }

594) The mother was coming to give her baby vaccination. Then she asked the nurse about her baby that he
has chronic constipation and defecate ribbon like stool. The stool after bass is bad smell. Which of the
following questions should the nurse ask the mother??
A. What are the fluids is provide to your baby this week?
B. What is the fiber diet do you give to your baby?
C. Have your baby delayed passing meconium after delivery? Answer: C

595)What is the. Advantage of Nursing documentation system??


A. Cost for hospitals B. Spent time with patient.
C. Decrease of nurse’s satisfaction D. Decrease of documentation quality Answer: B

596) the patient with cancer. He is on I. V infusion the nurse observed there is Extravasations. What should
the nurse do?
A. Stop the Infusion immediately ✅ B. Slow the rate C. Report the doctor Answer: A

597) The patient is post-operative. After the nurse taking the vital signs what should the nurse assess next??
A pain assessment B. incision assessment C. Glasgow assessment Answer: B

598) Patient after laryngectomy and mastoidectomy. What is the priority nursing diagnosis?
A. Impaired verbal communication B. Airway clearance ✅ Answer: B
599) Physician order three times the normal dose (triple dose) - when the nurse calls him, he insists on giving
the ordered medication - what does the nurse do?
a. Call pharmacy b. Call another doctor.
C. Withhold medication and inform charge nurse.
D. Withhold medication and inform medical office. Answer: C
600) Patient was complaining sever pain. The nurse gave to him I. V morphine but there is no effect with him.
What should the nurse do?
A. Ask the patient for drug abuse ✅ B. Give another dose
C. Ignore the patient Answer: A
601) A patient is one-day post-operative repair of a large umbilical hernia. The patient complains of
abdominal pain and described feeling the sutures give way. Upon assessment of the abdomen the nurse
observes an evisceration. The nurse’s IMMEDIATE response should be to:
A. Medicate the patient for pain B. Instruct the patient to cough hard
C. Have the patient perform the Valsalva maneuver
D. Cover the abdomen with a sterile soaked dressing✅ Answer: D
602)The surgery and insisting on the nurse to explain to explain the procedure. Which of the following
nursing? intervention is the most appropriate?
A. Assess anxiety level. B. Assure he will be better after surgery.
C. Call doctor to Explain surgery. D. Give patient some sedative medication Answer: C
603)New nurse during orientation period she feel that her preceptor isolated from her. What should she do?
A-Ask questions as much as she can ✅ B-Ask to increase orientation days.
C- came to the work late. D. Ask preceptor for important summary lectures in Nursing. Answer: A
604) woman patient admitted with indwelling Foley catheter with a closed drainage system was order
collection for sterile urine specimen. Which of the following steps is considered the best way to collect the
sterile urine specimen?
A-Obtain the specimen from the drainage bag

ِ‫يِاْلَ أرض‬
‫أ‬ ُ ‫َم‬
ُ ‫ك‬
‫ثِِف‬ َ ‫اس‬
‫ِِفي أ‬َ َّ ‫ع‬
‫ِالن‬ ُ ‫ف‬
َ ‫ماِيَ أن‬ َّ َ‫وأ‬
َ ِ‫ما‬ َ
{ 65 }

B-Obtain the specimen from the aspiration port✅


C-Obtain the specimen from the first voiding in the morning
D-Obtain a clean mid-stream catch cleaning the perineum Answer: B
605) Which of the following lab. test must be done on a patient with major burns, prior to administration of
antibiotics??
A. complete blood account B. wound culture
C. type and cross match D. sensitivity studies✅ Answer: D

606) Couple asked the nurse which of the first investigation they should do for Infertility. Which of the
following should be the proper a nurse answer?
A. Hysterosalpingogram B. Serum progesterone
C. Semen analysis ✅ D. Endometrial biopsy Answer: C

607) The patient came to ER with second degree of burn on his dorsal left arm and dorsal of both leg TBSA.
The patient weight is 62 Kg. What is the total intravenous fluids the patient need?
A. 5580 ✅ B. 6580 C. 6680 D. 7680 Answer: A
608)Child with VSD. What is blood flow characteristic?
A- mixes atrium blood B-decreased pulmonary blood flow
C-increase pulmonary blood flow ✅ Answer: C
610) The community nurse was instructing for two partners will marry and advise them about health life and
obligations. What is this considering?
A. Counseling B. Education ✅ C. Screening Answer: B
611) How we can reduce chronic cases in the community?
A. Evaluate the strategies with outcomes B. Monitor risk in each family members ✅ Answer: B
612) Patient with abdominal cancer. During diagnosis we cannot examine accurately and can't detect. What
should use for him?
A. laparoscopy B. MRI ✅ Answer: B
613) For proper evaluation actions. What should the manager use?
A. field B. group✅ Answer: B
Dexamethasone function >>>promote lung Maturation.
614) Which of the following is the most affected drug for The psychiatric patient and cause more problem?
A. Atypical antipsychotic
B. Typical antipsychotic because it has side effects more than atypical ✅
C. Serotonin inhibitor D. Dopamine inhibitor Answer: B
615) When the nurse knows the strengths and weaknesses point in her personality. What is considering?
A. Self-awareness B. Self-understanding C. Self-concept✅ Answer: C
616) Getting birth for mother who has gestational diabetes. How will this affect delivery process?
A. It will be Easy B. It will be Difficult ✅
C. It will be fast D. It will be slow Answer: B
617) client is diagnosed as having secondary Cushing’s syndrome. The nurse knows that the client has most
likely been taking which medication:
A. Estrogen B. Penicillin

ِ‫يِاْلَ أرض‬
‫أ‬ ُ ‫َم‬
ُ ‫ك‬
‫ثِِف‬ َ ‫اس‬
‫ِِفي أ‬َ َّ ‫ع‬
‫ِالن‬ ُ ‫ف‬
َ ‫ماِيَ أن‬ َّ َ‫وأ‬
َ ِ‫ما‬ َ
{ 66 }

C. Lovastatin D. Prednisone Answer: D


618) A 7-year-old child is assessed with a urinary tract infection BP 99/55 TEMP 36.4. which of the following
is? the most appropriate nursing action?
A. Notify the physician immediately.
B. Notify the nurse manager immediately.
C. Document the findings in the nurse note
D. Put the patient in Trendelenburg position. Answer: C
619)Patient with decrease Glomerular filtration rate (GFR) and small kidney size 8cm, creatinine 8.5 mg/dl.
What is the appropriate treatment?
1-nephrotomy 2-nephroctomy 3-heamodialysis Answer: C\
620) Which of the following assess effective liver and consider liver function test?
A. Creatinine B. BUN
C. Urea D. Urine gravity Answer: B
621)The nurse is preparing to give injection to patient. The patient refused to get medication. Then the nurse
forced him and gave the injection to him aggressively. What is the description of nurse action?
A. Assault B. Battery
C. Abuse D. Malpractice Answer: B
622)The patient asked nurse to check his temperature then the nurse ignore the patient and she did not give
him care?
A. Malpractice B. Negligence C. Slander Answer: B

623)Hospital starts in implementation of EHR.EHR (Electronic health record). What is the first step before
implementation the EHC program?
A. Set plan for implementation program B. Check workflow
C. Check provided supplies for EHC. D. Explain for workers EHC concept and program. Answer: D
624)How to prevent heat loss via evaporation for neonate after delivery?
A. Avoid exposure to air draft. B. Avoid contact to wall.
C. Dry neonate and cover him, avoid any cold objects.
D. Warm any equipment's before touching neonate. Answer: C
625)Woman has experienced recurrent vaginal infection and she is asking the nurse how to prevent the
infection. What is the response if she says the nurse expect she need further instructions and consider alter
preventive measures that kill good bacteria?
A. Wear cotton untied underwear B. Wipes front to back each toilet.
C. Wear condom to prevent infection. D. Avoid douche and perfume spray. Answer: C
626)What is the Position after thyroidectomy??
A. Lateral flexed B. Semi fowler with slight neck flexed.
C. Prone head extended. D. High Fowler with neck extended. Answer: B
627)What is the Position during thyroidectomy??
A. Supine with hyper flexion of neck B. Supine with hyperextended neck
C. Semi fowler D. Lateral with slightly flexion neck Answer: B
628)Which of the following category of CTG?? See image.
A. Normal B. Category 1
C. Category 2 D. Category 3 Answer: A

ِ‫يِاْلَ أرض‬
‫أ‬ ُ ‫َم‬
ُ ‫ك‬
‫ثِِف‬ َ ‫اس‬
‫ِِفي أ‬َ َّ ‫ع‬
‫ِالن‬ ُ ‫ف‬
َ ‫ماِيَ أن‬ َّ َ‫وأ‬
َ ِ‫ما‬ َ
{ 67 }

629) Which of the following category of CTG?? See image� � �


A. Normal B. Category 1 C. Category 2 D. Category 3 Answer: D

630)Which of the following category of CTG?? See image. � � �


A. Normal B. Category 1 C. Category 2 D. Category 3 Answer: B

631)Which of the following category of CTG?? See image. � � �


A. Normal B. Category 1 C. Category 2 D. Category 3 Answer: D
CTG Categories ‫شرح‬

ِ‫يِاْلَ أرض‬
‫أ‬ ُ ‫َم‬
ُ ‫ك‬
‫ثِِف‬ َ ‫اس‬
‫ِِفي أ‬َ َّ ‫ع‬
‫ِالن‬ ُ ‫ف‬
َ ‫ماِيَ أن‬ َّ َ‫وأ‬
َ ِ‫ما‬ َ
{ 68 }

632)Category 2 : C II C 2 : Variable decelerations � �


*Vitamin A deficiency cause? Night Blindness
633) Low fat diet. Which of the following is?
A. Skimmed milk with yolk✅ B. Evaporated milk with whole egg Answer: A

ِ‫يِاْلَ أرض‬
‫أ‬ ُ ‫َم‬
ُ ‫ك‬
‫ثِِف‬ َ ‫اس‬
‫ِِفي أ‬َ َّ ‫ع‬
‫ِالن‬ ُ ‫ف‬
َ ‫ماِيَ أن‬ َّ َ‫وأ‬
َ ِ‫ما‬ َ
{ 69 }

634)year-old man presents to the Emergency Department complaints of breathing difficulties. He appears
restless and reports.that he has had a cough with thick green sputum for days. The nurse performs
auscultation over the lung fields. Blood pressure 130/80 mmHg Heart rate 100 /min, irregular Respiratory
rate 24 /min, irregular Temperature 39.0C Which clinical finding is most likely heard over the right lower?
A. Decreased vocal resonance. B. Decreased fremitus
C. Tympani D. Bronchial Answer: D
635)A 15-month-old child is admitted for hypospadias repair. After surgery, which of the following
instruction should be given to the parents?
A. Limit activity for 2 weeks B. Avoid apply ointment or powder.
C. Give a child a diet that is high in protein.
D. Isolate the child from other children with the infection Answer: A
636)A patient has a history of severe, uncontrolled epistaxis. The patient's blood. pressure and platelet count
are normal. To minimize the occurrence of bleeding episodes the nurse should teach the patient to.
a- Sleep with the head elevated on at least two to three pillows.
b- Apply firm pressure to the nostrils four times a day.
c- Apply a water- soluble lubricant to the nasal septum twice daily.
d- Minimize the intake of caffeine and increase fluids intake. Answer: C

ِ‫يِاْلَ أرض‬
‫أ‬ ُ ‫َم‬
ُ ‫ك‬
‫ثِِف‬ َ ‫اس‬
‫ِِفي أ‬َ َّ ‫ع‬
‫ِالن‬ ُ ‫ف‬
َ ‫ماِيَ أن‬ َّ َ‫وأ‬
َ ِ‫ما‬ َ
{ 70 }

637). 27 years old female brought to the Emergency Room accompanied by her husband. He described that
she had marked Weight loss with episodes of emesis in the past three months. She is diagnosed as having
anorexia. She reported feeling Febrile but had not measured her temperature. Her White Blood Count was
11,000/mm3. Which of the following most Likely describe her diagnostic criteria for her anorexia?
A. Restricting food intake B. Fear of gaining weight.
C. Problems with body image D. Binge eating disorder. Answer: B
638) 11-month baby how many ml orally he can receive?
A. 200 B. 400
C. 800 D. 1200 Answer: B
639) A woman is at 30 weeks' gestational age admitted to antenatal with premature rupture of membrane. the
nurse administered Dexamethasone to her according doctor prescription. She asks what the Drug is for.
Which of the following the best answer.
A. To promote fetal lung maturation. B. Prevention of chorioamnionitis
C. To increase uteroplacental exchange. D. Treatment of fetal respiratory distress Answer: A
640)A 71-year-old woman who resides in a long-term nursing home fell while walking downstairs. The
attending nurse arrives to find the patient sitting motionless on the stairs. She is alert and oriented but wishes
to rest. While she rests, the nurse reviews the chart and notes that her medication regimen includes
metformin, loratadine, warfarin and diclofenac. Which medication is most likely to increase the patient's risk
of injury?
A. Metformin B. Loratadine
C. Warfarin D. Diclofenac Answer: B
641)A nurse is giving discharge planning instructions to the parents of a one-year-old child acute otitis media
which of the following discharge instruction takes priority?
A. Administrate antibiotics as prescribed.
B. Administer influenza vaccination.
C. Breast feeding as long as possible.
D. Continue using of pacifier. Answer: A
642)Which of the following is the primary level of prevention?
A. Detect and treat existing disease.
B. Prevent illness or injury occurring.
C. Reduce the extent and severity of health problem.
D. Minimize disability and restore to prevent function. Answer: B
643)Patient with NGT and intermittent suction, suddenly the patient complaining of pain and the abdomen is
distended, at 10:00am NGT output=120 cc, 14:00 NGT output= 60 cc, what is the first action should be taken
by the nurse?
A. Increase the suction B. Administer pain medication.
C. Irrigate NGT to check patency. D. Notify the physician Answer: C
644)A 45-year-old woman presents to the Emergency Room and reported that she has lost her husband due to
lung cancer three months aga. She believes that she has colon cancer, despite of negative results in repetitive
and extensive diagnostic tests which were negative. What is the most likely diagnosis?
A. Pain disorder B. Hypochondriasis
C. Depersonalization D. Body dysmorphic disorder Answer: B
645)Patient with ABG PH 7.33 ‫ و‬HCO3 30, PCO2 50?
A. Compensate respiratory B. Compensate metabolic
C. Uncompensated respiratory D. Uncompensated metabolic Answer: C

ِ‫يِاْلَ أرض‬
‫أ‬ ُ ‫َم‬
ُ ‫ك‬
‫ثِِف‬ َ ‫اس‬
‫ِِفي أ‬َ َّ ‫ع‬
‫ِالن‬ ُ ‫ف‬
َ ‫ماِيَ أن‬ َّ َ‫وأ‬
َ ِ‫ما‬ َ
{ 71 }

646) Fastest and most sanitary thermometer is:


A. Glass B. Electronic
C. Chemical D. Tympanic Answer: D
647)Considered as the most accessible and convenient method for
temperature taking is:
a. Oral b. Rectal c. Tympanic d. Axillary Answer: A
648)The nurse is performing a prenatal examination on a client in the third. trimester. The nurse begins an
abdominal examination that includes Leopold. maneuvers. What information should the nurse be able to
determine after? performing the assessment’s first maneuver?
1. Fetal descent 2. Placenta previa
3. Fetal lie and presentation 4. Strength of uterine contractions Answer: C
649)The nurse prepares the client for the removal of a nasogastric tube. During the tube removal, the nurse
instructs the client to take which action?
1. Inhale deeply. 2. Exhale slowly.
3. Hold in a deep breath. 4. Pause between breaths. Answer: C
Valsalva Maneuver
‫ كمان مشهور اكتر وانا بركب او بشيل‬CVP
650)25 y. Patient came to ER with family history of ADHD try to suicide by ingesting 225 mg
(Methylphenidate) concerta since 3-hour, heart rate 190beat /min, what is the nursing priority:
1. Gastric lavage 2. Put pt. on cardiac monitor
3. Give activated charcoal 4. Check blood pressure Answer: C

651)A neonatal nurse performs Apgar assessment at 1 minute of birth to evaluate the physical condition
of the newborn and immediate need for resuscitation. At 1 minute, Apgar score is 7. At 5 minutes Apgar
score is to the progression of scores suggests
A. A healthy newborn ✅ B. The need for supplement oxygen
C. A genetic defect D. The infant is becoming stable Answer: A
652)Neonate fontanelle shape??
A. Soft and flat B. Soft and sunken Answer: A
653) Why do we give the patient plasma?
A-Immunity B-stability ✅ C-Fighting Virus Answer: B

ِ‫يِاْلَ أرض‬
‫أ‬ ُ ‫َم‬
ُ ‫ك‬
‫ثِِف‬ َ ‫اس‬
‫ِِفي أ‬َ َّ ‫ع‬
‫ِالن‬ ُ ‫ف‬
َ ‫ماِيَ أن‬ َّ َ‫وأ‬
َ ِ‫ما‬ َ
{ 72 }

654) Which hormone causes the release of the follicle from the ovary?
A. FSH✅ B. Progesterone C. Estrogen Answer: A
655)A newly assigned nurse to the Home Health Care Department was conducting pre-entry phase of home
visit. Which of the following activities should the nurse do?
A. Call physician for a referral order B. Collect information and schedule a visit.
C. Establish mutually acceptable goals for care.
D. Conduct a health history and physical assessment Answer: B
656) A nurse is planning to assess the corneal reflex on unconscious client. Which of the following is the
safest? stimulus to touch the client’s cornea.
a. Cotton buds b. Sterile glove
c. Sterile tongue depressor d. Wisp of cotton L Answers & Rationale at Answer: D
657)Nursing manager her unscheduled staff absence what done
1_self-staffing 2_on call staffing Answer: B
658)35years old female patient after taking breast biopsy she diagnosed with 3rd stage breast cancer. She
asked what is the purpose of chemotherapy‫؟‬
A-it make no need for surgery. B- destroy cancer cells and prevent it from spreading.
C- prevent recurrence of cancer. Answer: B
659)nurse conducting research by using qualitative data.Which of the following methods she used to gather
this data?
A. weekly interview or one to one interview B. surveys C. data analysis Answer: A
660)What is the function of Maslow hierarchy for nursing??
A. Set priority B. Helping establish nursing diagnosis NANDA. Answer: A
661) The patient has skull fracture. The patient is complaining racoon eye, pain,and posterior neck
fracture what is the type of skull fracture?
A. Basilar✅ B. Depressed
C. Compound D. Linear Answer: A
662)An older-adult patient has developed acute confusion. The patient has been on tranquilizers for the past
week. The patient’s vital signs are normal. What should the nurse do?
a. Consider age-related changes in body systems that affect pharmacokinetic activity.
b. Increase the dose of tranquilizer if the cause of the confusion is an infection.
c. Note when the confusion occurs and medicate before that time.
d. Restrict phone calls to prevent further confusion. Answer: A
663). A30week gestational preterm admitted to NICU 2hours ago the neonate starts to have grunting, nasal
flaring. which of the following the nurse recognize regarding signs and symptoms?
A. Neonate has RDS. B. It is normally in the first 24 hours of birth.
C. This is not significant unless become cyanosis. D. Neonate has hypoglycemia. Answer: A

After umbilical hernia / inguinal hernia repair


Avoid coughing. Avoid straining. Avoid heavy weight.
664)While a nurse is assessing an infant born 11 hours ago full term by caesarean section, she auscultated
moist lung sounds.Which of the following is the most likely interpretation?
A. Abnormal finding B. Normal finding
C. Pneumothorax D. Surfactant aspiration Answer: B
665)All the following are normally present in urine result EXCEPT:
a) Urea. b) Creatinine c) Albumin. ✅ d) Sodium. Answer: C

ِ‫يِاْلَ أرض‬
‫أ‬ ُ ‫َم‬
ُ ‫ك‬
‫ثِِف‬ َ ‫اس‬
‫ِِفي أ‬َ َّ ‫ع‬
‫ِالن‬ ُ ‫ف‬
َ ‫ماِيَ أن‬ َّ َ‫وأ‬
َ ِ‫ما‬ َ
{ 73 }

666)A patient has had open heart surgery and is in a coma. The patient’s spouse brings suit against the
physician, alleging that the patient was not informed of the possible complications from use of general
anesthesia. The suit will be filed on the following basis.
A. malfeasance. B. misfeasance.
C. nonfeasance D. negligence Answer: B
Malfeasance - - - - intentional to do harm .‫قاصد أضر المريض‬
Misfeasance - - - - - - unintentional action ‫عملت خطأ بدون قصد‬
667) Patient with paranoid schizophrenia most sign and symptoms
A⁃ Grandiose/granduer delusions ✅ B ⁃ - psychomotor Answer: A
-pyloric stenosis (metabolic alkalosis)
668)The mother brought her child to hospital then the child admitted to NICU. After leaving the mother for
him. He started crying. what stage of anxiety separation?
A. Protest, despair, denial✅ B. Protest, despair and denail
C. Denial, protest, despair D. Protest, denial, despair Answer: A
669)Nephrotic syndrome what color urine?
a. Brown✅ b. Red Answer: A
670)Nephritic syndrome urine color?
a. Bright red✅ b. Dark Brown Answer: A
671)pregnancy patches contradiction?
a. Over wright 80 ✅ b. Have diarrhea .c. With antibiotic Answer: A
Hypertension 177/80 after 180/60! Risk for? Stroke
Catheter when remove? Ask patient deep breath.
Chest tube or CVP removal Valsalva maneuver
672)Patient have coped diagnosis high priority.
A. Ineffective Breath pattern B. Gas exchange Answer: A
673) The nurse was giving drug to the patient it was tablet, the patient said that is not my medication.
Why my drug is changed in color? What should the nurse response??
A. Check the medication list B. Check the doctor order ✅
C. Say this is your medication D. Say the doctor wrote new order new drug Answer: B
674)500mg diluted in ml 250ml D5NS with infusion pump then she modified the dose to 20mg/hr. How many
ml/hr. should the nurse give??
A)5 B) 10 C) 15 D) 20 Answer: B
675)Nurse manager is doing change in her unite. What is third stage of change??
A) select strategies B) solution C) Change strategies Answer: Bbb
676)The nurse is evaluating the patient with end stage chronic. obstructive pulmonary disease (COPD). The
patient has not achieved any of the goals in the plan of care. The spouse reports. Concerns about the patient’s
mood and increased dependency. What action should the nurse take FIRST?
a. Continue the care plan for 1more month. b. Refer the patient to psychiatric services.
c. Collaborate with the patient and spouse to revise the care plan.
d. Revise the care plan based on the spouse’s input. Answer: C

ِ‫يِاْلَ أرض‬
‫أ‬ ُ ‫َم‬
ُ ‫ك‬
‫ثِِف‬ َ ‫اس‬
‫ِِفي أ‬َ َّ ‫ع‬
‫ِالن‬ ُ ‫ف‬
َ ‫ماِيَ أن‬ َّ َ‫وأ‬
َ ِ‫ما‬ َ
{ 74 }

677)A nurse is caring for a patient receiving total parenteral nutrition. (TPN). The patient reports the sudden
onset of feeling short of breath and anxious. The nurse hears crackles in bilateral lower.lobes of the lungs and
the patient’s O2 saturation is 90%on room air. The nurse must IMMEDIATELY:
a. Turn off the TPN. b. Notify the physician.
c. Asses the patient’s capillary blood glucose level d. Attempt to suction the patient’s airway.Answer: B
Tocolytics drug :(also called anti-contraction medications or labor suppressants) are medications used to
suppress premature labor.
Oxytocin drug :used to cause contraction of the uterus to start labor, increase the speed of labor, and to stop
bleeding following delivery.
678) newly RN nurse is about to remove a nasogastric tube to a client with Guillen Barre Syndrome. To
determine the proper Time of removing the tube, the nurse should do it when.
A-when the client feeling hungry B-when the client wants to remove it.
C-when the client totally conscious
D- when the client exits flatulence or gases✅ Answer: D
679)A 25yearold woman presents to the Emergency Room with problem. Her main complaint is that she
woke up that morning see anything. All medical examination was normal. However, reported that two days
ago her husband had asked for a divorce. What is the most likely disorder?
A. Pain B. Conversion
C. Somatization D. Body Dimorphic Answer: C
680) What is the updated technique for CPR According to AHA?
A. 30:2120at5cm✅ B. 15:2120at5cm
C. 15:2120at5cm D. 30:2100at4cm Answer: A
681) A patient is being prepared for discharge following hip replacement surgery. The nurse is providing him
with discharge education. Which of the following information should be taught to him as an effective pain
management principle?
A. Avoid giving pain medication prior to participating in physical therapy
B. Give a double dose of pain medication if pain is intolerable
C. Give pain medication before pain becomes severe✅
D. Delay giving pain medication as long as possible Answer: C
682) Skeletal traction urinary catheter were discontinued for a patient who was immobilized in traction for
sex weeks. The patient developed a problem with urinary incontinence. Which of the following is the most
appropriate intervention?
A. Scheduled toileting B. Bladder retraining✅
C. Promoted voiding D. Behavioral training Answer: B
What is the process that sperm starting penetrates the ovum?? - Fusion process
Capacitation process: is the penultimate step in the maturation of mammalian spermatozoa and is required to
render them competent to fertilize an oocyte. This step is a biochemical event the sperm move normally and look
mature prior to Capacitation.

ِ‫يِاْلَ أرض‬
‫أ‬ ُ ‫َم‬
ُ ‫ك‬
‫ثِِف‬ َ ‫اس‬
‫ِِفي أ‬َ َّ ‫ع‬
‫ِالن‬ ُ ‫ف‬
َ ‫ماِيَ أن‬ َّ َ‫وأ‬
َ ِ‫ما‬ َ
{ 75 }

683) mother of a patient who is on antipsychotic drug asked a doctor about some more detail about the drug.
The doctor told the mother that her son is taking an atypical antipsychotic drug. Which of the following is an
examples of this type of drug?
A. Thioridazine B. Clozapine✅
C. Chlorpromazine D. Haloperidol Answer: B
684). The nurse is assisting a doctor with the removal of a central venous catheter. To prevent complications,
the patient should be instructed to
A. Turn his head to the left side and hyperextend the neck while looking up.
B. Take slow, deep breaths as the catheter is advanced.
C. Perform the Valsalva maneuver as the catheter is pulled.
D. Turn his head to the right while grasping the siderails. Answer: C

Noninvasive test of Hirsch rung disease anorectal mammography.


685)Which of the following of maternal age and other factors that affect negatively on labor and cause severe
complications?
A. Female 50and above B. Female 16 or less
C. Hypertension D. Anemia Answer: A
686)The hospital director attended to ward for evaluation of staff regarding vital signs follow up during blood
transfusion. What is the most appropriate action for monitoring vital signs during blood transfusion depends
on?
A. According to physician order B. According to nursing supervisor instructions
C. According to hospital policy
D. According to blood bank staff instruction Answer: C
687)female patient is diagnosed with deep-vein thrombosis. Which nursing diagnosis should receive the
highest priority at this time?
A. Impaired gas exchange related to increased blood flow.
B. Fluid volume excess related to peripheral vascular disease.
C. Risk for injury related to edema.
D. Altered peripheral tissue perfusion related to venous congestion. Answer: D
688)A nurse is collecting a urine of a 4-year-old child with nephrotic syndrome. Which of following
observation about the color of the child's urine the nurse expected to? will chart.
A. Bright red B. Amber
C. Dark, frothy D. Orang Answer: C
689)A nurse is collecting a urine of a 4-year-old child with nephritic syndrome. Which of following
observation about the color of the child's urine the nurse expected to will chart.
A. Bright red B. Amber
C. Dark, frothy D. Orange Answer: A
690) Patient diagnosed with UTI. He has frequent urination. The urine analysis is rescheduled to the next
day. What is the best action for the patient?
A. Increase fluid intake✅ B. Decrease fluid intake
C. Normal fluid intake D. Avoid fluid intake Answer: A
691)A female client who received general anesthesia returns from surgery. Postoperatively, which nursing
diagnosis takes highest priority for this client?
A. Acute pain R/T surgery
B. Deficient fluid volume R/T blood and fluid loss from surgery
C. Impaired physical mobility R/T surgery D. Risk for aspiration R/T anesthesia Answer: D

ِ‫يِاْلَ أرض‬
‫أ‬ ُ ‫َم‬
ُ ‫ك‬
‫ثِِف‬ َ ‫اس‬
‫ِِفي أ‬َ َّ ‫ع‬
‫ِالن‬ ُ ‫ف‬
َ ‫ماِيَ أن‬ َّ َ‫وأ‬
َ ِ‫ما‬ َ
{ 76 }

692)What is the Contraindication of baby vaccine?


A. Antiemetic B. Antibiotics C. Steroid ✅ Answer: C
693)A 70-year-old man presents to the clinic with difficulty sleeping at night. He has not had a good night's
rest for several months and feels exhausted. He needs to place three pillows behind his back in order to sleep.
Examination of the lungs reveals crackles and wheel Auscultation of the heart confirms an S gallop. Which of
the following is the most likely underlying health problem?
A. Asthma B. Pulmonary stenosis
C. Right-sided heart failure D. Left-sided heart failure Answer: D
694)nurse makes a clinical judgment that an African American male patient in a stressful job is more
vulnerable to developing hypertension than White male patients in the same or similar situation. The nurse
has formulated what type of nursing diagnosis?
A. Actual B. Risk
C. Possible D. Wellness Answer: B
695) What is the difference between measles and German measles?
Koplik spot and high fever ✅
696) What is signs and symptoms of Esophagial atrasia?
A. Crying B. Excessive frothy salivation from nose and mouth ✅✅
C. Absent respiratory sound Answer: B

Spina Bifida Occulta: A small hairy patch on back, or a soft skin colored mass called a lipoma,
That is the only external signs for diagnosis of spina bifida occult.
697)According to Maslow's hierarchy of needs, which nursing diagnosis has the lowest priority for a client
admitted to the intensive care unit with a diagnosis of congestive heart failure?
a) Impaired urinary elimination b) Ineffective airway clearance
c) Ineffective coping d) Risk for body image disturbance Answer: D
698)Which of the following nursing diagnoses has the highest priority when caring for an older adult client
with Alzheimer's disease?
a) Impaired physical mobility b) Impaired memory
c) Self-care deficit d) Risk for injury Answer: D
699)A nurse is reviewing the plan of care for a client with a breathing problem. Which of the following would
the nurse most likely expect to identify as a relevant nursing diagnostic statement for this client?
a) Altered airway b) Impaired respiration
c) Impaired breathing rate d) Ineffective airway clearance Answer: D\
700)planning the care for a client who has pneumonia, the nurse collects data and develops nursing diagnoses.
Which of the following is an example of a properly developed nursing diagnosis?
a) Ineffective health maintenance as evidenced by unhealthy habits
b) Ineffective airway clearance as evidenced by inability to clear secretions
c) Ineffective therapeutic regimen management due to smoking
d) Ineffective breathing pattern related to pneumonia Answer: B
701)Using Maslow's hierarchy of needs, rank the following nursing diagnoses in order of importance,
beginning with the highest-priority diagnosis.
1)Anxiety 2)Risk for infection
3)Disturbed body image 4)Sleep deprivation Answer: Arrangement 4213

ِ‫يِاْلَ أرض‬
‫أ‬ ُ ‫َم‬
ُ ‫ك‬
‫ثِِف‬ َ ‫اس‬
‫ِِفي أ‬َ َّ ‫ع‬
‫ِالن‬ ُ ‫ف‬
َ ‫ماِيَ أن‬ َّ َ‫وأ‬
َ ِ‫ما‬ َ
{ 77 }

702)patient with a stroke is paralyzed on the left side of the body and has developed a pressure ulcer on the
left hip. The best nursing diagnoses for this patient is:
a) Impaired physical mobility related to left-sided paralysis
b) Risk for impaired tissue integrity related to left-sided weakness
c) Impaired skin integrity related to altered circulation and pressure
d) Ineffective tissue perfusion related to inability to move independently Answer: C

703)A nurse is performing an initial assessment for a client. Which of the following would be considered
subjective information received during the assessment?
A. The client rates pain at a level of 6 on the numeric rating scale
B. The client has a pinpoint rash on the face and trunk.
C. The client's blood pressure increases when the provider enters the room.
D. The client weighs 186 pounds Answer: A
704) You are preparing the nursing care plan for a middle-aged patient admitted to the intensive care unit for
an acute myocardial infarction (heart attack). His symptoms include tachycardia, palpitations, anxiety,
jugular vein distention, and fatigue. Which of the following nursing diagnoses is most appropriate?
1)Decreased Cardiac Output 2)Impaired Tissue Perfusion
3)Impaired Cardiac Contractility 4)Impaired Activity Tolerance Answer: A
705)The nurse performs an assessment of a 23-year-old man who believes that. people are spying on him.
During the interview, he keeps his eyes to the floor. and answers questions awkwardly. He has never had an
intimate relationship. and avoids contact with his family members. He has never been employed and tells the
nurse that he is not looking for a job. The nurse considers Erickson’s. theory of psychosocial development.
Which stage is this patient most likely experiencing?
a- Autonomy versus shame and doubt b- Initiative versus guilt
c- Trust versus mistrust d- Identity versus confusion Answer: C\
706) A nurse is assessing the uterus of a G5P4 patient immediately after delivery. The nurse notes the fundus
is not contracted. Which of the following is the most appropriate immediate action should be taken?
A. Massage the fundus B. Assess the bladder
C. Elevated the mother's legs D. Encourage the mother to void Answer: A

707) *What is Early sign of portal hypertension?


A. Bradycardia B. Hypotension✅ C. Flat jugular vein Answer: B
708) A 39-year-old female is prepared for endoscopic craniotomy for. removal the accumulated fluid in the
dura matter and a biopsy. She is prepared preoperatively. What essential knowledge about the? procedure
that should be explained to her.
A. Small skull bone is removed to biopsy brain tissue.
B. Needle is guided to remove abnormal brain tissue.
C. Specialized tools are used to remove bones section.
D. Lighted tube camera is used to look inside the skull. Answer: D
709) *Isolation for RSV>>> Contact isolation
710) Pregnant mother come to ER complain of painless vaginal discharge ?
A. placenta previa,:* Pregnant mother 36 weeks of gestational vitally stable what nursing should instruct the pt
? A. wait untill 40 weeks
What is the Role of head nurse?
A. Ambulate the pt. B. Monitor ecg for the patient
C. Recruiting the nurses D. Allocate the nurse to the patient ✅ Answer: D

ِ‫يِاْلَ أرض‬
‫أ‬ ُ ‫َم‬
ُ ‫ك‬
‫ثِِف‬ َ ‫اس‬
‫ِِفي أ‬َ َّ ‫ع‬
‫ِالن‬ ُ ‫ف‬
َ ‫ماِيَ أن‬ َّ َ‫وأ‬
َ ِ‫ما‬ َ
{ 78 }

711)Which of the following mothers is High risk for laceration during labor?
A. 1 para mother for 12-hour delivery
B. 3 para mother for 9 hours' delivery
C. 2 para mother for 9 hours' delivery
D. 2 para mother for 6 hours' delivery Answer: B
712)Incubation period of infection is:
A) Person is most infectious and nonspecific sign and symptom.
B (Organism growing and multiplying.
C) Recovery from infection.
D) Presence of specific sign and symptom. Answer: A
713) 25 weeks-pregnant, primary gravid woman is on her first antenatal visit, completing physical
examination and history, the midwife found out her husband has sickle cell anemia minor. What should be the
most appropriate plan of care?
A. Genetic counselling ✅ B. Identify severity of disease
C. Discuss the chances of transferring
D. Amniocentesis to identify genetic abnormality Answer: A

714) Nurses work together in team project... suddenly there conflict between two nurses ....what should nurse
supery do?
A. Make nurses work independently B. Make nurses continue work together
C. Make nurse confront team members
D. Inform nurse supervisor to resolve issues ✅ Answer: D

715)Patient exposure to second degree burn what electrolytes imbalance?


A. Sodium depletion due to shift water✅✅
B. Increase potassium due to cell destruction Answer: A
716) Patient recieves (Psychotrophic medication ). The Patient develop agranulocytosis.
This side effects to which medication cause ?
A. Typical antipsychotic B. Atypical antipsychotic ✅
C. Sertonin reuptake inhibitor
D. Noradrenaline reuptake inhibitor Answer: B

717) Woman came to ER with right abdomenal pain ...vaginal bleeding.. AFter Ultrasound. THEY Detect she
is pregnant in 11، weeks .... there is adenxal tenderness ...this most likely ?
A. Appendecitis B. Ectopic pregnancy ✅ C. Threatened abortion Answer: B
718) How to differentiate between hypovolemic shock and postpartum hemorrhage?
A. Hemoglobin B. Hematocrit
C. Cold clammy skin D. Increase Pulse✅ Answer: D
719) During providing care to the patient. There was emergency situation in the ward. She already given
medication to patient but still no documentation yet. What should the nurse do to prevent any error result
from delayed documentation?
A. Document at the end of the shift B. Write note that drug done and the dose✅
C. Don't go to emergency situation until you finish documentation Answer: B

ِ‫يِاْلَ أرض‬
‫أ‬ ُ ‫َم‬
ُ ‫ك‬
‫ثِِف‬ َ ‫اس‬
‫ِِفي أ‬َ َّ ‫ع‬
‫ِالن‬ ُ ‫ف‬
َ ‫ماِيَ أن‬ َّ َ‫وأ‬
َ ِ‫ما‬ َ
{ 79 }

720) During operation procedure for patient with inguinal hernia. The doctor explored another site for hernia
not included in the informed consent. What is the most appropriate action?
A. Do the extra operation then get consent later after patient aware
B. Wait for procedure until patient signed another consent
C. Call for client's medical power attorney to provide additional informed consent For additional procedure✅
D. Additional informed consent and document in patient care Answer: C
721) What is hormons for biological difference between male and female?
A. Estrogen and HCG B. Testosterone and progestron✅✅ Answer: B
722)Child with Pyloric stenosis. What is the expected signs and symptoms postoperative?
A. Abdominal pain B. Watery stool
C. Vomiting ✅ D. Urinary Answer: C
723)-A 40 years old complains of severe pain, scheduled for surgery of abdomen, medical history he smokes
one packed of cigarettes per day. Which of the following complications he might develop?
A- atelectasis ✅ B- DVT C- constipation Answer: A
724)-A nurses is providing teaching session for pre-operative patient before appendectomy about how to use
spirometer, which of the following nursing process?
A- assessment B- evaluation
C- implementation ✅ D- diagnoses Answer: C
725)A parent asks the nurse how dissociative disorder can be treated?
A- psychotherapy ✅ B- psychology
C- electro convulsive therapy (ECT) Answer: A
726)Patient post, the goal for him to be mobile at 9 but he mobilized at 12, what should the nurse do?
A- delay discharge process B- discharge and write goal met✅
C- collaboration with therapist to review goal and plan. Answer: B
727) What is the Ministry of health in saudi arabia precutions regarding H5N1?
A. Hand washing before enter patient room ✅ B. Be cautious with patient things
C. Wear gloves and gown D. Don't follow precautions Answer: A

728) A 17-year-old mother presented to the primary health center ten after delivery. She is suffering from
fatigue, anemia, fever and vaginal discharge (see lab results)
Blood pressure 80/50 mmHg -Heart rate 112 /min
Respiratory rate 35 /min - Temperature 39.6 C
Test Result Normal Values - RBC 4 4.7-6.1 × 1012 /L (male) 4.2-5.4 × 1012 /L (female)
Hb 90 130-170 g/L 120-160 g/L (female) HCT 0.29 0.42-0.52 (male) 0.37-0.48 (female)
WBC 12.8 4.5-10.5 × 109/L . Which of the following is the best diagnosis of health problem in this case?
A. Severe urinary track infection B. Vesical-vaginal fistula
C. Puerperal sepsis ✅✅ D. Post-partum hemorrhage Answer: C
729) Nurse was documenting, and she documented wrongly (Error), what she should do?
A- Review policy of how to correct wrong documentation.
B- A- use liquid and Waite until dry then write the correct one above it.
C- use liquid and write the correct one beside.
D- do line then write the correct beside ✅ Answer: D

ِ‫يِاْلَ أرض‬
‫أ‬ ُ ‫َم‬
ُ ‫ك‬
‫ثِِف‬ َ ‫اس‬
‫ِِفي أ‬َ َّ ‫ع‬
‫ِالن‬ ُ ‫ف‬
َ ‫ماِيَ أن‬ َّ َ‫وأ‬
َ ِ‫ما‬ َ
{ 80 }

730) A nurse wants to delegate a post op patient from major surgery to assistant nurse, what task she will
delegate?
A- ambulate the patient ✅✅ B- obtain vital signs.
C- check patency of NGT Answer: A
731(A nurse is providing session regarding lung cancer, which statement indicates teaching was not effective?
A- I will eat fruit and vegetables B- if I have COPD does not mean I will have lung cancer.
C- no need to worry because I do not smoke, my husband does. ✔️ Answer: C
732) A nurse is obtaining vital signs for patient, which of the following she will document
Bp; 138/78 -HR: 108 -RR: 18 -T: 36.5
A- tachycardia✅ B- arrhythmia
C- tachypnea D- hypertension Answer: A
733)which of the following statements regarding SLE nurse should provide to group of people?
A- hypertension must be anticipated ✅ B- hypertension is not common with SLE.
C- the symptom will exacerbate more in winter and spring. Answer: A

734) position of patient with endoscopy? Supine


735) first thing for nurse after needle stick injury will do.
A- squeeze her finger. B- write incident report✅ Answer: B
736)When communicating with a client who speaks a different language, which best practice should the nurse
implement?
A. Speak loudly and slowly. B. Arrange for an interpreter to translate.
C. Speak to the client and family together.
D. Stand close to the client and speak loudly. Answer: B
737)Doctor order new antibiotics first thing to do before administering medication.
1- check the order in the system 2- do CBC
3- Obtain blood cultures ✅ 4- administration first dose when standard medication time. Answer: C
738)Infant with breast feeding associate jaundice came to clinic. The Infant has yellowish sclera discoloration.
What should the nurse do?
A. Encourage frequent breast feeding✅ B. Admitted baby and do phototherapy.
C. Withhold breast feeding. Answer: A
739)Patient attached with mechanical ventilator. The doctor assesses the patient to weaning off ventilator.
What is weaning mood must be adjusted?
A) assist control mood B) CPAP MODE✅ Answer: B
740)Multiple myeloma is a neoplastic proliferation of Which of the following??
a- lymphocytes b- Granulocytes
c- Plasma cells✅ d- Monocytes Answer: C
741)The Diagnosis is the reduction in the amount of circulating. hemoglobin, red blood cells or both:
a- polycythemia b- Anemia✅
c- Hemophilia d- Leucopenia Answer: B
742)A nurse is caring for a client with head injury and monitoring him for decerebrate posturing.
Which of the following is the characteristic of the posture?
A. Flexion of the extremities after stimulus B. Extension of the extremities after a stimulus✅
C. Upper extremity flexion with lower extremity flexion
D. Upper extremity flexion with lower extremity extension Answer: B

ِ‫يِاْلَ أرض‬
‫أ‬ ُ ‫َم‬
ُ ‫ك‬
‫ثِِف‬ َ ‫اس‬
‫ِِفي أ‬َ َّ ‫ع‬
‫ِالن‬ ُ ‫ف‬
َ ‫ماِيَ أن‬ َّ َ‫وأ‬
َ ِ‫ما‬ َ
{ 81 }

743) A 62-year-old male patient, admitted in the surgical Ward is scheduled for the surgical removal of polyps
from his descending colon under general anesthesia. he is experiencing fatigue, abdominal pain and blood
streaked. stools for a couple of months. he is worried whether the bleeding in his stools is going to stop after
surgery. What is most appropriate response by the nurse for the patient concern?
A. Surgery often relieves the symptoms.
B. Let us have a detail discussion with your physician✅
C. Your condition may or may not resolve, it depends.
D. In fact surgery is the only treatment for the problem. Answer: B
744) Cardiac patient smoking. He has blue lower extremity, pale skin and very exhausted. the nurse observe
that lower limb without hair. No hair growth in lower limb. What is the appropriate cause of hair loss in his
leg ?
A. Hermonal disturbance B. Impaired tissue perfusion✅ Answer: B
745)side effects of bCG vaccination
A-cold and small scar✅ B-diarrhea C- rash for three days Answer: A
746) mother has infant 6 months came to the clinic and afraid that her baby May will have meningitis as his
brother already have.
What should the nurse tell the mother?
1-hib vaccine can decrease the meningitis ✅✅
2-there are now vaccinations for all meningitis type
3- tell the mother the disease is will not come Answer: A
747) The patient with burn in the face and hand what will be site to assess the pulse‫؟‬
A.radial B.apical C.femoral✅ Answer: C
748) The patient see black dots flying what does it mean?
A. Glaucoma B. Retina deatachment ✅ Answer: B
749)Which influenza virus strain currently presents the greatest risk to human health?
A. H1N1 B. H2N2
C. H3N2 D. H5N1✅ Answer: D
750)Signs and symptoms of early fluid volume deficit, except.
A. Decreased urine output B. Decreased pulse rate ✅
C. Concentrated urine D. Decreased skin turgor Answer: B
751)A 45-year-old man is admitted to the neurosurgery ward for the surgical elevation of depressed skull
fracture. He has episodic severe headache with seizure and is unable to concentrate. Which of the following
initial nursing problem needs more attention?
A. Disturbed coping and anger spells
B. Risk of injury to seizure
C. Disturbed communication and irritability
D. Pain management and comfort measure Answer: B
752)A patient is seen in the emergency department with complaints of angina. Nitroglycerin (Nitro-stat) is
ordered by the physician. This medication is. to be administration via which of the following routs?
A. Intradermal B. Buccal C. Parental Answer: B
753) How can we know that this child has intussusception??
A. Baby crying &vomiting B. Baby pull knee to chest ✅ Answer: B

ِ‫يِاْلَ أرض‬
‫أ‬ ُ ‫َم‬
ُ ‫ك‬
‫ثِِف‬ َ ‫اس‬
‫ِِفي أ‬َ َّ ‫ع‬
‫ِالن‬ ُ ‫ف‬
َ ‫ماِيَ أن‬ َّ َ‫وأ‬
َ ِ‫ما‬ َ
{ 82 }

754) The patient had adrenal insufficiency & take corticosteroid she will have?
A. Pigment skin & dry ✅ B. Pale skin & dry
C. Pink skin & healthy Answer: A
755)Before sending a client for a CT with contrast dye, what the nurse's most important action?
A. Teach about the need for post-procedure hydration. B. Verify that the informed consent is complete.
C. Place the side rails of the bed up before transport.
D. Check the client's health record for allergies Answer: D
756)A midwife is conducting a health a health education session to the prim gravid mothers. The session is
about. antepartum care to ensure the health of the mother their babies. Which of the following statement
made by? mother suggest their appropriate understandings of anti-partum periods.
A. Beginning of labor till the baby birth B. Diagnosis of pregnancy till the baby birth
C. Last three months of pregnancy D. Forty days after the child is born. Answer: C
757) A patient has a hip fracture complaining of severe pain. He nurses finds no analgesic was ordered. She
calls the orthopedic doctor by telephone and received a medication order.
What is the most appropriate nursing action to ensure correct medication order taken?
A. Write down the order and get pharmacy o supply.
B. Write down the order and get medication immediately with another registered nurse.
C. Get another registered to witness, nurse write down the order and read back to the doctor
D. Write down the medication and after serving get the doctor to witness. Answer: C
758) Following lumbar surgery a patient has a 4 millimeter (mm) surgical incision. The incision is clean, and
the edges are well appropriate. This type of tissue healing is classified as which of the following?
a. Primary intention b. Secondary intention
c. Tertiary intention d. Superficial epidermal Answer: A
759) A surgeon instructs a nurse to serve as a witness to an elderly patient’s informed consent for surgery.
During the explanations to the patient, it becomes clear that the patient is confused and does not. understand
the procedure, but reluctantly sign the consent form. The nurse should:
A. Sign the form as a witness, making a nation that the patient did not. appear to understand.
B. Not sign the form as a witness and notify the nurse supervisor.
C. Not sign the form and answer the patient’s questions after the surgeon. leaves he room.
D. Sign the form and tell surgeon that the patient does not understand the procedure Answer: B
760) Which week lung (surfactant) will be mature?
A. 20 week B. 24 week C. 16 week D . 28 week✅ Answer: D
761. What is the main organism that cause meningitis for a child?
A. Meningococcal B. Staphylococcus
C. H. Influenza D. Streptococcal Pneumonia Answer: D

ِ‫يِاْلَ أرض‬
‫أ‬ ُ ‫َم‬
ُ ‫ك‬
‫ثِِف‬ َ ‫اس‬
‫ِِفي أ‬َ َّ ‫ع‬
‫ِالن‬ ُ ‫ف‬
َ ‫ماِيَ أن‬ َّ َ‫وأ‬
َ ِ‫ما‬ َ
{ 83 }

762)A13-weeks-pregnant, multigravida women is anxious, and apprehensive she has five children and is not.
willing to continue with this pregnancy she is requesting the midwife to about the fetus, she is underweight,
malnutrition and is over worked, BMI 17kg/m2, what intervention is desired immediately?
A. Admission and intravenous line management for induction.
B. Family planning and birth control measures
C. Dietary management and supplements.
D. Support, reassurance and counselling. ✅ Answer: D
763) When does mature surfactant begin to be Produced?
A. 21 week B. 24 week✅
C. 28 week D. 16 week Answer: B
764) 1 month-old infant is admitted to the surgical unit with hypertrophic pyloric stenosis and scheduled for
the surgery. Which of the following is the findings of abdominal examination?
A. palpable olive-like mass in the left side
B. palpable olive-like mass in the right side ✅
C. Palpable olive-like mass moved from left to right.
D. Palpable olive-like mass moved from right to left. Answer: B

765) PYLORIC stenosis peristalsis movement?


A. From right to left. B. From left to right✅ Answer: B

766) Surgery for pyloric stenosis movement?


A. Pylorotomy
B. Pylorectomy
C. Pylorostomy
D. Pyloromyotomy✅ Answer: D

767)A 5-week-old newborn was admitted to pediatric Ward with pyloric stenosis, the newborn has weight loss,
and projectile vomiting during feeding. They scheduled surgical repair of pyloric stenosis Which of the
following. postoperative intervention for this
A. IV fluid infant is retaining adequate amount by mouth ✅
B. Administration of proper analgesia until infant discomfort resolve
C. Start feeding immediately after postoperative.
D. Vomiting is uncommon in the first24-48 hrs. Answer: A
768) Which of the following can be considered as a major development in nursing home visits in terms of
information technology?
A. Medication dispensing B. Telehealth
C. Patient monitoring
D. Prevention of epidemiological Answer: B
769) Best management for fracture..??
A. Immbolization ✅ B. Moblization
C. Control pain Answer: A
770) Nurse manger want to make organizational structure which allow communication to flow in all lines of
structure and involving workers in decision making?
A. Centralized B. Decentralized ✔️
C. Infomal Answer: B

ِ‫يِاْلَ أرض‬
‫أ‬ ُ ‫َم‬
ُ ‫ك‬
‫ثِِف‬ َ ‫اس‬
‫ِِفي أ‬َ َّ ‫ع‬
‫ِالن‬ ُ ‫ف‬
َ ‫ماِيَ أن‬ َّ َ‫وأ‬
َ ِ‫ما‬ َ
{ 84 }

771) At is the recommended weight gain during pregnancy of a woman with MI of < 18.5?
A. 12.5 -18 Kg ✅ B. 11.5 -16 Kg
C. 7 -11.5 Kg D. 5- 9 Kg Answer: A
772)Normal weight gain during pregnancy.?
A. 12.5 -18 Kg B. 11.5 -16 Kg ✅✅
C. 7 -11.5 Kg D. 5- 9 Kg Answer: B
773) 21 years old patient exhibit nervousness .. isolated from all family and friends ... difficult falling sleep ...
develop this after having crash train accident and have been trapped for aday in train but wihout injury?
A. Post traumatic stress disorder Answer: A

774)The nurse educator presents an in-service training session on case management to nurses on the clinical
unit. During the presentation, the nurse educator clarifies that what is a characteristic. of case management?
A. Requires that 1 nurse take care of 1 client.
B. Promotes appropriate use of hospital personnel.
C. Requires a case manager who plans the care for all clients.
D. Uses a team approach, but 1 nurse supervises all other employees. Answer: B
775)a child with tracheoesophageal fistula is scheduled for an operation. what should the nurse do
preoperatively?
A) Insert NGT B) Suction periodically ✅✅
C) Prepare tracheostomy set if necessary Answer: B
776) The doctor touches the bed by the sterile gloves the nurse should:
A. Ask him to change the gloves and to give him new one ✅✅
B. Let him to complete the Central line procedure. Answer: A

777) A child with asthma has an order for albuterol, before administration of the medication the nurse MUST.
a- Pre-oxygenate the patient b- Assess the patient's heart rate ✔️✔️
c- Obtain venous Access d- Feed the patient a snack Answer: B
778)Steal syndrome in hemodialysis symptoms?
A. Pain and hotness of skin
B. Pain and coldness of skin ✔️✔️ Answer: B

779)The patient scheduled for operation. The doctor informed the patient about expected complication and
the outcomes May be not acceptable. What is the non-preferable ethical principles?
A. Malpractice B. Negligence
C. Ethical dilemma ✅ D. Molars concussi Answer: C

780)Initially and important physical examination for Which patient?


A. Pneumonia B. Pulsating in abdomen ✅ Answer: B\
781)Classification system forth category???
A. Close monitoring complete assistance most activities ✅
B. Recovering serious illness and need to some assistance. Answer: A
782)Which of the following is consider sputum expectorant?
A. Emphysema B. Asthma
C. Bronchitis ✅ Answer: C

ِ‫يِاْلَ أرض‬
‫أ‬ ُ ‫َم‬
ُ ‫ك‬
‫ثِِف‬ َ ‫اس‬
‫ِِفي أ‬َ َّ ‫ع‬
‫ِالن‬ ُ ‫ف‬
َ ‫ماِيَ أن‬ َّ َ‫وأ‬
َ ِ‫ما‬ َ
{ 85 }

783)) preeclampsia suspected patient is presented to the ER. what level of triage?
A. Emergency✅ B. Urgent Answer: A
784)postpartum priority assessment episiotomy for
A. Colour ✅ B. Edema
C. Discharge D. Appearance Answer: A
According to REEDA Redness Edema ecchymosis discharge Approximation
785)Complication DVT deep vein thrombosis:
A. pulmonary embolism ✅ B. Thrombophlebitis Answer: A
786) Patient with burn >>>> Reverse isolation
787)Position that increases heart Murmur.
A. High Fowler B. Sideline
C. Trendelenburg✅ Answer: C
788) Definition of effacement >> thining of cervix
789) The Height of enema is ?
A. 50 cm ✅ B. 150 cm C. Blow red edge Answer: A
790)A 1-year-old girl admitted to podiatric medical unit significant weight loss, diminished mid-arm.
circumference diarrhea, and red hair. Which of the following type of malnutrition do the nurse suspect?
A. Marasmus B . Spitting up
C. Kwashiorkor ✅ D. Rickets Answer: C
Stick like muscle is Marasmus.
791)A 12- year- old boy was brought to the Emergency respiratory arrest due to drowning. Cardiac
resuscitation.what is the major complication that might happen if treated after drowning quickly?
A. Sepsis B. Alkalosis
C. Acidosis D. Hypothermia Answer: C
792)A 40-year-old man. smoker, presents to the clinic. On examination, the toes are cold to the touch.
Extremities are pale to blue. The pedal pulse examination of the fingers shows small ulceration the skin. Blood
glucose testing is normal and then history of diabetes. On admission Blood pressure 140/90 mmHg Heart rate
86 /min Respiratory rate 22 /min Oxygen saturation 98% room air Which of the following would be most
effective?
A. Antibiotic administration B. Reduced fat intake
C. Smoking cessation D. Regular exercise Answer: C
793)A postpartum patient was in labor for 30 hours and had ruptured membranes for 24 hours. For which of
the following would the nurse be alert?
A. Endometritis B. Endometriosis
C. Salpingitis D. Pelvic thrombophlebitis Answer: A

794)schizophrenia patient started shouting loudly in the ward to anyone who speaks to him. what should the
nurse do?
A) administer tranquilizer IM B) call the security
C) isolate the patient
D) speak quietly to the patient and be cautious Answer: D

ِ‫يِاْلَ أرض‬
‫أ‬ ُ ‫َم‬
ُ ‫ك‬
‫ثِِف‬ َ ‫اس‬
‫ِِفي أ‬َ َّ ‫ع‬
‫ِالن‬ ُ ‫ف‬
َ ‫ماِيَ أن‬ َّ َ‫وأ‬
َ ِ‫ما‬ َ
{ 86 }

795)patient with Alzheimer's disease has been hospitalized after sustaining a fall and fractured hip. The
patient attempts to get out of bed and has fallen twice since admission. What is the most likely nursing action?
A. Continue the care plan as written.
B. Revise the goals and interventions✅
C. Add restraints to the interventions.
D. Add a new nursing diagnosis of noncompliance. Answer: B
796)Patient with (HF) he is on Digoxin, when the nurse checks vital signs, she noticed pulse was 110. what is
the nurse intervention?
A) hold digoxin medication B) inform doctor
C) recheck pulse D) give medication ✅ Answer: D
797)When examining the fetal monitor strip after the rupture of the
membranes in a laboring client. the nurse notes variable decelerations in the fetal heart rate. The nurse should:
A. Stop the oxytocin infusion. B. Change the client’s position✅
C. Prepare for immediate delivery. D. Take the client’s blood pressure. Answer: B
798)When providing care for and collecting evidence from a victim of sexual assault, which initial step should
the nurse focus on?
A- Obtaining consent ✅ B- Collecting specimens
C- Assessing the client's emotional state. Answer: A\
799).: Offer ....... medical equipment to encourage the child to expression of his feelings about new
hospitalization experience and illness.
A. Imitation B. Toys✅ C. Material D. Rules Answer: B
800)The wrong management for marasmus may lead.
A. CNS complication B. Kwashiorkor
C. Rickets D. Diarrhea✅ Answer: D
801)the child fears of being alone and some machine sound during hospitalization at age.
A.5 months B.8 months
C.9 months D.7years✅ Answer: D
802)The immunity system of the child reaches normal full mature level, at age.
A. 10 years B. 9 years
C. 12 months✅ D. 15 years Answer: C
803)A student taking the Saudi Commission exam in Dublin for the first time was optimistic and happy
accompanied by his friend, a nurse specialist, and after the exam, his mood was sour, anxious and frustrated.
Which of the following is the first intervention that his friend makes?
1 let him cry✅✅ 2 Hold him and lie down on his back.
3 share a hot drink. 4 drinks a cold beer Answer: A
804) the doctor writes a new order (PRN RESTRAIN) for an aggressive patient, how can doctor know the
nurse understand the order:
A. Restrain patient once if need. B. Call the doctor if patient be aggressive to restrain.
C. Do not follow the order and call supervisor.
D. Isolate the patient and not restrain. Answer: A

ِ‫يِاْلَ أرض‬
‫أ‬ ُ ‫َم‬
ُ ‫ك‬
‫ثِِف‬ َ ‫اس‬
‫ِِفي أ‬َ َّ ‫ع‬
‫ِالن‬ ُ ‫ف‬
َ ‫ماِيَ أن‬ َّ َ‫وأ‬
َ ِ‫ما‬ َ
{ 87 }

805)Disadvantages lntra Uterine Contrastive (IUD) A. Expensive B. Increase Bleeding✅✅ Answer: B


806) A 60-year-old patient was admitted with hepatic coma in the intensive care unit. The physician has
ordered. protein restriction diet for the patient. Which of the following substances is most likely causes
harmful effects when? the patient increases protein intake.
A. urea B. creatinine
C. ammonia D. amino acid Answer: C
807)A patient complains of severe pain which he stated to be 9/10, the physician ordered morphine 50 mg IV
every 4 hours, the last dose was given 2 hours ago, what is the best action his caring nurse would take:
A. Give another dose of morphine. B. Inform the doctor to change the order.
C. Distract the patient by TV, radio or games for 2 hours.
D. Ignore the patient completely. Answer: B
808)The infant 1 month with colic. What should nurse do??
A. Inform mother to increase milk product.
B. Put infant in prone position✅✅ Answer: B
809)Spina pifida risk for Which of the following complication??
A. Infection ✅✅ B.Fever C. Bleeding

Which of the following Noninvasive test used to assess fetus??


A. Contraction stress B.Non stress test✅✅ Answer: B
Aspirin should be stopped at least 7 to 10 days before surgery.
Heparin should be held 6 hours before surgery.
Warfarin should be stopped at least 2 to 3 days before operation.
810)The most risk of death??
A.hemorrhage ✅ B.pulmonary embolism
C-cervical cancer D.Cardiac disease Answer: A
VF - - - - - - Life Threatening
Stress incontinence - - - - - Kegel exercise
811)Which of the following we consider Congenital diaphragmatic hernia is related to any conditions or under
cases??
A-Sepsis B-Pulmonary
C-Heart D-Alkalosis Answer: B
812)a nurse is putting together a presentation Meningitis. on Which of the following has not been linked to
Meningitis in?
A-S. Pneumonia B-H. Influenza
C-n. Meningitis D-C. difficile Answer: D

ِ‫يِاْلَ أرض‬
‫أ‬ ُ ‫َم‬
ُ ‫ك‬
‫ثِِف‬ َ ‫اس‬
‫ِِفي أ‬َ َّ ‫ع‬
‫ِالن‬ ُ ‫ف‬
َ ‫ماِيَ أن‬ َّ َ‫وأ‬
َ ِ‫ما‬ َ
{ 88 }

813)A patient who is in isolation needs a temperature taken several times a day. Where is the appropriate
place for the thermometer to be kept?
A. At the nurses’ station. B. On the isolation cart outside the patient’s room.
C. In the dirty utility room. D. In the patient’s room Answer: D
814) The nurse is caring for an adult patient who is admitted with. Chest pain that started four hours ago.
Which test will be? most specific in identifying acute heart damage.
A. CKP B. Troponin level C. CK-MB D. Cholesterol level Answer: B
815)The nurse is caring for a patient who is admitted with chest pain and diagnosed with MI. Which test is
assisting in diagnosis?
A. CK-MR B. CK-MH
C. CK-MB D. CK-HM Answer: C

816)The purpose of the Adam's forward bend test is detecting structural or functional
scoliosis. This test is most often used during school screening for scoliosis.
‫تيجي باالختبار‬ ‫� �أشهر تعريفات‬
817) Evaluation: -a systematic process by which the worth or value of something is judged and a basis for those
judgments is defined. -a continuous feedback process whereby information is used to guide and direct decisions. -
requires measurement.

818)Surveillance : Systematic continuous observation of populations, and collection and analysis of data from
many varied sources. FOR: Rapid detection and timely, appropriate response to important health events. Social
surveillance for public health in community. Health surveillance for hospitals and health facilities.

819) Incubation period: Person is most infectious and nonspecific signs and symptoms.
820) Hirschsprung disease: Absence of ganglion cells of intestine. OR is a congenital condition
characterized by partial or complete colonic obstruction associated with the absence of intramural ganglion cells.
Because of the ganglionitis,

821) Placenta previa : Occurs when the placenta abnormally implants in the lower segment of the uterus near
o over the cervical os instead of attaching to the fundus.

822)Abruptio placenta: Premature Separation of a normally implanted placenta before the fetus is born.
823) OVR report.: Incident reports write on clear legible handwriting.
824)Nosocomial infection : Hospital acquired infection.
825)Blood pressure : Force of blood against arterial wall
826)Blood deficit :The difference between apical pulse and and peripheral pulse

ِ‫يِاْلَ أرض‬
‫أ‬ ُ ‫َم‬
ُ ‫ك‬
‫ثِِف‬ َ ‫اس‬
‫ِِفي أ‬َ َّ ‫ع‬
‫ِالن‬ ُ ‫ف‬
َ ‫ماِيَ أن‬ َّ َ‫وأ‬
َ ِ‫ما‬ َ
{ 89 }

827)Fusion process :The process that sperm starting penetrates the ovum. Process of egg and sperm fusing to
form a new individual.
828)Capacitation process: Switching on of sperm to become fertilizing; it’s a biochemical event as sperm
essentially demonstrates expected morphology and motility.This step is a biochemical event the sperm move
normally and look mature prior to Capacitation.

829) Pulse pressure: The difference between systolic blood pressure and diastolic blood pressure
Cranial Nerves
‫اسم العصب‬ ‫الوظيفة‬ ‫األدوات المستخدمة‬
I - Olfactory ‫االول‬ smell sensory Coffee or soap
II- Optic ‫الثاني‬ Vision distances sensory Vision distances chart
chart and cotton. Torch
III-Oculomotor ‫الثالث‬ Eyelid elevation motor
IV-Trochlear ‫الرابع‬ Turns eyes motor
downward and
laterally
V- trigeminal ‫الخامس‬ Chewing facial both Pinprick and brushing
and mouth teeth & Hammer for
sensation jaw reflex
VI-Abducens ‫السادس‬ Turns eye Motor
laterally
VII-Facial ‫السابع‬ Facial expression both sweet & salt & hammer
and lip closure
VIII- ‫الثامن‬ Hearing sensory The weber and Rinne
vestibulocochlear equilibrium test) tuning fork
IX- ‫التاسع‬ Gaging and both tongue depressor &
glossopharyngeal swallowing penlight
X-vague ‫العاشر‬ Gaging and both tongue depressor &
swallowing penlight
XI-Accessory ‫الحادي عشر‬ Shoulder motor tongue depressor
movement and
head rotation
XII-Hypoglossal ‫الثاني عشر‬ Tongue motor tongue depressor
movement and
speech

ِ‫يِاْلَ أرض‬
‫أ‬ ُ ‫َم‬
ُ ‫ك‬
‫ثِِف‬ َ ‫اس‬
‫ِِفي أ‬َ َّ ‫ع‬
‫ِالن‬ ُ ‫ف‬
َ ‫ماِيَ أن‬ َّ َ‫وأ‬
َ ِ‫ما‬ َ
{ 90 }

830) A 29-year-old man is in the Surgical Ward on his first post-open thyroidectomy. He appears drowsy but
he is. able to respiration by nodding head. He is
developing mild dy restlessness. What is the initial
recommended goal? of care?
A. Monitor vital signs of thyroid storm
B. Assess for bilateral vocal fold mobility.
C. Monitor for swelling on the neck.
D. Monitor for vocal cord paralysis
Answer: B

ِ‫يِاْلَ أرض‬
‫أ‬ ُ ‫َم‬
ُ ‫ك‬
‫ثِِف‬ َ ‫اس‬
‫ِِفي أ‬َ َّ ‫ع‬
‫ِالن‬ ُ ‫ف‬
َ ‫ماِيَ أن‬ َّ َ‫وأ‬
َ ِ‫ما‬ َ
{ 91 }

831)The Barlow Maneuver is a physical examination performed on infants to screen for developmental dysplasia
of the hip. Barlow's test identifies posterior sublimations or dislocation. Ortolani test also for DDH

Answer: C
832)Nursing student training under the supervision of Staff Nurse 3-month-old infant came to a normal
checkup and the student assessed the pulse it is 165 b/m
What should you do?
A. Inform the doctor
B. Ask parents about a history of heart disease.
C. The staff nurse should check all the vital signs and repeat assessment✅
D. Ask the student nurse to check child's file.
Answer: C
833)A child with a diagnosis of tetralogy of Fallot is scheduled.
to be discharged from the hospital the nurse planning
discharge education should instruct the caregivers that.
during a hyper cyanotic spell, the position MOST likely to
benefit the child is:
Supine
Side-lying
Prone

ِ‫يِاْلَ أرض‬
‫أ‬ ُ ‫َم‬
ُ ‫ك‬
‫ثِِف‬ َ ‫اس‬
‫ِِفي أ‬َ َّ ‫ع‬
‫ِالن‬ ُ ‫ف‬
َ ‫ماِيَ أن‬ َّ َ‫وأ‬
َ ِ‫ما‬ َ
{ 92 }

Knee-chest✅
Answer: D
834) Nurse manager tells one of the staff, “I don’t have time to discuss the matter with you now. See me in my office
later” when the latter asks if they can talk about an issue. Which of the following conflict resolution strategies did
she use?
A. Smoothing
B. Compromise
C. Avoidance✅
D. Restriction
Answer: C
835)A COPD patient siting and leaning forward, what should be improve to patient?
A. position
B. oxygen exchange ✔️
C.respiration pattern
Answer: B
836)Collection of air in pleural space?
A. pneumothorax✔️
B. hemothorax
C. Atelectasis
C. Pleural effusion
Answer: A
837)Patient has barrel chest, dyspnea and pink skin. what should the nurse expect another symptom for patient?
A. Obese
B. Thin✅
Answer: B
838)An 8-month-old patient has gastroenteritis, and He is treated, now what do you instruct the mother about?
A. weaning
B. Nutritional requirements✅
C. Toilet training
D. Accidents
Answer: B
839)Mothers of children aged 1-4 years, what should you teach them about for their child safety?
A. Pick up medicine and sharps✅
B. Don't use violence because it turns out to be a homicide.
C. Accidents
Answer: A
840)Infant came to ER and during the examination, the doctor said he had spinal bifida occult. What does the doctor
know about this condition?
A. Infant has hair patch at back✅
B. Infant has open wound on back.
Answer: A

841)How much does a full-term baby weigh?


2.5-4 Kg
842) NICU child, when his mother came out, he sat shouting, and crying at Which stage of separation?
A. Despair
B. Denial
C. Protest✅
D. Anxiety

ِ‫يِاْلَ أرض‬
‫أ‬ ُ ‫َم‬
ُ ‫ك‬
‫ثِِف‬ َ ‫اس‬
‫ِِفي أ‬َ َّ ‫ع‬
‫ِالن‬ ُ ‫ف‬
َ ‫ماِيَ أن‬ َّ َ‫وأ‬
َ ِ‫ما‬ َ
{ 93 }

Answer: C
843) Which option helps the mother in the taking in stage?
A. let her talk about her experience✅
B. Let her take care of the baby independent
C. Give her educational papers and instruction
Answer: A

844)Vaccine temperature 2-8° c


845)-Patient with blood type AB-ve for blood transfusion what are the compatible blood group for this patient:
A-
B-
O-
AB-✅
Answer: D
846)-nurse manager told his staff if one of you demonstrate punctuality, will have extra day off, what are the method
of motivation used by the nurse manager?
A. Coercive
B. Rewarding✅
Answer: B
847) The healthcare provider is caring for a patient with a diagnosis of first- degree atrioventricular (AV) block.
Which of these waveform patterns identified on the cardiac monitor is consistent with this arrhythmia?

A. QRS complexes are dropped randomly


B. No association between the P waves and QRS complexes
C.Significant shortening of the PR interval
D. Slowed conduction through the AV node✅
Answer: D
848) When the nurse starting change dressing procedure. What is the first step for her?
A. Wear mask
B. Wear gown
C. Sterile gloves✅
Answer: C
849)pregnant women G1 P0 vaginal delivery observed in the second postpartum day that the perineal pad.
saturated with bright red lochia rubra what is the priority nursing intervention?
A. Massage fundus
B. Obtain vital signs.
C. Inform physician
D. Inquire about time of pervious saturated perineal pads.
Answer: D
850)A patient with sever varicose veins of the left leg presents to the clinic. The patient states that three days
ago the right leg became very swollen and the skin on the right half area was very darkly colored. The capillary
refill in the fingers is three seconds and on the right toes is four seconds.
A. Teach the patient self-blood monitoring.
B. Assess the patient to sided heart failure.
C. Encourage the patient to joint an exercise class.
D. Teach the patient how to apply thigh high anti embolic stocking✅
Answer: D
851)What is the most common hazard in hospitals?
A. Chemical

ِ‫يِاْلَ أرض‬
‫أ‬ ُ ‫َم‬
ُ ‫ك‬
‫ثِِف‬ َ ‫اس‬
‫ِِفي أ‬َ َّ ‫ع‬
‫ِالن‬ ُ ‫ف‬
َ ‫ماِيَ أن‬ َّ َ‫وأ‬
َ ِ‫ما‬ َ
{ 94 }

B. Physical✅
C. Biological
D. Psychological
Answer: B
852)Patient with bipolar disorder / schizophrenia. What should the nurse ask the patient to assess orientation for him?
A. Family history
B. The last events in country
C. Date, Time, Person name✅
D. Months from back to first
Answer: C
853)The head nurse needs to increase her staff nursing the next year. What is this considering??
A. Selection
B. Recruitment
C. Downsizing
D. Rightsizing ✅✅
Answer: D
854)The head nurse needs to increase her staff nursing the next year. What is this considering??
A. Selection
B. Recruitment
C. Downsizing
D. Organizing ✅✅
Answer: D
855) A nurse cares for a group of clients in a long-term care facility.
Which situation represents a situation in which the nurse supports the
client's autonomy?
A.A client falls and fractures a hip. The nurse contacts the health care.
provider for a prescription for pain medication prior to transfer for
treatment.
B.A client reports to the nurse regarding observing staff smoking on
facility grounds when it was banned for residents and family members.
C.A competent client who has received a terminal diagnosis request.
the nurse to not reveal the diagnosis to the family due to fear of them.
seeking long-term mechanical ventilation.
D. A client wishes to have a do not resuscitate (DNR) order to
prevent heroic measures by the health care team in the event of
cardiac or respiratory arrest. ✅
Answer: D

856) A nurse cares for a group of clients in a long-term care facility.


Which situation represents a situation in which the nurse supports the client’s autonomy?
A. Patient refused to get I. V line
B. Doctor order for patient no CPR but patient insists to do CPR ✅
Answer: B
857)Patient came to ER with dyspnea, Heart rate 136, Respiration rate 29. After doing x-ray the patient diagnosed
lung abscess. Which Unit should the nurse have admitted the patient?
A. ICU
B. Medical, Surgical ward✅
C. Cardiac ward
Answer: B

ِ‫يِاْلَ أرض‬
‫أ‬ ُ ‫َم‬
ُ ‫ك‬
‫ثِِف‬ َ ‫اس‬
‫ِِفي أ‬َ َّ ‫ع‬
‫ِالن‬ ُ ‫ف‬
َ ‫ماِيَ أن‬ َّ َ‫وأ‬
َ ِ‫ما‬ َ
{ 95 }

858) The patient with dementia and complaining of memory deficit. When the nurse enters patient room. She
found the patient on the floor beside the bed. When she asked him What happened he said I forgot to press the
call bell. She applied her nursing diagnosis and set intervention for that. See the table includes
Nursing diagnosis:
_Risk for fall related to cognitive impairment
_Steps for intervention (Nurse care plan) :
_Remind patient to use call bell continuously
_Immediate Response to the patient after calling
_Don't wait while patient is asking
What is the most appropriate action?
A. Continue for applying care plan that is enough interventions
B. Change care plan and add more intervention to punish patient and provide restrain for him to avoid
reoccurrence.
C. Change intervention to add bed sensor in the side rails for quick response and provide bedside cane If he forgot
call bell✅
Answer: C
859)A 19-year-old boy has been hospitalized with fracture in upper and lower extremities after accident then
provided with casts for upper and lower limbs. Which of these Nursing diagnoses should the nurse consider in the
Nursing plan of care According to his age??
A. Impaired social interaction
B. Alteration in body image✅
C. Risk for infection
D. Anxiety
Answer: B
Definition of hemiparesis /weakness of one entire side of the body
Osteoma / to remove waste from body.
860)DM patient at night shift with cold skin, tachycardia, diaphoresis what you will do fist?
A. check blood glucose.
B. give him cup of orange juice✅
Answer: B
861) Nurse said to patient ... your identity will not be linked to responses... What's the principal?
A. Privacy
B. Confidentiality
C. Anonymity ✅
D. Debrifeing
Answer: C
complication of COPD: (core pulmonale)
862)Patient with Crackle, wheezing, fever and dull sound after percussion. What is the Diagnosis?
A. Pneumonia✅
B. Tuberculosis
C. Cor pulmonale
Answer: A
863)Patient has ulcers in his foot. What is most important nursing diagnosis??
A. High risk for injury✅
B. Altered body image.
Answer: A
864)COPD patient attached on mechanical ventilation. We need to change his position to prevent bed sores. What is
the most appropriate position?

ِ‫يِاْلَ أرض‬
‫أ‬ ُ ‫َم‬
ُ ‫ك‬
‫ثِِف‬ َ ‫اس‬
‫ِِفي أ‬َ َّ ‫ع‬
‫ِالن‬ ُ ‫ف‬
َ ‫ماِيَ أن‬ َّ َ‫وأ‬
َ ِ‫ما‬ َ
{ 96 }

A. Supine
with
elevation
head of bed
B. Lateral
with elevated
HOB 40
degree✅
C. Setting
position
Answer: B
865)What
should the
nurse do post test for the patient after IVP intravenous pyelogram?
A. Doing exercise
B. Encourage oral intake fluids✅✅
C. Eat something
Answer: B
866)Woman came to ER with euphoric mood, hyperactive and talkative. But after physical examination and
assessment she does not has any psychotic symptoms. The symptoms of hyperactivity and talkative not effect on her
social life or job. What is the appropriate diagnosis?
A. Dysesthesia
B. Mania✅
C. Mood disorder
Answer: B
868)Woman with severe pain in left iliac fossa and tenderness. They request for her CT. What should the nurse
expect on CT??
A. Sigmoid diverticulosis✅
B. Sanitary diverticulosis
C. Diverticolum
Answer: A
869)Nurse supervising on assistant nurses and delegate assignment to him and say “diving up patient's assignment."
what type of leadership
A. Autocratic
B. laissez-faire
C. Situational
D.transformational2
Answer: B

870)A 46-year-old patient is in the male Urology Ward after the surgical removal of the stone from his left kidney.
through percutaneous nephrolithotomy under general anesthesia He
has nausea and dull aching pain in left lumbar region His nephrostomy.
bag is attached through a tube in his left kidney for a few days (see

ِ‫يِاْلَ أرض‬
‫أ‬ ُ ‫َم‬
ُ ‫ك‬
‫ثِِف‬ َ ‫اس‬
‫ِِفي أ‬َ َّ ‫ع‬
‫ِالن‬ ُ ‫ف‬
َ ‫ماِيَ أن‬ َّ َ‫وأ‬
َ ِ‫ما‬ َ
{ 97 }

image). What of the following problems is is needed to focus on?


A. Risk of impaired skin integrity due to infection✅
B. Disturbed life cycle related to nephrostomy bag.
C. Knowledge deficiency for self-care management
D. Impaired social interaction due to altered lifestyle
Answer: A
871)46-year-old patient is in the male Urology Ward after the surgical removal of the stone from his left kidney
through percutaneous nephrolithotomy under general anesthesia. He has nurse and dull acting pain in left lumbar
region. His nephrostomy bag is attached through a tube in his left kidney for a few days (see image)
What findings should alert the nurse to report to the physician immediately?
A. Abdominal discomfort
B. Patient Very exhausted
C. Presence of blood and stone gravels in urine
D. Urine output less than 30 ml/hr.✅
Answer: D
872) Pregnant mother always delivery on 36 weeks. She was asking the nurse What to do to prevent that. May
be because of frequent sexual intercourse. How many times should the nurse provide education for her?
A. Once per week✅
B. Once per month
Answer: A

873)Patient came to ER after motor accident. The patient diagnosed hemiparesis /hemiplegia. Which part of spinal
cord injury is affected?
A. Cervical✅
B. Lumbar
C. Thoracic
D. Sacral
Answer: A

874)Patient has sever itching all over the body. What medication should the nurse give?
A. Antibiotic
B. Diphenhydramine✅
C. Zinc Oxide
Rational Antihistamine
Answer: B

875)Patient came to ER complain from blood in stool. What is the most common diagnosis??
A. GERD
B. Gastroenteritis
C. BID bowel inflammatory disease ✅
Answer: C
876. Quinine sulphate drug side effects:
A. Ringing in ears ✅
B. Blindness
C. Hypotension
D. Insomnia
Answer: A
877. Measurements rates of epidemiology.. rate of birth in the year of 2016 is 19.5 per 1000 .. what is it considered?
A. Crude rate ✅

ِ‫يِاْلَ أرض‬
‫أ‬ ُ ‫َم‬
ُ ‫ك‬
‫ثِِف‬ َ ‫اس‬
‫ِِفي أ‬َ َّ ‫ع‬
‫ِالن‬ ُ ‫ف‬
َ ‫ماِيَ أن‬ َّ َ‫وأ‬
َ ِ‫ما‬ َ
{ 98 }

B. Observation rate
C. Specific rate
D. Standardized rate
Answer: A
878. Schizophrenic patients says “ i feel like my arms and limbs are deatcahed from my body and i don’t feel
like i am a alive and it made me avoid socializing” the nurse interpreted this as :
A. Depersonalization ✅ B. Loss of association
C. Dissociative identity disorder D. Dissociative amnesia Answer: A

879. the nurse is monitoring a post surgery patient with respiratory problems, she noticed that his urine
output has declined from 50 ml to 30 ml, she interprets this as which cause?
A. Anesthesia effects✅ B. Operation effect Answer: A
880. Which patient is a contraindicated for enema??
A. Gluocoma B. hypertensive C. renal failure D. liver disease Answer: C
881. A 48yearold patient in the male Surgical Ward had his gall bladder removed through laparoscopic
cholecystectomy 24 hours ago. While evaluating his general condition, the patient appears lethargic and
complains of severe nauseated feeling along with disco mfort in the abdomen. What nursing problem needs to
be prioritized?
A. Disturbed metabolism due to higher energy demand
B. Weak and lethargic due to low food and fluid intake
C. Nausea and vomiting due to slower gut movement ✅
D. Impaired comfort related to postsurgical effects
Answer: C
882)Patient with Manic bipolar disorder. He is hyperactive, too much moving, talk alot and he has loss of appetite,
very thin . The patient also don't sleep. What is the most appropriate nursing diagnosis?
A. High risk for suicide
B. High risk for injury✅
C. Nutritional disturbance
D. Alter sleep pattern
Answer: B
883)Psychiatric patient was crying. When the nurse asked him, He said I want to stay alone. I don’t want anyone
with me leave me cry. What is the best action for the nurse?
A. Maintain privacy and leave him alone
B. You appear sad. I am here to help you✅
C. Stay with him and be silent
D. Say Okay and let him cry
Answer: B
884) The nurse was transporting patient with H1N1. She was asking another nurse for use of N95 with this patient.
What is the action?
A. Use the mask only inside patient room✅
B. Use the mask if patient has cold symptoms
Answer: A
885)The nurse was discussing with group of women and providing educational session about breast self-
examination. Which of the following if said need furthur education?
A. Breast examination after age of 20 years' old✅
B. Doing the examination after the shower
C. Doing the examination either standing or sleeping

ِ‫يِاْلَ أرض‬
‫أ‬ ُ ‫َم‬
ُ ‫ك‬
‫ثِِف‬ َ ‫اس‬
‫ِِفي أ‬َ َّ ‫ع‬
‫ِالن‬ ُ ‫ف‬
َ ‫ماِيَ أن‬ َّ َ‫وأ‬
َ ِ‫ما‬ َ
{ 99 }

D. Doing the examination 5/6 days after period.


Answer: A

886)Patient with 88% O2 saturation What should use?


A. Facial mask✅
A. Nasal cannula
Answer: A
887)The patient in ICU weaned from ventilator and start endotracheal tube. What should the nurse use?
A. Ventru mask✅
B. trachestomy collar
Answer: A
888)Which of the following vitamin supplements can decrease the incidence of Neural tube defects such as
anencephaly and spina bifida new-born or congenital anomalies?
A. Vitamin A
B. Riboflavin
C. Folic Acid✅
D. Vitamin K
Answer: C
889) While caring for a neonate with a meningococcal, the nurse should avoid positioning the child on the:
A. Abdomen
B. Left side
C. Right side
D. Back✅
Answer: D
890) A nurse is assessing a 6-month-old infant that has retar reduced responsiveness and interaction with the
environment to smile or make eye contact. The nurse notices that the attempt to hold or comfort the crying infant.
What diagnosis should the nurse anticipate?
A. Celiac disease
B. Failure to thrive✅
C. Cystic fibrosis
D. Growth hormone deficiency
Answer: B
891)When assessing the funds level At 36 week it is at the >>>> Xiphoid process
Midway between symphysis pubis and Umblical level >>>> 16 week
At the level of Umblicus >>>>>> 20 week
At symphysis pubis level>>>>> 12 week
892)A 28-year-old pregnant woman at 9 weeks presents to the o with vaginal bleeding. During assessment, the nurse
found height is 12cm. Which of the following is the most likely diagnosis?
A. Placenta previa
B. Abruptio placenta
C. Ectopic pregnancy
D. Hydatidiform mole✅
Answer: D
893)Orthostatic hypotension:
A - 80/60.✅
B- 90/70.
C- 110/75.
Answer: A

ِ‫يِاْلَ أرض‬
‫أ‬ ُ ‫َم‬
ُ ‫ك‬
‫ثِِف‬ َ ‫اس‬
‫ِِفي أ‬َ َّ ‫ع‬
‫ِالن‬ ُ ‫ف‬
َ ‫ماِيَ أن‬ َّ َ‫وأ‬
َ ِ‫ما‬ َ
{ 100 }

894)while taking care of the a patient with a spinal cord injury, the patient suddenly complains of pounding headache
upon assessment the patient was found to have diaphoresis , drop in heart and elevated blood pressure , autonomic
dysreflexia is suspected and the head on the bed is elevate . Which of the following is the most appropriate to
important immediately?
A-Notify the physician
B-Assess bladder for distension ✅
C-Continue to monitor for next hour
Answer: B
895)During vaginal examination the nurse palpated the posterior fontanel to be at the right side and upper quadrant
of the maternal pelvis ?
A. ROP
B. LOP.
C. ROA✅
D. LOA
Answer: C
896)The nurse is assessing a child (an infant) with pyloric stenosis. which of the following is likely to note?
A. Diarrhea
B. Projectile vomiting✅
C. Swallowing difficulties
D. Currant jelly like stool
Answer: B
897) A 5- week old newborn admitted paediatric ward with pyloric stenosis the newborn has weight loss, and
projectile vomiting after feeing. Which of the following abdominal organs are directly affected when stenosis
diagnosed?
A. Stomach and duodenum✅
B. Stomach and esophagus
C. Liver and spleen
D. Liver and bile duct
Answer: A
898) A 5year-old child was seen to the Emergency Department abdominal pain, palpable sauge-shaped mass, and
Intussusception is suspected Which of the following is the best diagnostic evaluation to?
A. X-ray
B. endoscopy
C. Rectal biopsy
D. Ultrasonograph✅
Answer: D
899)57- A 30-year-old pregnant lady in her 33 weeks' pregnancy. When the nurse assesses the health condition of the
lady, she provides nursing care and health teaching. Which of the following types of home visiting is the best to be
conducted?
A. Follow up✅
B. Selective
C. Systematic routine
Answer: A
900) Complication of hyperthyroidism?
A. Bleeding
B. Thyrotoxicosis✅
Answer: B
901)A Rehabilitation nurse reviews post stroke patient immunization history
which immunization is a priority for a 72-year-old patient
A. hepatitis A vaccine

ِ‫يِاْلَ أرض‬
‫أ‬ ُ ‫َم‬
ُ ‫ك‬
‫ثِِف‬ َ ‫اس‬
‫ِِفي أ‬َ َّ ‫ع‬
‫ِالن‬ ُ ‫ف‬
َ ‫ماِيَ أن‬ َّ َ‫وأ‬
َ ِ‫ما‬ َ
{ 101 }

B. hepatitis B vaccine
C. rotavirus vaccine
D. pneumococcal vaccine
Answer: D
902) Pregnant woman in third trimaster is complaining fatigue, tired and chest heavieness and dificulty breath. The
nurse told her that is normal specially in the last months of pregnancy. What is the Medical cause and interpretation
filor that?
A. The fetus and uterus on exophoid and press on lung and limit its expand ✅
Answer: A
903)Polycystic ovary syndrome (PCOS) Complications?
A. Amenorrhea
B. Dysmenorrhea
C. Infertility✅
Answer: C
904)Polycystic ovary syndrome (PCOS) Common Signs and symptoms?
A. Dysmenorrhea
B. Infertility✅
C. Polymenorrhea
Answer: B
905)Nursing manager her scheduled staff absence. She had to float another staff to cover defect from another unit
What is the manner that manager use?
A. self-staffing
B. on call staffing
C. Borrowing staffing
D. Floating staffing✅
Answer: D
906)When nursing manager invite unscheduled staff from same unit to cover nursing shortage. What this manner?
A. Self-staffing
B. On call staffing✅
Answer: B
907)Patient with fever, cough with blood. The nurse call to infection center to provide precaution to the patient.
Which precaution should the nurse Apply?
A. Airborne✅
B. Droplet
C. Contact
D. Precaution until the confirm of diagnosis
Answer: A
908) A group of mothers, in their third trimesters, are attending the education session regarding the alert signs for
which they are immediately contact the hospital. The midwife is to assess the understanding of the session by asking
question. What response for the mothers helps assure the midwife about right understanding?
A. Gush of urinary outflow without any urge
B. Low back pain at night and difficulty in sleep
C. Activity intolerance and breathlessness on exertion
D. Absence of contraction after 42 week of pregnancy✅
Answer: D
909)What is the Assessment of data after patient discharge??
A. Audit
B. Retrospective✅
C. Concrete

ِ‫يِاْلَ أرض‬
‫أ‬ ُ ‫َم‬
ُ ‫ك‬
‫ثِِف‬ َ ‫اس‬
‫ِِفي أ‬َ َّ ‫ع‬
‫ِالن‬ ُ ‫ف‬
َ ‫ماِيَ أن‬ َّ َ‫وأ‬
َ ِ‫ما‬ َ
{ 102 }

Answer: B
910) Patient post heart valve replacement. What is the expected drug for the patient after operation?
A. Nitrates
B. Beta blockers
C. Anticoagulant✅
Answer: C
911) A patient with a history of atrial fibrillation has an order for 0.25 milligrams of drug (X), the available
supply/dose is
0.125 milligrams.
Which of the following doses in correct?
A. Dispense two tablets to the patient ✅
B. Administer one tablet twice daily
C. Dispense one half tablet to the patient
D. Return tablets to the pharmacy and re-order
Answer: A
912) Patient Bl. P 230/135. What would patient high risk for??
A. Stroke✅
B. Hypertensive crisis
Answer: A
913) Why should control of high blood pressure with hypertensive patient. What is the most common complication?
A. Hypertensive Encephalopcy✅
B. Cardiac decompensation
Answer: A
914) Patient with unstable angina. The chest pain is ruled out. What is the priority nursing diagnosis?
A. Anxiety from threatened disease
B. Pain related to Angina pectoris✅
Answer: B
915) Self limited crisis Duration for solving ??
A. 2 weeks
B. 4 weeks✅
C. 6 weeks
D. 8 weeks
Answer: B
916) Mother has 8 months child wants to give her baby an egg to eat what is the kind of egg she should give?
A. Give whole egg
B. Don’t give for child until 1 year
C. Give white egg without yolk
D. Give yolk egg without white✅
Answer: D
917) Patient on oxygen with nasal cannula. How should the nurse ensure effectiveness?
A. Increase HR to 90 b/m
B. Increase RR to 24
C. Increase O2 saturation 95%✅
D. Increase Bl. P 120/80 mmhg
Answer: C

918) A nurse manager is open minded, listens to the team, understands others, makes changes to improve unit
operations and procedures. What type of leadership style is this?
A. Situational
B. Democratic ✅

ِ‫يِاْلَ أرض‬
‫أ‬ ُ ‫َم‬
ُ ‫ك‬
‫ثِِف‬ َ ‫اس‬
‫ِِفي أ‬َ َّ ‫ع‬
‫ِالن‬ ُ ‫ف‬
َ ‫ماِيَ أن‬ َّ َ‫وأ‬
َ ِ‫ما‬ َ
{ 103 }

C. Compassionate
D. Transformational
Answer: B
919) The Postpartum mother was asking the nurse about timing for restating sexual intercourse activity. What should
the nurse response?
A. 3 weeks after delivery
B. As long as taking contraceptives
C. After stop of lochia discharges✅
D. Any time she wants
Answer: C
920)Chicken box vaccine - - - - - - Varicella vaccine
Patient with COPD came to ER. The patient was very exhausted. He was complaining shortness of breathing and
cough. What is the most appropriate PaO2 percentage should be given for him?
A. 55 or below mmhg✅
B. 80 or high
C. 70 mmhg
Answer: A
921) What is the best system that help nursing in data interpertation for patient care?
A. Patient documentation system✅
B. Nursing documentation system
Answer: A
922) Sulfa allergy patient should avoid - - - - - Sulfadiazine, sulfamethoxazole, sulfapyridine & sulfisoxazole are
Antibiotics.
923) Primary prevention - - - - prevent disease occurrence rather than cure
924) The nurse use internet to see patient file outside hospital?
A. Telehealth
B. Telecommunication✅
C. Nursing informatics
Answer: B
925)Child with pyloric stenosis and re-hydration. What should the nurse do?
A. Induces Vomiting for child
B. Give oral feeding
C. Start rehydration by nasogastric tube ✅
Answer: C
926)Patient in triage area. What is the first subjective data should triage nurse Obtain?
A. Chief compliance ✅
B. Level of pain
C. Family history
Answer: A
927) Person work in factory. Every day expose to hazard. What type of hazard?
A. Chemical✅
B. Environmental
C. Biological
D. Psychological
Answer: A
928) Psychiatric patient doesn’t like her nose and mouth shape. She has false belief of disturbed body image and
counselled many doctors' for doing operation. Doctor's not accepted her due to no need everything is normal. What is
type of disorder?
A. Pain
B. Conversion

ِ‫يِاْلَ أرض‬
‫أ‬ ُ ‫َم‬
ُ ‫ك‬
‫ثِِف‬ َ ‫اس‬
‫ِِفي أ‬َ َّ ‫ع‬
‫ِالن‬ ُ ‫ف‬
َ ‫ماِيَ أن‬ َّ َ‫وأ‬
َ ِ‫ما‬ َ
{ 104 }

C. Body dysmorphic disease✅


D. Hypochondriasis
Answer: C
929) A patient with congestive heart failure and severe peripheral edema has a
nursing diagnosis of fluid volume excess
What are the two MOST important interventions for the nurse to initiate?
a- Diuretic therapy and intake and output
b- Nutritional education and low-sodium diet
c- Daily weights and intake and output ✅✅
d- Low-sodium diet and elevate legs when in bed
Answer: C
930)Patient has phobia to get infection. He always do hand washing too much until his hand injured from washing.
What is the Diagnosis?
Obsessive compulsive disorder ✅
Answer: A
931)Psychiatric patient repeats unknown words for nurse but the patient can understand it. What is the best term
describing situation?
A. Word salad
B. Neologism ✅
C. Circumstantiality
D. Thought problem
Answer: B
932)Adult Patient (big Scenario with ECG finding at last situation) patient with HR : 55 b/m. What is the most
appropriate Diagnosis?
A. Sinus Bradycardia ✅
Answer: A
933) Patient is experiencing Myocardial ischemia. The ECG is shown ST depression. What is more changes in ECG
the nurse expects?
A. T waves inversion✅
B. Tall T wave
Answer: A
934) Hemoglobin low + Platelets low - - - - - - Risk for bleeding
935) Patient in the Medical ward is experiencing chronic disease. The patient condition become exacerbation.
What this means?
A. The patient condition is improved B.The Patient condition is worsen ✅
C. The patient condition is stable Answer: B
936) Infant 2 months with patent ductus arteriosus. What is the expected signs and symptoms for infant?
A. Acrocyanosis B. Central cyanosis C. Tachycardia and tachypnea✅ D. Tachypnea Answer: C
937) Late deceleration patterns are noted when assessing the monitor tracing of a woman whose labor is
being induced with an infusion of Pitocin. FHT go down from 140 b/m to 130 b/m. The woman is in a side-
lying position. and her vital signs are stable and fall within a normal range. Contractions are intense. last 90
seconds. and occur every 1 1/2 to 2 minutes. The nurse's immediate action would be to:
A. Change the woman’s position B. Stop the Pitocin
C. Elevate the woman’s legs D. Administer oxygen via a tight mask at 8 to 10 liters/minute Answer: B

938) Psychiatric patient came to ER, the patient use razor markes on his hand with letters of the word " i
didn't want to serve ever at this life", the patient wrote on his arm bye tattoo. What is the most appropriate
diagnosis?

ِ‫يِاْلَ أرض‬
‫أ‬ ُ ‫َم‬
ُ ‫ك‬
‫ثِِف‬ َ ‫اس‬
‫ِِفي أ‬َ َّ ‫ع‬
‫ِالن‬ ُ ‫ف‬
َ ‫ماِيَ أن‬ َّ َ‫وأ‬
َ ِ‫ما‬ َ
{ 105 }

A. Borderline personality disorder B. Dissociative disease


C. Major manic episodes Answer: A
939)Famous football player came to hospital. Many nurses want to get up his medical information. The nurse
refused ti give any data due to privacy. What is the best effective explanation?
A. Viewing patients files for only valid authorized people ✅ Answer: A
940)The normal weight gain during pregnancy to mother BMI =19 ...?
A. 6.5 Kg - 10.5 Kg B. 11.5 Kg 16 Kg C. 12 Kg - 18 Kg Answer: B
941) Surgical unit nurse assessed a 35 years old post appendectomy observe abdominal distention with absent
bowel sounds. Which of the following interventions is most appropriate?
A. Encourage ambulation ✅ B. Provide liquid diet as tolerated
C. Administer hemolytic enema D. Check surgical site for signs of infection Answer: A
942)How can the nurse give health education to prevent or reduce sinusitis by health education?
A. Increase carbohydrate intake B. Increase fluids intake
C. Avoid travel with air plane ✅ Answer: C
943)Pregnant woman with hydatidiform mole. She came to ER for x-ray. What is the signs that confirm
diagnosis?? A. Grapes like clusters shape Answer: A
944) The nurse begins a morning shift with the following see the doctor in the waiting room (see table). 1. A
first time mother who delivered two days ago and is having Difficulties breastfeeding 2. A60 year-old man
who has a history of deep vein Thrombosis and is taking daily heparin 3. A76 year-old women who was
hospitalized three days Previously with pneumonia 4. A 56-year-old man who has used all his diuretic pills
Coughing up sputum Which patient should be seen first?
A. One B. Two C. Three D. Four ✅ Answer: D
* Bonding :Relationship between parents and child from childbirth to few months after delivery

945) The nurse was taking history from the patient and she asked one question then repeated that question
again to patient. The patient said (Are you hearing me I answered before). What should the nurse do to
clarify?
A. Ask new question B. Use eye contact during communication ✅
C. Ignore the patient D. Repeat again the previous question Answer: B
946) Post pleural effusion position during physical examination?
A. Setting ✅ B. Supine Answer: A
947) Mother has child with Spastic cerebral palsy. The mother was asking the nurse when her child be adult
he will have mental retardation or impairment. What should the nurse reply?
A. Who has cerebral palsy get more than 70% in IQ.
B. Who has epilepsy and CP Limited for them to have mental problem
C. Who has CP the main cause genetic factors.
D. Who has CP most of them has cognitive impairment with some mental retardation✅ Answer: D

948)According to Ramsey sedation scale pt in icu on sedation on assessment patient deep sleep and not response
to light what is score
A. 2
B. 3
C. 4
D. 6 ✅

ِ‫يِاْلَ أرض‬
‫أ‬ ُ ‫َم‬
ُ ‫ك‬
‫ثِِف‬ َ ‫اس‬
‫ِِفي أ‬َ َّ ‫ع‬
‫ِالن‬ ُ ‫ف‬
َ ‫ماِيَ أن‬ َّ َ‫وأ‬
َ ِ‫ما‬ َ
{ 106 }

Answer: D
949) The nurses in medical unit decided to make research study about fall. What is the first step for doing?
A. Literature review ✅
B. Select research design
C. Research question
Answer: A
950) Multipara postpartum woman was experiencing hemorrhage. Which of the following complication most
common for her?
A. Infection
B. DVT
Answer: A
951)Nurse is caring for a client with a nasogastric tube that is attached to low suction
The nurse monitors the client, knowing that the client is at risk for which acid-base disorder?
A. Metabolic acidosis
B. Metabolic alkalosis
C. Respiratory acidosis
D. Respiratory alkalosis
Answer: B

952) 40-year-old women patient with Parkinsonism Medical Ward. The patient stated that she has the past two
weeks. The nurse was planning to Which type of diet is most suitable for parkin?
A. Solid
B. Liquid
C. Semi solid ✅✅
D. Clear liquid
Answer: C

953) What is the tool for skin assessment temperature?


A. Stethoscope
B. Thermometer
C. Back of fingers
Answer: C
954) A nurse manger is preparing and writing a plan for dealing disasters (code red). Which procedure is the top
priority for the nurse manager the plan?
A. Infection control
B. Staff orientation ✅✅
C. Patient education
D. Patient relocation
Answer: B
955) Father always use violence with his child. What should the nurse expect the cause?
A. Financial status
B. His father abuse him when he was young ✅
Answer: B

956) Schizophrenic patient say ( I will eat fish, I will drink milk, I will go to the sea). What is the most term for that?
A. Ward salad
B. Loss of association ✅
C. Neologism
Answer: B

ِ‫يِاْلَ أرض‬
‫أ‬ ُ ‫َم‬
ُ ‫ك‬
‫ثِِف‬ َ ‫اس‬
‫ِِفي أ‬َ َّ ‫ع‬
‫ِالن‬ ُ ‫ف‬
َ ‫ماِيَ أن‬ َّ َ‫وأ‬
َ ِ‫ما‬ َ
{ 107 }

957) Schizophrenic patient say (Sea, Airport, bus). What is the most term for that?
A. Ward salad ✅✅
B. Loss of association
C. Neologism
Answer: A
*Chicken box rash start from Trunk
*Measles rash start from Face
958) Multipara postpartum woman was experiencing hemorrhage. Which of the following complication most
common for her?
A. Infection
B. DVT
Answer: A
959) Which of the following from pregnents mothers is priority?
A. Vaginal bleeding
B. Eclampsia ✅
C. Diabetes
Answer: B
960)Which of the following drug contraindication during pregnancy?
A. Antibiotic
B. Analgesic
C. Steroids✅
Answer: C
961)Which of the following drug should the doctor give Instruction for patient when using?
A. Antiemetic's
B. Steroids ✅
C. Antibiotic
D. Paracetamol
Answer: B
962) The charge nurse noticed that there is staff shortage during shift. For Whom should the charge nurse report?
A. Supervisor ✅
B. Chief Nurse
C. Director office
Answer: A
963) Child post total adenoctomy / tonsillectomy position
A. Prone or side lying ✅
Answer: A
964) There is one Senior evening shift has been promoted to supervisor. What is the most effective cause that helped
to take decision and accept new position
A. Numbers of staffing during shift
B. Financial promotion
C. His Leadership skills ✅✅
Answer: C
965(The patient came to receive his Lab results from the nurse. The patient has diagnosed with cancer. The patient
said to nurse I am not believe that is my diagnosis, that is not mine. What is the stage considering?
A. Denial✅
B. Anger
C. Depression
D. Bargaining
Answer: A

ِ‫يِاْلَ أرض‬
‫أ‬ ُ ‫َم‬
ُ ‫ك‬
‫ثِِف‬ َ ‫اس‬
‫ِِفي أ‬َ َّ ‫ع‬
‫ِالن‬ ُ ‫ف‬
َ ‫ماِيَ أن‬ َّ َ‫وأ‬
َ ِ‫ما‬ َ
{ 108 }

966) Scrub nurse was counting sheets during operation before doctor closing. The doctor asked her to give surgical
threads. What should the nurse do?
A. Don 'reply and Complete counting
B. Give him threads then Complete counting
C. Say for him wait I am still counting not finished yet ✅✅
Answer: C
967) Patient with active TB ...what about his family?
A. Carriers
B. Susceptible host ✅
Answer: B
968) How to teach a child to give insulin injection?
A. Teach him to give a stuffed animal
B. it is too early to teach him to give✅
Answer: B
‫ ولو موجود اني أعلم‬teach to parent ‫هتكون أصح‬

*Pancytopenia - - - - RBCs and platelet deficiency

969) The doctor order 600 ml of drug during 10 hours. How many minutes for 15 ml of the drug?
A. 15✅
B. 20
C. 25
D. 30
Answer: A

*Schizophrenia Signs Hallucination and delusion

970) The child always fears from nurses while entering the room. What is the most appropriate nursing diagnosis??
A. - Refusal effect
B- Anxiety related to exposure of strange environment
C- Anxiety / Fear related to medical staff✅
Answer: C
971) Woman came to ER with severe Vomiting. Vomiting not stop and the patient was anxious and crying. She said
I will die. What is the most appropriate nursing diagnosis?
A. Fear to unknown disease ✅
B. Distribution body image
Answer: A
972)Which of the following nurses best for providing support during labor?
A Nurse skilled to support ✅
B. Older nurse to support
C. Mother with Epidural no need for support
Answer: A
973) how to maintain patient free of infection during labor?
A. Use clean technique
B. Change pad
C. Limit vaginal examination ✅
Answer: C

ِ‫يِاْلَ أرض‬
‫أ‬ ُ ‫َم‬
ُ ‫ك‬
‫ثِِف‬ َ ‫اس‬
‫ِِفي أ‬َ َّ ‫ع‬
‫ِالن‬ ُ ‫ف‬
َ ‫ماِيَ أن‬ َّ َ‫وأ‬
َ ِ‫ما‬ َ
{ 109 }

974) The nurse care for a 60-year-old woman who history hypertension, hypothyroidism and elevated cholesterol
levels. She takes tablets daily for each of the health problem. The doctor orders a routine dual- x-ray absorptiometry
test that shows decrease bone density. Which medication most likely contributed the test result?
A. Statins
B. Anti-hypertensive
C. Synthetic thyroid hormones ✅
D. Cholesterol absorption inhibitors
Answer: C
975) Infant with tracheoesophageal fistula TOF. What is the signs that need suction for him?
A. Crying
B. Chocking and cyanosis ✅✅
C. Increase heart rate
Answer: B
976) How to prevent infant colic :
A- give infant warm shower then Trendelenburg position
B- Put a warm water bottle on your baby's belly✅
Answer: B
977) Why should the nurse while doing CPR ensure from complete chest recoil?
A - Allow the lung to expand ✅
B- increase cardiac output
Answer: A
978) What is the best nutrition teaching instruction for patient with
Parkinson disease?
A- Choking
B- Drooling ✅✅
C- Aspirations
D- Dysphasia
Answer: B
979) Position for lumbar puncture?!
A. Lateral recumbent position ✅
B. Prone position
C. Supine position
Answer: A
980) The relation between organism and environment is :
A. Ecology ✅✅
B. Pathology
C. Bacteriolog
D. Parasitolog
Answer: A
981) What is the treatment for Rheumatic Fever?
A. Antireumatic ✅
B. Anticoagulent
C. Corticol
Answer: A
982) During tonic clonic seizure patient moving and has difficulty of breathing and cyanosis what is the appropriate
nursing intervention:
A- support arm and extremities
B- put soft material under patient
C- put tongue depressed between teeth
D- protect patient head from injury ✅✅

ِ‫يِاْلَ أرض‬
‫أ‬ ُ ‫َم‬
ُ ‫ك‬
‫ثِِف‬ َ ‫اس‬
‫ِِفي أ‬َ َّ ‫ع‬
‫ِالن‬ ُ ‫ف‬
َ ‫ماِيَ أن‬ َّ َ‫وأ‬
َ ِ‫ما‬ َ
{ 110 }

Answer: D
983) Elderly patient referred to home health care department ...nurse observe home environment for any harms .?
A. Benefiece
B. Non malifecnce ✅
C. Privacy
Answer: B
984) A pregnant mother at early pregnancy was admitted in Emergency Room with leakage of amniotic fluid,
vaginal bleeding and lower abdominal cramping pain. What is the possible diagnosis should the nurse suspected?
A. Missed
B. Inevitable ✅
C. Incomplete
D. Threatened
Answer: B
985) The nurse is assigned to care for a patient with Paget’s dis complaining of bone pain. Which of the following
conditions are this patient high risk for?
A. Hypomagnesaemia
B. Hypernatremia
C. Hypercalcemia ✅
D. Hyperkalemia
Answer:
986) Patient came to Rr after traffic accident and have sever bleeding ...The doctor order she must enter operating
room immediately.... what is appropriate actions to do to obtain consent?
A. Call his husband to obtain telephone consent from him
B. Obtain order from nurse supervisor to enter operation room and do surgery
C. Enter pateint to operation immediately according to dr order without obtaining consent ✅
D. Obtain consent from her friend who come with her to hospital
Answer: C
987) A 4 - year - old boy is admitted with diabetes insidious. The nurse is doing assessment on the boy. Which of the
following findings should the nurse anticipate?
A. bradycardia
B. excessive thirst ✅✅
C low urine outpatient
D. high systolic blood pressure
Answer: B
988) The patient with leukemia. There is development of large numbers of immature / premature lymphocytes. What
is the type of leukemia?
A. Acute ✔️✔️
B. Chronic
C. Leukopnia
Answer: A
989) Patient underwent for endoscopic GIT. What is the expected symptoms that patient complain?
A. Fever
B. Sore throat ✅
C. Abdominal pain
Answer: B
990) Patient with SLE. What is the patient statement indicating that need furthered education?
A. Exposure to sun will harm my skin
B- Hypertension must be anticipated
C- Hypertension is not common with SLE. ✔️✔️

ِ‫يِاْلَ أرض‬
‫أ‬ ُ ‫َم‬
ُ ‫ك‬
‫ثِِف‬ َ ‫اس‬
‫ِِفي أ‬َ َّ ‫ع‬
‫ِالن‬ ُ ‫ف‬
َ ‫ماِيَ أن‬ َّ َ‫وأ‬
َ ِ‫ما‬ َ
{ 111 }

D- The symptom will exacerbate more in winter and spring.


Answer: C
Intentional tort example - - - - - Battery

991) Nurse during in emergency situation ...dr order to administer medication verbally..
What nurse do ?
A. Write order repeat it to dr and administer
B. Repeat order again to dr and administer it ✅
C. Have another nurse to hear order and administer
Answer: B

992) A 40-year-old client underwent an exploratory laparotomy an aesthesia. An assessment of the abdomen 36-
hours showed abdominal distension and an absence of bowel movement in all quadrants. Which complication is most
likely?
A. Paralytic ileus ✅✅
B. Hemorrhage
C. Ruptured colon
D. Intussusception
Answer: A
993) The nurse is preparing a preoperative client for cholecystectomy and transfer him to the operating room. The
nurse should take which of the following actions at this time?
A. Skin preparation ✅
B. Patient identification
Answer: A
994) While transferring patient to operation room. Why should the nurse be applying hospital policy during
transferring?
A. Confidentiality
B. Continuity of care ✅
C. Ethical principles
Answer: B
995) A 32-year-old woman being hospitalized since six days for obesity treatment she reported to the nurse “I feel
burning in my left calf for the last two days “after nursing assessment the nurse found that a left thigh edema, local
tenderness, thigh circumference is more by 8 cm than right thigh and it is warmer to touch, no trauma or wounds.
What is the most possible cause for that?
A. Obesity
B. Local cellulitis
C. Hypertension
D. Deep vein thrombosis ✅
Answer: D
996) A nurse is caring for a newborn in Well Born Nursery she warps the baby with blanket and ensures the
nursey temperature is suitable for the babies. What type of heat loss is the nurse preventing?
A. Radiation
B. Conduction
C. Convention ✅
D. Evaporation
Answer: C
997) Which of the following thermometer accuracy is affected by eating drinking and mouth breathing?
A. Glass ✅
B. Chemical

ِ‫يِاْلَ أرض‬
‫أ‬ ُ ‫َم‬
ُ ‫ك‬
‫ثِِف‬ َ ‫اس‬
‫ِِفي أ‬َ َّ ‫ع‬
‫ِالن‬ ُ ‫ف‬
َ ‫ماِيَ أن‬ َّ َ‫وأ‬
َ ِ‫ما‬ َ
{ 112 }

C. Electronic
D. Tympanic
Answer: A
998)Patient is receiving chemotherapy. When should the nurse give antiemetic to avoid side effect?
A. Before session ✅
B. During session
C. Half hour After session
Answer: A
999)Mother has inevitable abortion. What should the nurse monitor?
A. Hemorrhage ✅
B. Uterine contractions
Answer: A
1000)Child came to ER with projectile vomiting and dehydration. The child diagnosed with pyloric stenosis. What
should the nurse expect developing for?
A. Metabolic alkalosis
B. Metabolic acidosis ✅
C. Respiratory acidosis
Answer: B
1001)What is the signs should the nurse observed for preterm newborn during assessment?
A-Ear cartilage well
B-Sole(foot)well deep crease
C-have more subcutaneous fat
D-Prominent labia minor and smaller labia major ✅
Answer: D
1002)Alzheimer's patient high risk for Fall and admitted with fractures hip during hospitalization the patient fell
twice. What should the nurse do?
A. Continue same plan
B. Impair skin integrity
C. Risk for pressure ulcer
D. Noncompliance add new nursing diagnosis ✔️✔️
Answer: D
1003)Patient with asthma. What is the most appropriate position?
A. Semi fowler
B. High Fowler ✅
Answer: B
1004)Smallpox isolation nurse wear?
A. Gloves and mask
B. Gloves and gown
C. Gloves and eye protector
D. Gloves, gown and N95 mask ✅
Answer: D
1005)The neonate after delivery with normal heart rate and normal respiration rate. The neonate with Pink skin,
active reflexes and Lusty cry. What is the most appropriate Apgar score?
A. 9
B. 10 ✅
C. 8
D. 7
Answer: B
1006) Primipara mother ask nurse about importance of breastfeeding to her baby

ِ‫يِاْلَ أرض‬
‫أ‬ ُ ‫َم‬
ُ ‫ك‬
‫ثِِف‬ َ ‫اس‬
‫ِِفي أ‬َ َّ ‫ع‬
‫ِالن‬ ُ ‫ف‬
َ ‫ماِيَ أن‬ َّ َ‫وأ‬
َ ِ‫ما‬ َ
{ 113 }

Nurse response should be?


A. Lowers risk of asthma and allergies ✅
B. Lower risk of haemolytic diseae
C. Decreass rate of infant linear growth
D. Increase child abstract cognition
Answer: A
1007) A nurse is taking care of a patient who underwent abdominal surgery three days ago. The patient has not been
able to breathe deeply and uses to get out of bed since the surgery due to pain. Also, the patient complains of
shortness of breath, and the lung sounds are diminished on auscultation.
Blood pressure 120/70 mmHg
Heart rate 75 /min
Respiratory rate 22 /min
Temperature 36.4 ℃
Oxygen saturation 89%
Which of the following conditions should the nurse suspect?
A. Sepsis
B. Atelectasis ✅
C. Congestive heart failure
D. Emphysema
Answer: B
1008)CSF contains NOT all of the following except:
A. Insulin. ✅
B. RBC.
C. WBC.
D. Protein.
Answer: A
1009)An elderly client is experiencing an alteration in his
equilibrium and coordinated muscle movements. The
nurse realizes that these functions are controlled by which
area of the nervous system?
a-Brain stem
b-Cerebrum
c-Diencephalon
d-Cerebellum ✔️
Answer: D
1010)A patient is having difficulty with cognitive abilities after a stroke. What part of the brain was MOST likely
affected?
a. Midbrain
b. Cerebrum ✔️
c. Medulla oblongata
d. Cerebellum
Answer: B
1011)10 years old child die is expected. What should the nurse do?
A. Withdraw medication die anyway
B. Give medication even if there is pain ✅
C. Withdraw medication to prevent pain
Answer: B
1012) 42 multpara arrive to ER on active labour with rupture of membranes and she want to have mormal delivery
without medication ...her husband come with her and appear anxtious..

ِ‫يِاْلَ أرض‬
‫أ‬ ُ ‫َم‬
ُ ‫ك‬
‫ثِِف‬ َ ‫اس‬
‫ِِفي أ‬َ َّ ‫ع‬
‫ِالن‬ ُ ‫ف‬
َ ‫ماِيَ أن‬ َّ َ‫وأ‬
َ ِ‫ما‬ َ
{ 114 }

What is appropriate response from nurse?


A. Offer her non pharmacological method ...as she can do
B. Support her decision and offer non pharmacological method when needed ✅
C. Speak with her husband to convince her about other method...
D. Use pharmacological method
Answer: C
1013)Pregnant woman with cervical dilatation 6 cm and effacement 100%. What is the stage of labor?
A. Second stage
B. Active phase ✅
C. Transition phase
D. Latent phase
Answer: B
1014)Pregnant woman with cervical dilatation 7 cm and fetus head on zero station. What is the stage of labor?
A. Second stage
B. Active phase ✅
C. Transition phase
D. Latent phase
Answer: B
1015)What is collaboration method in conflict?
A. Organization system affected
B. Cooperation between two sides ✅
C. Cooperation on one side
Answer: B
1016)Patient came to ER with acute chest pain, restlessness. The patient has cardiac surgery before. He has some
injuries on his body
Bl. P 110/70 mmhg, HR 14 b/m, RR 22. What is the most appropriate nursing intervention?
A. Obtain 12 lead ECG ✅
B. Assess for bed sores
C. Administer Pain medication
D. Start I. V fluids
Answer: A
1017)Patient with acute renal failure (ARF)
Lab result
NA 120
Potassium 6
Calcium normal result
What is the most appropriate diet should nurse provide in food?
A. Low NA ✅
B. High potassium
C. High phosphate
D. Low carbohydrate
Answer: A
1018) Patient after inguinal hernia
What is best indicator for dischrage ?
A. Patient eat very well
B. Patient free of pain
C. Patient move without assistance ✅
D. Patient urine output500 ml
Answer: c

ِ‫يِاْلَ أرض‬
‫أ‬ ُ ‫َم‬
ُ ‫ك‬
‫ثِِف‬ َ ‫اس‬
‫ِِفي أ‬َ َّ ‫ع‬
‫ِالن‬ ُ ‫ف‬
َ ‫ماِيَ أن‬ َّ َ‫وأ‬
َ ِ‫ما‬ َ
{ 115 }

1019) The nurse is assessing 50-year-old woman whose thyroid enlarged. A blood sample was collected and an
analysis confirm diagnosis.
TSH 0.12 normal 0.4-6.5
Free 210 normal 50-140.
Thyroxin T4 normal 4.5-11.2
A. Thyroidectomy ✅
B. Incision and drainage
C. polythyroidectomy
D. adrenalectomy
Answer: A
1020) patient diagnosed with delirium sees the intravenous (IV) tubing and believes it to be a snake. How should the
healthcare provider document this behavior?
A. Hallucination
B. Illusion ✅
C. Confusion
D. Delusion
Answer: B
1021)What is the position during liver biopsy?
A-Right side lying
B-Prone
C-Lateral
D-supine and right hand below head✔️
Answer: D
1022)Community health nurse is providing health education for diabetic patient during home visit regarding self-
injection of insulin. Which of the following statement need further teaching?
A. Recap /Cover the needle and throw it in normal home garbage ✅
Answer: A
1023)Psychiatric patient is complaining from extrapyramidal side effect with continuous spasms and muscle
contractions), motor restlessness, rigidity, slowness of movement, tremor, and irregular, jerky movements, upward
eye movement, smacking and sucking of lips and protruding tongue. What is the most common drug cause that?
A. Clozapine
B. Haloperidol✔️
Answer: B
1024)Patient has been diagnosed with urinary tract infection. What is the
most likely cause of this infection?
A. Staphylococcus aureus
B. Neisseria gonorrhea
C. Escherichia coli enterococci ✅
D. Streptococcal beta-hemolytic A and B
Answer: C
1025)Amenorrhea for 5 weeks and breast tenderness. What should she do now?
A- HCG home test ✅
B- Ultrasound Sonar to see embryo
Answer: A
1026)At 6:00 PM while admitting a woman for a scheduled repeat C section1, a patient tells a nurse that she drank a
cup of coffee at 4:00OM because she wanted to avoid getting a headache. Which of the following actions should the
take first?
A. Inform anesthesia care provider. ✅
B. Ensure preoperative lab results are available.

ِ‫يِاْلَ أرض‬
‫أ‬ ُ ‫َم‬
ُ ‫ك‬
‫ثِِف‬ َ ‫اس‬
‫ِِفي أ‬َ َّ ‫ع‬
‫ِالن‬ ُ ‫ف‬
َ ‫ماِيَ أن‬ َّ َ‫وأ‬
َ ِ‫ما‬ َ
{ 116 }

C. Start prescribed IV with lactated ringers.


D. Contact patient's obstetrician
Answer: A
1027)Which of the following discharge planning instructions takes PRIORITY in patient with congestive heart
failure?
a. Maintaining a low cholesterol, low sodium and low potassium diet
b. Recognizing signs and symptoms that require immediate medical attention ✅
c. The importance of remaining physically active
d. Smoking Cessation
Answer: B
1028)37-year-old woman with inflammatory bowel disease is scheduled to undergo a procedure in which a stoma
will be formed in the right lower quadrant, five centimeters below the waistline. The nurse advises the patient on
how to avoid potential post-operative intestinal obstruction. Which of the following types of food best recommended
post-operatively?
A. Broccoli and fish
B. Meats and cauliflower
C. Yogurt and parsley ✅
D. Corn and seeds
Answer: C
1029)The community nurse visited Postpartum mother and she complained her. She feels abdominal pain during
breastfeeding and she never feel that on previous delivery before. What is the most appropriate action?
A. It is not important. No action
B. It is important. Stop breastfeeding
C. It is important. Continuous breastfeeding ✅
D. It is not important. Continuous breastfeeding
Answer: C
1030)Sigma theta tau theory in research. What is first priority guideline for Sigma theory?
A. Patient safety ✅
B. Health Team
C. Nursing staff
D. Patient satisfaction
Answer: A
1031)The head nurse wants to improve quality system in her department among nursing staff. What is this
considering?
A. Case management
B. Quality roles team ✅
Answer: B
1032)Why should recommend food with acid base (increase acidity)?
A. Prevent kidney stones
B. Prevent urinary retention
C. Increase urine output ✅
Answer: C
1033)Patient came to ER and diagnosed with meningitis. How should the nurse handle with patient?
A. Keep patient in same isolation room ✅
B. Use mask N95 all times
C. Limit the visitors
Answer: A
1034)How decrease heat loss for neonate by convection?
A. Dry baby
B. Air condition AC ✅

ِ‫يِاْلَ أرض‬
‫أ‬ ُ ‫َم‬
ُ ‫ك‬
‫ثِِف‬ َ ‫اس‬
‫ِِفي أ‬َ َّ ‫ع‬
‫ِالن‬ ُ ‫ف‬
َ ‫ماِيَ أن‬ َّ َ‫وأ‬
َ ِ‫ما‬ َ
{ 117 }

Answer: B
1035)Patient admitted to medical unit with paraplegia. The patient can transfer himself by wheel chair but need some
assistance. The nurse inserted foley catheter. And made urine analysis
Ph of urine 6 normal range 2-8
Urine color yellowish
What is the priority nursing diagnosis for him?
A. High risk for fall ✅
B. Risk for UTI
Answer: A
1036)What is the most common complications of varicose vein?
A. DVT
B. Varicose ulcer ✅
Answer: B
1037)What is the drug to minimize intracranial pressure?
A. Warfarin
B. Morphine
C. Potassium
D. Dulcolax
Answer: D
1038)Pregnant Woman with diabetes melitus. She came for periodic checkup and follow up. The mother tested for
lab investigations. Lab results high protein in urine and confirmed preeclampsia
Bl. P 164 /100
Glucose level 324 g/dl
What is the toxic effect for preeclampsia?
A. Hypertension ✅
B. Increase heart rate
C. Increase Glucose rate
Answer: A
1039)Patient with smallpox. He became fine and no symptoms but still isolated for 12-14 days. What is the type of
the period?
A. Incubation period
B. Infective period
C. Latency period
D. Communicable period
Answer: A
1040) nurse is assigned to the antenatal clinic to give health education to a group of pregnant women on the danger
signs in pregnancy Which of the following would the nurse instruct women to immediately come to hospital?
A. If they experience some headaches
B. If they start bleeding from the vagina ✅
C. If their estimated data of delivery has passed
D. If they experience more than 10 fetal movements per day
Answer: B
1041)Amnion. inner membrane surrounding the fetus. The amniotic fluid, fetus, and umbilical cord are all found
within the amnion.
Chorion. Outermost embryonic membrane and forms part of the placenta.
To prevent and releive fetal distress related to maternal hypotension. Which of the position should apply?
A. Left side ✅
B. Right side
C. Semi fowler

ِ‫يِاْلَ أرض‬
‫أ‬ ُ ‫َم‬
ُ ‫ك‬
‫ثِِف‬ َ ‫اس‬
‫ِِفي أ‬َ َّ ‫ع‬
‫ِالن‬ ُ ‫ف‬
َ ‫ماِيَ أن‬ َّ َ‫وأ‬
َ ِ‫ما‬ َ
{ 118 }

D. Knee chest position


Answer: A
1042)37 years old patient with spinal paralysis. He can't do self-care. What is the long term goal?
A. Strength of body muscles ✅
B. Provide socialization and prevent depression
C. Provide self-care
Answer: A
1043)Woman with tabia fracture. What is the priority action for her?
A. Elevate and support leg ✅
B. Pain management
Answer: A
1044)45-year-old woman is receiving chemotherapy for breast cancer. Two weeks after the initial treatment she
telephoned the nurse at the cancer center and reports she has hair loss, nausea, tiredness, a body temperature is 38.1
C, and air hunger. Which finding most likely indicates she needs to report the clinic?
A. Pyrexia
B. Nausea
C. Hair loss
D. Air hunger ✅
Answer: D
1045)A midwife visits a mother four weeks after delivery. The mother is breastfeeding her baby but she requests the
midwife to suggest alternative formula milk as she has to return back to her job and her baby will stay in a day care
center. Which of the following teaching plans is suitable for the mother?
A. Supplementary medications with bottle feeding
B. Combined schedule of the breastfeed and top feed
C. Hygiene practices with bottle feeding
D. How to express and save milk ✅
Answer: D
1046)What is insomnia drug?
A. Clozapine
B. Zolpidem ✅
Answer: B
1047)The community health nurse is giving instructions for Postpartum mother about oral contraceptive pills. What
is the most appropriate instructions?
A. Start taking pill after 7 days from menstruation for 21 days
B. Take one bill each day at same time ✅
Answer: B
1048)Mother of child came to ER. Her child has 7 times diarrhea. How can we detect category of dehydration?
A. Do Physical examination ✅
B. Assess how many times of bowel movement
Answer: A
1049)Which statement best describes case management?
Clinical outcomes should occur within a prescribed time frame case manager definition?
a-invasion of living tissues by pathogenic microorganism.
b- collaborative process of assessment, planning, facilitation, care coordination, evaluation, and advocacy for options
and services to meet an individual's and family's comprehensive health needs through communication and available
resources to promote quality, cost-effective outcome. ✅
c- the person who leads or commands a group, organization, or country
d-non.
Answer: B

ِ‫يِاْلَ أرض‬
‫أ‬ ُ ‫َم‬
ُ ‫ك‬
‫ثِِف‬ َ ‫اس‬
‫ِِفي أ‬َ َّ ‫ع‬
‫ِالن‬ ُ ‫ف‬
َ ‫ماِيَ أن‬ َّ َ‫وأ‬
َ ِ‫ما‬ َ
{ 119 }

1050) 5-year-old boy has dietary modification due to his diabetes, In his parents' encourage him to value good
nutritional habits, they decided to deny him playing favorite toys when he eats food that is not on his diet plan. What
this mode of value transmission is called?
A. Modelling
B. Moralising
C. Responsible choice
D. Rewarding and punishing ✅
Answer: D
1051)What is the priority care for patient during bathing?
A. Wear sterile gloves
B. Lower the bed and left side rails ✅
C. Put gauze on any wound
Answer: B
1052)What is signs for positive pregnancy confirmation?
A. Fetal heart sound by Doppler ✔️
B. Fetal movement by health care provider
C. Ultrasound
Answer: A
1053) A 10-month-old infant was admitted with recurrent otitis made with in months. On assessment, the nurse
noticed that the parents lock knowledge about the condition. Health teaching is planned. Which risk factor should the
nurse include in health teaching?
A. High fever
B. Painful ear
C. Foreign body
D. Respiratory infection ✅
Answer: D
1054)What is the thing that prevent overlapping in duty of work?
A. Organizational structure ✅
B. Job description
C. Need for assessment
Answer: A
1055) Nurse manager conduct survey about patient fall
What first thing to do ?
A⁃ Review patients file
B⁃ Review literature ✅
C⁃ Interview with patient
Answer: B
1056) Women wit Ectopic pregnancy most confirm sign ?
A ⁃ throbbing pain in right upper quadrant
B⁃ Stabbing pain in lower abdomen quadrant ✅
Answer: B
1057) Which structure secret HCG hormones to sign pregnancy ?
A⁃ decidua
B ⁃ Zygote
C ⁃ Morula
D⁃ Blastocyst ✅
Answer: D

ِ‫يِاْلَ أرض‬
‫أ‬ ُ ‫َم‬
ُ ‫ك‬
‫ثِِف‬ َ ‫اس‬
‫ِِفي أ‬َ َّ ‫ع‬
‫ِالن‬ ُ ‫ف‬
َ ‫ماِيَ أن‬ َّ َ‫وأ‬
َ ِ‫ما‬ َ
{ 120 }

1058) Patient came to ER after motor accident. The patient has sever bleeding with urine output 30 ml /hr and
normal vital signs Normal Bl. P, normal HR, Normal respiration). The doctor ask the nurse to assess good tissue
perfusion. Which of the following signs detect that? ‫؟‬
A⁃ heart rate
B⁃ Respiration rate
C ⁃ Urine output ✅
D ⁃ Blood pressure
Answer: c
1059) Neurological shock sign ?
A⁃ increase BP
B⁃ Decrease heart rate ✅
Answer: B
1060) Full bladder is important to evacuated it during pregnancy before sonar and during labour and after. What is
the Major complications of full bladder?
A⁃ Pain
B ⁃ False reading
C⁃ Bleeding ✅
D. Infection
Answer: c
1061) Child with recurrent UTI , What is the most risk factors ?
A ⁃ abdominal distention
B⁃ Abdominal pain
C ⁃ Chronic constipation ✅
Answer: c
1062 ) Patient came to ER has chronic COPD with dyspnea, wheezing and large amount of expectoration . Which
type of COPD ?
A⁃ Bronchitis ✅
B⁃ Bronchitis asthma
C ⁃ Emphysema
Answer: A
1063 ) Child came to ER and complaining from abdominal pain. The child is diagnosed willms. What is the first
reaction for parent?
A⁃ Anger ✅
B-Bargaining
Answer: A
1064 ) Maslow's hierarchy of needs is an idea in psychology........
The priority nursing care of maslow hierarchy is :
A. Nursing diagnosis
B. Nursing care
C. Nursing planning ✅
D. Nursing care with scientification
Answer: C
1065 ) Community health nurse was visiting people in certain area for observation and assessment. She found their
high risk for lead poison. She refer all to get lab invistigation and doing tests. What is the type for that?
A. Screening ✅✅

ِ‫يِاْلَ أرض‬
‫أ‬ ُ ‫َم‬
ُ ‫ك‬
‫ثِِف‬ َ ‫اس‬
‫ِِفي أ‬َ َّ ‫ع‬
‫ِالن‬ ُ ‫ف‬
َ ‫ماِيَ أن‬ َّ َ‫وأ‬
َ ِ‫ما‬ َ
{ 121 }

B. Survillance
C. Assessment
Answer: A

1066) Psychiatric patient use MAOI inhibitors. He should avoid tyramin in food. What is the risk for tyramin
intake?
A. GIT upset
B. Hypertension ✅
C. Kidney problems
Answer: B
1067 ) Scrub nurse experienced wound in her hand. She came to ER and care is done. The next day she has duty.
What should she do? ‫؟‬
A⁃ Report to head nurse ✅
B ⁃ Change role with circulating nurse
C⁃ Wear double gloves in right hand
D⁃ Wash five minutes before gloving
Answer: A
1068 ) Patient with large liver tumor. The doctor decide to him cryosurgery. What does this type of surgery means,
Cryosurgery was preformed on a patient with cancer tumors, this surgery ?
A. Freezes the tumor or abnormal or diseased tissue using cold liquid nitrogen or argon gas ✅
B. Heat therapy
C. Hepatomegaly
Answer: A
1069 ) The neonate is delivered from diabetic mother. He is high risk for hypoglycemia What should the nurse do to
avoid it?
A⁃ Give insulin
B⁃ Oral feeding ✅
C ⁃ Nothing to do
Answer: B
1070 ) A patient is scheduled for an abdominal aneurysm repair .
This is what type of surgical intervention ?
A. Diagnostic
B. Transplant
C. Curative ✅
D. Palliative
Answer: c
1071 ) 26.A Patient has a dissection aortic aneurysm. The patient’s surgery
would be categorized as:
a. Elective
b. Urgent
c. Emergency ✅
d. Diagnosed
Answer: c
1072 ) Patient with I. V infusion . The patient is complaining redness swelling pain at infusion site. What should the
nurse do?
A. Slow rate

ِ‫يِاْلَ أرض‬
‫أ‬ ُ ‫َم‬
ُ ‫ك‬
‫ثِِف‬ َ ‫اس‬
‫ِِفي أ‬َ َّ ‫ع‬
‫ِالن‬ ُ ‫ف‬
َ ‫ماِيَ أن‬ َّ َ‫وأ‬
َ ِ‫ما‬ َ
{ 122 }

B. Stop and inform doctor ✅


C. Cold compress
Answer: B
1073 ) Community health nurse is providing public health survillance. What is the method of problem solution
should use to solve several reasons?
A. Tree
B. Brain storming ✅
C. Delphi
Answer: B
1074) A young Unit Manager of the Pediatric Ward. Most of her staff nurses are senior to her, very articulate,
confident and sometimes aggressive. She feels uncomfortable believing that she is the scapegoat of everything that
goes wrong in her department. Which of the following is the best action that she must take?

A. Identify the source of the conflict and understand the points of friction ✅
B. Disregard what she feels and continue to work independently
C. Seek help from the Director of Nursing
D. Quit her job and look for another employment.
Answer: A
1075) As a young manager, she knows that conflict occurs in any organization. Which of the following statements
regarding conflict is NOT true?
A. Can be destructive if the level is too high
B. Is not beneficial; hence it should be prevented at all times ✅
C. May result in poor performance
D. May create leaders
Answer: B
1076) The head nurse knows that one of her staff is ‫اد‬burnout. Which of the following is the best thing for her to do?
A. Advise her staff to go on vacation.
B. Ignore her observations; it will be resolved even without intervention
C. Remind her to show loyalty to the institution.
D. Let the staff ventilate her feelings and ask how she can be of help. ✅
Answer: D
1077) Nurse manager plans of assigning competent people to fill the roles designed in the hierarchy. Which process
refers to this?
A. Staffing ✅
B. Scheduling
C. Recruitment
D. Induction
Answer: A
Barrel chest find in :
A. Lung abcess
B. Emphesema ✅
C. Cor pulmonale
Answer: B
1078) o An 18-year-old client with acute lymphocytic leukemia is admitted to the bone marrow transplantation unit.
His family is having trouble
dealing with the emotional and financial pres-sures of his disease.

ِ‫يِاْلَ أرض‬
‫أ‬ ُ ‫َم‬
ُ ‫ك‬
‫ثِِف‬ َ ‫اس‬
‫ِِفي أ‬َ َّ ‫ع‬
‫ِالن‬ ُ ‫ف‬
َ ‫ماِيَ أن‬ َّ َ‫وأ‬
َ ِ‫ما‬ َ
{ 123 }

The Pharmacist, nurse, doctor, Physiotherapy meet to discuss the care plan for the patient. What is the type of care
interaction?
A- Case management
B- Interdisciplinary ✅
C- Cooperation
D- Collegiality
Answer: B
1079) Patient with somatization. What is the best action?
A. Cognitive development ✅
Answer: A
1080) 4 years old Child with cast. What is the nursing care for cast?
A. Put in hot water
B. Put infront of fan or cooller to dry fast
C. Put in cold water to dry fast
D. Put powder on the edges of the cast to prevent itching
Answer: B
1081) pregnant lady she come to gynocolical word she have vaginal bleeding and cervex not dialted the problem
ocuured before she followed bed rest and when she did effort it returned back to her . They diagnosed her with
threatened abortion. What is the treatment should the nutse expect for this lady :
A-surgical treatment
B-induction of labor(syntocinon)
C-dialitation and curtage
D. Repeate Bed rest ✅
Answer: D
1082) A 23-year-old vaginal delivery primigravida mother was discomfort due to breast engorgement on the second
post The mother complained of pain on the breast site and the baby not able to suck the milk. Which of the following
will relief the mother's discomfort?
A. Breast binder
B. Well-fitting brassiere
C. Encourage breast feeding ✅
D. Lactation suppressing medication
Answer: C
1083) Female patient has been advised that laboratory tests confirm genital warts and recurrent many times . The
nurse should teach the patient that a Papanicolaou test (Pap smear) is recommended:
A. Once Every 6 months ✅
B. Once Every year
C. Once Every 3 years
Answer: A
1084) A 38-year- old patient is about to have lumber disk surgery. during preoperative care, the nurse instructs the
patient including the family.
members how to do "log rolling" to change patient position. One of the family
members ask why they must do such action in turning the patient.
postoperative.
Which of the following is the nurse best response?
A. Facility good circulation
B. Avoid spinal movement ✅
C. Prevent post-operative bed sore.

ِ‫يِاْلَ أرض‬
‫أ‬ ُ ‫َم‬
ُ ‫ك‬
‫ثِِف‬ َ ‫اس‬
‫ِِفي أ‬َ َّ ‫ع‬
‫ِالن‬ ُ ‫ف‬
َ ‫ماِيَ أن‬ َّ َ‫وأ‬
َ ِ‫ما‬ َ
{ 124 }

D. Makes changing of patient position easier.


Answer: B

1085) Which of the following needle's angle for insulin injection?


A. A
B. B ✅
C. C
Answer: B
1086) Mother want to ensure her fetus health is well. What is the most appropraite method?
A. Measure fundal hight ✅ B. Ultrasonography C. Check Mother heart rate Answer: A
1087) Nurse administer digoxin to patient and dr order subsequent dose of digitalis ...nurse assess apical pulse before
administering medication
When nurse should withholding it and notify dr?
A. heart rate 57✅
B. 120
C. 87
Answer: A

1088) Acute Respiratory distress syndrom is relates to?


A. Disease that cause chronic obstruction in airway
B. Disease that affect alveolar capilly permeability ✅
C. Disease caused due to excorine dysfunction
Answer: B
1089) Most appropraite method to explain ambulatory surgery for patient. which the best method?
A. Lectury and video

ِ‫يِاْلَ أرض‬
‫أ‬ ُ ‫َم‬
ُ ‫ك‬
‫ثِِف‬ َ ‫اس‬
‫ِِفي أ‬َ َّ ‫ع‬
‫ِالن‬ ُ ‫ف‬
َ ‫ماِيَ أن‬ َّ َ‫وأ‬
َ ِ‫ما‬ َ
{ 125 }

B. Audio vision
C. Discussion ✅
D. Written instructions
Answer: C
1090) Newly promoted (nurse manager) ... what's priority for her?
A. Supervise patient care delivered by staff
B. Complete evaluation of nursing staff ✅
C. Make unit plan and operational plan
Answer: B
1091)Multicultural team work on saudi Arabia what's important thing for the all team member to have?
A. Have the same value and belief
B. Have the same professional guidelines
C. Have the same goal and objectives toward work ✅
D. Have the same qualifications
Answer: C
1092) Patient fall and has injury ... what should nurse do to decrease liability to fall?
A. Just inform the coming nurse
B. Write incident report
C. Document what happened ... inform...and follow up patient ✅
Answer: C
1093) Quality team in meeting to set the Best measure to decrease liability for fall...What is the best action for that?
A. Apply fall risk band around wrist
B. Lower bed level
C. Round observations on patient
D. Apply fall risk sticker on patient door ✅
Answer: D
1094) Action of comined oral contraceptives?
A. Affect lining of uterus making it unfavorable for implementation
B. Supress lutenizing hormone and follicle stimulating hormone
and prevent ovulation ✔️
Answer: B
1095) Women come to ER with her husband and the husband demonstrate she not talk or voluntary eat since the son
died in accident, She isolated since she has seen his son body. What the long term care for her ?
A. Physiological needs
B. psychological Care ✅
C. spiritual support
D. family support
Answer: B
1096) The patient with head injury will be discharges. What is the best head injury advice after discharge?
A. Repeat CT scan after 48 hours
B. Return immediately if drowsy ✅
Answer: B
1097) A 42 year old man with thalassemia received a packed cell transfusion. The nurse assess the stability of his
condition after transfusion by monitoring the vital signs and general condition every two hour. When should the
nurse immediately report the patient condition ?
A. Sever headache and raised blood pressure

ِ‫يِاْلَ أرض‬
‫أ‬ ُ ‫َم‬
ُ ‫ك‬
‫ثِِف‬ َ ‫اس‬
‫ِِفي أ‬َ َّ ‫ع‬
‫ِالن‬ ُ ‫ف‬
َ ‫ماِيَ أن‬ َّ َ‫وأ‬
َ ِ‫ما‬ َ
{ 126 }

B. Raised body temperature and flushed skin ✅


C. Joint pain, body ache listlessness
D. Restlessness & bradycardia
Answer: B
1098) Girl after eye surgery. She is irritable and try to remove sheld from eye. The nurse observed sweeling, Redness
and tears around eye. What is the Nursing diagnosis?
A. Anxiety
B. Pain, sweeling after surgery ✅
C. Risk for injury
Answer: B
1099) The baby has asthma and complaining with cough, fever and secretions . He is lethargic and cyanosis around
his dry and crackles lips and anorexic. Wahati the first action?
A. Assess skin turger and dehydration
B. Assess Oxygen saturation and child behavior ✅
C. The last 24 hour intake and output
Answer: B
1100) A client diagnosed with somatization disorder visits multiple physicians because of var- ious, vague symptoms
involving many body systems. Which nursing diagnosis takes priority?
A. Risk for injury R / T treatment from multiple physician. ✅
B. Anxiety R / T unexplained multiple somatic symptoms.
c. Ineffective coping R / T psychosocial distress.
D. Fear R / T multiple physiological complaints.
Answer: A
1101) What is the most common cause for peptic ulcer?
A. pylori Answer: A
1102) When assessing the monitor tracing of a woman whose labor is being induced with an infusion of
oxytocin. her vital signs are stable and fall within a normal range. Contractions are intense last 90 seconds.
and occur every 1 1/2 to 2 minutes but with moderate strength . The fetah heart rate drop from 140 to 130 b/m
abd return to base line within 15-20 second. The nurse's immediate action would be to:
A. Increase rate and continue
B. Slow rate and put the pt left lateral to assess fatal heart rate✅
C. Slow rate, give oxygen and inform the doctor about the fatal heartrate irregularities and hypercontractlity
Answer: B
1103) 60-year-old man is being discharged from the post-operative Care Unit following a transurethral
resection of the prostate. The nurse provides discharge information regarding the care of the bladder
catheter. Which method would be most effective in bladder retraining for this patient?
A. Scheduled urination every 2-3 hours ✅
B. Limit fluid intake before sleeping time
C. Perform pelvic floor exercises daily
D. Increase fluid intake during the daytime Answer: A
1104)Alteration in nutrition: More than body requirements for 14 months. The patient has reached ideal body
weight and reports enjoying a new life style(see table)
*Alteration in nutrition, more than body requirements. Short term goals. The Patient will :
*Lose 4 pounds each week.
*Eat a healthy well-balanced diet daily

ِ‫يِاْلَ أرض‬
‫أ‬ ُ ‫َم‬
ُ ‫ك‬
‫ثِِف‬ َ ‫اس‬
‫ِِفي أ‬َ َّ ‫ع‬
‫ِالن‬ ُ ‫ف‬
َ ‫ماِيَ أن‬ َّ َ‫وأ‬
َ ِ‫ما‬ َ
{ 127 }

*Exercise for 30 minutes 5 days each week.


Which of the following is the best action should the nurse take?
A. Continue the care plan as written for another year
B. Discontinue the care plan since the patient has met all goals
C. Change the long-term goal for altered nutrition to maintain current weight ✅
D. Discontinue the care plan and add new diagnosis of health seeking behavior Answer: C

1105) 17 years old patient Newly diagnosed with type 1 diabetes melitus. He need to inject insulin twice per
day. What the most appropriate action?
A. Discharge instructions
B. Insulin injection ✅
C. life style change to mange disease Answer: B
1106) Doctor order 1000 unit heparin in 5 ml. The available 5000 unit. How many ml should the nurse give?
A. 1 ✅
B. 2
C. 3
Answer: A
1107) Nurse manger pull out (float) nurse from medical ward to CCU which of the following Patients should be
assigned to the nurse:
A. Patient just transferred from coronary artry surgery
B. Patient just received with unstable angina on heparin infusion ✅
C. Patient need discharge education about coronary stenting
D. Patient on admission need assessment
Answer: B
1108) A nurse is performing physical examination on a newborn. She that the baby has developed cephalohematoma.
This baby is at risk of which of the following?
A. Sudden death
B. Pathological jaundice ✅
C. Infected umbilical cot
D. Increased intracranial pressure
Answer: B
1109) Contraindication for oral contraceptives?
A. Hypotension
B. Hypertension ✅
C. Anemia
D. Infection
Answer: B
1110 ) Which of the following diagnostic studies is essential to differentiate
between renal failure and lower renal obstruction that cause urinary
retention?
A. Cholesterol level
B. Abdominal X-ray
C. Complete blood tests
D. Blood urea nitrogen and serum Creatinine. ✅
Answer: D
1111) Nurse is caring for a 58-year-old patient (See lab result).

ِ‫يِاْلَ أرض‬
‫أ‬ ُ ‫َم‬
ُ ‫ك‬
‫ثِِف‬ َ ‫اس‬
‫ِِفي أ‬َ َّ ‫ع‬
‫ِالن‬ ُ ‫ف‬
َ ‫ماِيَ أن‬ َّ َ‫وأ‬
َ ِ‫ما‬ َ
{ 128 }

Test Result Magnesium 2.8


Normal Value 0.7-1.2mmoI/L
Which ECG change is the nurse expected to note?
A. Prolonged QRS ✅
B. Multiple P waves
C. Prominent U waves
D. Depressed ST segment
Answer: A
1112) What is the goal for cardiac catheterization?
A. Obtain venous pressure
B. Assess Oxygen for heart chambers ✅
Answer: B
1113) The patient ask him self too much if he close the home door or no. What is the most appropriate Diagnosis?
A. Obsessive compulsive disorder ✅
B. illusion
C. Hallucinations
Answer: A
1114) Gremaer measles isolation period after rash ?
A. 5 days
B. 1 week ✅
C. 2 weeks
D. 3 weeks
Answer: B
1115) Patient has given digoxin then the doctor ordered Digibind which of the following vital signs is most
likely the patient have:
A. Bp ….. HR 69
B. Bp ….. HR 80
C. Bp ….. HR 90
D. Bp ….. HR 120
Answer: D
1116) Which patient is contraindicated from MRI ?
A. Patient who has dye contract allergy
B Patient who have cardiac pacemaker ✅
Answer: B
1117) is the dose of vitamin k according to WHO 2017?
A. 0.2
B. 0.5
C. 1 ✅
D. 2
Answer: C
1118) Postpartum mother breastfeeding weight 46 kg height 164 cm daily intake 1800 kcal what is the Nursing
diagnosis?
A. Nutritional imbalance less than body requirement ✅
B. Altered parent-infant relationship
C. Ineffective breastfeeding Answer: A

ِ‫يِاْلَ أرض‬
‫أ‬ ُ ‫َم‬
ُ ‫ك‬
‫ثِِف‬ َ ‫اس‬
‫ِِفي أ‬َ َّ ‫ع‬
‫ِالن‬ ُ ‫ف‬
َ ‫ماِيَ أن‬ َّ َ‫وأ‬
َ ِ‫ما‬ َ
{ 129 }

1119) Which goal is an aim of public surveillance?


A. To rapidly detect the introduction and early cases of a pandemic disease ✅
B. To serve as an early warning system to detect increases in illness in the community
C. To monitor a pandemic's impact on health
D. To track trends in community disease activity and identify populations that are severely affected
Answer: A
1120) Why would a hospital use surveillance?
a. To protect the public against isolated patients
b. To eliminate pathogens from the environment
c. To improve quality of care and outcomes ✅
d. To decrease the incidence of ventilator-acquired pneumonia Answer: C
1121) What is the position during Chemotherapy session?
A. Supine
B. Semi fowler ✅
Answer: B
1122) Elderly patient is receiving chemotherapy. The next day while he is having the dinner, He can't
complete eating. What is the most appropriate cause?
A.. Fatigue B.. Altered sensation related to inflamation in gum ✅ Answer: B

1123) The nurse is giving health education about cooper iud disadvantages. Which of the following is consider
disadvantage for CIUD ?
A. Cause Iron deficiency anemia
B. Short term cost
C. Painful Insertion ✅
Answer: C
1124) The nurse Assess Neonat after delivery using Apgar score.
Sponteanous respiration
Prompt respond
Limitted cry
98 pulse
Pinkish body color execpt hands
What is the expected score?
A. 8
B. 7 ✅
C. 6
D. 5
Answer: B
1125) The comatosed patient with NGT for feeding and positioned in low fowler. The nurse enter the patient room.
She found the patient in supine position. She auscultated lung sound with diventure. What is the most appropriate
nursing diagnosis?
A. Risk for injury B. Risk for aspiration due to NGT ✅ Answer: B
1126) Pregnant 33 week came to ER with sudden vaginal beeding and soft abdomen painless. What is the
most appropriate Diagnosis?
A. Threatened Abortion
B. Abruptio placenta C. Placenta previa ✅ Answer: C

ِ‫يِاْلَ أرض‬
‫أ‬ ُ ‫َم‬
ُ ‫ك‬
‫ثِِف‬ َ ‫اس‬
‫ِِفي أ‬َ َّ ‫ع‬
‫ِالن‬ ُ ‫ف‬
َ ‫ماِيَ أن‬ َّ َ‫وأ‬
َ ِ‫ما‬ َ
{ 130 }

1127) The patient has fever, nuasea, vomiting and rebound tenderness. He is complaining diarrhea with blood
and abdominal pain. What is the most appropriate Diagnosis?
A. Lower right pain
B. Appendicitis
C. Gastroenteritis
D. Devirticulitis ✅Answer: D
1128) 17 year old mother after normal delivery have anemia and bleeding
which of the following education the nurse do?
a. Mass media
b. Group education
c. Individual education ✅ Answer: c
1129) A newborn baby is being evaluated on the APGAR score immediately after his birth. The score
measures the quality of newborn's pulse rate, reflexes, muscles tone and respiration. His APGAR score is
graded as 7 out 10. What is the significance of measuring the APGAR score immediately after the birth?
A. It helps to plan the treatment for congenital diseases
B. It serves as a permanent record for the newborn babies
C. It provides the bases for the comparison, a few minutes later ✅
D. It helps identifying the abnormalities related to muscular tone Answer: c
1130) o As per of a neurological assessment, which of the following is associated with the higher score on the
Glasgow coma scale?
A. Eye opening to pain, no verbalization.
B. Confused, obey command. ✅
C. Localized pain, abnormal extension.
D. Eye opening to speech, confused Answer: B

1131) Newly nurse checked v/s for postoperative pt 2 days ago. The patient had high blood pressure. What
should the nurse do next ?
A. Check medication list if contain anti hypertensive
B. Check patient medical record ✅
Answer: B
1132 ) Which of the following APGAR score is consider moderate risk at the first minutes and after 5 minutes
of assessment?
A. 3-4 B. 3-5
C. 5-7 ✅ D. 8-10 Answer: C
1133) Patient came to ER breath rapidly. What is the first action?
A. Notify the doctor B. Adminster oxygen
C. Assess oxygen saturation ✅ Answer: C

1134) Pregnant woman 12 week after motor accident. She came to ER with vaginal bleeding. What is the
most appropriate Diagnosis?
A-Placenta previa B-inevitable abortion ✅
C. Abruptio placenta Answer: B

1135)Woman come to follow up in the third week after delivery ...nurse observe that uterus is slight palpable
Perineal pad full soaked with blood ...Bp 100/60 temperature 39 what sould nurse anticipate?
A. Perineal laceration B. Retained placental fragment ✅ Answer: B

ِ‫يِاْلَ أرض‬
‫أ‬ ُ ‫َم‬
ُ ‫ك‬
‫ثِِف‬ َ ‫اس‬
‫ِِفي أ‬َ َّ ‫ع‬
‫ِالن‬ ُ ‫ف‬
َ ‫ماِيَ أن‬ َّ َ‫وأ‬
َ ِ‫ما‬ َ
{ 131 }

1136)Pregnant woman in labor. She has sever abdominal pain extend to symphysis pubis. Contraction
frequency every 3 minutes and Duration 40 - 50 seconds. Cervical dilatation 3 cm. What is the most
appropriate action?
A. Encourage her to walk ✅ B. Give nitro oxide
C. Give analgesic Answer: A

1137) Mother with 11 month infant is asking for wrong action for her baby. Which of the following is
consider risk for him?
A. Giving him Cow milk ✅ B. Using Rear face car seat Answer: A
1138) Psychiatric patient with depression. The doctor order antidepressant with low side effect. Which of the
following is the most appropriate type of antidepressant?
A. Anti depressant nor Adrenaline B. Monoamine oxidase inhibitors (MAOIs)
C. Selective serotonin reuptake inhibitors (SSRIs) ✅ Answer: C

1139) Nurse manager provide evaluation for all staff. How can we detect that she acheive good feed back
during evaluation?
A. Apply with all staff same criteria during evaluation ✅ Answer: A

1140)Patient hospitalized for surgical procedure. The nurse informed the patient that she will be discharged.
On the next shift another nurse received her. The nurse has discussed with patient and family and conflict is
happened. What is the most appropriate action?
A. Check the patient file ✅
B. Call supervisor inform her about previous nurse action Answer: A

1141)Patient came to ER after motor accident with hip fracture. What is the first important sign should be
concerned?
A. Hematurea B. Hypotension ✅ Answer: B
1142)When giving analgesic for pregnant mother during labor. What is the most important consideration
should the nurse taken?
A. Mother B. Fetus
C. Contraction D. It will affect and cause prolonged labor ✅ Answer: D

1143) Child diagnosed with appendicitis. He will be have appendectomy. He was anxious and crying. The
nurse was asking mother to calm him. His mother do behavior that indicates there is misunderstanding for
the procedure. Which of the following statement should the nurse correct to wrong behavior for the mother?
A. You should not cry. You are older.
B. Because You will be punished, you have the surgery ✅
C. Explain to child pre-operative and postoperative procedure Answer: B
1144)The nurse stated to Psychiatric patient " I will stay here with you". Which of the following therapeutic
communication thechniques that the nurse used.?
A. Accepting B. Exploring
C. Offering self ✅ D. Reflecting Answer: C

1145)New manager promoted to position. Which of the following is the most appropriate action?
A. Assess patient care B. Interview for new staff ✅

ِ‫يِاْلَ أرض‬
‫أ‬ ُ ‫َم‬
ُ ‫ك‬
‫ثِِف‬ َ ‫اس‬
‫ِِفي أ‬َ َّ ‫ع‬
‫ِالن‬ ُ ‫ف‬
َ ‫ماِيَ أن‬ َّ َ‫وأ‬
َ ِ‫ما‬ َ
{ 132 }

C. Ask help from head nurse Answer: B


1146) Alzheimer's patient high risk for Fall and admitted with fractures hip during hospitalization the patient
fell twice. What should the nurse do?
A. Continue same plan B. Impair skin integrity
C. Risk for pressure ulcer D. Non compliance add new nursing diagnosis ✔✔ Answer: D

1147) The nurse is preparing patient with knee replacement surgery. During physical examination for patient,
The nurse observed abdominal pulsating mass in epigastria. The patient with high blood pressure. What is the
first action for nurse?
A. Continue in surgery preparation B. Make abdominal Ultrasound ✅ Answer: B

1148) Patient came to ER with lower back pain and numbness of leg: Multiple sclerosis.
1149)Patient with vital signs: Blood pressure 100/70., T 40 °c, HR 89, Hgb 11.7 g/dl. What is the first action
for the nurse?
A. Corticosteroid for inflammation B. Antibiotics for infection ✅
C. I. V fluids for blood pressure D. Blood transfusion for Hgb Answer: B
1150) What is the initial sign should the nurse Assess indicates that ICP for patient not increased?
A. Patient tolerate lower head of bed B. Normal blood pressure
C. Absence of confusion and restlessness ✅ Answer: C
1151) a Patient takes a full course of treatment Levofloxacin for pneumonia what confirmed that treatment
was effective?
A-decrease haemoglobin level B- decrease wbc count ✅
C-low platelets count D- high hematocrite Answer: B
1152) What is the difference between Transational stool and meconium?

ِ‫يِاْلَ أرض‬
‫أ‬ ُ ‫َم‬
ُ ‫ك‬
‫ثِِف‬ َ ‫اس‬
‫ِِفي أ‬َ َّ ‫ع‬
‫ِالن‬ ُ ‫ف‬
َ ‫ماِيَ أن‬ َّ َ‫وأ‬
َ ِ‫ما‬ َ
{ 133 }

1153 ) Which of the following category for CTG?


A. Normal
B. Category I
C. Category II
D. Category III ✅
Answer: D
1154) Patient with somatization. The patient will experiencing which of the following?
A. With clinical symptoms ✅
B. No clinical symptoms
Answer: A
1155)Girl diagnosed with somatization. What should the nurse expect for this patient high risk for ?
A. Anorexia nervosa
B. Anxiety disorder ✅
Answer: B
1156)Community health nurse visit diabetic patient. One of family member complaining for her that patient
unmoved. what should the nurse do?
A. Forced the patient to move
B. Walk with the patient and take round
C. Ask patient express feeling and the cause ✅
Answer: C
1157)The nurse found the patient fell down on the floor. After she helped the patient to get the bed. Do assessment
and the patient is good. What is the next first action?
A. Write incident report
B. Set bed alarm ✅
C. Call the Witness client to be side the patient
D. Inform the manager
Answer: B
1158)The nurse research ER need data for study. She called to another nurse in the ward to ask about Information for
DM patient. What should the nurse response ?
A. Give her information
B. This my patient not give you information
C. No problem to give you information but not on the phone
D. Ask patient to get informed consent to use this information ✅
Answer: D
1159)Physician came to the ward and he checked patient medical record. After finishing the nurse find that the
physician not related to the patient. What should the nurse do?
A. Nothing to do that is patient and normal
B. Call patient to clarify for reasons and inform the patient
C. Inform nurse supervisor ✅
Answer: C
1160)What is chest assessment for patient with Vitamin D deficiency ?
A. Dull sound
B. Rosary ✅
C. Crackle
D. Chest deformity Answer: B
1161)Pregnant woman had an accident....and come to ER with brown bleeding...abdominal
distension...cramps. In abdomen fetaus heart rate not heard
A. Inevitable
B. Threatened
C. Abruptio placenta Answer: A

ِ‫يِاْلَ أرض‬
‫أ‬ ُ ‫َم‬
ُ ‫ك‬
‫ثِِف‬ َ ‫اس‬
‫ِِفي أ‬َ َّ ‫ع‬
‫ِالن‬ ُ ‫ف‬
َ ‫ماِيَ أن‬ َّ َ‫وأ‬
َ ِ‫ما‬ َ
{ 134 }

1162) Unit manager conducts a 6-month performance review session with a staff member. Which of the
following actions is appropriate?
A. She asks another nurse to attest the session as a witness.
B. She informs the staff that she may ask another nurse to read the appraisal before the session is over.
C. She tells the staff that the session is manager-centered.
D. The session is private between the two members. ✅ Answer: D
1163) Nurse enter patient room ...found him crying ...she asked him question about why he crying ....he didn't
respond...?
A. Get out of room to maintain privacy
B. Sit with him and not say anything ✔️
C. Say don't cry Answer: B
1164)What is the most common complication during TPN insertion?
A. Rebound hypoglycemia
B. Air emoblism
C. Pneumothorax ✅ Answer: C

0101-8:‫تجميعات شهر أغسطس‬


1165. The nurse is assessing a 65-year-old patient, who reports the fatigue, Weight loss, night
sweats, and a productive cough with thick sputum The nurse should immediately initiate isolation
precautions for which of the following?
A. Influenza
B. Pertussis
C. Bacterial pneumonia
D. Pulmonary tuberculosis ✅ Answer : D

1166. A nurse researcher wrote in her explanatory statement: "The researcher will -35 Use a
questionnaire to measure nursing job satisfaction," which ethics principle is the researcher
addressing here?
A. Justice ✅
B. Beneficence
C. Confidentiality
D. Non – maleficencge Answer :" A

1167. A patient fell in bathroom and his left leg was fractured, in order to communicate
information about the patient to next shift.Which of the following documentation should be used
by the nurse at the end of the shift?
A.Kardex record✅
B.Assignment record
C.Shift report
D.Incident report Answer : A

1168. Nurse community visit school students what the most important topic to focus on.

ِ‫يِاْلَ أرض‬
‫أ‬ ُ ‫َم‬
ُ ‫ك‬
‫ثِِف‬ َ ‫اس‬
‫ِِفي أ‬َ َّ ‫ع‬
‫ِالن‬ ُ ‫ف‬
َ ‫ماِيَ أن‬ َّ َ‫وأ‬
َ ِ‫ما‬ َ
{ 135 }

A. Sport safety
B. Bicycle safety✅
C. Road safety Answer : B

1169. Multipara postpartum woman was experiencing hemorrhage. Which of the following
complication most common for her?
A. Infection✅
B. DVT
Answer : A

1170. Which of the following should the midwife do for her episiotomy?
A. Mother her fetus with posterior position
B. Delayed Second stage of labor Answer :B

1171. Patient with yellowish sputum. What is expected lung sound?


A. Crackle✅
B. Wheezing
C. Bronchial Answer : A

1172. The nurse is providing health education for parent has child after shunt. What is the
teaching should included to family ?
A. Increase protein diet
B. Restrict activity ✅
C. Isolate Child From Others Answer : B

1173. Patient has AV fistula in his leg. The patient came to dialysis unit for session. When the nurse
check the fistula, She found that fistula not working. What is the first action?
A. Auscultate the fistula
B. Notify the physician✅
C. Monitor vital signs Answer : B

1174. A child is treated for bacterial meningitis with an intravenous antimicrobial agent. Which of
the following BEST indicates effectiveness of the treatment?
A. Increased appetite
B. Low Temperature ✅
C. Decrease pulse
D. Nodul of skin disappear Answer : B

1175. A 3 year is brought by the mother to the emergency Department with fever diarrhea and
vomiting. She passed four loose motions and three vomiting the last 24 hours , she is anorexic,
irritable , has dry lips and moderate skin turgor. She is given ORS to drink, but she refused it after
the first stip. O2 sat 96 HR 36 TEM 38.8 What is the immediate nursing intervention is required to
encourage the baby to drink the ORS?
A. Give In a cup once cold a day
B. Help her to drink with a syringe
C. Give in a small amount frequently ✅

ِ‫يِاْلَ أرض‬
‫أ‬ ُ ‫َم‬
ُ ‫ك‬
‫ثِِف‬ َ ‫اس‬
‫ِِفي أ‬َ َّ ‫ع‬
‫ِالن‬ ُ ‫ف‬
َ ‫ماِيَ أن‬ َّ َ‫وأ‬
َ ِ‫ما‬ َ
{ 136 }

D. Engage in playing and help to her drink it Answer : C

1176. When administering medication to a four year-old child, which of the following actions
should a nurse take?
A. Ask the child to take the medication
B. Indicate to the child a firm expectation to take the medication
C. Mix the medication with a large amount of the child's favourite food or
drink ✅
D. Inform the child that an injection will be necessary, if the child refuses to
take oral medication Answer : C

1177. A 7-year-old insulin dependent diabetic mother has delivered normally in 38 gestational
weeks. The nurse was assessing the insulin requirement this mother after delivery. What is the
insulin requirement for this patient?
A. Higher than before pregnancy
B. No changes in insulin requirement
C. Lower than when she was pregnant ✅
D. Slightly increased than before deliver Answer : C

1178. 30 years old women absent her menstruation for 5 months and her menstrual cycle come
every 28 days, she controlled her diet and do heavy exercise this
women condition?
A. Pregnancy
B. Primary amenorrhea
C. Secondary amenorrhea ✅ Answer : C

1189. The doctor ordered medication 5000 mg. The nurse deluted in 250 ml of D5N5 and modified
dose on infusion pump to 20mg/hr. How many ml should the nurse give per hour?
A. 5 B. 10
C. 15 D. 20 ✅ Answer : D

1190. 572 A 26 years old married woman is admitted in the plastic surgery for the correction of
burn strictures and skin g rafting on her neck andface under general anesthesia. While discussing
the treatment with her, the plastic surgeon explained that she will have a series of surgeries but
she needs to be on family planning until the treatment is completed. the patient aske d the nurse
whether she will be normal again. What initial assessment is required?
A. Detailed history and physical examination
B. Patient’s acceptance for the treatment plan
C. Need for psychological support to reduce anxiety ✅
D. Family’s involvement and consent for her treatment .
Answer: C

ِ‫يِاْلَ أرض‬
‫أ‬ ُ ‫َم‬
ُ ‫ك‬
‫ثِِف‬ َ ‫اس‬
‫ِِفي أ‬َ َّ ‫ع‬
‫ِالن‬ ُ ‫ف‬
َ ‫ماِيَ أن‬ َّ َ‫وأ‬
َ ِ‫ما‬ َ
{ 137 }

1191. What is the name of procedure?


A. Myringoplasty
B. Tympanoplasty ✅
C. Tympanotomy Answer : B

1192. Interdisciplinary client care rounds and hand-off communication are examples of
strategies used to improve communication in health care settings.What is the most important
outcome of effective communication among caregivers?
A. Decreased length of hospital stay
B. Less obvious needs of clients met accordingly
C. Properly educated clients
D. Reduced number of medical errors✅ Answer : D

1193. Which issue would a unit quality improvement committee address?


A. A 10% decrease in client satisfaction in the registration process
B. A nurse who made 3 medication errors in the past quarter
C. An increase in catheter-associated urinary tract infections ✅
D. Staff perception of hospital laboratory personnel incivility Answer : C

1194. 70 years- old woman presents with increasing dyspnea on exertion. She feels breathless
and restless while performing household tasks, such as making the bed and sweeping the floor.
Her previous medical history includes a myocardial infarction at 57 years old. She sleeps with her
head elevated on three pillows. Examination reveals bilateral basal crackles and cold, damp
skin.
Blood pressure 172/94 mmhg Heart rate 94/min Respiratory rate 36/min
Temperature 37.1 C O2 saturation 90% on room air
Which heart chamber most likely failed first?
a- Right atrial
b- Right ventricle
c- Left ventricle ✅
d- Left atrial Answer : C

1195. Which type of study design provides the strongest evidence?


A. Qualitative study

ِ‫يِاْلَ أرض‬
‫أ‬ ُ ‫َم‬
ُ ‫ك‬
‫ثِِف‬ َ ‫اس‬
‫ِِفي أ‬َ َّ ‫ع‬
‫ِالن‬ ُ ‫ف‬
َ ‫ماِيَ أن‬ َّ َ‫وأ‬
َ ِ‫ما‬ َ
{ 138 }

B. Randomized control trial ✅


C. Systematic review of descriptive studies
D. Systematic review of correlational studies Answer: B

1196. The LPN is preparing to administer an injection of vitamin K to the newborn. The nurse
should administer the injection in the:
A. Dorsogluteal muscle.
B. Rectus femoris muscle.
C. Vastus lateralis muscle. ✅
D. Deltoid muscle. Answer: C

1197. When administering an intramuscular injection to an infant, which of the following sites
appropriate for the nurse to use?
a. Rectus femoris ✅
b. Deltoid
c. Dorsogluteal
d. Ventrogluteal Answer : A

1198. A hospitalized 72-years-old man who uses a walker is received medication and must use
the bathroom several times each night. To promote the safety of the patient, which of the
following appropriate nursing action?
A. Keep the side rails up
B. Leave the bathroom light on
C. Provide a bedside commode ✅
D. Withhold the patient’s diuretic medication Answer :C

1199. When the bag of water ruptures, the nurse should watch for which of the following risks?
A. Fetal distress
B. Cord prolapse✅
C. Respiratory distress
D. Bleeding Answer : B

1200. A nurse in the labor room is performing a vaginal assessment on a pregnant client in labor.
The nurse notes the presence of the umbilical cord protruding from the vagina. Which of the
following would be the initial nursing action?
A. Place the client in Trendelenburg’s position ✅
B. Call the delivery room to notify the staff that the client will be
transported immediately
C. Gently push the cord into the vagina
D. Find the closest telephone and stat page the physician Answer : A

1201. A pregnant mother is in the active labour. Her cervix is fully effaced dilated with crowing of
the fetal head. The mother is exhausted, perspiring, and is having increasingly painful and severe
uterine contractions. Which of the following intervention is best ?desired by the assisting midwife

ِ‫يِاْلَ أرض‬
‫أ‬ ُ ‫َم‬
ُ ‫ك‬
‫ثِِف‬ َ ‫اس‬
‫ِِفي أ‬َ َّ ‫ع‬
‫ِالن‬ ُ ‫ف‬
َ ‫ماِيَ أن‬ َّ َ‫وأ‬
َ ِ‫ما‬ َ
{ 139 }

A. exert pressure on abdomen to aid in child birth


B. increase the rate of intravenous medication
C. wipe face with wet towel and hold hands
D. instruct mother to push harder ✅ Answer : D

1202. 42 year-old patient went to the clinic for an eye consult . Patient complained of blurred
vision , ocular pain and head active . During assessment tonometry was done ( see results ) . Test
Result Normal Values Intraocular pressure 34 10-20mmHg . Which surgical procedure is the most
appropriate ?
a. Laminectomy
b. Laser trabeculoplasty ✅
c. Incision and drainage
d. Extra capsular cataract extraction Answer: B

1203. A child with deformity (broken) nose, the child went to the school and his friends find this
funny, the child was upset and went to the nurse in the school and told him, he will stop coming
to school, the nurse tokes a paper and draw the child face and nose and tell him that ‘he will
look like them after the procedure’. The step the nurse perform is called?
A. Self conception
B. Self confidence ✅ \ Self deception ✅
C. Self esteem
Answer : B

1204. . A child with deformity (broken) nose, the child went to the school and his friends find this
funny, the child was upset and went to the nurse in the school and told him, he will stop coming
to school, the nurse tokes a paper and draw the child face and nose and tell him that ‘he will
look like them after the procedure’Which of the following step in Maslow Hiarrechy missed for
child ?
A. Self confidence
B. Self esteem
C. Self deception
Answer : B
1205. Community health nurse is making preparation for crisis plan. What is the most accurate
information to make a disaster plan in community ?
A. Check stored water every year and change it if expired
B. Store bag with 3 gallons of water for each person for 3 days ( 1 gallon
per day for each person). ✅
C. Inform that animals don't need plan. It can survive
D. Prepare bag and Storing of food water and other stuff for 7 days
Answer : B

1206. 3 month Pregnant woman came with her husband to ER with fatigue and confusion. Her
husband inform the nurse that she doesn't have any disease .
The doctor order maternal drug screening test (drug abuse) Test for Her.
What should the nurse do before the Test?
A. Request for blood test

ِ‫يِاْلَ أرض‬
‫أ‬ ُ ‫َم‬
ُ ‫ك‬
‫ثِِف‬ َ ‫اس‬
‫ِِفي أ‬َ َّ ‫ع‬
‫ِالن‬ ُ ‫ف‬
َ ‫ماِيَ أن‬ َّ َ‫وأ‬
َ ِ‫ما‬ َ
{ 140 }

B. Get informed consent from husband ✅


C. Ask for medical history
Answer : B

1207. While the nurse checking patient file to transfer him to operation room. She found that the
doctor didn't get the patient's signature in informed consent. What is this consider?
A. Negligence ✅
B. Malpractice
Answer : A

1208. The doctor was busy. He requested from the nurse to see certain patient. What is this
called?
A. Practitioner
B. Responsibility ✅
Answer: B

1209. The Doctor prescribed 75 ml /hr of 0.9 NS. How much will infused through 12 hour?
A. 1500
B. 900 ✅
C. 1000
Answer: B

1210. What is Signs of ICP?


A. Tachypnia
B. Intermittent tachycardia
C. Restlessness ✅
Answer : C

1211. The patient come to ER with head injury after 1 hr the expected Vital/Signs for patient is?
A. Bp 126/85 RR 12 HR 78
B. Bp 158/92 RR 9 HR 60 ✅
Answer : B

1212. Patient with ABG ph low, pco2 high, hco3 normal what is expected signs andsymptoms for
the patient ?
A. Vomiting and difficult breathing SOB ✅
B. Headache and vomiting
C. Shallow, rapid breath and vomiting
D. Kaussmal respiration Answer: A

1213. What is the development of six months infant?


A. Startle reflex improve
B. Move from prone to supine ✅
C. Get ready to crawling Answer: B

1214. What is the transmission method of HAb virus?

ِ‫يِاْلَ أرض‬
‫أ‬ ُ ‫َم‬
ُ ‫ك‬
‫ثِِف‬ َ ‫اس‬
‫ِِفي أ‬َ َّ ‫ع‬
‫ِالن‬ ُ ‫ف‬
َ ‫ماِيَ أن‬ َّ َ‫وأ‬
َ ِ‫ما‬ َ
{ 141 }

A. Food ✅
B. Blood
C. Sexual transmission Answer: A

1215. Wich of the following Most of the time sign associated with Alzhiemer's
disease?
A. Depressed mood ✅
B. Act out of behavior Answer: A

1216. A pregnant woman has previous preterm labor and she was asking the nurse if sexual
intercourse is safe during her pregnancy?
A. Its safe
B. It’s not safe
C. It’s safe until 3rd trimaster ✅ Answer: C

1217. Which of the following food rich in potasium?


A. Orange ✅
B. Tofu
C. Butter
D. Milk Answer: A

1218. Why Postpartum hemorrhage cause infection?


A. Body not able to defense infection
B. Good media of bacteria ✅
C. Body loss blood component that help for prevent infection Answer: B

1219. There is single parents. One of both has one child. They will married. What is the type of
family?
A. Nuclear
B. Extended
C. Blended ✅ Answer: C
1220. Patient with alzheimer's disease. What is the cause that affect brain function (the defect)?
A. Absent of nerve cells
B. Destruction of neuronal ✅ Answer : B

1221. The hospital managers observed that staff using personal smart phones too much during
work time to get information related job. What should hospital
do?
A. Prevent staff to accompany the phones or bring inside work area
B. Provide staff with another phones from the hospital
C. Set new policy to reduce staff using smart phones ✅ Answer: C

1222. 72 year-old patient bedridden. His neighbour found that patient has loss of conciousness
and he brought him to hospital. The nurse provide long term nursing care plan for patient. What is
the most important consideration?

ِ‫يِاْلَ أرض‬
‫أ‬ ُ ‫َم‬
ُ ‫ك‬
‫ثِِف‬ َ ‫اس‬
‫ِِفي أ‬َ َّ ‫ع‬
‫ِالن‬ ُ ‫ف‬
َ ‫ماِيَ أن‬ َّ َ‫وأ‬
َ ِ‫ما‬ َ
{ 142 }

A. Previous injuries on his body ✅


B. Identify Other medical illness
C. Bedridden status Answer: A

1223. Patient scheduled to perform venacava filter surgery. The doctor ordered to stop
medication before one day of operation for preoperative preparation. Which of the following
drug should be stopped?
A. Warfarin ✅
B. Sodium bicarb
C. Potassium Answer: A

1224. What is the impairement definition?


A. Social isolation
B. Difficulties in executing activities
C. Difficulties in body functioning such as exetremity loss ✅ Answer: C

1225. 72 year-old patient bedriddend. He is complaining sever abdominal pain and suffering
from loss of skin integrity. What is the most appropriate Diagnosis?
A. Addison's crisis ✅
B. Thyrotoxicosis
C. Cushing syndrome Answer: A

1226. A 44 years year old obese patient was subjected for a surgery called bariatric surgery,
gastric bypass to be specific. The nurse and the physician are giving information about the
procedure to the patient. The patient understands the procedure if he stated which among the
following?
A. ''Same effect with liposuction''
B. ''An opening will be placed in my abdomen''
C. ''The surgery will reduced the size of my stomach'' ✅
D. ''Easiest way to lose weight no need to exercise after surgery'' Answer: C

1227. What is the goal for giving Corticosteroids in Asthma?


A. Bronchodilator
B. Decrease airway swelling ✅
C. Chest clear and Remove secretions
D. Vasodilation Answer: B

1228. A primigravida mother is having her baby through normal vaginal delivery. The baby is
completely delivered but the mother is still experiencing the uterine contractions.
A. Beginning of the third stage of labour ✅
B. Indication of increase in vaginal bleeding
C. Need for reducing the rate of intravenous oxytocin
D. Uterine contraction will gradually reduce then stop. Answer: A

ِ‫يِاْلَ أرض‬
‫أ‬ ُ ‫َم‬
ُ ‫ك‬
‫ثِِف‬ َ ‫اس‬
‫ِِفي أ‬َ َّ ‫ع‬
‫ِالن‬ ُ ‫ف‬
َ ‫ماِيَ أن‬ َّ َ‫وأ‬
َ ِ‫ما‬ َ
{ 143 }

1229. Patient admitted to inpatient ward diagnosed with schizophrenia. What is the first
intervention for patient?
A. Observe patient behavior ✅
B. Give health education about medication Answer: A

1230. A pregnant woman visits the Outpatient clinic complaining of excessive vaginal secretion.
Which of the following is the appropriate nursing assessment?
A. Fetal heart rate
B. Fundal height
C. Signs of infection of labor ✅
D. Fetal presentations and position Answer: C

1231. The mother and her husband are sickle cell anemia carrier. What is the child percentage to
be carrier?
A. 25 %
B. 50 % ✅
C. 75%
D. 100 % Answer: B

1232. Patient during intraoperative period while administering anesethia. The patient complained
from reaction. The vital signs Bl. P 110/70 mmhg, HR 140 b/m, RR 24 b/m. Which of the following
drug should the nurse administer?
A. I. M epinephrine ✅
B. I. V atropine Answer: A

1233. The nurse is providing Session about ( disease process, coping, dietary management) for
patient. Which is the initial important long term goal?
A. Increase understanding of disease process
B. Have a healthy life style ✅
C. Increase coping
D. Dietary management Answer: B

1234. Patient with expressive aphasia has impaired communication What is the short term goal
for him to increase verbalization?
A. Able to communicate effectively within a week
B. Anticipate all patients needs
C. Able to determine basic needs ✅ Answer: C

1235. a nurse is caring for a patient who had a cerebrovascular accident 30 minutes ago with
residual right-side hemiparesis. The nurse places a trochanter roll next ending from the patient's
iliac crest to the mid. What is the most likely
complication?
A. Adduction of the leg
B. External rotation of the hip
C. Muscle spasms in the thigh ✅

ِ‫يِاْلَ أرض‬
‫أ‬ ُ ‫َم‬
ُ ‫ك‬
‫ثِِف‬ َ ‫اس‬
‫ِِفي أ‬َ َّ ‫ع‬
‫ِالن‬ ُ ‫ف‬
َ ‫ماِيَ أن‬ َّ َ‫وأ‬
َ ِ‫ما‬ َ
{ 144 }

D. Flexion contractures of the knee Answer: C

1236. The patient has exhausted, fatigue due to severe pain the level 6 /10, temp 37,5. What is
the most appropriate Short term goals?
A. Alleviate exhaustion
B. Manage pain and decrease temperature ✅ Answer: B

1237. Patient complained from Loss of consciousness, The patient diagnosed with meningitis.
What's the first action for the patient
A. Neurological assessment ✅
B. Observe for seizures Answer: A

1238. The patient ate around 20% of meal. He can not compelete food. He was experiencing Too
tired after eating. What should the nurse do?
A. Encourage fluids intakes
B. Offer food after rest ✅
C. Encourage him to take the protein part of diet Answer: B

1239. Head nurse want to increase size of staff next year , what is the first thing done by human
resources?
A. Downsizing
B. Recruitment ✅ Answer: B

1240. Patient postoperative after hip surgery. What's the realistic goal should the nurse
accomplish ?
A. Pain control within one hour of medication
B. Sitting three times per day ✅
C. Able to drive a car Answer: B

1241. What is COPD position?


A. Semi fowler
B. Supine
C. Prone
D. High fowler ✅ Answer:D

1242. What is the action /function of Ringer Lactate?


A. Increase intracellular fluids
B. Decrease intracellular fluids
C. Expend extracellular fluids ✅
D. Decrease extracellular fluids Answer: C

1243. Patient ABG PH 7.38, Paco2 50, Hco3 6 normal. What's the ABGinterpretation?
A. Compensated Metabolic alkalosis
B. Uncompensated Metabolic acidosis
C. Compensated Respiratory acidosis ✅
D. Uncompensated Respiratory alkalosis Answer: C

ِ‫يِاْلَ أرض‬
‫أ‬ ُ ‫َم‬
ُ ‫ك‬
‫ثِِف‬ َ ‫اس‬
‫ِِفي أ‬َ َّ ‫ع‬
‫ِالن‬ ُ ‫ف‬
َ ‫ماِيَ أن‬ َّ َ‫وأ‬
َ ِ‫ما‬ َ
{ 145 }

1244. Woman came to outpatient for follow up. She complained from dizzness and fatigue. The
nurse refered her to relax inside empty room. She suddenly fell down on the floor then she has
faintaing and loss of conciousness. The nurse examined her and assessed vital signs. Bl. P 90/50
mmhg , HR 120 b/m, RR 23 b/m.What is the first intervention for her?
A. Notify the doctor
B. Elevate her leg ✅
C. Check Physical assessment and vital signs Answer: B

1245. The nurse has been set priorities for health problem. She is starting to management of time
board. Which of the following step of nursing process?
A. Planning✅
B. Diagnosis
C. Intervention
D. Assessment Answer: A

1246. A 57 year-old woman presents with complaints of pain in the bones of her toes, foot and
swelling on knees . What type of musculoskeletal disorder is most likely?
A. Osteoarthritis.
B. Rheumatic arthritis
C. Osteomyelitis
D. Gout ✅ Answer: D

1247. The nurse examined Pregnant woman for the five factors affecting labor process. The five Ps
(passenger, passage, powers, placenta, and.......? Which of the following additional factor?
A. Pathway
B. Psychology ✅
C. Process Answer: B

1248. The psychaitric patient is receiving Benzodiazepine drug. The nurse should instruct patient
that drug affected by which of the following?
A. Alcohol ✅
B. Tobacco
C. Nicotine
D. Caffiene Answer: A

1249. The patient is experiencing burn 20% TBSA. Which of the following hormone affected?
A. Ranin
B. Cortisol ✅
C. Anti diuretic Answer: B

1250. Patient diagnosed with nephrotic syndrome. See lab result Hgb 12. 5 g/dl What is the most
common complication for nephrotic syndrome?
A. UTI
B. Peritonitis ✅

ِ‫يِاْلَ أرض‬
‫أ‬ ُ ‫َم‬
ُ ‫ك‬
‫ثِِف‬ َ ‫اس‬
‫ِِفي أ‬َ َّ ‫ع‬
‫ِالن‬ ُ ‫ف‬
َ ‫ماِيَ أن‬ َّ َ‫وأ‬
َ ِ‫ما‬ َ
{ 146 }

C. Bladder infection
D. Hypertension encephalopathy Answer: B

1251. The patient has refused to take the medication, what the nurse will be response?
A. Give me a good reason for not taking the medication
B. Tell me why you don't want to take the medication
C. Convinced me why you shouldn't take the mediction
D. If you will not take the medication, you will sign a waiver ✅ Answer: D

1252. What is the expected symptoms for uterine fibroid?


A. Irregular menstruation
B. Back pain✅
C. Pressure in pelvic Answer: B

1253. Who will teach about car /road safety ?


A. Doctor
B. Nurse✅
C. Social worker
D. Administrator Answer: B

1254. 3 months baby diagnosed with hirschsprung disease what is the disease?
a- Symptoms occur after 6 months
b- affect both small and large intestine
c- absences of prestalisis movement in the distal part of large intestine /colon ✅
d- telescoping of the intestine Answer: C

1255. Girl come hospital with thalassemia . She is given blood transfusion every 3 weeks. She
came to hospital for blood transfusion and the doctor ordered lab investigation before
administeration, He found on results increase ferritin level. What should the nurse give for her to
prevent complication?
A. deferasirox ✅
B. Iron supplement
C. Sodium choloride Answer: A

1256. Postpartum Client called to midwife after 10 days of discharge. She told her that lochia
become before 3 days brown. What is the most appropriate action?
A. Informe her to return hospital to reassesment
B. Tell her that’s normal and lochia changes in color from day to day ✅
C. Ask her about if she breastfeed her baby Answer: B

1257. What is the side effects of isoniazid?


A. Urecemia
B. Photosensitivity ✅
C. Nerve inflammation Answer: B

ِ‫يِاْلَ أرض‬
‫أ‬ ُ ‫َم‬
ُ ‫ك‬
‫ثِِف‬ َ ‫اس‬
‫ِِفي أ‬َ َّ ‫ع‬
‫ِالن‬ ُ ‫ف‬
َ ‫ماِيَ أن‬ َّ َ‫وأ‬
َ ِ‫ما‬ َ
{ 147 }

1258. A client with cold flu . He is suspected of having systemic lupus erythematous. The nurse
monitors the client, knowing that which of the following is one of the initial characteristic sign of
systemic lupus erythematous?
A. Weight gain
B. Fever, malaise
C. Elevated red blood cell count
D. Rash on exetremties and the face ✅ Answer: D

1259. The nurse is giving health education for mother about oral contraceptive pills. The mother
asked the nurse about side effects. What should the nurse response?
A. Don't ask
B. Do you want to clarify about something
C. Nausea, water retention and weight gain ✅ Answer: C

1260. Neonate in NICU with spina bifida. What is intervention regarding feeding ?
A. Give feeding at scheduled time ✅
B. Limit feeding for neonate
C. Stop feeding. When baby feel back pain during feeding Answer: A

1261. Patient preoperative take midazolam hydrochloride. The patient become restless and
suffered from respiratory distress and confusion. What should the nurse and surgical team do?
A. Defibrillate patient
B. Administration antidote (activated characol) ✅
C. Administration epinephrine
D. Ventilation patient with oxygen therapy Answer: B

1262. Postoperative patient is complaining pain. The pain level is 8 from 10. The nurse is busy.
What's the task should the nurse delegate for assistant nurse?
A. Determine pain level
B. Assess pain while checking vitals
C. Give anelgesic
D. Teach patient breathing exercise to releive pain ✅ Answer: D

1263. The Vitro fertlization is related to (indications) ?


A. Blockage of fallobian tube ✅
B. Donner of sperm
C. Absence of spearm
D. imnunolergic hormone Answer: A

1264. The patient is postoperative and sedated. Which of the following signs require immidiate
intervention?
A. Urine bag is full
B. Hemovac bag is full with serosanguineous drainage
C. Intermediate nasogastric suction is not connected ✅
D. Nasal cannula 4L without humidification Answer: C

ِ‫يِاْلَ أرض‬
‫أ‬ ُ ‫َم‬
ُ ‫ك‬
‫ثِِف‬ َ ‫اس‬
‫ِِفي أ‬َ َّ ‫ع‬
‫ِالن‬ ُ ‫ف‬
َ ‫ماِيَ أن‬ َّ َ‫وأ‬
َ ِ‫ما‬ َ
{ 148 }

1265. Pregnant woman in active phase with cervical dilitation 3 cm. The midwife induced labor
by amniotomy. What is the next action?
A. insertion catheter to empty bladder
B. Assess contraction for full minute
C. Assess FHR at least full minute ✅Answer: D

1266. For the baby immediately after head delivery to expect good crying . What should the
nurse provide ?
A. Endotrachial tube
B. Suction secretion from nose and mouth✅
C. Warm bath
D. Slap baby on his buttocks Answer: B

1267. Postpartum woman came to ER and complaining from fatigue . She told nurse that "I am so
exhausted, I am so worried, I am hoppeless and I can't do anything. The mother diagnosed with
postpartum blues . What is the next question The nurse should ask her ?
A. Are you blame yourself you are not be coping to motherhood ? ✅
B. Is there a family member you can talk with him?
C. Are you have any thoughts about harm yourself or harm your baby Answer: B

1268. Postpartum Mother 2 days after delivery. Her baby Blood collection from heel is done to
PKU. She has appointment next day for circumcission. What is the most important attention for
circumcission?
A. Avoid tighted daipper ✅
B. Notify first day of first voiding urination Answer: A

1269. Child complaining Wheezing, crackle and nasal flaring. The doctor ordered Pulmicort 0.5
mg and 1 mg salbutamol. What is the most appropriate cause or What should the nurse expect
diagnosis ?
A. Viral bronchitis ✅
B. Asthma
C. Rhinitis
D. Because his parents are smoking Answer: A

1270. Pregnant woman came to ER complain with sever pain in leg. When the nurse assessed her,
the nurse found that woman has varicose veins. What should the nurse do?
A. Instruct her to Put pillow behind back while setting
B. Instruct her to put pillow under head and elevate the shoulder
C. Instruct her to wear elastic stocking with suitable size ✅ Answer: C

1271.Woman diagnosed with breast cancer and she will be performed mastectomy. She asked
the nurse to not inform anyone except her husband. Which of the following ethical principle?
A. Confidentiality ✅
B. Privacy
C. Autonomy

ِ‫يِاْلَ أرض‬
‫أ‬ ُ ‫َم‬
ُ ‫ك‬
‫ثِِف‬ َ ‫اس‬
‫ِِفي أ‬َ َّ ‫ع‬
‫ِالن‬ ُ ‫ف‬
َ ‫ماِيَ أن‬ َّ َ‫وأ‬
َ ِ‫ما‬ َ
{ 149 }

D. Justice Answer: A.

1272.A patient treated an insect bite with traditional alternative therapy of row garlic juice
caused strong skin reaction. The patient agreed not use it again after the nurse explained its
harmful effects. What should the nurse instruct him on discharge?
A. Refrain self treatment ✅
B. Medical treatment better than traditional
C. Avoid communication injury
D. Change dressing frequently\ Answer: A

1273.Soldier was brought to triage area after being exposed to chemical weapons. Signs and
symptoms of nerve gas exposure were noticed. A nurse prepares for medical management.
Which medication should the nurse prepare for the patient?
A. Atropine ✅
B. Adrenaline
C. Sodium nitrate
D. Sodium thiosulphate Answer: A

1274.During a night shift a medical doctor complains of back pain and asks the t nurse to give
him morphine 5 mg IM. Which of the following actions indicates professionalism in handing the
ation by the nurse?
A. Call another doctor to manage
B. Refer him to Emergency Room ✅
C. Administer morphine to doctor
D. Ask him to write a prescription first Answer: B

1275. A mother is the outpatient Clinic for her first post natal visit on the 15th day her normal
vaginal delivery. Her physical examination reveals a stable condition, breasts are soft and her
sanitary napkin has bright coloured rubra. Which of the following needs further evaluation?
A. Amount and frequency of breast feeding
B. Hydration level and bleeding breast feeding
C. Activity, exercise and resting periods
D. Uterine size and position ✅ Answer: D

1276. Which of the following measures can reduce or prevent the incidence of atelectasis in a
post-operative client?
A. Chest physiotherapy
B. Mechanical ventilation
C. Reducing oxygen requirements
D. Use of an incentive spirometer ✅ Answer: D

1277.Most recommended position for a client with severe ascites?


A Side lying.
B.Sim’s.
C.Dorsal.

ِ‫يِاْلَ أرض‬
‫أ‬ ُ ‫َم‬
ُ ‫ك‬
‫ثِِف‬ َ ‫اس‬
‫ِِفي أ‬َ َّ ‫ع‬
‫ِالن‬ ُ ‫ف‬
َ ‫ماِيَ أن‬ َّ َ‫وأ‬
َ ِ‫ما‬ َ
{ 150 }

D.Fowler’s. ✅ Answer: D

1278.Patient with cardiacTemponade diseaes. What is the expected symptoms?


A. Tachypnia, bradycardia and fever
B. Muffled heart sound ✅
C. Cyanosis in extremities Answer: B
1279.What is the point of maximum pulse monitoring while auscultated apical pulse?
A. 5th intercostal space left midclavicular ✅
B. 5th right
C. 4th left
D. 4th right Answer: A

1280.The community health centers want to improve condition for people with disabilities . What
is Primary prevention for disabilities patients?
A. Contact with families and supporting organization✅
B. Provide extra educational programs Answer: A

1281.Patient came to ER with cold clammy skin and loss of hair in leg. He has pressure ulcer in
both leg. What is the most appropriate nursing diagnosis?
A. Risk for immobility
B. Risk for injury ✅
C. Risk for fall Answer: B

1282.The nurse is instructing and learning patient and his relatives how to change patient
dressing. What is the step confirm for evaluation?
A. Verblize the tools that used in dressing change
B. Verbalize the steps of dressing change oral
C. Demonestrate wound dressing and observe for them ✅ Answer: C

1283.Postpartum woman with normal vital signs and the fundus was boggy and soft. The nurse
performed massage for the fundus. The woman has been passed large amount of vaginal blood.
The fundus become firm. What is the next first action?
A. Notify the physician
B. Check vital signs ✅
C. Remassage the fundus Answer: B.

1284.Patient with pacemaker. What is the dischrage instructions should the nurse give?
A. Report doctor if doing heavy activity or carry more than 20 kg
B. Inform doctor if the ring out reading of normal rate ✅
C. Do light exercise Answer: B

1285.Child admitted to hospital. He was axnxious and crying too much. What is the most
appropriate cause?

ِ‫يِاْلَ أرض‬
‫أ‬ ُ ‫َم‬
ُ ‫ك‬
‫ثِِف‬ َ ‫اس‬
‫ِِفي أ‬َ َّ ‫ع‬
‫ِالن‬ ُ ‫ف‬
َ ‫ماِيَ أن‬ َّ َ‫وأ‬
َ ِ‫ما‬ َ
{ 151 }

A. Unfamiliar environment, and anxiety ✅


B. Pain and Fear from staff Answer: B

1286.Child came to ER with some bruising and discoloration in the skin. They try to took with him
but not responding or talking with anyone. What should the nurse suspect the child have?
A. Fear from people
B. Physical abuse ✅
C.Mother and child relation Answer: B

1287.Post care recovery for mother of infection in vulva what is the important instructions for
discharge?
A. prevent recurrence of infection
B. maintain hygiene and prevent complications ✅ Answer: B

1288.The patient came to ER with hip fracture. After surgery she is high risk for DVT. To prevent dvt
she wear antiembolism stockings .What’s the additional instruction nurse needed to focus on ?
A. reposition patient frequently ✅
B. assess calves for warmth and tenderness Answer: A

1289.A healthy baby came to clinic with jaundice. What is the first action for the nurse?
A. Ask about time of appearance of jaundice ✅
B. Assess the billirubin level by test
C. Assess hemoglobin for mother Answer: A

1290.A nurse who has been working in the unit for 15 years has been receiving a lot of
complaints from patient and staff. She’s meeting with the nurse manger. Which statement the
nurse manger would start with the appraisal?
A. “How do you view your nursing care?” ✅
B. “You don’t deserve to be in this unit”
C. “I’ve received a lot of good feedback back about the way you administer
medication” Answer: A

1291.Patient developed ascites which procedure will help relieve this condition:
A. Paracenthesis ✅
B. Hemodialysis
C. Lumber puncture Answer: A

1292.Diabetic Patient with history of hypertension hyperthyroidism hyperlipidemia is going for a


surgery. Which medication can be hold while the patient is NPO
A. Lispro
B. Atorvastatin
C. Glucophage ✅
D. Thyroxine Answer: C

1293.During home visits nurse suspects that mother and child are being abused by husband.
Which question should the nurse ask?

ِ‫يِاْلَ أرض‬
‫أ‬ ُ ‫َم‬
ُ ‫ك‬
‫ثِِف‬ َ ‫اس‬
‫ِِفي أ‬َ َّ ‫ع‬
‫ِالن‬ ُ ‫ف‬
َ ‫ماِيَ أن‬ َّ َ‫وأ‬
َ ِ‫ما‬ َ
{ 152 }

A. “Is there anything else bothering you you want to tell me about?”
B. “Are you being abused by husband?”
C. “Does your husband argue with you beat you and your son?” ✅
D. “Is your husband abusive?” Answer: C

1294.30 year old man newly diagnosed with Diabetes. The patient has great knowledge on how
to monitor his blood glucose levels. But needs education on insulin administration, dose, discard
of medication. What should the nurse’s outcome be?
A. “ the patient will show knowledge about insulin and insulin administration and discard of
needle”
B. “ the patient will return correct demonstration of aspirating the medication for vial,
administration of insulin and how, discarding needles in sharp box.” ✅ Answer: B

1295.Why does hypertonic uterine contractions cause fetal hypoxia


A. It compress the cord and prevent exchange of oxygen wastes ✅
B. It interrupts placental blood flow” Answer: A

1296.A nurse is caring for a patient receiving skeletal traction. Due to the patients severe limits on
mobility, the nurse has identified a risk for atelectasis or pneumonia. What intervention should the
nurse provide in order to prevent these complications?
A) Perform chest physiotherapy once per shift and as needed.
B) Teach the patient to perform deep breathing and coughing exercises. ✅
C) Administer prophylactic antibiotics as ordered.
D) Administer nebulized bronchodilators and corticosteroids as ordered Answer: B

1297.Postpartum mother after 4 days of Cesarean Section. She has BMI 29. The nurse wants to
give health instructions for her. Which of the following teachings should the nurse provide about ?
A. Infection
B. Puerperal sepsis
C. Diet and exercises ✅
D. Bleeding Answer: C

1298.High reading of Blood Pressure what is the reason?


A. Small cuff ✅
B. Arm is elevated above heart level Answer:A.

1299.What is symptoms of fluid excess ‫؟‬


A. High BP ✅
B. Low BP
C. Dizziness Answer: A

ِ‫يِاْلَ أرض‬
‫أ‬ ُ ‫َم‬
ُ ‫ك‬
‫ثِِف‬ َ ‫اس‬
‫ِِفي أ‬َ َّ ‫ع‬
‫ِالن‬ ُ ‫ف‬
َ ‫ماِيَ أن‬ َّ َ‫وأ‬
َ ِ‫ما‬ َ
{ 153 }

0101-9: ‫تجميعات شهر سبتمبر‬


1300.What is complications after partial gastrectomy needs more teaching and reporting-to
hospital ‫؟‬
A. Fullness quickly
B. Gastric spasm ✅
C. Colic pain Answer: B

1301.Which of following support nurse assumptions that pt risk for recurrence, uncontrollable, lack
of sleep
A- Patient avoid taking king makreal B- Patient taking oatmeal for 2 years
C- Patient taking vitamin E for 2 year ✅ Answer: C

1302.What's the Function of patient classification system?


A. Administration of Nursing care according to pt requirements need
B. Grouping Patient according to length of care needed ✅ Answer: B

1303.The patient has huntington's disease. Which of the following is the most expected symptoms
of disease ?
A- Weight gain B-SeverHeadache
C- CNS depression ✅ Answer: C

1304.Which of the following is the most appropriate Health appraisal in school?


A-Screening and lab tests
B-Immunization ✅ Answer: B

1305.Women with bipolar taking carbamazepine. Which of the following side effects?
A-False positive pregnancy test
B- Sexual dysfunction ✅ Answer: B

1306.Patient came to ER after motor accident. The patient has sever bleeding. Postoperative
head surgery the vital signs are Bl. P 90/60, HR 126 b/m , respiration 24 b/m). The doctor ask the
nurse to assess good tissue perfusion. Which of What is Indication of Improve tissue perfusion?
A- decrease HR to 100 B - increase pulmonary wedge pressure
C -Increase Systolic BP to 86 D- 100 ml dark urine ✅ Answer: D

1307.The patient has raynaud's syndrome on his toes. The patient has yellow ulceration changed
from blue to pale. What is the most common cause?
A-Ascites B- palptation
C- digital sensitivity to cold ✅ Answer: C

ِ‫يِاْلَ أرض‬
‫أ‬ ُ ‫َم‬
ُ ‫ك‬
‫ثِِف‬ َ ‫اس‬
‫ِِفي أ‬َ َّ ‫ع‬
‫ِالن‬ ُ ‫ف‬
َ ‫ماِيَ أن‬ َّ َ‫وأ‬
َ ِ‫ما‬ َ
{ 154 }

1308.The patient complained from stomachache continue after eating for 2 to 3 hours. The
patient has increased weight 3 kg during short time. Which of the following is the most important
nursing diagnosis?
A-Imbalance nutrition B- Acute pain ✅
C-Fluid volume deficit Answer: B

1309.A 48yearold patient in the male Surgical Ward had his gallbladder removed through
laparoscopic cholecystectomy 24 hours ago. While evaluating his general condition, the patient
appears lethargic and complains of severe nauseated feeling along with disco mfort in the
abdomen. What nursing discharge instructions ?
A. return to hospital if there is no bowel movement for1 week ✅ B. increase fluid intake
C. consume normal diet and restrict fiber Answer: A

1310.After left retinal detachment surgery. Which of the following is the most appropriate health
education ?
A. avoid bowel straining ✅
B. talk with pt from right side Answer: A

1311.patient hospitalized then develop dysphagia and he has tablets, how to give him the drug?
A. Dissolve it in water ✅ B. Divide the pill in half and give him one half
C. Open the capsule and place it under his tongue
D. Looking for a liquid alternative to medication? Answer: A

1312.patient with liver disease and take lactulose enema. what type of enema?
A-feeding B-cleaning
C-medicated ✅ Answer: C

1313.. nurse take patient radial pulse half minute and found irregular what will the nurse do?
A-check apical pulse B-check radial pulse for full minute ✅
C-call the doctor Answer: B

1314.child with sickle cell crisis and take oxygen, how to know oxygen was effective?
A-by heart rate B-pain level
C-respiratory rate ✅ Answer: C

1315.baby with tracheoesophageal fistula and has scheduled for surgery, how to feed the baby
before surgery?
A-gastrostomy B-Brest feeding
C. NG tube ✅ D-bottle feeding Answer: C

1316.After delivery estrogen and progesterone drop down there is increase in which hormone ?
A. FSH
B. LH
C. prolactin ✅ Answer: C

ِ‫يِاْلَ أرض‬
‫أ‬ ُ ‫َم‬
ُ ‫ك‬
‫ثِِف‬ َ ‫اس‬
‫ِِفي أ‬َ َّ ‫ع‬
‫ِالن‬ ُ ‫ف‬
َ ‫ماِيَ أن‬ َّ َ‫وأ‬
َ ِ‫ما‬ َ
{ 155 }

1317.40 y old women she diagnosis with 4 cm fibroid mass what is the best interpreting by the
nurse ?
A. Carcinoma need hysterectomy ✅ B. Carcinoma need chemotherapy
C. Benign disappear if no symptoms menopause Answer: A

1318.Pregnant women dignoised with herpis .Which is the best Method for delivery ?
A. Cesarean section✅ B. Normal delivery
C. Forceps Answer: A

1319.Which of the following hirschsprung disease causes?


A. Congenital ✅ B. Constipation
C. The child is infected at the age of 6 months Answer: A

1320.Which step in community organizing involves training of potential leaders in the


community?
A. Integration B. Community organization
C. Community study D. Core group formation ✅ Answer: D

1321. In which step are plans formulated for solving community problems?
A. Mobilization B. Community organization ✅
C. Follow-up/extension D. Core group formation Answer: B

1322.The nurse received the patient after hip surgery. The nurse is unable to locate an older
client's left popliteal pulse. Which action will the nurse take next?
A. Check for the femoral pulse.
B. Check for the pedal pulse. ✅
C. Use doppler ultrasound for location .
D. Measure the blood pressure on the left thigh Answer: B

1323.Which of the following ethical principle define that the nurse should not allowing personal
beliefs or feelings or pressure from others or patients to affect decisions that are made?
A. Subjectivity B. Objectivity ✅
C. Reliability D. Validity Answer: B

1324.The patient with gastroenteritis and he on I. V fluids N. S 500 ml starting at 13:00 pm. The
doctor ordered the rate on infusion pump 125 ml/ hr. The nurse checked the solution after 2
hours. She found empty bottle. The solution finished before timing. What should the nurse do?
A. Check for Infusion pumps settings ✅ B. Stop solution and start next at 17:00 pm
C. Administer new N. S now Answer: A
1325.A patient with server diverticulitis had surgery for placement of a colostomy. The patient is
upset, crying, and will not look at the colostomy. Which of the following would be the highest
priority nursing diagnosis at this time?
A. Knowledge deficit, colostomy care B. Distorted body image ✅
C. Self-care deficit, toileting D. Alteration in comfort Answer: B

ِ‫يِاْلَ أرض‬
‫أ‬ ُ ‫َم‬
ُ ‫ك‬
‫ثِِف‬ َ ‫اس‬
‫ِِفي أ‬َ َّ ‫ع‬
‫ِالن‬ ُ ‫ف‬
َ ‫ماِيَ أن‬ َّ َ‫وأ‬
َ ِ‫ما‬ َ
{ 156 }

1326.During pre- operative phase nursing intervention/ nursing role is ?


A. explain surgery to patient B. obtain informed consent from
patient
C. verify that patient and family understand informed consent form ✅ Answer: C

1327.The hospital observed MERS-COV infection spreed to many persons. The hospital not
identified the affected from non-affected person. What should do to reduce the risk?
A. Isolate all patients B. Prevent visiting
C. Apply standard precautions to all patients ✅ D. Wear protective equipments
Answer: C

1328.Which of the following MARSA medication?


A. Vancomycin ✅ B. Gentamycin
C. Ampicillin D. Streptom Answer: A

1329.What is the best time for corticosteroids drug intake ?


A. Before night sleep B. Early morning 7am ✅
C. After noon D. 7pm Answer: B

1330.Child admitted to surgical ward for thyroidectomy .Which position for child with
thyroidectomy ?
A. Semi fowler with slight flexion neck ✅
B. Supine with hyperextened neck Answer: A

1331.Which abnormal lung sound would require immediate intervention?


A. Stridor ✅ B. Crackles
C. Wheezing Answer: A

1332.A 24 years old patient brought to emergency room after an accident and the nurse noticed
a leaking of clear fluid from the nose. A nurse anticipated the injury is :
A. Basilar ✅ B. Frontal lobe
C. Temporary lobe D. Neuropathic Answer: A
1333.Patient with thyroid storm after hyperthyroidism. He has high grade fever.
What is the additional expected symptoms?
A-protroted eyeballs
B-increase sensitive to heat ✅ Answer: B

1334.Patient came to ER with complaints of dark spots in eye. He diagnosed with retina
detachment. What should the nurse ask to patient that cause case?
A.Do you have history of cataract ✅ B. Do you have hypertension
C. Do you have facial surgery? D. Are you smoker ? Answer: A

1335.The community health nurse visit the patient to provide health education about colostomy
care,. She found the patient with his relatives and will go outside. What should the nurse do?
A. Write the patieng refuse the visit B. Don't go out, you will be late
C. Arrange for another visit ✅ Answer: C

ِ‫يِاْلَ أرض‬
‫أ‬ ُ ‫َم‬
ُ ‫ك‬
‫ثِِف‬ َ ‫اس‬
‫ِِفي أ‬َ َّ ‫ع‬
‫ِالن‬ ُ ‫ف‬
َ ‫ماِيَ أن‬ َّ َ‫وأ‬
َ ِ‫ما‬ َ
{ 157 }

1336.Patient with Dissociative disorder. Which of the following nursing diagnosis should the nurse
expect?
A. Impaired cognitive process
B. Self image deficit ✅ Answer: B

1337.Patient postoperative for gasterectomy. What is the expected color of drainage first 12 to 24
hours after operation?
A - green
B-bright red ✅ Answer: B

1338. A nurse is preparing scheduled medications due at 6 pm. If a doctor orders paracetamol
tab 1g QID, and it was supplied from the pharmacy in 250 mg tablets. Which of the following is
the most appropriate nursing actions?
A- Ask the pharmacy to provide 1g tablets
B- Call the doctor to recheck the dosage
C- Give the patient four 250 mg tablets ✅
D- Hold the medication and document in nursing notes Answer: C

1339.Which of the following sound gurgling breath sound at the end of expiration ?
a. Bronchovesicular
b. Adventitious breath ✅ Answer: B

1340.Patient with systemic Lepus arthritis complain pain in the joint She has butterfly rash in her
face what is most appropriate nursing action ?
A.maintain skin integrity
B.manage pain discomfort relief ✅ Answer: B

1341.How many times shower the baby?


A.twice a week with warm water B.Every day with warm water
C.Every day with warm water and acidic soap
D.twice a week with warm water and alkaline soap ✅ Answer: D

1342.Postoperative patient with cataract procedure. What is the symptoms should the nurse
concern?
A-Blurred vision ✅
B-Slight redness
C-slight discharge Answer: A

1343.What is The term that use to describe people communicate their own belief same culture,
goals, values and ethical?
A.Interpersonal communication B.Intrapersonal communication ✅
C.Intergroup communication Answer: B

ِ‫يِاْلَ أرض‬
‫أ‬ ُ ‫َم‬
ُ ‫ك‬
‫ثِِف‬ َ ‫اس‬
‫ِِفي أ‬َ َّ ‫ع‬
‫ِالن‬ ُ ‫ف‬
َ ‫ماِيَ أن‬ َّ َ‫وأ‬
َ ِ‫ما‬ َ
{ 158 }

1344.A lot of theories took about early skin contact in 3rd stage of labour what is the purpose
from early skin to skin contact between mother and child?
A-at warm baby B-improve bonding ✅
C-improve uterine contraction. D-decrease maternal pulse rate Answer: B

1345.neonate after one day of delivery diagnosed with tracheoesophageal fistula.What should
the nurse observe for neonate?
A-continuous crying ✅
B-Vomiting Answer: A

1346. A nurse is preparing to administer 25 mg iron dextran inject patient with iron deficiency
anemia .the nurse knows this d to subcutaneous tissue and wants to administer the drug safely
which of the best administration techniques ?
A.Z-track ✅ B.deep im
C. use large gauge D.insert needle at 45 angle Answer: A

1347.A 6 months old infant mother decided to wean her child. Which of the following is the best
principle of weaning process?
A.Start the weaning process by 8 month of life
B.Gradually replace one breast session at a time ✅
C.Discontinues the nighttime feeding first
D.Allow the child to take a bottle of milk or juice bed Answer: B

1348.Postpartum Mother delivered since 5 hours. She complained of pain from breastfeeding.
She never experienced pain before. What should the nurse instruct her?
A.Stop breast feeding B.It is Emergency C .Normal and take pain medication ✅ Answer: C
1349.2 years old girl came to ER with perant. She experienced frequent urination. She lost 3 kg of
her weight. Which of the following is the best diagnostic tests?
A.CT B.CBC C.Fat D.Blood glucose level ✅ Answer: D

1350.Patient came to ER with burn on his face and chest. What is the first action for nurse?
A.Access I. V line B.Give oxygen ✅ Answer: B

1351.In the middle of providing care for the patient by the nurse. Father of child came to her from
another room and told her that his child has sever abdominal pain. What should the nurse do?
A. Ask him to wait for 10 minutes B. Leave the patient and go to child ✅
C. Go to room after the shift Answer: B

1352.A 71 year-old woman who resides in a long-term nursing home fell while walking down
stairs. The attending nurse arrives to find the patient sitting motionless on the stairs. She is alert
and oriented but wishes to rest. While she rests, the nurse reviews the chart and notes that her
medication regimen includes metformin, loratadine, warfarin and diclofenac. Which of the
following sign most concern and should the nurse be alert?
A. Bleeding B. Bone fracture
C. Brain concussion ✅ D. Hypoglycemia Answer: C

ِ‫يِاْلَ أرض‬
‫أ‬ ُ ‫َم‬
ُ ‫ك‬
‫ثِِف‬ َ ‫اس‬
‫ِِفي أ‬َ َّ ‫ع‬
‫ِالن‬ ُ ‫ف‬
َ ‫ماِيَ أن‬ َّ َ‫وأ‬
َ ِ‫ما‬ َ
{ 159 }

1353.A 65 year-old woman presents to her care provider with complaints of bright red -15 blood
in the stool, a loss of appetite, a feeling of fullness and fatigue. She has lost five kilograms in the
past three weeks without dieting. A faecal occult blood test is positive and the patient scheduled
for an additional screening test. Which of the following ?screening tests is the most appropriate
A. barium enema B. colonoscopy ✅
C. endoscopy D. computed tomography scan Answer: B

1354.What is the definition of hemiparesis?


A.Weakness on one side of the body ✅
B.Pralysis on one side of the body Answer: A

1355.Community health nurse visited alzheimer's disease patient. The patient complained from
bruises and many injuries. The caregiver denied any fallness caused for patient. What is the nurse
response?
A.Restrain the patient
B.Blame to patient that he is the reason
C.The cause is a cognitive impairement
D.Arrange room and provide lightnight in room ✅ Answer: D

1356.Postpartum Mother was complaining from hemorrhage. The bleeding controlled and stop 4
hours ago. The mother is asking now the nurse to go bathroom for urination. What should the
nurse response?
A. Don't let her go to bathroom
B. Let her to go bathroom
C. Instruct mother to set on the bed then put her leg down gradually✅
D. Keep her in bed and Provide bed pan Answer: C

1357.Child came to the ER with his parents to report as a witness about one victim from their
neighbours. What should the nurse consider when question the child?
A. Developmental activity
B. His awareness of the incident ✅ Answer: B

1358.Patient admitted to hospital for umbilical hernia surgery. After doing physical examination
and Lab investigation the doctor decided to reschedule the surgery after 2 months because of
result abnormalities. The informed consent already signed by the patient. What is the best action
regarding consent?
A. Keep the old consent it is enough
B. We need to take verbal consent later
C. Provide new informed consent ✅
D. Need to take consent from the husband Answer: C

1359.Community health nurse is providing Education for secondery schoole girls.Which of the
following is the best topic to discuss?
A.Annualy examination importance B.Personal Hygiene
C.Ovarian cancer ✅ Answer: C

1360.Patient admitted to hospital with COPD. The patient will dischrage. What is the dischrage
instructions should the nurse give?

ِ‫يِاْلَ أرض‬
‫أ‬ ُ ‫َم‬
ُ ‫ك‬
‫ثِِف‬ َ ‫اس‬
‫ِِفي أ‬َ َّ ‫ع‬
‫ِالن‬ ُ ‫ف‬
َ ‫ماِيَ أن‬ َّ َ‫وأ‬
َ ِ‫ما‬ َ
{ 160 }

A.Avoid and remove every thing that irritate him at home ✅ B.Report any signs of cyanosis
C.Provide dietary management D.Avoid exercise Answer: A
1361.Community health nurse visited patient in home with alzheimer's disease. The patient
complained from bruises and several injuries on his body. The community nurse noticed patient
fall and The caregiver denied any fallness occurred for the patient. What should the nurse do?
A.Provide restrain to the patient
B.Blame the patient that he is the reason
C.Arrange the room and provide nightlight ✅
D.The cause of fall is cognitive impairement Answer: C

1362.Mother want to use hormonal patch as contraceptive device. She is obese. What will
reduce the effect of this device?
A. Weight over 80 ✅ B. Using Antibiotic
C. Age less than 35 Answers: A

1363. How to collect sputum culture?


A. Collect the sputum early morning around 30 ml
B. Instruct patient to deep breathing then cough out slowly in cup ✅ Answer: B

1364.Patient with abdominal pain and abdominal distention and unable to pass stool. When
preforming rectal examination there is no stool and rectum clean. After that the nurse inserted
nasogastric tube for patient. What is the purpose for Nasogastric tube For this patient?
A. obstruction B. enteral feeding
C. decompression ✅ Answer: C

1365. A 57 year-old woman presents with complaints of pain in the bones of her both hands .
What type of musculoskeletal disorder is most likely?
A. Osteoarthritis. B. Rheumatic arthritis ✅
C. Osteomyelitis D. Gout Answer: B

1366. Which of the following patient care plan is the most appropriate for a 37 years old post
appendectomy woman who is at risk of pneumonia ?
A. Restrict fluid intake B. Teach how to use spirometer
C. Encourage ambulation as tolerated ✅ D. Avoid coughing and deep breathing
Answer: C

1367.Patient fell down on the floor. The patient found by nurse Assistant, What should the nurse do
?
A. file incident report B. Document in file and have assistant do an
incident report
C. Document in patient file and do incident report ✅ Answer: C

1368.Situation Doctor for followup and order called and nurse responded but she doesn’t know
the doctor and all nurses are busy what to do?
A. Take call and administer medication ordered B. Tell the nurse caring for patient to
immediately take the call
C. Transfer call to patient room and have the charge nurse takethe call ✅ Answer: C

ِ‫يِاْلَ أرض‬
‫أ‬ ُ ‫َم‬
ُ ‫ك‬
‫ثِِف‬ َ ‫اس‬
‫ِِفي أ‬َ َّ ‫ع‬
‫ِالن‬ ُ ‫ف‬
َ ‫ماِيَ أن‬ َّ َ‫وأ‬
َ ِ‫ما‬ َ
{ 161 }

1369.What key procedure for Occupational health nursing to protect health and security for
working population?
A. Health promotion and prevention B. Health hazard identification ✅
C. Cost containment , job productivity D. Eliminate complaints Answer: B
1370.Situation >> caridac tamponade typical signs and symptoms ?
A. Dyspnea , tachycardia ,chest pain
B. Cardiac muffeled murmur and hypothermia ✅ Answer : B

1371.Patient exhausted and feel pain when moving pain only in right leg what cause pf pain ?
A. Unstable arterial disease
B. Atherosclerosis ✅ Answer: B

1372.Patient told nurse researcher he wants to stop the research her response?
A. you can stop at any time ✅ B. You can’t stop
C. You must have good reason to stop Answer; A

1373.Personality disorders related to social >> ideal body being thin what is the
term?
A-Anxiety ✅
B-Somatoform Answer: A

1374.Which of following is Directing function ?


A.Lead and evaluate subordinates ✅
B. Organizing and planning work Answer: A
1375.Female Patient came very weak and tired -Not take folic acid or vitamins , low hemoglobin
, anorexic , skips meals and is recieving plasma transfusion . What priority diagnosis ?
A-Altered daily living due to weakness B-Disturbed metabolism due to
disease process
C-Altred dietary intake due to inability to eat ✅ Answer: C

1376.25-year-old man presents with a compound fracture in the left leg and profuse bleeding
What immediate action should be taken to control the bleeding?
A.Elevate the patient's leg B.Apply pressure on the femoral artery
C.Use a tourniquet above the fracture site D.Apply direct pressure on the fracture site ✅
Answer: D

1377.Child came to ER confirmed german measles. What is the best intervention for child ?
A.Antipyretics for low grade fever ✅ B. Start I. V antibiotics
C.Antihistamine for itching Answer: A
1378.49-year-old women presented to the Emergency Department complaint of severe chest
pain. The ECG showed that the patient myocardial infarction. The doctor ordered the nurse to
give the 800 mg of aspirin. What is the primary indication of aspirin in this case?
A.Breaks down the thrombus B. Decreases the formation of platelet plugs ✅
C.Inhibits the conversion of prothrombine to thrombin
D.Interferes with vitamin k to maintain clotting factors Answer: B

ِ‫يِاْلَ أرض‬
‫أ‬ ُ ‫َم‬
ُ ‫ك‬
‫ثِِف‬ َ ‫اس‬
‫ِِفي أ‬َ َّ ‫ع‬
‫ِالن‬ ُ ‫ف‬
َ ‫ماِيَ أن‬ َّ َ‫وأ‬
َ ِ‫ما‬ َ
{ 162 }

1379. RSV vaccine route for premature baby is ?


A) IM B) IV C) SC D) Oral ✅ Answer: D

1380.Psychaitric Patient diagnosed with histrionic personality disorder. What should the nurse
expect symptoms?
A. Self image deficit B. Overly dramatic behavior ✅ Answer: B

1381.Which of the following risk can be determined by Alpha fetoprotein analysis screening test
?
A.Neural tube defects ✅ B.Placental insufficiency
C.Hydrous fatalism D.Intra uterine growth retardation Answer: A

1382.A nurse I the medical surgical unital is review a plan of care elderly client with chronic
obstruction pulmonary disease ( COPD) limited mobility . the nursenotes that the physical therapist
change in the plan of care to progress ambulation from 50 to 100 times a day. Which action in
necessary to ensure that the client need are?
A.Inform physical therapist of client respiratory status before progressing to ambulation ✅
B.Instruct physical therapist not to proceed with ambulation nurse presence
C.Inform physician about physical decision to ambulation
D.Cancel referral to physical therapist Answer: A
1383.Which of the following lower posterior lung sound?
A.Bronchial B.Bronchovesicular
C.Vesicular ✅ D.Both bronchial and vesicular Answer: C

1384.Patient admitted hospital with Disuse syndrome. What should the nurse instruct him
regarding disease?
A.Instruct patient to use drug timely
B.Encourage patient to walk every 2 hours in room ✅ Answer: B

1385.What is symptoms of fluid excess?


A.Crackle lung sound ✅ B.Impaired skin integrity Answer: A

1386.What is the Nursing diagnosis of chest radiography tuberculosis ?


A.Ineffective breathing pattern related to tuberculosis B.Infection related disease
C.Difficult breathing related to secretion D.Ineffective breathing pattern related to secretion
✅ Answer: D

1387.Patient has an anxiety and turned into physical symptoms, What defense mechanism is this?
A.Social isolation B.Projetion C.Regresion D,Conversion ✅ Answer: D

1388.What is the pernicious anemia complications?


A.Prolonged bleeding ✅ B.Hepatomegaly Answer: A

1389. Patient have sore throat, lesions in mouth that didn't cure , disphagia , loss of weight. What's
the expected diagnosis?
A. Gastritis B.Tonsillitis C.Laryngeal cancer ✅ D..Emphysema Answer:
C

ِ‫يِاْلَ أرض‬
‫أ‬ ُ ‫َم‬
ُ ‫ك‬
‫ثِِف‬ َ ‫اس‬
‫ِِفي أ‬َ َّ ‫ع‬
‫ِالن‬ ُ ‫ف‬
َ ‫ماِيَ أن‬ َّ َ‫وأ‬
َ ِ‫ما‬ َ
{ 163 }

1390. Definition of advocate nurse: defined as protecting by expressing and defending the cause of
another. Representing the client's needs and wishes to other healthcare professionals. Helping
clients exercise their rights

1391. Patient admitted to hospital with hip fracture. The patient was submitted to surgery with
external fixation. The patient complained from pain 3 from 10 level and numbness in limbs. What
is the most appropriate nursing intervention for patient?
A. Give patient placebo drug analgesic B. Teach patient to take 5 deep breathing
exercise ✅
C. Give patient local anesethia D. Allow his family stay beside him Answer: B
1392. Defintion of myringotomy : Proced urethat makes an incision into the tympanic membrane
usually done to relieve inflammation in the middle ear

1393. Defintion of Myringoplasty : surgical repair of the eardrum with a tissue graft. this procedure is
performed to correct hearing loss. It is also called tympano plasty. Needle for aspiration of fluid
from ear .

1394.A male patient has an HIV and has a wife and son , his wife is depressed due to his
diagnosis What's the immediate goal ?
A.Risk for infection related to low immunity ✅ B.Risk for infective psychosocial support Answer:
A

1395.How many week fever sustain in rheumatic fever?


A.1 week✅ B.4 days C.5 days D.3 days Answer: A

1396. What is Definition of advocate nurse?


A. let patient express feelings and support with treatment
B. Let patient express feelings and help with taking choices of treatment
C. Let patient have autonomy on his own choices
D. Prevent patient from danger Answer: D

1397. Nurse administer medication to wrong patient, she wrote incident report .Which one is the
appropriate?
A. Put the incident report in patient file B. Give a copy to physician
C. Document entery in patient file the concern of incident D. Give incident to patient
family Answer: C

1398. Staff activate SBAR communication , What's the most appropriate action ?
A. follow organization structure B. Talk to practitioner C. Wait for response Answer:
A

1399.Patient told nurse researcher he wants to stop the research , what is called ?
A. Autonomy B. Confidentiality C. Justice Answer: A

1400.Patient came to ER with allergy from ventoline. She has 20% bleeding. What's the expected
type of isolation?
A. Strict isolation B. Reverse Isolation Answer: B

ِ‫يِاْلَ أرض‬
‫أ‬ ُ ‫َم‬
ُ ‫ك‬
‫ثِِف‬ َ ‫اس‬
‫ِِفي أ‬َ َّ ‫ع‬
‫ِالن‬ ُ ‫ف‬
َ ‫ماِيَ أن‬ َّ َ‫وأ‬
َ ِ‫ما‬ َ
{ 164 }

1401.A 44 year-old woman presented to the Emergency Department with eye pain and redness.
She was diagnosed with uveitis related to a viral infection. The doctor prescribe dexamethasone
eye drops.Where is the correct place to instill eye drops?
A. Above the eyelid B. Directly into the cornea
C. Into the conjunctiva sac D. Outside the conjunctiva sac Answer:
C

1402.What is the medical abbreviation used for At bedtime?


A. Hs B. Prn C. Stat Answer: A
1403.What is the Isonized side affect?
A. Hyperuricemia B. blured vision C. Prephral neuritis Answer: C
1404.Patient came to hospital with his wife. The wife asked the nurse to apply Saudi culture during
giving medical and nursing care. What should the nurse know about this culture?
A. Saudi use folk tradition B. Family has strong tight C. Most common is Nuclear family.
Answer: B
1405.Postoperative woman after vogotomy. What should the nurse instruct her on discharge?
A. High fiber Diet B. Increase fluid C. Low protein Answer: B
1406.New staff working for 5 month the supervisor offer for her new position supervisor of night
shift what to do before accept the new position?
A. Review mission and vision B. Leadership skills and experience Answer: B

1407.Schizophrenia paranoid has previous attempts to suicid which is most appropriate


diagnosis?
A. Risk to harm his self B. Risk to harm others Answer: A
1408. Addison disease patient admitted to hospital. The nurse should increase the rest to patient.
What will you do?
A. Warm the patient with blanket B. Increase fluids intakes Answer: A

1409.The nurse manger applied evaluation. She found that there is increase in recovery
percentage among patients. What is the expected evidence for this result?
A. Outcome B. Strategies Answer: A
1410. Which of the following diets would be most appropriate for the client with Crohn's disease?
A. High-calorie, low-protein. B. High-protein, low-residue.
C. Low-fat, high-fiber. D. Low-sodium, high-carbohydrate Answer: B

1411.What Is The a pimple that has white head on skin !!?


A. Muscle B. Papule C. Cyst D. Pastules Answer: D

1412..Encouraging fantasy play and participation by children in their own care is a useful
?developmental approach for which pediatric age-group
a. Preschool age (3 to 5 years) b. Adolescence (10 to 19 years)
c.School-age (5 to 10 years) d. Toddler (1 to 3 years) Answer: A
1413.Child came to ER with his mother complained from sever pain after playing football. The
pain is relieved. Which of the following cause expected?

ِ‫يِاْلَ أرض‬
‫أ‬ ُ ‫َم‬
ُ ‫ك‬
‫ثِِف‬ َ ‫اس‬
‫ِِفي أ‬َ َّ ‫ع‬
‫ِالن‬ ُ ‫ف‬
َ ‫ماِيَ أن‬ َّ َ‫وأ‬
َ ِ‫ما‬ َ
{ 165 }

A. Infection B. Exercise pressure Answer: B

1414.The community health nurse During a home visit, a client with AIDS tells the nurse hat he has
been exposed to measles from his brother 's son since 10 days . Which action by the nurse is
most appropriate?
A.Administer an antibiotic. B.Contact the physician for an order for immune globulin
C.Administer an antiviral. D.Tell the client that he should remain in isolation for two weeks.
Answer: B

1415. A 62 year old woman presents to the clinic with a primary compliant of fatigue. An assess
shows a smooth and reddened tongue with a loss of papillae and pallid mucous membranes.
She complaints of a sense of constant fullness of the stomach, with a decreased appetite and
two three loose bowel movement per day. A neurological assessment shows numbness in the
feet and lower legs. The patient type of test would the most likely?
A. Schilling B. Erythropoietin C. Folic acid levels D. Vitamin B12 levels Answer: D
1416.A 76-year-old woman presents to the clinic with complaints of fatigue. She feels her heart
skips beats and becomes irregular with activities such as climbing stairs and walking long
distances. She has chest pain with exertion that she rates at a level 5 on a 1-10 scale. The pain
subsides with rest. Her skin and nail beds appear pale. An assessment of gait shows imbalance
and she admits to episodes of numbness in the hands and feet. The nurse suspects anemia and
prepares a care plan. Which test would be the most appropriate?
A. Schilling B. Folic acid levels C. Lymph node biopsy D. Bone marrow aspiration
Answer: A

1417.A Patient is recovering following surgery for placement of a colostomy. The nurse goes to
the patient’s room to instruct the patient how to care for the colostomy. The patient’s roommate
has visitors and the patient does not want to participate at this time. What should the nurse do? a.
Document the patient’s refusal and add non-compliance to the care plan
b. Tell the patient that this is vital information and may delay discharged
c. Plan a time convenient to both the patient and the nurse
d. Pull the curtain around be bed and speak, ensuring privacy Answer: C
1418.The nurse found the Patient fell down on the floor beside the bed. Where shouldthe nurse
documentation the action?
A. Care plan B. Kardex C. Incident report D. Patient file Answer: C
1419.CHN is a community-based practice. Which best explains this statement?
A. The service is provided in the natural environment of people
B. The nurse has to conduct community diagnosis to determine nursing needs and problems
C. The service are based on the available resources within the community
D. Priority setting is based on the magnitude of the health problems identified Answer: B

1420.Which of the following is the most prominent feature of public health nursing?
A. It involves providing home care to sick people who are not confined in the hospital
B. Services are provided free of charge to people within the catchment area
C. The public health nurse functions as part of a team providing a public health nursing service
D. Public health nursing focuses on preventive, not curative services Answer: D

ِ‫يِاْلَ أرض‬
‫أ‬ ُ ‫َم‬
ُ ‫ك‬
‫ثِِف‬ َ ‫اس‬
‫ِِفي أ‬َ َّ ‫ع‬
‫ِالن‬ ُ ‫ف‬
َ ‫ماِيَ أن‬ َّ َ‫وأ‬
َ ِ‫ما‬ َ
{ 166 }

1421.multiparous 28 year old delivered a full term infant weight 3500 grams fetal heart monitoring
during the active phase of the first stage of labor showed variable decelerations. The midwife is
inspecting the placenta immediately following expulsion which type of placenta is most likely ?
A.Normal B. Circumvallate C. Succenturiate D. Velamentous
Answer: B

1422.Postoperative patient 2 days with intestinal obstruction. The patient complained from
edema ++, urine output 600 ml and the total fluid intake 2400 ml. What should the nurse expect
cause?
A. Hypokalemia B. Intracellular fluid osmolarity C. Hypertension Answer: B
1423. Infant with Spina Bafida admitted to NICU. What is the sac care should the nurse do?
A. Cover with sterile ointment (moist) B. Cover with sterile dry Answer: A

1424. Patient came to ER with cardiac problem. The patient need cpr suddenly. Who will do the
procedure?
A. Care giver B. Surgon Answer: A

1425. Child with Ductus arterioses. What should the nurse consulting parent??
A. follow up 6 month B. vital sign every day Answer: A

1426. The nurse is providing health teaching to educate mother about bottle feeding. Which of
the following if said by mother, she is understanding?
A. Put baby in sitting position during feeding B. Put bottle in microwave if it was inside fridge
Answer: A
1427. Patient with high cortisol. What should the nurse instruct for Food restriction?
A. White bread, chicken B. Egg hold,liver C. Fruits and vegitables Answer:
A

1428. 22 week pregnant women diagnosed with molar pregnancy. The nurse should expect On
ultrasound to see which of the following ??
A.empty of gastetional sac B.abnormal fetus C.Grape like cluster
Answer: C
1429. What is the characteristics of New born meconium???
A. brown green loosely B. dark green thickly Answer: B
1430. Patient with heart problem Plan for surgery. What type of surgery??
A. Urgent B. Emergancy. C. Elective Answer: C

ِ‫يِاْلَ أرض‬
‫أ‬ ُ ‫َم‬
ُ ‫ك‬
‫ثِِف‬ َ ‫اس‬
‫ِِفي أ‬َ َّ ‫ع‬
‫ِالن‬ ُ ‫ف‬
َ ‫ماِيَ أن‬ َّ َ‫وأ‬
َ ِ‫ما‬ َ
{ 167 }

1431. Mother in labor for late deceleration. What is the Nursing intervention
A. Administration of oxygen B. increase rate of pitucin
C. Put pt in supine position D. Documents fetal heart rate and continuous monitoring
Answer: A

1432. Which mother risk to have baby with cognitive impairment: Mother with
A. WBCs in urine B. Albumin in urine C. Folic acid deficit Answer: C

1433.Patient post surgery of ovarian removal have rapid increase in abdominal size and
complain of increased pain what the complication developed?
A. Pneumonia B. Expulsions of surgical content C. Internal hemorrhage
Answer: C
1434. Patient with HBV admitted to hospital with restlessness what‘s the most important nursing
management
A. Breathing B. Diet Answer: A
1435.Patient admitted to hospital diagnosed with meningitis.. GCS 9 E2 V3 M4. The Patient is
drowsy and loss of weight. : Bp 163/80 -Temp 39 -HR 106 -Height 165 -Wight 56 ,Most
important finding ?
A. vital signs instability B. cognitive impairment C. Mental state change Answer: C
1436.The Wassermann reaction test should done before married to early identification which of
the following?
A. Immunosuppressant B. Toxoplasmosis Answer: A

1437.A 70-year-old man presents to the clinic with difficulty sleeping at night. He has not had a
good night's rest for several months and feels exhausted. He needs to place three pillows behind
his back in order to sleep. Examination of the lungs reveals crackles and wheel Auscultation of
the heart confirms an S gallop. Which of the following is the most likely underlying health
problem?
A. Apnea B. Rest dyspnea C. Orthopnea D. Dyspnea at night
Answer; C
1438. The nurse is teaching a patient who was just diagnosed with narcolepsy. The nurse should
teach the patient that which of the following typically increases the level of fatigue?
A. Eating large meal B. Participating in an exercise program
C. Taking brief naps D. Working in a cool environment Answer: A

1439. A patient is diagnosed with agoraphobia. Which of the following would the healthcare
identify as a characteristic of this disorder?
A. Refuses to use a public restroom B. Fear from crowded area
C. Avoids interacting with strangers D. Fear from dealing with animals
Answer: B
1440.A 33-year-old patient in her 13th week of pregnancy has come for her routine
gynecological Appointment. According to the ultrasound, her fetus is a female. She already has
3 daughters but her family wants the male child, so she asked the nurse guidance to terminate
her pregnancy. The Nurse explained that baby's gender is determined by the father. She later
involved her husband in the Discharge planning and finally helped the father agree to continue

ِ‫يِاْلَ أرض‬
‫أ‬ ُ ‫َم‬
ُ ‫ك‬
‫ثِِف‬ َ ‫اس‬
‫ِِفي أ‬َ َّ ‫ع‬
‫ِالن‬ ُ ‫ف‬
َ ‫ماِيَ أن‬ َّ َ‫وأ‬
َ ِ‫ما‬ َ
{ 168 }

the mother's pregnancy. What professional obligation is fulfilled through the nursing
implementation?
A. Influenced decision making B. Incident prevention beforehand
C. Empathetic leadership approach D. Sympathetic knowledge sharing
Answer: A
1441. 36-year-old son is the primary caregiver to his 76-year-old has many chronic diseases and
need full time assistance discussed with the community nurse the idea of referring his of the
elderly day care centers. The nurse explains the case to such services which of the following
elderly groups this patient belongs to this service?
A. With busy caregivers who need assistance. B. Who are bored staying at home and need
socialization
C. Who have been diagnosed with Alzheimer D. Who want to engage in handcraft activities
and art Answer: A

1442.A nurse is documenting in patient records several events that occurred during home health
visits. Which of the following is an example of the correct way to document patient information?
A-Patient fell walking to bathroom. Busy preparing for sterile dressing change, when the patient
left the Room
B- Patient got out of bed without assistance. Denise any symptoms
C- Patient sitting in chair. Strict bed rest orders ignored
D- Patient showering. Electronic epidural infusion pump on floor by shower stall Answer: B
1443.When descending the stairs, what crutch leg order should be followed?
A. Crutches B. Affected leg C. Unaffected leg Answer: B
1444.The midwife is conducting a health education session to pre and post-menopausal woman .
She has discussed the importance of having Pap-smear test regularly . Which of the following
nursing problem is being considered?
A. Ineffective coping B. Disease prevention
C. Health maintenance D. Therapeutic regimen Answer: D
1445.A 19 year-old patient is admitted to the Male Surgical Word after the road traffic accident in
the car racing with his friends. He had fractured bones in his left sided extremities including fibula
in his leg. His wrist. The cast is applied on his arm as well as on his leg. His physiotherapy sessions
will be started according to the evaluation of his treating surgeon. What short term goal should be
planned while considering the age of the patient?
A. Personal counseling for safe driving practice B. Importance of body image for the
patient
C. Respect patient privacy needs D. Encourage activity and exercise
Answer: B

1446. A patient has been experiencing recurrence of this media antibiotic therapy and the
patient was scheduled for preparative teaching, the patient asked the nurse about the purpose
of the procedure. What is the best answer that the nurse should provide?
A. Stimulates motion of the ossicles B. Detects permanent hearing loss
C. Allows drainage of purulent fluid from the middle ear
D. Enables medication administration directly to the affected ear Answer: C

ِ‫يِاْلَ أرض‬
‫أ‬ ُ ‫َم‬
ُ ‫ك‬
‫ثِِف‬ َ ‫اس‬
‫ِِفي أ‬َ َّ ‫ع‬
‫ِالن‬ ُ ‫ف‬
َ ‫ماِيَ أن‬ َّ َ‫وأ‬
َ ِ‫ما‬ َ
{ 169 }

1447. Post- operative patient to be discharged and his temperature is 37.6 at 08:00 - everything
else is normal. What should the nurse do?
a. inform the physician to delay discharge b. give aspirin dose earlier than scheduled
c. Increase fluids intake. Measure temperature twice in morning and evening Answer: C
1448.Patient with alzheimer's disease. The doctor ordered restrain for the patient. The nurse got
the informed consent from his family. What is the first measurement of safety to prevent injuries for
patient ?
A. Tie the extremities to the side rails B. Tie the extremities to fixed part of the bed
C. Put pads on bony prominence D. Allow restrain to tightly to Avoid the patient move
freely Answer: C
1449.Patient admitted to inpatient for surgical procedure. The doctor ordered lab investigation
and performed physical examination. The patient diagnosed with HIV. The patient Disbelieve that
and said that he is good. What is the long term planning ?
A. Let patient accept the disease B. Let the patient adapt and coping the disease
Answer: B

1450. Woman admitted to hospital, she was harming her brother, she diagnosed with
schizophrenia. What's the first action for nurse?
A. Limited behavior B. Build nurse and patient relationship
Answer: B
1451. Registered nurse is educating new nurse how to do nasal suction. What‘s the first action
should the nurse teach her?
A. Check the nares by penlight to ensure patency
B. Connect the External portion of the tube with suction machine Answer: A
1452.Neonate delivered in 34 week. The baby weight 1300 g. The neonate diagnosed with
jaundice and starting phototherapy. What is the albumin\ billirubin rate require for baby?
A. Less than 5 B. From 4 to 5 C. From 5 to 8 Answer: C

1453. the Dr ordered I. V fluids, patient connected with CVL, taking I. V solution. What should the
nurse do?
A. Slow the rate for the first solution B. Discard the first solution
C. Inform doctor about complication as patient receiving I. V fluid Answer: C
1454.Which of the following is the article for assessment of sensory?
A. Hammar B. Tunnig fork C. Tounge depressor Answer: B
1455. for prevention of Hepatitis A, you decided to conduct health education activities. Which of
the following is Irrelevant?
A. Use of sterile syringes and needles B. Safe food preparation and food handling by
vendors
C. Proper disposal of human excreta and personal hygiene
D. Immediate reporting of water pipe leaks and illegal water connections Answer: A
1456.A 43 year old woman accompanied by her son to the primary health clinic and she speaks
different language of the nurse. The nurse in the clinic in the clinic does not speak the same

ِ‫يِاْلَ أرض‬
‫أ‬ ُ ‫َم‬
ُ ‫ك‬
‫ثِِف‬ َ ‫اس‬
‫ِِفي أ‬َ َّ ‫ع‬
‫ِالن‬ ُ ‫ف‬
َ ‫ماِيَ أن‬ َّ َ‫وأ‬
َ ِ‫ما‬ َ
{ 170 }

language of the patient. In this situation, what should the nurse do to assess the needs of this
patient?
A. Ask the patient if it is OK to let her son translate for her
B. Try to use an Urdu translation application in her phone
C. Find out an interpreter employed the clinic
D. Try to make flash with common phrases used Answer: C

0101-11: ‫تجميعات شهر أكتوبر‬


1457. Which of the following the color of urine that indicates well hydration for the patient?
A. Light yellow B. Dark yellow C. Orange D. Amber Answer: A

1458. A 57 year-old patient with diabetes mellitus. The nurse is instructing patient how to care for his foot.
Which of the following is appropriate instruction?
A. Use antibiotic ointment when injured B. Wearing open slipper
C. Keep foot dry properly after bathing D. Apply closed socks Answer: C
1459. Patient after cardiac surgery came to ER with dizziness, swelling and numbness in leg
What is the expected Complication after cardiac surgery..?
a. Damage phrenic nerve b. Pulmonary embolism C. Cardiac Tymopnade Answer: C
1460. Pateint with severe bleeding the doctor ordered plasma. The patient receives plasma. What is the
function of the plasma?
A. Increase HGB and hematocrit. B. Increase WBCs.
C. Increase fluid volume. D. Increase clotting factor. Answer: C
1461. Child with disintegrative disorder. How the nurse identifies that is progress in child condition. What is
the First sign that indicate improvement in disintegrative disorder?
A. Child no longer steal things B. Child no longer blame others
C. Persist aggressive behavior D. Child no longer loss of bladder control Answer : D
1462.Computerized order system. What is the primary function for informatic nurse?
A. System designer B. Soft ware C. Liaison D. Purchasing Answer : C
1463. A newborn born by elective caesarian section under general anesthesia term 30 weeks of pregnancy. His
weight is 1340 gm, and he is in (20) the percentile in intrauterine growth chart. He is admitted to Neonatal
intensive Care Unit. Which of the following is the classification of this newborn according to gestational age
and birth weight using intrauterine growth chart?
A. He is appropriate for gestational age B. He is extremely low birth weight
C. He is small for gestational age D. He is very low birth weight. Answer: A
1464.A newborn born by elective caesarian section under general anesthesia term 30 weeks of pregnancy. His
weight is 1480 gm, and he is in (30) the percentile in intrauterine growth chart. He is admitted to Neonatal
intensive Care Unit. Which of the following is the classification of this newborn according to gestational age
and birth weight using intrauterine growth chart?
A. He is appropriate for gestational age B. He is extremely low birth weight
C. He is small for gestational age D. He is very low birth weight. Answer: A
1465. Which of the following is patient more risk for fluid volume Excess?
A. Ileostomy B. Patient with chronic renal failure

ِ‫يِاْلَ أرض‬
‫أ‬ ُ ‫َم‬
ُ ‫ك‬
‫ثِِف‬ َ ‫اس‬
‫ِِفي أ‬َ َّ ‫ع‬
‫ِالن‬ ُ ‫ف‬
َ ‫ماِيَ أن‬ َّ َ‫وأ‬
َ ِ‫ما‬ َ
{ 171 }

C. Patient with attach to intestinal suctioning D. Patient used loop diuretics Answer: B
1466.Nurse started shift and received two patient. One with thoractomy and the 2nd with pneumonia. Which of
the following assess first?
A. Head to toe assessment for patient with pneumonia
B. Assess level of consciousness and vital signs for both patients
C. Review for care plan and medication for both patients Answer: B
1467. A patient presents to the clinic with “pins and needles” sensations of the left foot and complains that
objects appear “Shimmering”. The patient is diagnosed with optic neuritis and referred for further testing.
The patient is MOST likely to be tested for:
a. Glaucoma b. Multiple sclerosis c. Lesion of brain stem d. Psychosis Answer: B
1468.Postpartum mother came to ER with colic pain and dysurea. She has high temperature 38°c and WBCs
12,000. What is the initial nursing intervention?
A. Administer antipyretics B. Administer antibiotics as ordered by doctor
C. Collect midstream urine sample D. Encourage her to drink plenty of fluids Answer: B
1489. The most common normal position of the fetus in utero in 36 week is:
A. Transverse position B. Vertical position C. Oblique position D. Diagonal position Answer: B
1490.A 25 year old woman present to the emergency room with decrease reflexes, hypoventilation,
hypotension, and fuced pupils, a family member who is accompanying the patient has bottle of diazepam
which the label states was recently the family member also indicates that the patient has a depression. What
intervention should the nurse expect to administer?
A. Naloxone B. Active charcoal C. Tap water enema
D. Magnesium sulfate to reduce the risk seizure Answer: B
1491. Patient with Addison's disease. What is appropriate action?
A. Calm environment B. Bed Rest C. Daily activities should be maintained Answer: A
1492.Child post appendectomy with severe pain. What is the most appropriate nursing intervention?
A. Give pain medication as ordered B. Rest read book
C. Walk with parents D. Talk with same group age child Answer: A
1493.Doctor order sputum culture how to collect?
A. Cough deeply B. Collect 30ml C. Cough the first sputum in the throat
D. Cough in the morning and send it in the night Answer: A
1494. What is the ethical principle of child with poor prognosis
A. Withhold treatment that cause pain and suffering B. Give treatment regardless pain Answer: B
1495. Manger tell new staff (I will evaluate you after 3 month).Which aspect of practice referred to that?
A-assignment B-delegaation C-feedback Answer: C
1496. Staff nurse ask manger what is the effect of patient system or electronic system. Because they have
concern from finding difficulties during learning. What is the best response?
A. You can ask information technology department
B. The hospital will do workshops before implement Answer: B
1497. Pregnant woman she have 4 children, was abortion 2 times, first pregnancy it was twins, what should
report note by nurse
A. G3 p3 +2 B. G5 p4+2
C. G6 p5+2 D. G7 p4+2 Answer: C

ِ‫يِاْلَ أرض‬
‫أ‬ ُ ‫َم‬
ُ ‫ك‬
‫ثِِف‬ َ ‫اس‬
‫ِِفي أ‬َ َّ ‫ع‬
‫ِالن‬ ُ ‫ف‬
َ ‫ماِيَ أن‬ َّ َ‫وأ‬
َ ِ‫ما‬ َ
{ 172 }

1498. What is most common symptoms of COPD??


A. Air hunger B. Dyspnea C. Cough with sweat night D. Persistent cough Answer: D
1499. What is a characteristic of case method?
The Case study gives researchers the possibility to investigate cases which could not be set up in research
laboratories.
Strength: Case studies may contradict established theory and helps to develop new theories
Purpose of surveillance: find way to detect early disease and exposure for others.
1500. Which of the following is considering manger function?
A. formulating or making budget B. Take blood sample Answer: A
1501. Urine sample sent to lab for test and assess urine gravity and osmolarity. What is indicating?
A. Index capacity B. Urine volume Answer: A
1502.A 26 year old married, female patient is admitted in the plastic surgery ward for the correction of burn
strictures and skin grafting on her neck and lower face under general anesthesia. While discussing the
treatment plan with her, the plastic surgeon explained that she will have a series of similar surgeries but she
needs to be on family planning until the treatment ends. The patient asked the nurse whether she will be
normal again.
A-Lack of understanding of surgical procedure B-Chronic low self-confidence due to disfigurement
C-Possible discontinuation from long term treatment D-Lack of family's support and carin Answer: A
1503. What is the OVR incident characteristic?
A. Reported to nursing supervisor immediately B. Small issue not reported
C. Patient education D. Reported within or every 24 hours Answer: D
1504. What is the most common Causes of central venous line infection?
A-Heparin infusion B- Fluid replacement C- Iatrogenic Answer: C
H1N1 - - - - - - - - - Airborne precaution
H5N1 - - - - - - - - - Contact precaution
1505. The nurse is assessing the social history of a patient. The patient reports recent unemployment with
subsequent no income. In addition, the patient has developed a sleep disturbance associated with financial
worries. Which combination of stressors is this patient experiencing?
a. Environmental and sociocultural b. Physiological and environmental
c. Physiological and psychological d. Environmental and cognitive Answer: C
1506. Patient with pacemaker ECG and chest X-ray has done. Patient has slight dyspnea. What is the Nursing
intervention?
A. Position client properly B. Monitor pacemaker functioning Answer: A
1507. A client tells the nurse that the television newscaster is sending a secret message to her. The nurse
suspects the client is experiencing:
A. Delusion. B. Flight of ideas.
C. Ideas of reference. D. Hallucination. Answer: C
1508. A patient admitted to hospital for a exploration surgery. Liver sample is taken after investigation tests
results the patient is diagnosed as positive for Human Immunodeficiency Virus (HIV). The patient expresses
Disbelieve and refuse the result, there is mistake, and impossible to have this disease. Which of the following
short term nursing goal should?
A. Reduce anxiety B. Health teaching disease
C. Refer to health care facilities Answer: A

ِ‫يِاْلَ أرض‬
‫أ‬ ُ ‫َم‬
ُ ‫ك‬
‫ثِِف‬ َ ‫اس‬
‫ِِفي أ‬َ َّ ‫ع‬
‫ِالن‬ ُ ‫ف‬
َ ‫ماِيَ أن‬ َّ َ‫وأ‬
َ ِ‫ما‬ َ
{ 173 }

1509. A patient admitted to hospital for a exploration surgery. Liver sample is taken after investigation tests
results the patient is diagnosed as positive for Human Immunodeficiency Virus (HIV). The patient expresses
Disbelieve and refuse the result, there is mistake, and impossible to have this disease. Which of the following
long term nursing goal should?
A. Help patient for Coping B. Refer to health care facilities Answer: A
1510. A nurse has taught a patient with iron deficiency anemia about eating als that are high in iron. Which of
the following meal is low in iron?
A. Dried beans and brown rice B. Eggs and whole wheat toast
C. Steak and a salad made with fresh spinach D. Cheese pizza and pasta with tomato sauce Answer: D
1511.While caring for a patient with in effective airway clearance related to increased mucus production, the
nurse should encourage fluids to:
A. Maintain nutrition B. Prevent boredom C. Stimulate coughing D. Thin secretion Answer :D
1512. Postoperative Patient with hip replacement surgery ,which position of the bed?
A. Supine è slight elevation of head B. Trendelenburg
C. Raise side rails Answer : A
1513. Child came to ER with broken noes, bleeding , mouth breathing , difficulty breathing .Which of the
following Position .?
A. Supine è elevation head and clean nose B. Flexion head downward and clean nose Answer: B
1514. Pregnant women with varicose vein. What is the Nursing intervention?
A. Elastic stock B. Elevation leg on pillow
C. Wear flat shoes ( not wearing heels ) D. appropriate position while sitting with support back Answer: A
1515.What should you do if I. V line infltration happen ?
A. Remove line B. Apply Cold compress Answer: A
1516. During blood transfusion. The patient suffering from low back pain and chills. What should the nurse to
do ?
A. Give normal saline B. Slow rate
C . Review identification of blood Answer : B
1517. Pregnant women admitted to ward for controlling BP, She having PIH. Women anxious, crying, fear
that condition may be harm to her baby, The Nurse educated her, Nurse calm her. What is goal of nursing
action?
A. Reduce anxiety B. Reduce fear Answer : A
1518.Preoperative patient scheduled for surgery. Before 4 hours of operation. Which of the following if done
will risk for complications?
A- Heparin B- oral beta blocker C- analgesic I. V Answer: A
1519.Immediately after a craniotomy for head trauma, the nurse must monitor the drainage on the dressing.
Which of the following should be reported?
A. Blood tinged B. Straw colored C. Clotted D. Foul-smelling Answer: B
1520.What is the function of glomerular filtration? :
A. Filtration of blood in the kidney B. Absorbing fluid in kidney Answer: A

ِ‫يِاْلَ أرض‬
‫أ‬ ُ ‫َم‬
ُ ‫ك‬
‫ثِِف‬ َ ‫اس‬
‫ِِفي أ‬َ َّ ‫ع‬
‫ِالن‬ ُ ‫ف‬
َ ‫ماِيَ أن‬ َّ َ‫وأ‬
َ ِ‫ما‬ َ
{ 174 }

1521. A newborn born by elective caesarian section under general anesthesia 30 weeks of pregnancy. His
weight is 1000 gm. He is admitted to Neonatal Intensive Care Unit. Which of the following is the percentile
gestation of this newborn on the chart?
A. 20 % B. 10 % C. 90 % D. 95 % Answer: A

1522. A newborn born by elective caesarian section under general anesthesia 32 weeks of pregnancy. His
weight is 1000 gm. He is admitted to Neonatal intensive Care Unit. Which of the following is the percentile
gestation of this newborn on the chart?
A. 10 % B. 20 % C. 90 % D. 95 % Answer: A
1523. A newborn born by elective caesarian section under general anesthesia 22/24 weeks of pregnancy. His
weight is 1000 gm. He is admitted to Neonatal intensive Care Unit. Which of the following is the percentile
gestation of this newborn on the chart?
A. 20 % B. 70 % C. 90 % D. 25 % Answer: C
1524. What should the nurse measure for patient on mannitol during increase ICP ?
A. Serum glucose B. Serum electrolytes Answer: B
1525.What is the organism that causes cervical cancer?
A. HPV B. Herpes 2 C. HIV Answer: A
1526. What causes spontaneous abortion in 12 gestational age?
A. Maternal stress B. infection C. fetal abnormalities Answer: C
1527.Agitated man with history of schizoprenia. He tries to kill his wife, people caught him and took him to
ER, What do you observe on his behavior that indicate active schizophrenia?
A. Blunt B. Hallucination C. Poor speech Answer: B
1528. There is 20 cm laceration on right forearm of girl in ER why type of anesthesia to use before repair?
A. Local B. General C. Regional Answer: C
1529. Name of opening where feces and urine are drained?
A.Stoma B.Ostomy C.Fistula Answer: C
1530.Nurse want to change colostomy bag for patient , what should the nurse do to prevent injury for
peristoma area?
A.Apply gently pressure when removing the stoma slowly B. Apply warm pad to stoma Answer: A
1531.Patient with CVC on fluid therapy. He told the nurse that bacteria are getting inside the CVC. What is
the cause of possible infection?

ِ‫يِاْلَ أرض‬
‫أ‬ ُ ‫َم‬
ُ ‫ك‬
‫ثِِف‬ َ ‫اس‬
‫ِِفي أ‬َ َّ ‫ع‬
‫ِالن‬ ُ ‫ف‬
َ ‫ماِيَ أن‬ َّ َ‫وأ‬
َ ِ‫ما‬ َ
{ 175 }

A. Many injection administration B. Many Frequent blood sampling from CVC Answer: B
1532. During fluid therapy on infusion, the infusion pump or Blood pressure device is malfunctioned. What
should the nurse do?
A. Put the pump aside inside the pt room B. Put pump in utility room,
C. Remove the pump and put it with other pumps,
D. Place Broken Label on pump and put it with other malfunctioned pumps Answer: D
1533.What is the Minimal PPE for aids?
A. Gloves + non sterile gown, B. Sterile gloves +sterile gown ,
C. Mask +sterile gloves Answer: A
1534.Patient said to the nurse i felt sick when my wife wanted to get divorced, What should the nurse reply?
A. Why are you sad B. Its okay anyone can divorce
C. How did you feel when you got divorced Answer: C
1535.67 year old Female pt has stress continence related to poor muscle tone, what is the plan of nurse?
A. Put adult diaper B. put catheter,
C. perform kegel excercise regularly Answer: C
1536.40 year-old female is diagnosed to have stress Incontinence , one of nursing diagnosis stated by the nurse
is "Stress incontinence related to decrease pelvic muscle tone" What is the most appropriate nursing
intervention ?
a. Apply adult diapers b. Catheterize the patient
c. Teach Kegel exercises d. Initiate a bladder emptying program Answer:C
1537.Postpartum Mother complaining of breast engorgement and pain during feeding what to do?
A.Apply ice pack, before feeding B. Apply warm pack, before feeding
C. Massage breast D. Apply analgesic cream Answer:B
1538.What is the cause of cord prolapse and infection?
A. Cordiocentesis B. Amniocentesis C. Trendelenburg position Answer: B
1539.Mother of 10 month old baby is complaining of breast engorgement. What should the pt avoid in the
dietary recommendation for baby?
A. Increase fluid intake B. Start Weaning C. Give baby cows milk Answer: C
1540.The nurse should assess which of the following tests before giving I. V furosemide?
A. Bp , Bun , liver enzyme B. Bp , Bun , potassium C. Bp , HR , urine output Answer: C
1541.Neonate borned with vernix caseosa. The nurse should identify it protects baby from what?
A. From injuries and diseases B. From trauma during delivery or infection Answer: B
1542.Which of the following electrolyte cause cardiac arrest?
A. CLoride B. Potassium C. Soudim D. Calcium Answer: B
1543.Patient with chlamydia she delivers then baby given erythromycin ointment to prevent:
A. Bilateral cataracts B. Opthalmic neontorum Answer: B
1544.Patient with ulcerative colitis the doctor order to give him 300ml through NGT. What is nursing
intervention?
A.Warm the feed before giving B.Give every 15 minute Answer: A
1545.The patient ICP 10 : 15 mmhg. What should the nurse expect to do?
A. Normal B. Slight elevation Answer: B

ِ‫يِاْلَ أرض‬
‫أ‬ ُ ‫َم‬
ُ ‫ك‬
‫ثِِف‬ َ ‫اس‬
‫ِِفي أ‬َ َّ ‫ع‬
‫ِالن‬ ُ ‫ف‬
َ ‫ماِيَ أن‬ َّ َ‫وأ‬
َ ِ‫ما‬ َ
{ 176 }

1546.When the nurse taking medical and personal history for the patient. The nurse should ask patient all of
the following Except?
A. Name B. Religion C. System review D. Race Answer: C

1547. A nurse is providing postpartum care for a GSP4 mother who had a rapid labor of an infant weighing
4000 gm. Assessment revealed a boggy uterus, heavy lochia and stable vital signs. After fundal massage and
bladder evacuation. the fundus remains soft. which of the following is the most appropriate next nursing
action?
A. Inform the physician . B. Reassess the vital signs
C. Continue fundal massage D. Take venous blood sample Answer: A
1548.The postpartum mother is breastfeeding newborn. The neonate took 5 Ounces. The mother is worry
about the amount and She is asking the nurse. What should the nurse reply ?
A. That’s normal capacity B. The capacity for baby stomach same amount
C. That’s proper amount in first time Answer: C
1549. Which of the following tests is important assessment before operation?
A. HIV screening B. Blood grouping and cross matching Answer: B
1550. The doctor ordered morphine to patient. The nurse should assess which of the following with the
medication?
A. O2 saturation 95% B. Respiratory rate 11 b/m
C. Hear rate 65 b/m D. Blood pressure 118/75 Answer: B
1551.Student nurse is asking the head nurse about touching communications. What should the Manager
response?
A. Don't touch patient head B. You can touch patient as much you need
C. Each patient has different congnition for touching Answer: C
1552. When selecting activities to help develop a child’s fine motor skills, which of the following would best
meet this goal?
A. Sorting cardboard objects that are in different shapes
B. Singing while turning the pages of a book that plays music.
C. Jumping rope D. Riding a three-wheeled bicycle. Answer: A
1553. Child admitted to hospital for tonsillectomy. The child has been discharged. The child has complained
from ear pain at home. What is the most common cause?
A. Pain B. Infection C. Septicemia Answer: B
1554. Patient with ABG results. The patient has been diagnosed with acidemia, what is the expected
interpretation?
A. Compensate respiratory B. Compensate metabolic
C. Uncompensated respiratory D. Uncompensated metabolic Answer: D
1555. Psychatric patient has suicidal ideation and tried to end his life. The patient told the nurse that he has
been set limits to his life. What should the nurse response?
A. I can understand that you are sad B. Do you have plans for suicide
C. The life is go id try to cope Answer: B
1556. What is the function of the fibers of endometrial?
A. Help in expulsion of fetus during delivery B. Help in implantation of the fetus in the uterus
C. Help in preventing tumors Answer: B
1557.12 years old child diagnosed with Diabetes mellitus. The doctor ordered insulin for him. The nurse
should teach how to take insulin. Which of the following is the most appropriate nursing intervention?

ِ‫يِاْلَ أرض‬
‫أ‬ ُ ‫َم‬
ُ ‫ك‬
‫ثِِف‬ َ ‫اس‬
‫ِِفي أ‬َ َّ ‫ع‬
‫ِالن‬ ُ ‫ف‬
َ ‫ماِيَ أن‬ َّ َ‫وأ‬
َ ِ‫ما‬ َ
{ 177 }

A. Teach parent how to inject B. Teach child how to inject himself Answer: B
1558.The nurse was disscussing with psychatric patient. During meeting the patient was getting close to her.
What should the nurse do?
A. Shout and Push him B. Change the topic of discussion
C. Keep silent and move steps back Answer: C
Rural community Definition: A rural area is an open swath of land that has few homes or other buildings, and not
very many people. A rural areas population density is very low. ... In a rural area, there are fewer people,
and their homes and businesses are located far away from one another. Agriculture is the primary
industry in most rural areas
1559. Which of the following herid of uterine involution for postpartum mother?
A. Course of contraceptive B. Uncomplicated C&D C. Breast feeding Answer: C
1560.A 56-year-old man with a history of COPD was rushed to the Emergency Department with chest pain,
shortness of breath, fever and a cough. Upon assessment, crackles can be heard over the low lobes. The
patient looks pale and lethargic (see image) . Blood pressure 110/70 mmHg Heart rate 130 /min Respiratory
rate 9 /min Temperature 38.1C Oxygen saturation 85 % What is the most likely condition?
A. Bronchial asthma B. Respiratory failure
C. Pulmonary embolism D. Myocardial infarction Answer: B
1561. Older patient came to ER with shortness of breathing. The nurse applied oxygen by venture mask.
Suddenly the patient condition changed and deteriorated. What is the most common diagnosis?
A. MI B. Carcinogenic shock
C. Septic shock D. Respiratory failure Answer: D
1562. Patient with urine output less than 400 ml for 24h. What is the common diagnosis?
A. Dysuria B. Polyuria C. Oliguria Answer: C
1563.Which of the following to detect the effect of using incentive spirometer?
A. Lung sound B. Oxygen saturation C. Heart rate Answer: A
1564. What is the Location of rheumatic arthritis?
A. Wrist B. Elbow C. Knee D. Shoulder Answer: A
1565.What is the complication of disc?
A. Continue muscle spasm B. Impaired motor response
C. Cerebrovascular accident Answer: B
1566. Patient takes anticoagulant daily. What is the common risk factor should the nurse concern?
A. Infection B. Bleeding C. Pain Answer: B
1567. Patient admitted to ICU with tracheotomy and Nasogastric tube. The vital signs are normal. The nurse
was doing suction for patient and yellow secretion coming out. What is the cause?
A. Diet imbalance B. Infection around tracheotomy C. iatrogenic infection Answer: B
1568. The doctor ordered for patient TPN. How can we prevent infection from central line if the Nurse
suspects that there are bacteria inside?
A. Give antibiotics 3 times per day B. Apply dressing with alcohol on central line
C. Apply one lumen for TPN D. Apply dressing with antibiotics TPN daily Answer: B
1569. What is THE Side effects of Depakote ( Depakine) ?
A. Nausea B. Vomiting C. Hair loss Answer: C
1570.Long Situation , teacher don’t want be tired front his student Maslow's hierarchy of need??

ِ‫يِاْلَ أرض‬
‫أ‬ ُ ‫َم‬
ُ ‫ك‬
‫ثِِف‬ َ ‫اس‬
‫ِِفي أ‬َ َّ ‫ع‬
‫ِالن‬ ُ ‫ف‬
َ ‫ماِيَ أن‬ َّ َ‫وأ‬
َ ِ‫ما‬ َ
{ 178 }

A. Self esteem B. Safety and Security Answer: B


1571.14 years old diabetic patient went to party. He came back with lethargic and tired what is the First
action?
A. Check blood glucose levels B. Adminster juice Answer: A
1572. Community Heath services in Saudi Arabia should be developed. What is the best way for better
outcomes??
A. Improve the health program B. Using Education technology Answer: B
1573. Which of the following is food high in purines?
A. Rice B. Red meat Answer: B
1574.A 36-year-old man who is a cardiac patient presents Beck's triad manifestations. Which of the following
is the most likely occurs in Beck's triad?
A. Flushing, pruritus and urticarial B. Tachypnea, tachycardia and hypoxia
C. Hypotension, muffled heart sounds and jugular venous distension
D. Swelling of lips and tongue, Shortness of breath and edema of larynx and epiglottis Answer: C
1575. In the scope of community nursing, epidemiology and vital statistics are very essential in controlling
disease spreading, disease impact on the given population and effectiveness of the preventive measures done.
An outbreak of measles has been reported in the community. Which of the following epidemiological actions
should the nurse perform initially?
a. Identify the disease that was reported b. Classify the outbreak if sporadic or pandemic
c. Determine the date when the disease first occurred
d. Report the incident to the nearest primary hospital Answer: A
Vital statistics definition ?facts systematically collected and compiled in numerical form relating to or derived from
records vital events , namely live births , fetal death.
Data analysis Definition - the process of systematically applying statistical and/or logical techniques to describe and
illustrate, condense and recap, and evaluate data Giving meaning to data. -bringing understanding to data. -making
decisions from meaning and understanding
Nephritis Definition :
Nephritis is a condition in which the nephrons, the functional units of the kidneys, become inflamed. This
inflammation, which is also known as glomerulonephritis, can adversely affect kidney function.
Condition in which the tissues in the kidney become inflamed and have problems filtering waste from the blood.
Nephritis may be caused by infection, inflammatory conditions (such as lupus), certain genetic conditions, and other
diseases or conditions
1576. Patient eats regular meals with high calories daily and she has weight loss, pulging eye. She is asking
what is the cause of weight loss however eating?
A. Increase metabolism rate B. Decrease metabolism rate
C. Nutrition disorder Answer: A
1577.The float nurse is transefering from surgical word to neurological ward. Which of the following cases
should delegate for her?
A. Patient with myathenia gravis B. Patient stable with fracture hip and casted
C. Patient with chronic disease and on oxygen Answer: B
1578.The nurse is receiving Post operative patient from operating room after general anesthesia to transefer
him to the ward. Which of the following is the priority nursing care for the patient?
A. Wear comfortable, loosely clothes B. Ensure to keep side rails up
C. Adminstration of morphine Answer: B

ِ‫يِاْلَ أرض‬
‫أ‬ ُ ‫َم‬
ُ ‫ك‬
‫ثِِف‬ َ ‫اس‬
‫ِِفي أ‬َ َّ ‫ع‬
‫ِالن‬ ُ ‫ف‬
َ ‫ماِيَ أن‬ َّ َ‫وأ‬
َ ِ‫ما‬ َ
{ 179 }

1579.17 year old mother after normal delivery have anemia and bleeding which of the following education the
nurse do?
a. Mass media b. Group education c. Individual education Answer : C

1580.What is the location of colostomy


1581. What is the name of anatomy site or location?
A. Esophagus B. Duodenum C. Stomach Answer: B
1582. What is the name of procedure? A. Abdominal Aortic Aneurysm

0101/10+11:‫ ديسمبر‬+‫تجميعات شهرنوفمبر‬


1583. Mechanical ventilator used for patient with respiratory problems. When can we use Mechanical
ventilator ?
A.To assess respiratory muscle fuction at the the end of expiration
B. To maintain tidal volume if HO2 less than 30 per minute
C. To protect lung from accidity Answer : B
1584.The nurse has been assigned to care for a 60-year-old critically ill patient with a triple-lumen central
venous line. The doctor's orders include daily care of the insertion site and catheter device. The nurse creates
care plane based on the nursing diagnosis, Risk for infection related to insertion of avenous catheter. Which
intervention is most likely to prevent infection?
A. Re-cap access hub after drawing blood B. Maintain clean technique
C. Wash hands before performing catheter care D. Clean catheter tubing with isopropyl alcohol Answer: C
1585. What is the Main meaning for primary health care?
A. Patient rather than population B. Individual rather than people
C. Disease rather than health D. People rather than disease Answer: D

ِ‫يِاْلَ أرض‬
‫أ‬ ُ ‫َم‬
ُ ‫ك‬
‫ثِِف‬ َ ‫اس‬
‫ِِفي أ‬َ َّ ‫ع‬
‫ِالن‬ ُ ‫ف‬
َ ‫ماِيَ أن‬ َّ َ‫وأ‬
َ ِ‫ما‬ َ
{ 180 }

1586.A critical care nurse is using a computerized decision support system to correctly position her ventilated
patients to reduce pneumonia cause by accumulated respiratory secretions. This is an example of which
Quality and Safety in the Education of Nurses (QSEN) competency?
A. Patient centered care B. Safety
C. Teamwork and collaboration D. Informatics Answer: D
1587. Child came to ER diagnosed with laryngitis (croup). Child vital signs: Respiration 50, Temperature
38.5. Which of the following should the nurse concern for this disease?
A. Monitor respiration B. ICP position C. Rise the HOB D. Heparin antidote Answer: C
1588. Pregnant 29 week with transverse position. What should the nurse instruct her?
A. Should deliver by Cesarean section B. The possibility of Normal delivery Answer: B
1589. Which of the following patient should the nurse see first?
A. Patient after bone marrow biopsy suspected leukemia
B. Patient has convulsion since 20 minute after receiving drug Answer: B

1590. The nurse can assess abdominal tenderness by? A. Palpation B. Percussion Answer: A
1591. Multipara mother in labour the cervix dilated 7 cm 2 hours ago. The mother asked nurse if she can go
bathroom for bowel movement. What is the nurse response?
A. Help mother to go bathroom B. Give mother bed pan
C. Do vaginal examination Answer: B
1592. A nurse is conducting health screening for osteoporosis. Which of the following clients is at greatest risk
of developing this disorder?
a)A 25-year-old woman who jogs b)A 36-year-old man who has asthma
c)A 70-year-old man who consumes excess alcohol
d)A sedentary 65-year-old woman who smokes cigarettes Answer: D
1593. Coomb's test used to detect and screening for which of the following?
A. Sickle cell anemia B. congenital anomaly C. neural tube deficient Answer: A
1594. MAOI drug connected to patient then develop severe nausea, vomiting during infusion. What should the
nurse do?
A. Stop infusion B Give thiamine C. Notify physician Answer: A
1595. What us the name of this (color patterns of a newborn's skin may be startling to some parents. Mottling
of the skin, a lacy pattern of small reddish and pale areas (red or blue) is common because of the normal
instability of the blood circulation at the skin's surface?
A. Mottling in newborn B. Startle reflex Answer: A
1596. Post operative Mother still under sedation, she have two boys 5 and 7 years are anxious and they want
to see their mother. What is your response?
A. Your mom may be not open her eyes but she will be so happy to hearing you
B. You will see your mom but not now Answer: A
1597. The patient came to ER. He complained from exacerbation of bronchial
asthma. What sign of improvement for the patient?
A. Respiratory Rate 50 b/m B. O2 saturation 97% at room air Answer: B
1598. The NG tube (Gastric lavage) is used to which of the following?
A. Prevent nausea, vomiting (prevent aspiration) B. Gastric distension.
C. Wash the stomach of toxin. D. All of the above Answer: D

ِ‫يِاْلَ أرض‬
‫أ‬ ُ ‫َم‬
ُ ‫ك‬
‫ثِِف‬ َ ‫اس‬
‫ِِفي أ‬َ َّ ‫ع‬
‫ِالن‬ ُ ‫ف‬
َ ‫ماِيَ أن‬ َّ َ‫وأ‬
َ ِ‫ما‬ َ
{ 181 }

1599. What is the purpose of self help group for psychiatric patients?
A. Support B. Psychotherapy Answer: B
1600. What is the Risk factor of osteoporosis?
A. Vitamin D deficiency B. Menopause Answer: A
1601. Pregnant woman has convulsions associated with eclampsia. Which of the following drug should give to
her?
A. Midazolam B. Diazepam C Magnesium sulfate Answer: B
1602. Patient woman with cholecystectomy elective procedure. She complained from pain 8/10. What should
the nurse do?
A. Administer morphine IM B. Notify doctor C. Encourage fluid intake D. Full set vital Answer: B
1603. Child with decrease in growth, low vitamin D and high calcium level. What is the most appropriate
intervention?
A. Treat Hypocalcaemia B. Expose child to sun C. Monitor growth chart Answer: B
1604. 80 years old patient married from 70 years old woman. He suffers from Inability to do daily activities,
bedridden with diaper and NGT for feeding. The patient need long term care. Which of the following
developmental stage for both of them?
A. Aging B. Middle age C. Adulthood Answer: A
1605. Which of the following is produced oxytocin hormone?
A. pituitary gland B.Hypothalamus
C. Postrior pituiyary gland D.Anterior pituitary gland Answer: B
1606. Mother’s after vaginal delivery she-complains of dysuria with frequent small amount urine. What is
nursing interventions?
A-Intake plenty of fluid B-take urine sample (midstream) Answer: A
1607. Patient came to ER with burns on his face, neck and hand.Which of the following first action should do?
A. Check the airway B. Told the patient to take breath C. Administer O2 Answer: A
1608. There is a family that was exposed to a fire in their house when they were sleep but nothing happens to
them but there is elderly man can’t walk lives with them, and they ask the nurse how can the do to protect the
older man from the fire if it occur :
A Put someone sleep with him B. Put fire sensors on all the house
C. Keep his room near the exit D. Keep the door and window opening Answer: B
1609. 5 years old child has humorous complete fracture. According to type of fracture what should the nurse
expect signs?
A. Generalized edema and pain or tenderness B. Headache Answer: A
1610. What is the time nurse should follow Glasgow's coma scale ?
A. Every quarter of an hour B. Every hour
C. Every 2 to 4 hours D. Every shift Answer : C
1611. When should we do cleft palate and lip surgery?
A. After one year B. After two years
C. After the child is able to hold a cup D. After improving his awareness Answer : C

ِ‫يِاْلَ أرض‬
‫أ‬ ُ ‫َم‬
ُ ‫ك‬
‫ثِِف‬ َ ‫اس‬
‫ِِفي أ‬َ َّ ‫ع‬
‫ِالن‬ ُ ‫ف‬
َ ‫ماِيَ أن‬ َّ َ‫وأ‬
َ ِ‫ما‬ َ
{ 182 }

1612. according to information provider in the accompanying graphic, a transverse colostomy


would be expected to produce which of the following (see image)
A. 1 B. 2 C. 3 D. 4 Answer: B
1613. Why is better for preventing use of powder for children after bathing?
A. Respiratory problems B. Itching skin C. Diarrhea D. Skin dryness Answer: A
1614. Patient came to ER asked to perform Pcr corona test. Which of the following indications should the
nurse educate him to do the test?
A. If you have friend with positive test
B. If you has contacted closely with affected relative Positive corona
C. If you have suspect with symptoms . Answer: B
1615. Infant borned with normal delivery. After 2 months during follow up visit, the nurse has observed the
infant has one thigh shorter than the second one. The nurse performed Barlow test and the affected leg turn
outward. What should the nurse expect?
A. Fracture B. Dislocation Answer: B
1616. The nurse is caring for a critically ill patient on mechanical ventilation. The physician identifies the
need for a bronchoscopy, which requires informed consent. If the physician were to obtain consent from the
patient, the patient must be able to:
A. Be weaned from mechanical ventilation. B. Have knowledge and competence to make the decision.
C. Nod his head that it is okay to proceed. D. Read and write in English. Answer: B
1617. Infant borned with low developing nasal bones and his nose flat with the face. What is the expected
caused diagnosis?
A. Iron deficiency anemia B. Sickle cell anemia
C. A plastic anemia D. Hemolytic anemia Answer: B
1618. Patient with right sided hemi paresis . What is the brain part is affected?
A. Brain stem B. Frontal lope C. Partial lobe D. Brain ganglia Answer: D
1619. Child with type 1 diabetes mellitus. The nurse is instructing mother. Which of the following signs should
be alert?
A. Polyuria B. Vomiting C. Dizziness D. Thirsty Answer: C
1620. 11 month infant came to ER with rash all over the body, itchy and fluid filled blisters.
Which of the following is the most appropriate diagnosis?
A. Measles B. Chicken pox C. German measles Answer: B

ِ‫يِاْلَ أرض‬
‫أ‬ ُ ‫َم‬
ُ ‫ك‬
‫ثِِف‬ َ ‫اس‬
‫ِِفي أ‬َ َّ ‫ع‬
‫ِالن‬ ُ ‫ف‬
َ ‫ماِيَ أن‬ َّ َ‫وأ‬
َ ِ‫ما‬ َ
{ 183 }

1621. Breast feeding mother with left breast engorgement has diagnosed with mastitis. What should the nurse
tell the mother to do?
A. Stop feeding and take antibiotics B. Continue breast feeding on right breast
C. Use breast pump D. Stop feeding and wait until its gone Answer: C
1622. 5 years old child complained from nausea, vomiting and diarrhea 5 times . The doctor order I.V fluids .
How many fluids should give to child?
A. 900 ml B. 1200 ml C. 1500 ml D. 1700 ml Answer: D
1623. 7 years old Child scheduled for operation. The child weight 50 kg. What is the fluid replacement should
the child need during 24 hour?
A. 1900 B. 2100 Answer: B
1624. What is the first assessment for neonate after cutting umbilical cord?
A. Cover baby with towel and keep warming B. Encourage early contact with mother
C. Check baby from cyanosis D. Check score assessment Answer: D
1625. Neonate borned with normal Apgar score. What should the nurse do?
A. Administer oxygen B. Protect neonate from heat loss and keep warmed Answer: B
1626. Pregnant women with diabetes mellitus. She use insulin and came to clinic for follow up everything is
normal . What should the nurse do?
A. Higher dose of insulin B. Lower dose of insulin C. Increase dose of insulin after delivery Answer: A

1627. CHN (Community Health Nursing) is a community-based practice. Which best explains this statement?
A. The service is provided in the natural environment of people
B. The nurse has to conduct community diagnosis to determine nursing needs and problems
C. The service are based on the available resources within the community
D. Priority setting is based on the magnitude of the health problems identified Answer: B
1628. 43 years female 12 weeks pregnant presents to her OB-GYN for HIV testing following recent knowledge of
her husband's infidelity. She tests positive and becomes increasingly emotional stating "There is no point in me
continuing this pregnancy since my baby will be born HIV positive." Given the client's statement what is the best
response by the nurse?
A. If that is how you feel we can discuss different options for abortion.
B. While it is true your baby will be born HIV positive, this doesn't mean him or her can't live a happy life.
C. If we start you on a series of medications called antiretrovirals and you stick to your prescribed treatment there
is a very small chance your baby will be born HIV positive.
D. There is no reason to worry; your baby can only become HIV positive if he or she is breastfed once born Answer: C
1629. Nurse Manager is a young Unit Manager of the Pediatric Ward. Most of her staff nurses are senior to
her, very articulate, confident and sometimes aggressive. Katherine feels uncomfortable believing that she is
the scapegoat of everything that goes wrong in her department. Which of the following is the best action that
she must take?
A. Identify the source of the conflict and understand the points of friction
B. Disregard what she feels and continue to work independently
C. Seek help from the Director of Nursing D. Quit her job and look for another employment. Answer: A
1630. As a young manager, she knows that conflict occurs in any organization. Which of the following
statements regarding conflict is NOT true?
A. Can be destructive if the level is too high B. Is not beneficial; hence it should be prevented at all times
C. May result in poor performance D. May create leaders Answer: B
1631. Manager knows that one of her staff is experiencing burnout. Which of the following is the best thing
for her to do?

ِ‫يِاْلَ أرض‬
‫أ‬ ُ ‫َم‬
ُ ‫ك‬
‫ثِِف‬ َ ‫اس‬
‫ِِفي أ‬َ َّ ‫ع‬
‫ِالن‬ ُ ‫ف‬
َ ‫ماِيَ أن‬ َّ َ‫وأ‬
َ ِ‫ما‬ َ
{ 184 }

A. Advise her staff to go on vacation. B. Ignore her observations; it will be resolved even without intervention
C. Remind her to show loyalty to the institution.
D. Let the staff ventilate her feelings and ask how she can be of help . Answer: D
1632. She knows that performance appraisal consists of all the following activities EXCEPT:
A. Setting specific standards and activities for individual performance.
B. Using agency standards as a guide. C. Determine areas of strength and weaknesses
D. Focusing activity on the correction of identified behavior. Answer: D
1633. Which of the following statements is NOT true about performance appraisal?
A. Informing the staff about the specific impressions of their work help improve their performance.
B. A verbal appraisal is an acceptable substitute for a written report
C. Patients are the best source of information regarding personnel appraisal.
D. The outcome of performance appraisal rests primarily with the staff. Answer: C
1634. A physician has ordered gavage feeding every 4 hours for a 12- week-old infant with failure to thrive. In
order to know how far to insert the feeding tube. The nurse should measure the distance from:
a. The infant’s mouth to the xiphoid process of the sternum
b. The tip of the infant’s nose to the ear and then to the umbilicus
c. The infant’s mouth to the ear and then to the umbilicus
d. The tip of the infant’s nose to the ear and then to the xiphoid process of the sternum Answer: D
1635.11 month old infant 8 kg . The vital signs is normal except heart rate 170 b/m . Calculate the amount of
fluids the girl needs ?
A. 600 ml B. 700 ml C. 800 ml Answer: C
1636. Which of the following food should the cancer patient avoid ?
A. Fatty food B. High soduim food
C. Fiber food D. Bran products containing food Answer: A
1637. There are times when manager evaluates her staff as she makes her daily rounds. Which of the following
is NOT a benefit of conducting an informal appraisal?
A. The staff member is observed in natural setting. B. Incidental confrontation and collaboration is allowed.
C. The evaluation is focused on objective data systematically
D. The evaluation may provide valid information for compilation of a formal report Answer: C
1638. She plans of assigning competent people to fill the roles designed in the hierarchy. Which process
refers to this?
A. Staffing B. Scheduling C. Recruitment D. Induction Answer: A
1639. Scenario mother and her child came to ER with rheumatic fever. What should you expect?
A- Fever for the last week B- Recent pharyngitis C- Headache Answer: A
1640. What is the type of Polio vaccine?
A- live vaccine B- killed C- mature Answer: B
1641. Patient came with Covid-19 which statement by him regarding Covid-19 is incorrect?
A- Transmit with hand Shaking B- Needs washing hands C- I got Covid-19 while eating Answer: C
1642. Lochia Alba normally disappears after how many days postpartum?
A. 5 days B. 7-10 days C. 18-21 days D. 28-30 days Answer: B
1643. Pregnant woman with abortion. The Hcg result is negative. Products of concept: positive. What is the
type of abortion? A. Missed B. Incomplete Answer: A
1644. A 28-year-old man and his wife involved in which his wife was killed. The client is being Care Unit for

ِ‫يِاْلَ أرض‬
‫أ‬ ُ ‫َم‬
ُ ‫ك‬
‫ثِِف‬ َ ‫اس‬
‫ِِفي أ‬َ َّ ‫ع‬
‫ِالن‬ ُ ‫ف‬
َ ‫ماِيَ أن‬ َّ َ‫وأ‬
َ ِ‫ما‬ َ
{ 185 }

multiple rib fractures and a broken which room his wife is located. Which of the following is the most
appropriate?
A. "Your wife is not in the hospital" B." I'm sorry, but your wife did not survive the
C. "I need to get your family so that you can wife” D." The doctor will be talking to you about located” Answer: D
1645. Which of the following is low fat diet?
A. Lemon Juice B. Beans, fish C. Chicken, egg yolk Answer: B
1646. A primi-gravida client at 25 weeks gestation visits the clinic and tells the nurse that her lower back
aches when she arrives home from work. The nurse should suggest that the client perform:
A. Tailor sitting B. Leg lifting C. Shoulder circling D. Squatting exercises Answer:
1647. The head nurse ensures that nursing staff give nursing care to patients according to criteria. What is
this consider?
A. Leading B. Supervisor C. Delegation D. Give orders Answer: A
1648. When the nurse was doing Barlow and ortolani test for baby , the test is positive . What should the nurse
expect the diagnosis?
A. Fracture B. Dysplasia Answer: B
1649. The nurse is giving health teaching for patient with stage II pressure ulcer for discharge. What should
the nurse tell?
A. Clean the wound with normal saline B. Avoid using bedpan Answer: B
1650. Patient with open heart surgery. The nurse is giving health teaching about wound care for discharge.
Which of the following is most appropriate intervention?
A. Inform the doctor in case of bleeding B. Do not raise the arm above shoulder level
C. Apply dressing for 2-3 weeks to reduce swelling) Answer: A
1651. Which of the following vitamins, if deficient, causes sleep disorder?
A. Vitamin D B. Vitamin K C. Vitamin C D. Vitamin E Answer: A
1652. A home care nurse makes a follow-up visit to a patient who had shingles. A month since the onset, the
patient’s pain level in six on a scale of one to ten, where one is no pain and ten is greatest pain. Two weeks
ago, the pain level was at seven. The patient is treated with pain medication as needed, and skin vesicles are
completely resolved without any scarring. The patient’s condition has which of the following?
A. Met the expected outcome B. Partially met the expected outcome
C. Not improved D. Worsened Answer: A
1653. Patient after liver biopsy position on right sided .Why should the nurse keep patient on operation site?
A . Enhance blood circulation B. Prevent bleeding
C. Increase comfortable D. Prevent trapped fluid Answer: B

1654. How to ensure proper suction in chest tube?


A- Bubbling B- No kinks Answer: A
1655. Patient with pneumonia. The doctor wants to ventilate the patient. What is the expected stage of
pneumonia?
A- Moderate B- Sever C- Normal D- Mild Answer: B
1656. Patient with Bp: 145/95. What stage of hypertension?
A- Stage 1 B- Stage 2 C- Elevated D- Crisis Answer: B
1657. Patient with Mitral valve prolapsed. The doctor ordered prolonged antibiotics. Why give prophylaxis
antibiotics to patient? A. Prevent endocarditic B. Prevent infection Answer: A

ِ‫يِاْلَ أرض‬
‫أ‬ ُ ‫َم‬
ُ ‫ك‬
‫ثِِف‬ َ ‫اس‬
‫ِِفي أ‬َ َّ ‫ع‬
‫ِالن‬ ُ ‫ف‬
َ ‫ماِيَ أن‬ َّ َ‫وأ‬
َ ِ‫ما‬ َ
{ 186 }

1658. Patient with rheumatoid takes ant malarial. What is common side effect?
A. Tinnitus B. Visual changes C. Something in the GIT Answer: A
1659. Child came to ER after head injury there is nose leak. What to do first?
A. Check for glucose B. Report physician C. Assess patient condition Answer: A
1660.40 year woman should screen every 2 years for which of the following?
A. Mammogram B. Breast biopsy C. Self check cancer Answer: A
1661. What are the characteristics of renal colic pain?
A. Dull B. Thropping C. Cramping D. Sharp / Stabbing Answer: D
1662. How to prevent fall risk in cognitive impairment patients?
A. Instruct to slowly set legs down then stand B. keep environment free and rugs tucked
C. Physical activity and muscle strength Answer: C
1663. During Hajji timing there is outbreak of meningitis in KSA. The Saudi health authority requests the
vaccine. What is the type of disease?
A. Epidemic B. Pandemic C. Endemic Answer: A

1664. Elderly Patient with paralytic ileus signs rectal exam done and there was no findings. The doctor
ordered NGT tube inserted. What is the purpose?
A. Decompression B. Enternal feeding C. Fluids specimen D. Suction Answer: A
1665. Newborn delivered with 950gm at 29weeks. What is expectation of gestational age?
A. Small for gastrointestinal age B. Appropriate for gestational age Answer: B
1666. One day newborn after delivery. What sign should report?
A. Grunt in expiration B. Blood in diaper C. Red spot on left side Answer: A
1667. Patient comes to ER with cough, weight loss, fever and night sweats, after chest x – ray the doctor
diagnosed the case tuberculosis and order to give hypertonic solution, what the suitable should be given?
A. Dextrose 10% B. Normal saline 9%
C. Normal saline 45% D. Dextrose 5 % % NaCl 0.9% Answer: D
1668. The patient came to hospital for examination. Patient tested for HIV and the result is positive. Which
statement is wrong and means that patient need more teaching?
A This doesn’t mean I have aids B. Antiviral will decrease the chance of developing Aids
C. When the HIV positive that mean i develop immunity against this disease
D. My test is positive that mean I have ability to transmited the disease for others Answer: C
1669. Child with Marasmus. What is type of deficiency for him?
A. Low vitamis and low calories B. Low protein and low calories
C. Enough protein and low calories D. Enough calories and low protein Answer: B
1670. What is the Intervention for open fracture?
A. Mobilize as much as possible B. Left uncovered
C. Splinting leg D. Rise leg 90 degree Answer: C
1671. Which of the following risk factors of pregnancy above a maternal age?
A. Hypertension B. Multipara 50 age C. 16 age and less D. Anemia Answer: A
1672. 22 years old patient came to ER after a quarrel and a strike between another people. The vital signs: Bb
110/90, Hr/130, Rr 36 -ABG result Hco3 20, Pco2:10.6 PH: 7.41. What should the nurse do regarding ABG
expectation? A. Admit patient to cardiac unit B. Give patient paper Bag to push on it
C. Inform the police D. Sedation and intubation Answer: B

ِ‫يِاْلَ أرض‬
‫أ‬ ُ ‫َم‬
ُ ‫ك‬
‫ثِِف‬ َ ‫اس‬
‫ِِفي أ‬َ َّ ‫ع‬
‫ِالن‬ ُ ‫ف‬
َ ‫ماِيَ أن‬ َّ َ‫وأ‬
َ ِ‫ما‬ َ
{ 187 }

1673. Patient admitted to hospital with head injury to perform craniotomy after surgical procedure. What
should the nurse monitor? A. Bleeding B. ICP Answer: B
1674. 21years diagnosis with head injury. After craniotomy. What is the nursing intervention?
A. Transfer patient to ICU B. Transfer patient to medical ward
C. Do CT scan after 48 hour Answer: C
1675. What is the term Oliguria meaning?
A. Inability to pass urine B. Reduced capacity to form urine Answer: B
1676. What should be done Breast exam?
A. Biannually B. Annually C. Monthly Answer: C
1677. What are the expected signs and symptoms of premenstrual syndrome?
A. Weight loss B. Dizziness C. Mood fluctuation Answer: C
1678. What is the location of liver in abdomen?
A. Left hypochondriac B. Right hypochondroic Answer: B
1679. Man wroks in company after the manager /director anger and shout him. He went to his house and
shouing and screaming for his wife and children. Which of the following defense mechanism?
A. Sublimation B. Displacement Answer: B
1680. What is the gland that inside urethral canal that responsible from secretion of discharge/mucus?
A. Seminal B. Bartholin C. Cowper's Answer: C
1681. There is a conflict between 2 staff nurse about annual leave. Each of them needs to take this year. The
nurse supervisor decided that one takes her annual leave this year and the other one wait to next year. What is
the type of conflict solutions?
A. Accommodating B. Avoiding C. Compromise Answer: A

1682. How many millitters of fluids should the breastfeeding women increase more than her needs?
A. 100 ml B. 400 ml C. 600 ml D. 800 ml Answer: D
1683. What is the assessment for cyst?
A. Inspection B. Palpation C. Percussion Answer: B
1684. When using an internal fetal heart rate monitor, you must ensure first for which of the following?
A. Complete dilatation of cervix B. Rupture of membranes Answer: B
1685. Night shift staff complaint to Nurse Manger the day shift staff and versa. What should the nurse
manager do?
A. Schedule a meeting B. gradual shifting for the staffing between night and day shift Answer: B
1686. Patient with casted leg on discharge. Which of the following instructions should the nurse include?
A. Rise your leg 90 degrees B. Remove cast after finishing treatment
C. Inform doctor for any abnormalities in circulation Answer: C
1687. What is the purpose of taking history from the patient during examination?
A. Put in the patient file B. Improve detection of diagnosis
C. Put the goal of management Answer: B
1688. Patient came to ER diagnosed with COPD. Which of the following diagnosis consider same COPD in
characteristics? A. Air hunger B. Asthma C. Pneumonia Answer: A
1689. Most of patients admitted to hospital doctors ordered DAST-10 test for them. What is this test for?
A. Misuse B. Overuse C. Lowuse Answer: A

ِ‫يِاْلَ أرض‬
‫أ‬ ُ ‫َم‬
ُ ‫ك‬
‫ثِِف‬ َ ‫اس‬
‫ِِفي أ‬َ َّ ‫ع‬
‫ِالن‬ ُ ‫ف‬
َ ‫ماِيَ أن‬ َّ َ‫وأ‬
َ ِ‫ما‬ َ
{ 188 }

1690.Which of the following test result for hyperthyroidism diagnosis?


A. T4 increased T3 decreased B. T4 decreased T3 increased Answer: A
*Hyperthyroidism test: * T4 increased T3 decreased -TSH Low
*Hypothyroidism test: * T4 decreased T3 increased -TSH High
1691. One nurse refused to give care for patient. What should the supervisor do?
A. The supervisor gives the care to patient B. Ask the nurse about the cause and problem
C. Ask the staff about the problem Answer: B
1692. Mother has a son with seizure what the nurse should teach her before discharge:
A. Remove hazardous things from the house to prevent injury B. Stable the movement if the child Answer: A

1693. Patient old and with heart disease and hypoxia. He is 70kgm. Which tidal volume should you start?
A. 400 ml B. 620 ml C. 540 ml D. 700 ml Answer: C

0100-1 :‫تجميعات شهر يناير‬


1694. Doctor order medication for psychiatric patient, the drug is not for therapeutic cause but to decrease
the concern of the patient. The nurse should expect which effect?
A. Placebo effect B. Tolerance level C. Pain control D. Addictive level Answer: A
1695. Patient with Boold transfusions start to have blood transfusion reactions the nurse stop the infusion,
what should the nurse do next?
A. Check vital signs B. Insert new IV set C. Start normal saline D. Notify physician Answer: C
1696. Elderly with sensory impairment, what the priority assessment should the nurse to do?
A- Agitation and confusion B - sensory overload C- Cognitive decline Answer: B
1697. Patient for first time of Dialysis session, what should nurse monitor for baseline
A. Weight B. Blood pressure C. Edema Answer: B
1698. Patient take erythropoietin ( epogen) how the nurse assess the effectiveness of the drug?
A. Check BP B. Hemoglobin Answer: B
1699. Child after Ventriculoperitoneal shunt surgery and the nurse note the shunt is not working, What sign
of increase ICP ?
A. Nuseae and refund to eat B. Change in neurological state C. Blood drain from ear Answer: B
1700. Postoperative patient after epidural anesthesia for surgery she is on semi fowler position and start
feeling nausea and dizziness BP 98/60 . What should the nurse do for her?
A. Change position to side lying position B. Infusion bolus of normal saline Answer: B
1701. The nurse noticed that another nurse gave false medication for her friend . The nurse didn't write
incident report or inform supervisor because both are friends. What is indicate the nurse action ?
A. Professional responsibility B. Professional misconduct Answer: B
* Raynaud phenomenon is taking Calcium channel blockers Why? Calcium channel blockers, such as
nifedipine, are confirmed as useful in reducing the frequency, duration, severity of attacks, pain and disability
associated with Raynaud's phenomenon.
1702. Every day the hospital has received many corona virus cases. They should record affected, critical and
healing persons. Where to sace all data?

ِ‫يِاْلَ أرض‬
‫أ‬ ُ ‫َم‬
ُ ‫ك‬
‫ثِِف‬ َ ‫اس‬
‫ِِفي أ‬َ َّ ‫ع‬
‫ِالن‬ ُ ‫ف‬
َ ‫ماِيَ أن‬ َّ َ‫وأ‬
َ ِ‫ما‬ َ
{ 189 }

A. Dashboard B. Nursing notes C. Nising reports Answer: A


1703. Patient with rheumatoid arthritis. The nurse instruct patient for pain relive measures . Which
intervention decrease pain if occurred?
A. During pain do flexion for your fingers B. Put warm pads early morning
C. Take pain medication frequently even if no pain Answer: B
1704. There is conflict with nurses inside the unit. The manager has solved it. Which of the following type of
leadership style?
A. Autocratic B. Democratic C. laissez-faire D. Situational Answer: A
1705. Patient with burn. The burn include epidermis and dermis layers of skin and reach to tissue. Which of
the following stage of burn?
A. First B. Second C. Third D. Fourth Answer: C
1706. Weber test for which cranial nerves? A. VI B. VIll vestibulico Answer: B
1707. The vegetarian people that not eat red meat. Which of deficiency is high risk?
A. Folic acid B. Vitamin B 12 C. Vitamin C Answer: B
1708. Pregnant woman takes folic acid supplements. What should the nurse instruct her to drink with it to
increase absorption? A. Citrus juice B. Apple juice Answer: A
1709. A 6 month old boy is admitted with weight loss below 5th percentile on growth chart. According to the
growth chart at months of age. What is the expected diagnosis?
A. Failure to thrive B. Shaken baby syndrome Answer: A
1710. Which of the following is Contraindications for tonsillectomy?
A. Sever otitis media with effusion B. Acute infection Answer: B
1711. During transferring for patient to operation room. The patient changed his mind to do surgery.
What should the nurse do?
A. Try to convince patient to do operation B. Talk with relatives
C. Say that is not according to your mind D. Inform operation staff and manager Answer: D
1712. The Saudi Ministry of health is developing education health program. All clerks not aware for the
program and not have skills to deal. What should the authority do?
A. Survey B. Gap analysis C. SWOT analysis Answer: A
1713. What Anthropometric measurements included?
A. Length ,growth and development
B. Weight, height, waist and arm circumference (body mass index) Answer: B
1714. When can late breastfeeding jaundice starts in newborn?
A. 2 - 4 weeks B. After 2 weeks C. After 3 months D. 4 - 7 weeks Answer: B
1715. The nurse was discussing with another nurse that she heard patients will make something in her lunch
break. What should the nurse do?
A. Ignore the situation B. Cancel breaks and wait to observe patients
C. One nurse goes to break and the other wait to observe Answer: C
1716. What are the complications of ruptured fallopian tube after ectopic pregnancy?
A . Peritonitis B. Intraperitoneal hemorrhage C. Decrease cardiac output Answer: B
1717. What is timing for Bone healing to child 3 years ago?
A. 2 - 4 weeks B. 4 - 6 weeks C. 6- 8 week Answer: A
1718. What are the unusual tracheotomy tube complications?

ِ‫يِاْلَ أرض‬
‫أ‬ ُ ‫َم‬
ُ ‫ك‬
‫ثِِف‬ َ ‫اس‬
‫ِِفي أ‬َ َّ ‫ع‬
‫ِالن‬ ُ ‫ف‬
َ ‫ماِيَ أن‬ َّ َ‫وأ‬
َ ِ‫ما‬ َ
{ 190 }

A. Respiratory distress syndrome B. Injury laryngeal nerve Answer: B

1719. Pregnant woman came to ER. She was worry and complained from there is no fetal movement since
night. What should the nurse do?
A. Monitor fetal heart rate B. Administration and observation C. Bed rest Answer: A
1720. Child with severe vomiting and dehydration. What is the expected color of his urine?
A. Cloudy B. Yellow C. Yellow brownish D. Bright red Answer: C
1721. What is the most common cause of CVP line infection?
A. Increase withdrawing of blood B. Increase Administering heparin
C. Increase Administering antibiotics Answer: C

1722. Patient with pneumonia on mechanical ventilator, the doctor ordered to change tidal volume from 450
to 500 and rate from 15 to 18 after see ABG results what is expected ABG result:
A. Increase Pco2 B. Decrease Po2 Answer: A

1723. Psychiatric patient diagnosed with depression. The nurse started antidepressant drug for her as doctor
order. On the third day of medication, the nurse noticed that the patient in her bed along day .Next day the
patient waked up at 4 am and planned for party in the unit. What should the nurse expect?
A. Side effects of medication B. False diagnosis of depression Answer: B

1724. Mother came to clinic for family planning methods and select contraceptives. Before the nurse telling
her which one is better, she should assess which of the following first??
A. Hypotension B. Menstruation cessation C. Hypertension Answer: C

1725. Mother admitted to hospital for surgery. She is worry about her baby at home. What should the nurse
do? A. Reassure her that her baby is fine B. Let her to call him by telephone
C. Ask her family to call Answer: A
1726. The nurse should encourage mother for breastfeeding. What is the type of immunity that acquired from
breastfeeding? A. Active B. Passive C. Natural Answer: B
1727. The hospital gave immunoglobulin for Hepatitis A to two nurses. What is the type of immunity?
A. Natural B. Active C. Passive Answer: C
1728. Child With bronchial asthma will discharge. He is on predinslone if drug exhibit. The child will
experience which of the following? A. Wight loss B. Tachycardia Answer: B
1729. Postoperative child after several days . The nurse want to do painful procedure for him . What should
the nurse do ?
A. Ask him to close his eyes and don't speak
B. Choose something to grasp his attention away to decrease pain
C. Make procedure in hospital garden Answer: B
1730. Child with pyloric stenosis. He is complained from severe vomiting. How should the nurse assess
progress condition? A. Vomiting times B. Weight loss Answer: A
1731. There is person came and took corona virus vaccine. The person experienced from pain in injection site
and he is to take the vaccine. Which of the following ethical principles?
A. Autonomy B. Beneficence C. Non-malfeasance Answer: A
1732. Postoperative patient the doctor ordered blood transfusion. The nurse assessed the patient after 15
minutes. The patient feels froozing and tight breathing. What should the nurse do first?
A. Stop infusion, start normal saline and notify the doctor

ِ‫يِاْلَ أرض‬
‫أ‬ ُ ‫َم‬
ُ ‫ك‬
‫ثِِف‬ َ ‫اس‬
‫ِِفي أ‬َ َّ ‫ع‬
‫ِالن‬ ُ ‫ف‬
َ ‫ماِيَ أن‬ َّ َ‫وأ‬
َ ِ‫ما‬ َ
{ 191 }

B. Stop infusion , cover patient with blanket and notify the doctor
C. Slow infusion rate to half and give anti allergic medication Answer: A
1733. The nurse manager trying to develop plans in future and modify mistakes. What is the term?
A. Proactive B. Inactive C. Reactive Answer: A
1734. Why should position patient in left lateral position during enema insertion?
A. Relax anal sphincter B. To facilitate solution insertion
C. This position is comfortable to patient Answer: A
1735. Pregnant woman with sever constipation. Why should pregnant woman avoid castor oil?
A. Stimulate the uterus contraction B. Increase absorption of fatty vitamins Answer: A
1736. Patient with chronic renal failure and apply hemodialysis twice per week. The nurse is giving
medication at 9 am before the session and she should start the session at 10 am. Which of the following drug
should stop after dialysis session?
A. Calcium channel blocker B. Intermittent Insulin
C. Proton pump inhibitors D. Laxatives for constipation Answer: D
1737. During physical examination and assessment for the patient. The nurse auscultate S3 heart sounds.
What is that mean?
A. This indicates a heart problem B. It's normal in children and adult
C. It's normal heart sound Answer: B
1738. Patient came to ER with motor accident. After assessment he has broken leg and arms and casted done.
What is immediately nursing intervention?
A. Check the extremities from discoloration B. Move the leg and hand to improve
C. Do exercise Answer: A
1739. Child came to ER with tea colored urine . What should the nurse expect for the diagnosis?
A. UTI B. Glomerulonephritis Answer: B
1740. Client was admitted in the emergency room due to a long-lasting chronic cough, shortness of breath. He
is diagnosed as COPD. Which of the following is a drug of choice to treat cerebral edema and increase
intracranial pressure?
A. Lasix B. Frusemide C. Digoxin D. Mannitol Answer: D
1741. A nurse is assigned to care for a client after a mastoidectomy. Which nursing intervention would be
appropriate in the care of this client?
A. Maintain a supine position. B. Position the client on the affected side to promote drainage.
C. Change the ear dressing daily. D. Monitor for signs of facial nerve injury. Answer: B
1742. Patient after kidney transplantation. The patient complained from rejection. Which of the following
signs indicate for that? A. Oliguria B. Hematuria Answer: A

1743. Postpartum neonate came for circumcision. What should the nurse give mother instruction?
A - Put pillow between legs B - Check diaper C - Clean with soap Answer: B
1744. Woman came to ER diagnosed with end stage of lung cancer. The patient is transferred to ICU. How
can the nurse communicate with her?
A. Clear instructions B. Discuss the procedures with her
C. Administer the medication without talking Answer: B
1745. Child with VPS shunt constructed. What is the expected sign?
A. nausea and refuse eat B. Neurological disturbances Answer: B
1746. What is the expected weight increasing for neonate after 6 months?

ِ‫يِاْلَ أرض‬
‫أ‬ ُ ‫َم‬
ُ ‫ك‬
‫ثِِف‬ َ ‫اس‬
‫ِِفي أ‬َ َّ ‫ع‬
‫ِالن‬ ُ ‫ف‬
َ ‫ماِيَ أن‬ َّ َ‫وأ‬
َ ِ‫ما‬ َ
{ 192 }

A. 480 gm B. 1800 gm C. 2000 gm Answer: C


1747. Patient came to ER with thrombotic stroke. The nurse should expect what is the cause?
A. Decrease of O2 In part of brain B. Obstruction by athreascelorosis Answer: B
1748. The nurse is doing CPR for patient. The nurse notes that patient chest not raise properly (no proper
recoil). What is the next action?
A. Squeeze oxygen bag harder B. Change with another nurse
C. Connect to oxygen Answer: B
1749. Woman after mastectomy transferred to ward. After doing assessment. Which of the following nursing
diagnosis is priority?
A. Inffective airway clearance related to anesthesia
B. Disturebed body image related to breast removed Answer: B
1750. How many cm should the baby increase length after 6 months of delivery ?
A. 0.5 B. 1,5 C. 2,5 D. 3,5 Answer: B
1751. Nurse has one patient. His father is the hospital manager. The nurse told patient to speak with his father
and let her work with him and get job. What is nurse action name?
A. Professional Accountability B. Crossing professional boundaries Answer: B
1752. Patient always takes I.M injection medication at same site. What could patient high risk for?
A. Deep Vein thrombosis B. Muscle Fibrosis C. Tissue damage D. Bleeding Answer: B
1753. The nurse is recording patient care. She is worry from making mistakes on computer internet during
documentation. What is the best action?
A. Review medical records before documentation.
B. After giving each care do documentation directly. Answer: B
1754. During documentation the nurse forgot to record important notes and doctor recommends in patient
chart. What should the nurse do?
A. Next time she will do correct documentation and not forget
B. Cut the paper and write again the missed point and doctor recommends
C. Write the missed notes and put late documentation beside and the reason for late Answer: C
1755. Child with extremely obese. What is the expected BMI range ?
A. 30-39 B. 18-25
C. 20-29 D. >40 Answer: D
Which of the following helps the kidney to control Blood pressure and urine output ?
A. Renin release B. ADH Answer: A

ِ‫يِاْلَ أرض‬
‫أ‬ ُ ‫َم‬
ُ ‫ك‬
‫ثِِف‬ َ ‫اس‬
‫ِِفي أ‬َ َّ ‫ع‬
‫ِالن‬ ُ ‫ف‬
َ ‫ماِيَ أن‬ َّ َ‫وأ‬
َ ِ‫ما‬ َ

You might also like